You are on page 1of 114

Tecson Vs. Comelec Case Digest Tecson Vs. Comelec 424 SCRA 277 G.R. No.

161434 March 3, 2004 Facts: Victorino X. Fornier, petitioner initiated a petition before the COMELEC to disqualify FPJ and to deny due course or to cancel his certificate of candidacy upon the thesis that FPJ made a material misrepresentation in his certificate of candidacy by claiming to be a naturalborn Filipino citizen when in truth, according to Fornier, his parents were foreigners; his mother, Bessie Kelley Poe, was an American, and his father, Allan Poe, was a Spanish national, being the son of Lorenzo Pou, a Spanish subject. Granting, petitioner asseverated, that Allan F. Poe was a Filipino citizen, he could not have transmitted his Filipino citizenship to FPJ, the latter being an illegitimate child of an alien mother. Petitioner based the allegation of the illegitimate birth of respondent on two assertions - first, Allan F. Poe contracted a prior marriage to a certain Paulita Gomez before his marriage to Bessie Kelley and, second, even if no such prior marriage had existed, Allan F. Poe, married Bessie Kelly only a year after the birth of respondent. Issue: Whether or Not FPJ is a natural born Filipino citizen. Held: It is necessary to take on the matter of whether or not respondent FPJ is a natural-born citizen, which, in turn, depended on whether or not the father of respondent, Allan F. Poe, would have himself been a Filipino citizen and, in the affirmative, whether or not the alleged illegitimacy of respondent prevents him from taking after the Filipino citizenship of his putative father. Any conclusion on the Filipino citizenship of Lorenzo Pou could only be drawn from the presumption that having died in 1954 at 84 years old, Lorenzo would have been born sometime in the year 1870, when the Philippines was under Spanish rule, and that San Carlos, Pangasinan, his place of residence upon his death in 1954, in the absence of any other evidence, could have well been his place of residence before death, such that Lorenzo Pou would have benefited from the "en masse Filipinization" that the Philippine Bill had effected in 1902. That citizenship (of Lorenzo Pou), if acquired, would thereby extend to his son, Allan F. Poe, father of respondent FPJ. The 1935 Constitution, during which regime respondent FPJ has seen first light, confers citizenship to all persons whose fathers are Filipino citizens regardless of whether such children are legitimate or illegitimate. But while the totality of the evidence may not establish conclusively that respondent FPJ is a natural-born citizen of the Philippines, the evidence on hand still would preponderate in his favor enough to hold that he cannot be held guilty of having made a material misrepresentation in his certificate of candidacy in violation of Section 78, in relation to Section 74, of the Omnibus Election Code.

EN BANC

[G.R. No. 161434. March 3, 2004]

MARIA JEANETTE C. TECSON and FELIX B. DESIDERIO, JR., petitioners, vs. The COMMISSION ON ELECTIONS, RONALD ALLAN KELLY POE (a.k.a. FERNANDO POE, JR.) and VICTORINO X. FORNIER, respondents.

[G.R. No. 161634. March 3, 2004]

ZOILO ANTONIO VELEZ, petitioner, vs. RONALD ALLAN KELLEY POE, a.k.a. FERNANDO POE, JR., respondent.

[G. R. No. 161824. March 3, 2004]

VICTORINO X. FORNIER, petitioner, vs. HON. COMMISSION ON ELECTIONS and RONALD ALLAN KELLEY POE, ALSO KNOWN AS FERNANDO POE JR., respondents. DECISION VITUG, J.:

Citizenship is a treasured right conferred on those whom the state believes are deserving of the privilege. It is a precious heritage, as well as an inestimable acquisition,[1] that cannot be taken lightly by anyone - either by those who enjoy it or by those who dispute it. Before the Court are three consolidated cases, all of which raise a single question of profound importance to the nation. The issue of citizenship is brought up to challenge the qualifications of a presidential candidate to hold the highest office of the land. Our people are waiting for the judgment of the Court with bated breath. Is Fernando Poe, Jr., the hero of silver screen, and now one of the main contenders for the presidency, a natural-born Filipino or is he not? The moment of introspection takes us face to face with Spanish and American colonial roots and reminds us of the rich heritage of civil law and common law traditions, the fusion resulting in a hybrid of laws and jurisprudence that could be no less than distinctly Filipino.

Antecedent Case Settings On 31 December 2003, respondent Ronald Allan Kelly Poe, also known as Fernando Poe, Jr. (hereinafter "FPJ"), filed his certificate of candidacy for the position of President of the Republic of the Philippines under the Koalisyon ng Nagkakaisang Pilipino (KNP) Party, in the forthcoming national elections. In his certificate of candidacy, FPJ, representing himself to be a natural-born citizen of the Philippines, stated his name to be "Fernando Jr.," or "Ronald Allan" Poe, his date of birth to be 20 August 1939 and his place of birth to beManila. Victorino X. Fornier, petitioner in G.R. No. 161824, entitled "Victorino X. Fornier, Petitioner, versus Hon. Commission on Elections and Ronald Allan Kelley Poe, also known as Fernando Poe, Jr., Respondents," initiated, on 09 January 2004, a petition docketed SPA No. 04-003 before the Commission on Elections ("COMELEC") to disqualify FPJ and to deny due course or to cancel his certificate of candidacy upon the thesis that FPJ made a material misrepresentation in his certificate of candidacy by claiming to be a natural-born Filipino citizen when in truth, according to Fornier, his parents were foreigners; his mother, Bessie Kelley Poe, was an American, and his father, Allan Poe, was a Spanish national, being the son of Lorenzo Pou, a Spanish subject. Granting, petitioner asseverated, that Allan F. Poe was a Filipino citizen, he could not have transmitted his Filipino citizenship to FPJ, the latter being an illegitimate child of an alien mother. Petitioner based the allegation of the illegitimate birth of respondent on two assertions - first, Allan F. Poe contracted a prior marriage to a certain Paulita Gomez before his marriage to Bessie Kelley and, second, even if no such prior marriage had existed, Allan F. Poe, married Bessie Kelly only a year after the birth of respondent. In the hearing before the Third Division of the COMELEC on 19 January 2004, petitioner, in support of his claim, presented several documentary exhibits - 1) a copy of the certificate of birth of FPJ, 2) a certified photocopy of an affidavit executed in Spanish by Paulita Poe y Gomez attesting to her having filed a case for bigamy and concubinage against the father of respondent, Allan F. Poe, after discovering his bigamous relationship with Bessie Kelley, 3) an English translation of the affidavit aforesaid, 4) a certified photocopy of the certificate of birth of Allan F. Poe, 5) a certification issued by the Director of the Records Management and Archives Office, attesting to the fact that there was no record in the National Archives that a Lorenzo Poe or Lorenzo Pou resided or entered the Philippines before 1907, and 6) a certification from the Officer-In-Charge of the Archives Division of the National Archives to the effect that no available information could be found in the files of the National Archives regarding the birth of Allan F. Poe. On his part, respondent, presented twenty-two documentary pieces of evidence, the more significant ones being - a) a certification issued by Estrella M. Domingo of the Archives Division of the National Archives that there appeared to be no available information regarding the birth of Allan F. Poe in the registry of births for San Carlos, Pangasinan, b) a certification issued by the Officer-In-Charge of the Archives Division of the National Archives that no available information about the marriage of Allan F. Poe and Paulita Gomez could be found, c) a certificate of birth of Ronald Allan Poe, d) Original Certificate of Title No. P-2247 of the Registry of Deeds for the Province of Pangasinan, in the name of Lorenzo Pou, e) copies of Tax Declaration No. 20844, No. 20643, No. 23477 and No. 23478 in the name of Lorenzo Pou, f) a copy of the certificate of death of Lorenzo Pou, g) a copy of the purported marriage contract between Fernando Pou and Bessie Kelley, and h) a certification issued by the City Civil Registrar of San Carlos City, Pangasinan, stating that the records of birth in the said office during the period of from 1900 until May 1946 were totally destroyed during World War II. On 23 January 2004, the COMELEC dismissed SPA No. 04-003 for lack of merit. Three days later, or on 26 January 2004, Fornier filed his motion for reconsideration. The motion was denied on 06 February 2004 by the COMELEC en banc. On 10 February 2004, petitioner assailed the decision of the COMELEC before this Court conformably with Rule 64, in relation to Rule 65, of the Revised Rules of Civil Procedure. The petition, docketed G. R. No. 161824, likewise prayed for a temporary restraining order, a writ of preliminary injunction or any other resolution that would stay the finality and/or execution of the COMELEC resolutions. The other petitions, later consolidated with G. R. No. 161824, would include G. R. No. 161434, entitled "Maria Jeanette C. Tecson, and Felix B. Desiderio, Jr., vs. The Commission on Elections, Ronald Allan Kelley Poe (a.k.a. Fernando Poe, Jr.), and Victorino X. Fornier," and the other, docketed G. R. No. 161634, entitled "Zoilo Antonio G. Velez, vs. Ronald Allan Kelley Poe, a.k.a. Fernando Poe, Jr.," both challenging the jurisdiction of the COMELEC and asserting that, under Article VII, Section 4, paragraph 7, of the 1987 Constitution, only the Supreme Court had original and exclusive jurisdiction to resolve the basic issue on the case.

Jurisdiction of the Court In G. R. No. 161824 In seeking the disqualification of the candidacy of FPJ and to have the COMELEC deny due course to or cancel FPJs certificate of candidacy for alleged misrepresentation of a material fact (i.e., that FPJ was a natural-born citizen) before the COMELEC, petitioner Fornier invoked Section 78 of the Omnibus Election Code Section 78. Petition to deny due course to or cancel a certificate of candidacy. --- A verified petition seeking to deny due course or to cancel a certificate of candidacy may be filed by any person exclusively on the ground that any material representation contained therein as required under Section 74 hereof is false

in consonance with the general powers of COMELEC expressed in Section 52 of the Omnibus Election Code Section 52. Powers and functions of the Commission on Elections. In addition to the powers and functions conferred upon it by the Constitution, the Commission shall have exclusive charge of the enforcement and administration of all laws relative to the conduct of elections for the purpose of ensuring free, orderly and honest elections and in relation to Article 69 of the Omnibus Election Code which would authorize "any interested party" to file a verified petition to deny or cancel the certificate of candidacy of any nuisance candidate. Decisions of the COMELEC on disqualification cases may be reviewed by the Supreme Court per Rule 64[2] in an action for certiorari under Rule 65[3] of the Revised Rules of Civil Procedure. Section 7, Article IX, of the 1987 Constitution also reads "Each Commission shall decide by a majority vote of all its Members any case or matter brought before it within sixty days from the date of its submission for decision or resolution. A case or matter is deemed submitted for decision or resolution upon the filing of the last pleading, brief, or memorandum, required by the rules of the Commission or by the Commission itself. Unless otherwise provided by this Constitution or by law, any decision, order, or ruling of each Commission may be brought to the Supreme Court on certiorari by the aggrieved party within thirty days from receipt of a copy thereof." Additionally, Section 1, Article VIII, of the same Constitution provides that judicial power is vested in one Supreme Court and in such lower courts as may be established by law which power includes the duty of the courts of justice to settle actual controvers ies involving rights which are legally demandable and enforceable, and to determine whether or not there has been a grave abuse of discretion amounting to lack or excess of jurisdiction on the part of any branch or instrumentality of the Government. It is sufficiently clear that the petition brought up in G. R. No. 161824 was aptly elevated to, and could well be taken cognizance of by, this Court. A contrary view could be a gross denial to our people of their fundamental right to be fully informed, and to make a proper choice, on who could or should be elected to occupy the highest government post in the land. In G. R. No. 161434 and G. R. No. 161634 Petitioners Tecson, et al., in G. R. No. 161434, and Velez, in G. R. No. 161634, invoke the provisions of Article VII, Section 4, paragraph 7, of the 1987 Constitution in assailing the jurisdiction of the COMELEC when it took cognizance of SPA No. 04-003 and in urging the Supreme Court to instead take on the petitions they directly instituted before it. The Constitutional provision cited reads: "The Supreme Court, sitting en banc, shall be the sole judge of all contests relating to the election, returns, and qualifications of the President or Vice-President, and may promulgate its rules for the purpose." The provision is an innovation of the 1987 Constitution. The omission in the 1935 and the 1973 Constitution to designate any tribunal to be the sole judge of presidential and vice-presidential contests, has constrained this Court to declare, in Lopez vs. Roxas,[4] as not (being) justiciable controversies or disputes involving contests on the elections, returns and qualifications of the President or Vice-President. The constitutional lapse prompted Congress, on 21 June 1957, to enact Republic Act No. 1793, "An Act Constituting an Independent Presidential Electoral Tribunal to Try, Hear and Decide Protests Contesting the Election of the President-Elect and the Vice-President-Elect of the Philippines and Providing for the Manner of Hearing the Same." Republic Act 1793 designated the Chief Justice and the Associate Justices of the Supreme Court to be the members of the tribunal. Although the subsequent adoption of the parliamentary form of government under the 1973 Constitution might have implicitly affected Republic Act No. 1793, the statutory set-up, nonetheless, would now be deemed revived under the present Section 4, paragraph 7, of the 1987 Constitution. Ordinary usage would characterize a "contest" in reference to a post-election scenario. Election contests consist of either an election protest or a quo warranto which, although two distinct remedies, would have one objective in view, i.e., to dislodge the winning candidate from office. A perusal of the phraseology in Rule 12, Rule 13, and Rule 14 of the"Rules of the Presidential Electoral Tribunal," promulgated by the Supreme Court en banc on 18 April 1992, would support this premise Rule 12. Jurisdiction. - The Tribunal shall be the sole judge of all contests relating to the election, returns, and qualifications of the President or Vice-President of the Philippines. Rule 13. How Initiated. - An election contest is initiated by the filing of an election protest or a petition for quo warranto against the President or Vice-President. An election protest shall not include a petition for quo warranto. A petition for quo warranto shall not include an election protest. Rule 14. Election Protest. - Only the registered candidate for President or for Vice-President of the Philippines who received the second or third highest number of votes may contest the election of the President or the Vice-President, as the case may be, by filing a verified petition with the Clerk of the Presidential Electoral Tribunal within thirty (30) days after the proclamation of the winner. The rules categorically speak of the jurisdiction of the tribunal over contests relating to the election, returns and qualifications of the "President" or "Vice-President", of thePhilippines, and not of "candidates" for President or Vice-President. A quo warranto proceeding is generally defined as being an action against a person who usurps, intrudes into, or unlawfully holds or exercises a public office.[5] In such context, the election contest can only contemplate a post-election scenario. In Rule 14, only a registered candidate who would have received either the second or third highest number of votes could file an election protest. This rule again presupposes a post-election scenario. It is fair to conclude that the jurisdiction of the Supreme Court, defined by Section 4, paragraph 7, of the 1987 Constitution, would not include cases directly brought before it, questioning the qualifications of a candidate for the presidency or vice-presidency before the elections are held. Accordingly, G. R. No. 161434, entitled "Maria Jeanette C. Tecson, et al., vs. Commission on Elections et al.," and G. R. No. 161634, entitled "Zoilo Antonio Velez vs. Ronald Allan Kelley Poe a.k.a. Fernando Poe, Jr." would have to be dismissed for want of jurisdiction.

The Citizenship Issue Now, to the basic issue; it should be helpful to first give a brief historical background on the concept of citizenship. Perhaps, the earliest understanding of citizenship was that given by Aristotle, who, sometime in 384 to 322 B.C., described the "citizen" to refer to a man who shared in the administration of justice and in the holding of an office. [6] Aristotle saw its significance if only to determine the constituency of the "State," which he described as being composed of such persons who would be adequate in number to achieve a selfsufficient existence.[7] The concept grew to include one who would both govern and be governed, for which qualifications like autonomy, judgment and loyalty could be expected. Citizenship was seen to deal with rights and entitlements, on the one hand, and with concomitant obligations, on the other.[8] In its ideal setting, a citizen was active in public life and fundamentally willing to submit his private interests to the general interest of society. The concept of citizenship had undergone changes over the centuries. In the 18th century, the concept was limited, by and large, to civil citizenship, which established the rights necessary for individual freedom, such as rights to property, personal liberty and justice. [9] Its meaning expanded during the 19th century to include political citizenship, which encompassed the right to participate in the exercise of political power.[10] The 20th century saw the next stage of the development of social citizenship, which laid emphasis on the right of the citizen to economic well-being and social security.[11] The idea of citizenship has gained expression in the modern welfare state as it so developed in Western Europe. An ongoing and final stage of development, in keeping with the rapidly shrinking global village, might well be the internationalization of citizenship.[12]

The Local Setting - from Spanish Times to the Present There was no such term as "Philippine citizens" during the Spanish regime but "subjects of Spain" or "Spanish subjects."[13] In church records, the natives were called 'indios', denoting a low regard for the inhabitants of the archipelago. Spanish laws on citizenship became highly codified during the 19th century but their sheer number made it difficult to point to one comprehensive law. Not all of these citizenship laws of Spain however, were made to apply to the Philippine Islands except for those explicitly extended by Royal Decrees. [14] Spanish laws on citizenship were traced back to the Novisima Recopilacion, promulgated in Spain on 16 July 1805 but as to whether the law was extended to the Philippines remained to be the subject of differing views among experts; [15] however, three royal decrees were undisputably made applicable to Spaniards in the Philippines - the Order de la Regencia of 14 August 1841,[16] the Royal Decree of 23 August 1868 specifically defining the political status of children born in the Philippine Islands, [17] and finally, the Ley Extranjera de Ultramar of 04 July 1870, which was expressly made applicable to the Philippines by the Royal Decree of 13 July 1870.[18] The Spanish Constitution of 1876 was never extended to the Philippine Islands because of the express mandate of its Article 89, according to which the provisions of theUltramar among which this country was included, would be governed by special laws.[19] It was only the Civil Code of Spain, made effective in this jurisdiction on 18 December 1889, which came out with the first categorical enumeration of who were Spanish citizens. (a) Persons born in Spanish territory, (b) Children of a Spanish father or mother, even if they were born outside of Spain, (c) Foreigners who have obtained naturalization papers, (d) Those who, without such papers, may have become domiciled inhabitants of any town of the Monarchy.[20] The year 1898 was another turning point in Philippine history. Already in the state of decline as a superpower, Spain was forced to so cede her sole colony in the East to an upcoming world power, the United States. An accepted principle of international law dictated that a change in sovereignty, while resulting in an abrogation of all political laws then in force, would have no effect on civil laws, which would remain virtually intact. The Treaty of Paris was entered into on 10 December 1898 between Spain and the United States.[21] Under Article IX of the treaty, the civil rights and political status of the native inhabitants of the territories ceded to the United States would be determined by its Congress "Spanish subjects, natives of the Peninsula, residing in the territory over which Spain by the present treaty relinquishes or cedes her sovereignty may remain in such territory or may remove therefrom, retaining in either event all their rights of property, including the right to sell or dispose of such property or of its proceeds; and they shall also have the right to carry on their industry, commerce, and professions, being subject in respect thereof to such laws as are applicable to foreigners. In case they remain in the territory they may preserve their allegiance to the Crown of Spain by making, before a court of record, within a year from the date of the exchange of ratifications of this treaty, a declaration of their decision to preserve such allegiance; in default of which declaration they shall be held to have renounced it and to have adopted the nationality of the territory in which they reside. Thus "The civil rights and political status of the native inhabitants of the territories hereby ceded to the United States shall be determined by the Congress."[22] Upon the ratification of the treaty, and pending legislation by the United States Congress on the subject, the native inhabitants of the Philippines ceased to be Spanish subjects. Although they did not become American citizens, they, however, also ceased to be "aliens" under American laws and were thus issued passports describing them to be citizens of the Philippines entitled to the protection of the United States.

The term "citizens of the Philippine Islands" appeared for the first time in the Philippine Bill of 1902, also commonly referred to as the Philippine Organic Act of 1902, the first comprehensive legislation of the Congress of the United States on the Philippines ".... that all inhabitants of the Philippine Islands continuing to reside therein, who were Spanish subjects on the 11th day of April, 1891, and then resided in said Islands, and their children born subsequent thereto, shall be deemed and held to be citizens of the Philippine Islands and as such entitled to the protection of the United States, except such as shall have elected to preserve their allegiance to the Crown of Spain in accordance with the provisions of the treaty of peace between the United States and Spain, signed at Paris, December tenth eighteen hundred and ninety eight."[23] Under the organic act, a citizen of the Philippines was one who was an inhabitant of the Philippines, and a Spanish subject on the 11th day of April 1899. The term inhabitant was taken to include 1) a native-born inhabitant, 2) an inhabitant who was a native of Peninsular Spain, and 3) an inhabitant who obtained Spanish papers on or before 11 April 1899.[24] Controversy arose on to the status of children born in the Philippines from 11 April 1899 to 01 July 1902, during which period no citizenship law was extant in the Philippines. Weight was given to the view, articulated in jurisprudential writing at the time, that the common law principle of jus soli, otherwise also known as the principle of territoriality, operative in the United States and England, governed those born in the Philippine Archipelago within that period.[25] More about this later. In 23 March 1912, the Congress of the United States made the following amendment to the Philippine Bill of 1902 "Provided, That the Philippine Legislature is hereby authorized to provide by law for the acquisition of Philippine citizenship by those natives of the Philippine Islands who do not come within the foregoing provisions, the natives of other insular possession of the United States, and such other persons residing in the Philippine Islands who would become citizens of the United States, under the laws of the United States, if residing therein."[26] With the adoption of the Philippine Bill of 1902, the concept of "Philippine citizens" had for the first time crystallized. The word "Filipino" was used by William H. Taft, the first Civil Governor General in the Philippines when he initially made mention of it in his slogan, "The Philippines for the Filipinos." In 1916, the Philippine Autonomy Act, also known as the Jones Law restated virtually the provisions of the Philippine Bill of 1902, as so amended by the Act of Congress in 1912 That all inhabitants of the Philippine Islands who were Spanish subjects on the eleventh day of April, eighteen hundred and ninetynine, and then resided in said Islands, and their children born subsequently thereto, shall be deemed and held to be citizens of the Philippine Islands, except such as shall have elected to preserve their allegiance to the Crown of Spain in accordance with the provisions of the treaty of peace between the United States and Spain, signed at Paris December tenth, eighteen hundred and ninety-eight and except such others as have since become citizens of some other country; Provided, That the Philippine Legislature, herein provided for, is hereby authorized to provide for the acquisition of Philippine citizenship by those natives of the Philippine Islands who do not come within the foregoing provisions, the natives of the insular possessions of the United States, and such other persons residing in the Philippine Islands who are citizens of the United States, or who could become citizens of the United States under the laws of the United States, if residing therein." Under the Jones Law, a native-born inhabitant of the Philippines was deemed to be a citizen of the Philippines as of 11 April 1899 if he was 1) a subject of Spain on 11 April 1899, 2) residing in the Philippines on said date, and, 3) since that date, not a citizen of some other country. While there was, at one brief time, divergent views on whether or not jus soli was a mode of acquiring citizenship, the 1935 Constitution brought to an end to any such link with common law, by adopting, once and for all, jus sanguinis or blood relationship as being the basis of Filipino citizenship Section 1, Article III, 1935 Constitution. The following are citizens of the Philippines (1) Those who are citizens of the Philippine Islands at the time of the adoption of this Constitution (2) Those born in the Philippines Islands of foreign parents who, before the adoption of this Constitution, had been elected to public office in the Philippine Islands. (3) Those whose fathers are citizens of the Philippines. (4) Those whose mothers are citizens of the Philippines and upon reaching the age of majority, elect Philippine citizenship. (5) Those who are naturalized in accordance with law. Subsection (4), Article III, of the 1935 Constitution, taken together with existing civil law provisions at the time, which provided that women would automatically lose their Filipino citizenship and acquire that of their foreign husbands, resulted in discriminatory situations that effectively incapacitated the women from transmitting their Filipino citizenship to their legitimate children and required illegitimate children of Filipino mothers to still elect Filipino citizenship upon reaching the age of majority. Seeking to correct this anomaly, as well as fully cognizant of the newly found status of Filipino women as equals to men, the framers of the 1973 Constitution crafted the provisions of the new Constitution on citizenship to reflect such concerns Section 1, Article III, 1973 Constitution - The following are citizens of the Philippines: (1) Those who are citizens of the Philippines at the time of the adoption of this Constitution.

(2) Those whose fathers or mothers are citizens of the Philippines. (3) Those who elect Philippine citizenship pursuant to the provisions of the Constitution of nineteen hundred and thirty-five. (4) Those who are naturalized in accordance with law. For good measure, Section 2 of the same article also further provided that "A female citizen of the Philippines who marries an alien retains her Philippine citizenship, unless by her act or omission she is deemed, under the law to have renounced her citizenship." The 1987 Constitution generally adopted the provisions of the 1973 Constitution, except for subsection (3) thereof that aimed to correct the irregular situation generated by the questionable proviso in the 1935 Constitution. Section I, Article IV, 1987 Constitution now provides: The following are citizens of the Philippines: (1) Those who are citizens of the Philippines at the time of the adoption of this Constitution. (2) Those whose fathers or mothers are citizens of the Philippines. (3) Those born before January 17, 1973 of Filipino mothers, who elect Philippine citizenship upon reaching the age of majority; and (4) Those who are naturalized in accordance with law.

The Case Of FPJ Section 2, Article VII, of the 1987 Constitution expresses: "No person may be elected President unless he is a natural-born citizen of the Philippines, a registered voter, able to read and write, at least forty years of age on the day of the election, and a resident of the Philippines for at least ten years immediately preceding such election." The term "natural-born citizens," is defined to include "those who are citizens of the Philippines from birth without having to perform any act to acquire or perfect their Philippine citizenship."[27] The date, month and year of birth of FPJ appeared to be 20 August 1939 during the regime of the 1935 Constitution. Through its history, four modes of acquiring citizenship - naturalization, jus soli, res judicata and jus sanguinis[28] had been in vogue. Only two, i.e., jus soli and jus sanguinis, could qualify a person to being a natural-born citizen of the Philippines. Jus soli, per Roa vs. Collector of Customs[29] (1912), did not last long. With the adoption of the 1935 Constitution and the reversal of Roa in Tan Chong vs. Secretary of Labor[30] (1947), jus sanguinis or blood relationship would now become the primary basis of citizenship by birth. Documentary evidence adduced by petitioner would tend to indicate that the earliest established direct ascendant of FPJ was his paternal grandfather Lorenzo Pou, married to Marta Reyes, the father of Allan F. Poe. While the record of birth of Lorenzo Pou had not been presented in evidence, his death certificate, however, identified him to be a Filipino, a resident of San Carlos, Pangasinan, and 84 years old at the time of his death on 11 September 1954. The certificate of birth of the father of FPJ, Allan F. Poe, showed that he was born on 17 May 1915 to an Espaol father, Lorenzo Pou, and a mestiza Espaol mother, Marta Reyes. Introduced by petitioner was an uncertified copy of a supposed certificate of the alleged marriage of Allan F. Poe and Paulita Gomez on 05 July 1936. The marriage certificate of Allan F. Poe and Bessie Kelley reflected the date of their marriage to be on 16 September 1940. In the same certificate, Allan F. Poe was stated to be twentyfive years old, unmarried, and a Filipino citizen, and Bessie Kelley to be twenty-two years old, unmarried, and an American citizen. The birth certificate of FPJ, would disclose that he was born on 20 August 1939 to Allan F. Poe, a Filipino, twenty-four years old, married to Bessie Kelly, an American citizen, twenty-one years old and married. Considering the reservations made by the parties on the veracity of some of the entries on the birth certificate of respondent and the marriage certificate of his parents, the only conclusions that could be drawn with some degree of certainty from the documents would be that 1. 2. 3. 4. 5. The parents of FPJ were Allan F. Poe and Bessie Kelley; FPJ was born to them on 20 August 1939; Allan F. Poe and Bessie Kelley were married to each other on 16 September, 1940; The father of Allan F. Poe was Lorenzo Poe; and At the time of his death on 11 September 1954, Lorenzo Poe was 84 years old.

Would the above facts be sufficient or insufficient to establish the fact that FPJ is a natural-born Filipino citizen? The marriage certificate of Allan F. Poe and Bessie Kelley, the birth certificate of FPJ, and the death certificate of Lorenzo Pou are documents of public record in the

custody of a public officer. The documents have been submitted in evidence by both contending parties during the proceedings before the COMELEC. The birth certificate of FPJ was marked Exhibit "A" for petitioner and Exhibit "3" for respondent. The marriage certificate of Allan F. Poe to Bessie Kelley was submitted as Exhibit "21" for respondent. The death certificate of Lorenzo Pou was submitted by respondent as his Exhibit "5." While the last two documents were submitted in evidence for respondent, the admissibility thereof, particularly in reference to the facts which they purported to show, i.e., the marriage certificate in relation to the date of marriage of Allan F. Poe to Bessie Kelley and the death certificate relative to the death of Lorenzo Pou on 11 September 1954 in San Carlos, Pangasinan, were all admitted by petitioner, who had utilized those material statements in his argument. All three documents were certified true copies of the originals. Section 3, Rule 130, Rules of Court states that Original document must be produced; exceptions. - When the subject of inquiry is the contents of a document, no evidence shall be admissible other than the original document itself, except in the following cases: x x x xxx xxx

(d) When the original is a public record in the custody of a public office or is recorded in a public office. Being public documents, the death certificate of Lorenzo Pou, the marriage certificate of Allan F. Poe and Bessie Kelly, and the birth certificate of FPJ, constitute prima facie proof of their contents. Section 44, Rule 130, of the Rules of Court provides: Entries in official records. Entries in official records made in the performance of his duty by a public officer of the Philippines, or by a person in the performance of a duty specially enjoined by law, areprima facie evidence of the facts therein stated. The trustworthiness of public documents and the value given to the entries made therein could be grounded on 1) the sense of official duty in the preparation of the statement made, 2) the penalty which is usually affixed to a breach of that duty, 3) the routine and disinterested origin of most such statements, and 4) the publicity of record which makes more likely the prior exposure of such errors as might have occurred.[31] The death certificate of Lorenzo Pou would indicate that he died on 11 September 1954, at the age of 84 years, in San Carlos, Pangasinan. It could thus be assumed that Lorenzo Pou was born sometime in the year 1870 when the Philippines was still a colony of Spain. Petitioner would argue that Lorenzo Pou was not in the Philippines during the crucial period of from 1898 to 1902 considering that there was no existing record about such fact in the Records Management and Archives Office. Petitioner, however, likewise failed to show that Lorenzo Pou was at any other place during the same period. In his death certificate, the residence of Lorenzo Pou was stated to be San Carlos, Pangasinan. In the absence of any evidence to the contrary, it should be sound to conclude, or at least to presume, that the place of residence of a person at the time of his death was also his residence before death. It would be extremely doubtful if the Records Management and Archives Office would have had complete records of all residents of the Philippines from 1898 to 1902.

Proof of Paternity and Filiation Under Civil Law. Petitioner submits, in any case, that in establishing filiation (relationship or civil status of the child to the father [or mother]) or paternity (relationship or civil status of the father to the child) of an illegitimate child, FPJ evidently being an illegitimate son according to petitioner, the mandatory rules under civil law must be used. Under the Civil Code of Spain, which was in force in the Philippines from 08 December 1889 up until the day prior to 30 August 1950 when the Civil Code of the Philippines took effect, acknowledgment was required to establish filiation or paternity. Acknowledgment was either judicial (compulsory) or voluntary. Judicial or compulsory acknowledgment was possible only if done during the lifetime of the putative parent; voluntary acknowledgment could only be had in a record of birth, a will, or a public document. [32] Complementary to the new code was Act No. 3753 or the Civil Registry Law expressing in Section 5 thereof, that In case of an illegitimate child, the birth certificate shall be signed and sworn to jointly by the parents of the infant or only by the mother if the father refuses. In the latter case, it shall not be permissible to state or reveal in the document the name of the father who refuses to acknowledge the child, or to give therein any information by which such father could be identified. In order that the birth certificate could then be utilized to prove voluntary acknowledgment of filiation or paternity, the certificate was required to be signed or sworn to by the father. The failure of such requirement rendered the same useless as being an authoritative document of recognition.[33] In Mendoza vs. Mella,[34] the Court ruled "Since Rodolfo was born in 1935, after the registry law was enacted, the question here really is whether or not his birth certificate (Exhibit 1), which is merely a certified copy of the registry record, may be relied upon as sufficient proof of his having been voluntarily recognized. No such reliance, in our judgment, may be placed upon it. While it contains the names of both parents, there is no showing that they signed the original, let alone swore to its contents as required in Section 5 of Act No. 3753. For all that might have happened, it was not even they or either of them who furnished the data to be entered in the civil register. Petitioners say that in any event the birth certificate is in the nature of a public document wherein voluntary recognition of a natural child may also be made, according to the same Article 131. True enough, but in such a case, there must be a clear statement in the document that the parent recognizes the child as his or her own." In the birth certificate of respondent FPJ, presented by both parties, nowhere in the document was the signature of Allan F. Poe found. There being no will apparently executed, or at least shown to have been executed, by decedent Allan F. Poe, the only other proof of

voluntary recognition remained to be "some other public document." In Pareja vs. Pareja,[35]this Court defined what could constitute such a document as proof of voluntary acknowledgment: "Under the Spanish Civil Code there are two classes of public documents, those executed by private individuals which must be authenticated by notaries, and those issued by competent public officials by reason of their office. The public document pointed out in Article 131 as one of the means by which recognition may be made belongs to the first class." Let us leave it at that for the moment. The 1950 Civil Code categorized the acknowledgment or recognition of illegitimate children into voluntary, legal or compulsory. Voluntary recognition was required to be expressedly made in a record of birth, a will, a statement before a court of record or in any authentic writing. Legal acknowledgment took place in favor of full blood brothers and sisters of an illegitimate child who was recognized or judicially declared as natural. Compulsory acknowledgment could be demanded generally in cases when the child had in his favor any evidence to prove filiation. Unlike an action to claim legitimacy which would last during the lifetime of the child, and might pass exceptionally to the heirs of the child, an action to claim acknowledgment, however, could only be brought during the lifetime of the presumed parent. Amicus Curiae Ruben F. Balane defined, during the oral argument, "authentic writing," so as to be an authentic writing for purposes of voluntary recognition, simply as being a genuine or indubitable writing of the father. The term would include a public instrument (one duly acknowledged before a notary public or other competent official) or a private writing admitted by the father to be his. The Family Code has further liberalized the rules; Article 172, Article 173, and Article 175 provide: Art. 172. The filiation of legitimate children is established by any of the following:

(1) The record of birth appearing in the civil register or a final judgment; or (2) An admission of legitimate filiation in a public document or a private handwritten instrument and signed by the parent concerned. In the absence of the foregoing evidence, the legitimate filiation shall be proved by: (1) The open and continuous possession of the status of a legitimate child; or (2) Any other means allowed by the Rules of Court and special laws. Art. 173. The action to claim legitimacy may be brought by the child during his or her lifetime and shall be transmitted to the heirs should the child die during minority or in a state of insanity. In these cases, the heirs shall have a period of five years within which to institute the action. The action already commenced by the child shall survive notwithstanding the death of either or both of the parties. x x x xxx x x x.

Art. 175. Illegitimate children may establish their illegitimate filiation in the same way and on the same, evidence as legitimate children. The action must be brought within the same period specified in Article 173, except when the action is based on the second pa ragraph of Article 172, in which case the action may be brought during the lifetime of the alleged parent. The provisions of the Family Code are retroactively applied; Article 256 of the code reads: "Art. 256. This Code shall have retroactive effect insofar as it does not prejudice or impair vested or acquired rights in accordance with the Civil Code or other laws. Thus, in Vda. de Sy-Quia vs. Court of Appeals,[36] the Court has ruled: "We hold that whether Jose was a voluntarily recognized natural child should be decided under Article 278 of the Civil Code of the Philippines. Article 2260 of that Code provides that 'the voluntary recognition of a natural child shall take place according to this Code, even if the child was born before the effectivity of this body of laws' or before August 30, 1950. Hence, Article 278 may be given retroactive effect." It should be apparent that the growing trend to liberalize the acknowledgment or recognition of illegitimate children is an attempt to break away from the traditional idea of keeping well apart legitimate and non-legitimate relationships within the family in favor of the greater interest and welfare of the child. The provisions are intended to merely govern the private and personal affairs of the family. There is little, if any, to indicate that the legitimate or illegitimate civil status of the individual would also affect his political rights or, in general, his relationship to the State. While, indeed, provisions on "citizenship" could be found in the Civil Code, such provisions must be taken in the context of private relations, the domain of civil law; particularly "Civil Law is that branch of law which has for its double purpose the organization of the family and the regulation of property. It has thus [been] defined as the mass of precepts which determine and regulate the relations of assistance, authority and obedience among members of a family, and those which exist among members of a society for the protection of private interests."[37] In Yaez de Barnuevo vs. Fuster,[38] the Court has held:

"In accordance with Article 9 of the Civil Code of Spain, x x x the laws relating to family rights and duties, or to the status, condition and legal capacity of persons, govern Spaniards although they reside in a foreign country; that, in consequence, 'all questions of a civil nature, such as those dealing with the validity or nullity of the matrimonial bond, the domicile of the husband and wife, their support, as between them, the separation of their properties, the rules governing property, marital authority, division of conjugal property, the classification of their property, legal causes for divorce, the extent of the latter, the authority to decree it, and, in general, the civil effects of marriage and divorce upon the persons and properties of the spouses, are questions that are governed exclusively by the national law of the husband and wife." The relevance of "citizenship" or "nationality" to Civil Law is best exemplified in Article 15 of the Civil Code, stating that "Laws relating to family rights and duties, or to the status, condition and legal capacity of persons are binding upon citizens of the Philippines, even though living abroad" that explains the need to incorporate in the code a reiteration of the Constitutional provisions on citizenship. Similarly, citizenship is significant in civil relationships found in different parts of the Civil Code,[39] such as on successional rights and family relations.[40] In adoption, for instance, an adopted child would be considered the child of his adoptive parents and accorded the same rights as their legitimate child but such legal fiction extended only to define his rights under civil law[41] and not his political status. Civil law provisions point to an obvious bias against illegitimacy. This discriminatory attitude may be traced to the Spanish family and property laws, which, while defining proprietary and successional rights of members of the family, provided distinctions in the rights of legitimate and illegitimate children. In the monarchial set-up of old Spain, the distribution and inheritance of titles and wealth were strictly according to bloodlines and the concern to keep these bloodlines uncontaminated by foreign blood was paramount. These distinctions between legitimacy and illegitimacy were codified in the Spanish Civil Code, and the invidious discrimination survived when the Spanish Civil Code became the primary source of our own Civil Code. Such distinction, however, remains and should remain only in the sphere of civil law and not unduly impede or impinge on the domain of political law. The proof of filiation or paternity for purposes of determining his citizenship status should thus be deemed independent from and not inextricably tied up with that prescribed for civil law purposes. The Civil Code or Family Code provisions on proof of filiation or paternity, although good law, do not have preclusive effects on matters alien to personal and family relations. The ordinary rules on evidence could well and should govern. For instance, the matter about pedigree is not necessarily precluded from being applicable by the Civil Code or Family Code provisions. Section 39, Rule 130, of the Rules of Court provides Act or Declaration about pedigree. The act or declaration of a person deceased, or unable to testify, in respect to the pedigree of another person related to him by birth or marriage, may be received in evidence where it occurred before the controversy, and the relationship between the two persons is shown by evidence other than such act or declaration. The word `pedigree includes relationship, family genealogy, birth, marriage, death, the dates when and the places where these facts occurred, and the names of the relatives. It embraces also facts of family history intimately connected with pedigree. For the above rule to apply, it would be necessary that (a) the declarant is already dead or unable to testify, (b) the pedigree of a person must be at issue, (c) the declarant must be a relative of the person whose pedigree is in question, (d) declaration must be made before the controversy has occurred, and (e) the relationship between the declarant and the person whose pedigree is in question must be shown by evidence other than such act or declaration. Thus, the duly notarized declaration made by Ruby Kelley Mangahas, sister of Bessie Kelley Poe submitted as Exhibit 20 before the COMELEC, might be accepted to prove the acts of Allan F. Poe, recognizing his own paternal relationship with FPJ, i.e, living together with Bessie Kelley and his children (including respondent FPJ) in one house, and as one family "I, Ruby Kelley Mangahas, of legal age and sound mind, presently residing in Stockton, California, U.S.A., after being sworn in accordance with law do hereby declare that: 1. 2. 3. I am the sister of the late Bessie Kelley Poe. Bessie Kelley Poe was the wife of Fernando Poe, Sr. Fernando and Bessie Poe had a son by the name of Ronald Allan Poe, more popularly known in the Philippines as `Fernando Poe, Jr., or `FPJ. Ronald Allan Poe `FPJ was born on August 20, 1939 at St. Luke's Hospital, Magdalena Street, Manila. x x x 7. xxx xxx

4.

Fernando Poe Sr., and my sister Bessie, met and became engaged while they were students at the University of the Philippines in 1936. I was also introduced to Fernando Poe, Sr., by my sister that same year. Fernando Poe, Sr., and my sister Bessie had their first child in 1938. Fernando Poe, Sr., my sister Bessie and their first three children, Elizabeth, Ronald, Allan and Fernando II, and myself lived together with our mother at our family's house on Dakota St. (now Jorge Bocobo St.), Malate until the liberation of Manila in 1945, except for some months between 1943-1944.

8. 9.

10. Fernando Poe, Sr., and my sister, Bessie, were blessed with four (4) more children after Ronald Allan Poe. x x x xxx xxx

18. I am executing this Declaration to attest to the fact that my nephew, Ronald Allan Poe is a natural born Filipino, and that he is the legitimate child of Fernando Poe, Sr. Done in City of Stockton, California, U.S.A., this 12th day of January 2004. Ruby Kelley Mangahas Declarant

DNA Testing In case proof of filiation or paternity would be unlikely to satisfactorily establish or would be difficult to obtain, DNA testing, which examines genetic codes obtained from body cells of the illegitimate child and any physical residue of the long dead parent could be resorted to. A positive match would clear up filiation or paternity. In Tijing vs. Court of Appeals,[42]this Court has acknowledged the strong weight of DNA testing "Parentage will still be resolved using conventional methods unless we adopt the modern and scientific ways available. Fortunately, we have now the facility and expertise in using DNA test for identification and parentage testing. The University of the Philippines Natural Science Research Institute (UP-NSRI) DNA Analysis Laboratory has now the capability to conduct DNA typing using short tandem repeat (STR) analysis. The analysis is based on the fact that the DNA of a child/person has two (2) copies, one copy from the mother and the other from the father. The DNA from the mother, the alleged father and the child are analyzed to establish parentage. Of course, being a novel scientific technique, the use of DNA test as evidence is still open to challenge. Eventually, as the appropriate case comes, courts should not hesitate to rule on the admissibility of DNA evidence. For it was said, that courts should apply the results of science when competently obtained in aid of situations presented, since to reject said result is to deny progress."

Petitioners Argument For Jurisprudential Conclusiveness Petitioner would have it that even if Allan F. Poe were a Filipino citizen, he could not have transmitted his citizenship to respondent FPJ, the latter being an illegitimate child. According to petitioner, prior to his marriage to Bessie Kelley, Allan F. Poe, on July 5, 1936, contracted marriage with a certain Paulita Gomez, making his subsequent marriage to Bessie Kelley bigamous and respondent FPJ an illegitimate child. The veracity of the supposed certificate of marriage between Allan F. Poe and Paulita Gomez could be most doubtful at best. But the documentary evidence introduced by no less than respondent himself, consisting of a birth certificate of respondent and a marriage certificate of his parents showed that FPJ was born on 20 August 1939 to a Filipino father and an American mother who were married to each other a year later, or on 16 September 1940. Birth to unmarried parents would make FPJ an illegitimate child. Petitioner contended that as an illegitimate child, FPJ so followed the citizenship of his mother, Bessie Kelley, an American citizen, basing his stand on the ruling of this Court in Morano vs. Vivo,[43] citing Chiongbian vs. de Leon[44] and Serra vs. Republic.[45] On the above score, the disquisition made by amicus curiae Joaquin G. Bernas, SJ, is most convincing; he states "We must analyze these cases and ask what the lis mota was in each of them. If the pronouncement of the Court on jus sanguinis was on the lis mota, the pronouncement would be a decision constituting doctrine under the rule of stare decisis. But if the pronouncement was irrelevant to the lis mota, the pronouncement would not be a decision but a mere obiter dictum which did not establish doctrine. I therefore invite the Court to look closely into these cases. First, Morano vs. Vivo. The case was not about an illegitimate child of a Filipino father. It was about a stepson of a Filipino, a stepson who was the child of a Chinese mother and a Chinese father. The issue was whether the stepson followed the naturalization of the stepfather. Nothing about jus sanguinis there. The stepson did not have the blood of the naturalized stepfather. Second, Chiongbian vs. de Leon. This case was not about the illegitimate son of a Filipino father. It was about a legitimate son of a father who had become Filipino by election to public office before the 1935 Constitution pursuant to Article IV, Section 1(2) of the 1935 Constitution. No one was illegitimate here. Third, Serra vs. Republic. The case was not about the illegitimate son of a Filipino father. Serra was an illegitimate child of a Chinese father and a Filipino mother. The issue was whether one who was already a Filipino because of his mother who still needed to be naturalized. There is nothing there about invidious jus sanguinis. Finally, Paa vs. Chan.[46] This is a more complicated case. The case was about the citizenship of Quintin Chan who was the son of Leoncio Chan. Quintin Chan claimed that his father, Leoncio, was the illegitimate son of a Chinese father and a Filipino mother. Quintin therefore argued that he got his citizenship from Leoncio, his father. But the Supreme Court said that there was no valid proof that Leoncio was in fact the son of a Filipina mother. The Court therefore concluded that Leoncio was not Filipino. If Leoncio was not Filipino, neither was his son Quintin. Quintin therefore was not only not a natural-born Filipino but was not even a Filipino.

The Court should have stopped there. But instead it followed with an obiter dictum. The Court said obiter that even if Leoncio, Quintin's father, were Filipino, Quintin would not be Filipino because Quintin was illegitimate. This statement about Quintin, based on a contrary to fact assumption, was absolutely unnecessary for the case. x x x It was obiter dictum, pure and simple, simply repeating the obiter dictum in Morano vs. Vivo. "Aside from the fact that such a pronouncement would have no textual foundation in the Constitution, it would also violate the equal protection clause of the Constitution not once but twice. First, it would make an illegitimate distinction between a legitimate child and an illegitimate child, and second, it would make an illegitimate distinction between the illegitimate child of a Filipino father and the illegitimate child of a Filipino mother. The doctrine on constitutionally allowable distinctions was established long ago by People vs. Cayat.[47] I would grant that the distinction between legitimate children and illegitimate children rests on real differences. x x x But real differences alone do not justify invidious distinction. Real differences may justify distinction for one purpose but not for another purpose. x x x What is the relevance of legitimacy or illegitimacy to elective public service? What possible state interest can there be for disqualifying an illegitimate child from becoming a public officer. It was not the fault of the child that his parents had illicit liaison. Why deprive the child of the fullness of political rights for no fault of his own? To disqualify an illegitimate child from holding an important public office is to punish him for the indiscretion of his parents. There is neither justice nor rationality in that. And if there is neither justice nor rationality in the distinction, then the distinction transgresses the equal protection clause and must be reprobated. The other amici curiae, Mr. Justice Vicente Mendoza (a former member of this Court), Professor Ruben Balane and Dean Martin Magallona, at bottom, have expressed similar views. The thesis of petitioner, unfortunately hinging solely on pure obiter dicta, should indeed fail. Where jurisprudence regarded an illegitimate child as taking after the citizenship of its mother, it did so for the benefit the child. It was to ensure a Filipino nationality for the illegitimate child of an alien father in line with the assumption that the mother had custody, would exercise parental authority and had the duty to support her illegitimate child. It was to help the child, not to prejudice or discriminate against him. The fact of the matter perhaps the most significant consideration is that the 1935 Constitution, the fundamental law prevailing on the day, month and year of birth of respondent FPJ, can never be more explicit than it is. Providing neither conditions nor distinctions, the Constitution states that among the citizens of the Philippines are those whose fathers are citizens of the Philippines. There utterly is no cogent justification to prescribe conditions or distinctions where there clearly are none provided.

In Sum (1) The Court, in the exercise of its power of judicial review, possesses jurisdiction over the petition in G. R. No. 161824, filed under Rule 64, in relation to Rule 65, of the Revised Rules of Civil Procedure. G.R. No. 161824 assails the resolution of the COMELEC for alleged grave abuse of discretion in dismissing, for lack of merit, the petition in SPA No. 04-003 which has prayed for the disqualification of respondent FPJ from running for the position of President in the 10th May 2004 national elections on the contention that FPJ has committed material representation in his certificate of candidacy by representing himself to be a natural-born citizen of the Philippines. (2) The Court must dismiss, for lack of jurisdiction and prematurity, the petitions in G. R. No. 161434 and No. 161634 both having been directly elevated to this Court in the latters capacity as the only tribunal to resolve a president ial and vice-presidential election contest under the Constitution. Evidently, the primary jurisdiction of the Court can directly be invoked only after, not before, the elections are held. (3) In ascertaining, in G.R. No. 161824, whether grave abuse of discretion has been committed by the COMELEC, it is necessary to take on the matter of whether or not respondent FPJ is a natural-born citizen, which, in turn, depended on whether or not the father of respondent, Allan F. Poe, would have himself been a Filipino citizen and, in the affirmative, whether or not the alleged illegitimacy of respondent prevents him from taking after the Filipino citizenship of his putative father. Any conclusion on the Filipino citizenship of Lorenzo Pou could only be drawn from the presumption that having died in 1954 at 84 years old, Lorenzo would have been born sometime in the year 1870, when the Philippines was under Spanish rule, and that San Carlos, Pangasinan, his place of residence upon his death in 1954, in the absence of any other evidence, could have well been his place of residence before death, such that Lorenzo Pou would have benefited from the en masse Filipinization that the Philippine Bill had effected in 1902. That citizenship (of Lorenzo Pou), if acquired, would thereby extend to his son, Allan F. Poe, father of respondent FPJ. The 1935 Constitution, during which regime respondent FPJ has seen first light, confers citizenship to all persons whose fathers are Filipino citizens regardless of whether such children are legitimate or illegitimate. (4) But while the totality of the evidence may not establish conclusively that respondent FPJ is a natural-born citizen of the Philippines, the evidence on hand still would preponderate in his favor enough to hold that he cannot be held guilty of having made a material misrepresentation in his certificate of candidacy in violation of Section 78, in relation to Section 74, of the Omnibus Election Code. Petitioner has utterly failed to substantiate his case before the Court, notwithstanding the ample opportunity given to the parties to present their position and evidence, and to prove whether or not there has been material misrepresentation, which, as so ruled in Romualdez-Marcos vs. COMELEC,[48] must not only be material, but also deliberate and willful. WHEREFORE, the Court RESOLVES to DISMISS 1. G. R. No. 161434, entitled "Maria Jeanette C. Tecson and Felix B. Desiderio, Jr., Petitioners, versus Commission on Elections, Ronald Allan Kelley Poe (a.k.a. "Fernando Poe, Jr.,) and Victorino X. Fornier, Respondents," and G. R. No. 161634, entitled "Zoilo Antonio Velez, Petitioner, versus Ronald Allan Kelley Poe, a.k.a. Fernando Poe, Jr., Respondent," for want of jurisdiction. 2. G. R. No. 161824, entitled Victorino X. Fornier, Petitioner, versus Hon. Commission on Elections and Ronald Allan Kelley Poe, also known as Fernando Poe, Jr., for failure to show grave abuse of discretion on the part of respondent Commission on Elections in dismissing the petition in SPA No. 04-003. No Costs.

SO ORDERED.

[1] [2]

Tan Chong vs. The Secretary of Labor, 45 O.G. No. 31, 1269. Sec. 2. Mode of review. A judgment or final order or resolution of the Commission on Elections and the Commission on Audit may be brought by the aggrieved party to the Supreme Court on certiorari under Rule 65, except as hereinafter provided. (Rule 64) Sec. 1. Petition for certiorari. When any tribunal, board or officer exercising judicial or quasi-judicial functions has acted without or in excess of its or his jurisdiction, or with grave abuse of discretion amounting to lack or excess of jurisdiction, and there is no appeal, or any plain, speedy, and adequate remedy in the ordinary course of law, a person aggrieved thereby may file a verified petition in the proper court, alleging the facts with certainty and praying that judgment be rendered annulling or modifying the proceedings of such tribunal, board or officer, and granting such incidental reliefs as law and justice may require. The petition shall be accompanied by a certified true copy of the judgment, order or resolution subject thereof, copies of all pleadings and documents relevant and pertinent thereto, and a sworn certification of non-forum shopping as provided in the third paragraph of section 3, Rule 46. (Rule 65)

[3]

[6] [8]

The Politics of Aristotle, edited and translated by Ernest Barker, Oxford University Press, London, 1946. Introduction, The Conditions of Citizenship, edited by Bart Van Steenbergen, Sage Publications, London, Thousand Oaks, New Delhi (1994). Under the codified Novisima Recopilacion promulgated in Spain in 1805, the following were considered denizens (vecinos) " all foreigners who obtained the privilege of naturalization, those who were born in these kingdoms, those who residing therein may be converted to the holy Catholic faith; those, being self-supporting, established their domicile therein; and in the case of a foreign woman who married a native man, she thereby becomes subject to the same laws and acquires the same domicile as her husband; those who establish themselves in the country by acquiring real property; those who have trade or profession and go there to practice the same; also those who practice some mechanical trade therein or keep a retail store;....those who reside for a period of ten years in a home of his own; and also those foreigners who, in accordance with the common law, royal orders and other laws of the kingdoms, may have become naturalized or acquired residence therein. (Leon T. Garcia, The Problems of Citizenship in the Philippines, Rex Bookstore, 1949, at p. 4) Garcia, supra., at p. 3. Justices Malcolm, Recto and Florentino Torres believed that the law was effective in the Philippines. Those who entertained the contrary view were Justices Imperial and Villareal. (Garcia, supra., at 4.). Under the Royal Decree of August 23, 1868, the following were considered foreigners --- (1) The legitimate and recognized natural children of a father who belongs to another independent state, and the unrecognized and natural and other illegitimate children of a mother belonging to another State born outside of the Spanish dominions, (2) The children specified in the preceding paragraph, born in the Spanish dominions or on board Spanish vessels on the high seas if they do not, on attaining the age of majority fixed in the laws of the Kingdom, elect Spanish nationality, (3) Those being Spaniards, acquire another nationality, as well by renouncing the first as by accepting employment, from another government without the authority of the sovereign and (4) The woman who contracts marriage with a subject of another State. (Garcia, supra., pp. 6-7) Under the law, the following were foreigners (a) All persons born of foreign parents outside of the Spanish territory; (b) Those born outside of the Spanish territory of foreign fathers and Spanish mothers while they do not claim Spanish nationality, (3) Those born in Spanish territory of foreign parents or foreign fathers and Spanish mothers while they do not make that claim, (4) Spaniards who may have lost their nationality, (5) Those born outside of the Spanish territory of parents who may have lost their Spanish nationality; and (6), the Spanish woman married to a foreigner. (Garcia, supra., p. 7) Article 16. Real property as well as personal property is subject to the law of the country where it is situated. However, intestate and testamentary successions, both with respect to the order of succession and to the amount of successional rights and to the intrinsic validity of testamentary provisions, shall be regulated by the national law of the person whose succession is under consideration, whatever may be the nature of the property and regardless of the country wherein said property may be found. Article 17. The forms and solemnities of contracts, wills, and other public instruments shall be governed by the laws of the country in which they are executed. When the acts referred to are executed before the diplomatic or consular officials of the Republic of the Philippines in a foreign country, the solemnities established by Philippine laws shall be observed in their execution. Prohibitive laws concerning persons, their acts or property, and those which have for their object public order, public policy and good customs, shall not be rendered ineffective by laws or judgments promulgated, or by determinations or conventions agreed upon in a foreign country. Article 815. When a Filipino is in a foreign country, he is authorized to make a will in any of the forms established by the law of the country in which he may be. Such will may be probated in the Philippines. Article 816. The will of an alien who is abroad produces effect in the Philippines if made with the formalities prescribed by the law of the place in which he resides, or according to the formalities observed in his country, or in conformity with those which this Code prescribes. Article 817. A will made in the Philippines by a citizen or subject of another country, which is executed in accordance with the law of the country of which he is a citizen or subject, and which might be proved and allowed by the law of his own country, shall have the same effect as if executed according to the laws of the Philippines.

[13]

[14] [15]

[17]

[18]

[39]

Article 819. Wills, prohibited by the preceding article, executed by Filipinos in a foreign country shall not be valid in the Philippines, even though authorized by the laws of the country where they may have been executed. Article 1039. Capacity to succeed is governed by the law of the nation of the decedent.
[40]

Article 10. Marriages between Filipino citizens abroad may be solemnized by a consul general, consul or vice-consul of the Republic of the Philippines. The issuance of the marriage license and the duties of the local civil registrar and of the solemnizing officer with regard to the celebration of marriage shall be performed by said consular official. Article 21. When either or both of the contracting parties are citizens of a foreign country, it shall be necessary for them before a marriage license can be obtained, to submit a certificate of legal capacity to contract marriage, issued by their respective diplomatic or consular officials. Stateless persons or refugees from other countries shall, in lieu of the certificate of legal capacity herein required, submit an affidavit stating the circumstances showing such capacity to contract marriage. Article 26. xxx

Where a marriage between a Filipino citizen and a foreigner is validly celebrated and a divorce is thereafter validly obtained abroad by the alien spouse capacitating him or her to remarry, the Filipino spouse shall have capacity to remarry under Philippine law. Article 80. In the absence of a contrary stipulation in the marriage settlements, the property relations of the spouses shall be governed by Philippine laws, regardless of the place of the celebration of the marriage and their residence. This rule shall not apply: (1) Where both spouses are aliens; (2) With respect to the extrinsic validity of contracts affecting property not situated in the Philippines and executed in the country where the property is located; and (3) With respect to the extrinsic validity of contracts entered into in the Philippines but affecting property situated in a foreign country whose laws require different formalities for their extrinsic validity.

Republic of the Philippines SUPREME COURT Manila EN BANC

G.R. No. L-21289 October 4, 1971 MOY YA LIM YAO alias EDILBERTO AGUINALDO LIM and LAU YUEN YEUNG, petitioners-appellants, vs. THE COMMISSIONER OF IMMIGRATION, respondent-appellee. Aruego, Mamaril & Associates for petitioners-appellants. Office of the Solicitor General Arturo A. Alafriz, Assistant Solicitor General Frine' C. Zaballero and Solicitor Sumilang V. Bernardo for respondent-appellee.

BARREDO, J.:

Appeal from the following decision of the Court of First Instance of Manila in its Civil Case No. 49705 entitled Moy Ya Lim Yao, etc., et al. vs. The Commissioner of Immigration which, brief as it is, sufficiently depicts the factual setting of and the fundamental issues involved in this case thus: In the instant case, petitioners seek the issuance of a writ of injunction against the Commissioner of Immigration, "restraining the latter and/or his authorized representative from ordering plaintiff Lau Yuen Yeung to leave the Philippines and causing her arrest and deportation and the confiscation of her bond, upon her failure to do so." The prayer for preliminary injunction embodied in the complaint, having been denied, the case was heard on the merits and the parties submitted their respective evidence. The facts of the case, as substantially and correctly stated by the Solicitor General are these: On February 8, 1961, Lau Yuen Yeung applied for a passport visa to enter the Philippines as a non-immigrant. In the interrogation made in connection with her application for a temporary visitor's visa to enter the Philippines, she stated that she was a Chinese residing at Kowloon, Hongkong, and that she desired to take a pleasure trip to the Philippines to visit her great (grand) uncle Lau Ching Ping for a period of one month (Exhibits "l," "1-a," and "2"). She was permitted to come into the Philippines on March 13, 1961, and was permitted to stay for a period of one month which would expire on April 13, 1961. On the date of her arrival, Asher Y, Cheng filed a bond in the amount of P1,000.00 to undertake, among others that said Lau Yuen Yeung would actually depart from the Philippines on or before the expiration of her authorized period of stay in this country or within the period as in his discretion the Commissioner of Immigration or his authorized representative might properly allow. After repeated extensions, petitioner Lau Yuen Yeung was allowed to stay in the Philippines up to February 13, 1962 (Exhibit "4"). On January 25, 1962, she contracted marriage with Moy Ya Lim Yao alias Edilberto Aguinaldo Lim an alleged Filipino citizen. Because of the contemplated action of respondent to confiscate her bond and order her arrest and immediate deportation, after the expiration of her authorized stay, she brought this action for injunction with preliminary injunction. At the hearing which took place one and a half years after her arrival, it was admitted that petitioner Lau Yuen Yeung could not write either English or Tagalog. Except for a few words, she could not speak either English or Tagalog. She could not name any Filipino neighbor, with a Filipino name except one, Rosa. She did not know the names of her brothers-in-law, or sisters-in-law. Under the facts unfolded above, the Court is of the considered opinion, and so holds, that the instant petition for injunction cannot be sustained for the same reason as set forth in the Order of this Court, dated March 19, 1962, the pertinent portions of which read: First, Section 15 of the Revised Naturalization Law provides: Effect of the naturalization on wife and children. Any woman who is now or may hereafter be married to a citizen of the Philippines, and who might herself be lawfully naturalized shall be deemed a citizen of the Philippines. The above-quoted provision is clear and its import unequivocal and hence it should be held to mean what it plainly and explicitly expresses in unmistakable terms. The clause "who might herself be lawfully naturalized" incontestably implies that an alien woman may be deemed a citizen of the Philippines by virtue of her marriage to a Filipino citizen only if she possesses all the qualifications and none of the disqualifications specified in the law, because these are the explicit requisites provided by law for an alien to be naturalized. (Lee Suan Ay, Alberto Tan and Lee Chiao vs. Emilio Galang, etc., G. R. No. L-11855). However, from the allegation of paragraph 3 of the complaint, to wit: 3. That plaintiff Lau Yuen Yeung, Chinese by birth, who might herself be lawfully naturalized as a Filipino citizen (not being disqualified to become such by naturalization), is a Filipino citizen by virtue of her marriage on January 25, 1962 to plaintiff MOY YA LIM YAO alias EDILBERTO AGUINALDO LIM, under the Naturalization Laws of the Philippines. it can be deduced beyond debate that petitioner Lau Yuen Yeung while claiming not to be disqualified, does not and cannot allege that she possesses all the qualifications to be naturalized, naturally because, having been admitted as a temporary visitor only on March 13, 1961, it is obvious at once that she lacks at least, the requisite length of residence in the Philippines (Revised Naturalization Law, Sec. 2, Case No. 2, Sec. 3, Case No. 3). Were if the intention of the law that the alien woman, to be deemed a citizen of the Philippines by virtue of marriage to a Filipino citizen, need only be not disqualified under the Naturalization Law, it would have been worded "and who herself is not disqualified to become a citizen of the Philippines." Second, Lau Yuen Yeung, a temporary Chinese woman visitor, whose authorized stay in the Philippines, after repeated extensions thereof, was to expire last February 28, 1962, having married her co-plaintiff only on January 25, 1962, or just a little over one month before the expiry date of her stay, it is evident that said marriage was effected merely for convenience to defeat or avoid her then impending compulsory departure, not to say deportation. This cannot be permitted.

Third, as the Solicitor General has well stated: 5. That petitioner Lau Yuen Yeung, having been admitted as a temporary alien visitor on the strength of a deliberate and voluntary representation that she will enter and stay only for a period of one month and thereby secured a visa, cannot go back on her representation to stay permanently without first departing from the Philippines as she had promised. (Chung Tiao Bing, et al. vs. Commissioner of Immigration, G. R. No. L-9966, September 29, 1956; Ong Se Lun vs. Board of Commissioners, G. R. No. L-6017, September 16, 1954; Sec. 9, last par., Phil. Immigration Law). The aforequoted argument of the Solicitor General is well buttressed not only by the decided cases of the Supreme Court on the point mentioned above, but also on the very provisions of Section 9, sub-paragraph (g) of the Philippine Immigration Act of 1940 which reads: An alien who is admitted as a non-immigrant cannot remain in the Philippines permanently. To obtain permanent admission, a non-immigrant alien must depart voluntarily to some foreign country and procure from the appropriate Philippine Consul the proper visa and thereafter undergo examination by the Officers of the Bureau of Immigration at a Philippine port of entry for determination of his admissibility in accordance with the requirements of this Act. (This paragraph is added by Republic Act 503). (Sec. 9, subparagraph (g) of the Philippine Immigration Act of 1940). And fourth, respondent Commissioner of Immigration is charged with the administration of all laws relating to immigration (Sec. 3, Com. Act No. 613) and in the performance of his duties in relation to alien immigrants, the law gives the Commissioner of Immigration a wide discretion, a quasi-judicial function in determining cases presented to him (Pedro Uy So vs. Commissioner of Immigration CA-G. R. No. 23336-R, Dec. 15, 1960), so that his decision thereon may not be disturbed unless he acted with abuse of discretion or in excess of his jurisdiction. It may also be not amiss to state that wife Lau Yuen Yeung, while she barely and insufficiently talk in broken Tagalog and English, she admitted that she cannot write either language. The only matter of fact not clearly passed upon by His Honor which could have some bearing in the resolution of this appeal is the allegation in the brief of petitioners-appellants, not denied in the governments brief, that "in the hearing ..., it was shown thru the testimony of the plaintiff Lau Yuen Yeung that she does not possess any of the disqualifications for naturalization." Of course, as an additional somehow relevant factual matter, it is also emphasized by said appellants that during the hearing in the lower court, held almost ten months after the alleged marriage of petitioners, "Lau Yuen Yeung was already carrying in her womb for seven months a child by her husband." Appellants have assigned six errors allegedly committed by the court a quo, thus: I THE LOWER COURT ERRED IN HOLDING THAT THE CLAUSE "WHO MIGHT HERSELF BE LAWFULLY NATURALIZED" (OF SECTION 15, REVISED NATURALIZATION LAW) INCONTESTABLY IMPLIES THAT AN ALIEN WOMAN MAY BE DEEMED A CITIZEN OF THE PHILIPPINES BY VIRTUE OF HER MARRIAGE TO A FILIPINO CITIZEN, ONLY IF SHE POSSESSES ALL THE QUALIFICATIONS AND NONE OF THE DISQUALIFICATIONS SPECIFIED IN THE LAW. II THE LOWER COURT ERRED IN HOLDING THAT A WOMAN FOREIGNER WHO DOES NOT POSSESS ANY OF THE DISQUALIFICATIONS FOR CITIZENSHIP AND WHO MARRIED A FILIPINO CITIZEN IS STILL CONSIDERED AN ALIEN EVEN AFTER SUCH MARRIAGE AS TO FALL WITHIN THE REQUIREMENT OF SECTION 9, SUBPARAGRAPH (9) OF THE PHILIPPINE IMMIGRATION ACT OF 1940. III THE COURT ERRED IN CONCLUDING THAT LAU YUEN YEUNG'S MARRIAGE TO A FILIPINO CITIZEN WAS ONLY FOR CONVENIENCE, MERELY BECAUSE THE SAME WAS CELEBRATED JUST OVER A MONTH BEFORE THE EXPIRY DATE OF HER AUTHORIZED STAY. IV THE LOWER COURT ERRED IN FAILING TO FIND THAT THE COMMISSIONER OF IMMIGRATION ACTED WITH ABUSE OF DISCRETION OR IN EXCESS OF HIS JURISDICTION WHEN SAID OFFICER THREATENED TO SEND OUT OF THE COUNTRY PLAINTIFF LAU YUEN YEUNG WITH WARNING THAT HER FAILURE TO DO SO WOULD MEAN CONFISCATION OF HER BOND, ARREST AND IMMEDIATE DEPORTATION, IN SPITE OF THE FACT THAT LAU YUEN YEUNG IS NOW A FILIPINO CITIZEN. V

THE LOWER COURT ERRED IN DISMISSING PLAINTIFFS-APPELLANTS' COMPLAINT AND IN REFUSING TO PERMANENTLY ENJOIN THE COMMISSIONER FROM ORDERING PLAINTIFF LAU YUEN YEUNG TO LEAVE THE PHILIPPINES AS A TEMPORARY VISITOR WHICH SHE IS NOT. VI THE LOWER COURT ERRED IN REFUSING TO GRANT PLAINTIFFS-APPELLANTS' MOTION FOR PRELIMINARY INJUNCTION EMBODIED IN THEIR COMPLAINT, IN AN ORDER DATED MARCH 19, 1962. (PAGES 36-41, RECORD ON APPEAL) . We need not discuss these assigned errors separately. In effect, the above decision upheld the two main grounds of objection of the Solicitor General to the petition in the court below, viz: That petitioner Lau Yuen Yeung, having been admitted as a temporary alien visitor on the strength of a deliberate and voluntary representation that she will enter and stay only for a period of one month and thereby secured a visa, cannot go back on her representation to stay permanently without first departing from the Philippines as she had promised. (Chung Tiao Bing, et al. vs. Commissioner of Immigration, G.R. No. L-9966, September 29, 1956; Ong Se Lun vs. Board of Commissioners, G.R. No. L-6017, Sept. 16, 1954, Sec. 9, last par. Phil. Immigration Law); That the mere marriage of a Filipino citizen to an alien does not automatically confer on the latter Philippine citizenship. The alien wife must possess all the qualifications required by law to become a Filipino citizen by naturalization and none of the disqualifications. (Lee Suan Ay, Alberto Tan and Lee Chiao vs. Galang, etc., G. R. No. L-11855, Dec. 25, 1959) It is obvious from the nature of these objection that their proper resolution would necessarily cover all the points raised in appellants' assignments of error, hence, We will base our discussions, more or less, on said objections. I The first objection of the Solicitor General which covers the matters dealt with in appellants' second and fourth assignments of error does not require any lengthy discussion. As a matter of fact, it seem evident that the Solicitor General's pose that an alien who has been admitted into the Philippines as a non-immigrant cannot remain here permanently unless he voluntarily leaves the country first and goes to a foreign country to secure thereat from the appropriate Philippine consul the proper visa and thereafter undergo examination by officers of the Bureau of Immigration at a Philippine port of entry for determination of his admissibility in accordance with the requirements of the Philippine Immigration Act of 1940, as amended by Republic Act 503, is premised on the assumption that petitioner Lau Yuen Yeung is not a Filipino citizen. We note the same line of reasoning in the appealed decision of the court a quo. Accordingly, it is but safe to assume that were the Solicitor General and His Honor of the view that said petitioner had become ipso facto a Filipina by virtue of her marriage to her Filipino husband, they would have held her as entitled to assume the status of a permanent resident without having to depart as required of aliens by Section 9 (g) of the law. In any event, to set this point at rest, We hereby hold that portion of Section 9 (g) of the Immigration Act providing: An alien who is admitted as a non-immigrant cannot remain in the Philippines permanently. To obtain permanent admission, a non-immigrant alien must depart voluntarily to some foreign country and procure from the appropriate Philippine consul the proper visa and thereafter undergo examination by the officers of the Bureau of Immigration at a Philippine port of entry for determination of his admissibility in accordance with the requirements of this Act. does not apply to aliens who after coming into the Philippines as temporary visitors, legitimately become Filipino citizens or acquire Filipino citizenship. Such change of nationality naturally bestows upon their the right to stay in the Philippines permanently or not, as they may choose, and if they elect to reside here, the immigration authorities may neither deport them nor confiscate their bonds. True it is that this Court has vehemently expressed disapproval of convenient ruses employed by alien to convert their status from temporary visitors to permanent residents in circumvention of the procedure prescribed by the legal provision already mentioned, such as inChiong Tiao Bing vs. Commissioner of Immigration, 99 Phil. 1020, wherein, thru Mr. Justice J.B.L. Reyes, the Court, reiterating the ruling in Ong Se Lun vs. Board of Immigration Commissioners, 95 PMI. 785, said: ... It is clear that if an alien gains admission to the Islands on the strength of a deliberate and voluntary representation that he will enter only for a limited time, and thereby secures the benefit of a temporary visa, the law will not allow him subsequently to go back on his representation and stay permanently, without first departing from the Philippines as he had promised. No officer can relieve him of the departure requirements of section 9 of the Immigration Act, under the guise of "change" or "correction", for the law makes no distinctions, and no officer is above the law. Any other ruling would, as stated in our previous decision, encourage aliens to enter the Islands on false pretences; every alien so permitted to enter for a limited time, might then claim a right to permanent admission, however flimsy such claim should be, and thereby compel our government to spend time, money and effort to examining and verifying whether or not every such alien really has a right to take up permanent residence here. In the meanwhile, the alien would be able to prolong his stay and evade his return to the port whence he came, contrary to what he promised to do when he entered. The damages inherent in such ruling are self-evident. On the other hand, however, We cannot see any reason why an alien who has been here as a temporary visitor but who has in the meanwhile become a Filipino should be required to still leave the Philippines for a foreign country, only to apply thereat for a re-entry here and undergo the process of showing that he is entitled to come back, when after all, such right has become incontestible as a necessary concomitant of his assumption of our nationality by whatever legal means this has been conferred upon him. Consider for example, precisely the case of the minor children of an alien who is naturalized. It is indubitable that they become ipso facto citizens of the Philippines. Could it be the law that before they can be allowed permanent residence, they still have to be taken abroad so that they may be processed to determine whether or not they have a right to have permanent residence here? The difficulties and hardships which such a requirement entails and its seeming

unreasonableness argue against such a rather absurd construction. Indeed, as early as 1957, in Ly Giok Ha vs. Galang, 101 Phil. 459, Mr. Justice Concepcion, our present Chief Justice, already ruled thus: ... (P)etitioners allege that, upon her marriage to a Filipino, Ly Giok Ha became also a citizen of the Philippines. Indeed, if this conclusion were correct, it would follow that, in consequence of her marriage, she had been naturalized as such citizen, and, hence the decision appealed from would have to be affirmed, for section 40(c) of Commonwealth Act 613 provides that "in the event of the naturalization as a Philippine citizen ... of the alien on whose behalf the bond deposit is given, the bond shall be cancelled or the sum deposited shall be returned to the depositor or his legal representative ." (At. pp. 462-463) In other words, the applicable statute itself more than implies that the naturalization of an alien visitor as a Philippine citizen logically produces the effect of conferring upon him ipso facto all the rights of citizenship including that of being entitled to permanently stay in the Philippines outside the orbit of authority of the Commissioner of Immigration vis-a-vis aliens, if only because by its very nature and express provisions, the Immigration Law is a law only for aliens and is inapplicable to citizens of the Philippines. In the sense thus discussed therefore, appellants' second and fourth assignments of error are well taken. II Precisely, the second objection, of the Solicitor General sustained by the trial judge is that appellant Lau Yuen Yeung's marriage to appellant Moya Lim Yao alias Edilberto Aguinaldo whose Filipino citizenship is not denied did not have the effect of making her a Filipino, since it has not been shown that she "might herself be lawfully naturalized," it appearing clearly in the record that she does not possess all the qualifications required of applicants for naturalization by the Revised Naturalization Law, Commonwealth Act 473, even if she has proven that she does not suffer from any of the disqualifications thereunder. In other words, the Solicitor General implicitly concedes that had it been established in the proceedings below that appellant Lau Yuen Yeung possesses all the qualifications required by the law of applicants for naturalization, she would have been recognized by the respondent as a Filipino citizen in the instant case, without requiring her to submit to the usual proceedings for naturalization. To be sure, this position of the Solicitor General is in accord with what used to be the view of this Court since Lee Suan Ay, et al. v. Emilio Galang, etc., et al., G.R. No. L-11855, promulgated December 23, 1959, 106 Phil., 706,713, 1 for it was only in Zita Ngo Burca vs. Republic, G.R. NO. L-24252 which was promulgated on January 30, 1967 (19 SCRA 186), that over the pen of Mr. Justice Conrado Sanchez, this Court held that for an alien woman who marries a Filipino to be deemed a Filipina, she has to apply for naturalization in accordance with the procedure prescribed by the Revised Naturalization Law and prove in said naturalization proceeding not only that she has all the qualifications and none of the disqualifications provided in the law but also that she has complied with all the formalities required thereby like any other applicant for naturalization, 2 albeit said decision is not yet part of our jurisprudence inasmuch as the motion for its reconsideration is still pending resolution. Appellants are in effect urging Us, however, in their first and second assignments of error, not only to reconsider Burca but to even reexamine Lee Suan Ay which, as a matter of fact, is the prevailing rule, having been reiterated in all subsequent decisions up to Go Im Ty. 3 Actually, the first case in which Section 15 of the Naturalization Law, Commonwealth Act 473, underwent judicial construction was in the first Ly Giok Ha case, 4 one almost identical to the one at bar. Ly Giok Ha, a woman of Chinese nationality, was a temporary visitor here whose authority to stay was to expire on March 14, 1956. She filed a bond to guaranty her timely departure. On March 8, 1956, eight days before the expiration of her authority to stay, she married a Filipino by the name of Restituto Lacasta. On March 9, 1956, her husband notified the Commissioner of Immigration of said marriage and, contending that his wife had become a Filipina by reason of said marriage, demanded for the cancellation of her bond, but instead of acceding to such request, the Commissioner required her to leave, and upon her failure to do so, on March 16, 1956, the Commissioner confiscated her bond; a suit was filed for the recovery of the bond; the lower court sustained her contention that she had no obligation to leave, because she had become Filipina by marriage, hence her bond should be returned. The Commissioner appealed to this Court. In the said appeal, Mr. Justice Roberto Concepcion, our present Chief Justice, spoke for the Court, thus: The next and most important question for determination is whether her marriage to a Filipino justified or, at least, excused the aforesaid failure of Ly Giok Ha to depart from the Philippines on or before March 14, 1956. In maintaining the affirmative view, petitioners alleged that, upon her marriage to a Filipino, Ly Giok Ha became, also, a citizen of the Philippines. Indeed, if this conclusion were correct, it would follow that, in consequence of her marriage, she had been naturalized as such citizen, and, hence, the decision appealed from would have to be affirmed, for section 40(c) of Commonwealth Act No. 613 provides that "in the event of the naturalization as a Philippine citizen ... of the alien on whose behalf the bond deposit is given, the bond shall be cancelled or the sum deposited shall be returned to the depositor or his legal representative." Thus the issue boils down to whether an alien female who marries a male citizen of the Philippines follows ipso facto his political status. The pertinent part of section 15 of Commonwealth Act No. 473, upon which petitioners rely, reads: Any woman who is now or may hereafter be married to a citizen of the Philippines, and who might herself be lawfully naturalized shall be deemed a citizen of the Philippines. Pursuant thereto, marriage to a male Filipino does not vest Philippine citizenship to his foreign wife, unless she "herself may be lawfully naturalized." As correctly held in an opinion of the Secretary of Justice (Op. No. 52, series of 1950),* this limitation of section 15 excludes, from the benefits of naturalization by marriage, those disqualified from being naturalized as citizens of the Philippines under section 4 of said Commonwealth Act No. 473, namely: (a) Persons opposed to organized government or affiliated with any association or group of persons who uphold and teach doctrines opposing all organized governments;

(b) Persons defending or teaching the necessity or propriety of violence, personal assault, or assassination for the success and predominance of their ideas; (c) Polygamists or believers in the practice of polygamy; (d) Persons convicted of crimes involving moral turpitude; (e) Persons suffering from mental alienation or incurable contagious diseases; (f) Persons who, during the period of their residence in the Philippines, have not mingled socially with the Filipinos, or who have not evinced a sincere desire to learn and embrace the customs, traditions, and ideals of the Filipinos; (g) Citizens or subjects of nations with whom the ... Philippines are at war, during the period of such war; (h) Citizens or subjects of a foreign country other than the United States, whose laws does not grant Filipinos the right to become naturalized citizens or subjects thereof. In the case at bar, there is neither proof nor allegation in the pleadings that Ly Giok Ha does not fall under any of the classes disqualified by law. Moreover, as the parties who claim that, despite her failure to depart from the Philippines within the period specified in the bond in question, there has been no breach thereof, petitioners have the burden of proving her alleged change of political status, from alien to citizen. Strictly speaking, petitioners have not made out, therefore a case against the respondents-appellants. Considering, however, that neither in the administrative proceedings, nor in the lower court, had the parties seemingly felt that there was an issue on whether Ly Giok Ha may "be lawfully naturalized," and this being a case of first impression in our courts, we are of the opinion that, in the interest of equity and justice, the parties herein should be given an opportunity to introduce evidence, if they have any, on said issue. (At pp. 462-464.) . As may be seen, although not specifically in so many words, no doubt was left in the above decision as regards the following propositions: . 1. That under Section 15 of Commonwealth Act 473, the Revised Naturalization Law, the marriage of an alien woman to a Filipino makes her a Filipina, if she "herself might be lawfully naturalized"; 2. That this Court declared as correct the opinion of the Secretary of Justice that the limitation of Section 15 of the Naturalization Law excludes from the benefits of naturalization by marriage, only those disqualified from being naturalized under Section 4 of the law qouted in the decision; 3. That evidence to the effect that she is not disqualified may be presented in the action to recover her bond confiscated by the Commissioner of Immigration; 4. That upon proof of such fact, she may be recognized as Filipina; and 5. That in referring to the disqualification enumerated in the law, the Court somehow left the impression that no inquiry need be made as to qualifications, 5 specially considering that the decision cited and footnotes several opinions of the Secretary of Justice, the immediate superior of the Commissioner of Immigration, the most important of which are the following: Paragraph (a), section 13 of Act No. 2927, as amended, (now section 15, Commonwealth Act No. 473), provided that "any woman who is now or may hereafter be married to a citizen of the Philippines, and who might herself be lawfully naturalized shall be deemed a citizen of the Philippines." A similar provision in the naturalization law of the United States has been construed as not requiring the woman to have the qualifications of residence, good character, etc., as in the case of naturalization by judicial proceedings, but merely that she is of the race of persons who may be naturalized. (Kelly v. Owen [Dist. Col. 1868] 7 Wall 496, 5F, 11, 12; ex parte Tryason [D. C. Wash. 1914] 215 F. 449, 27 Op. Atty. Gen. 507). (Op. No. 168, s. 1940 of Justice Sec. Jose Abad Santos.) In a previous opinion rendered for your Office, I stated that the clause "who might herself be lawfully naturalized", should be construed as not requiring the woman to have the qualifications of residence, good character, etc., as in cases of naturalization by judicial proceedings, but merely that she is of the race of persons who may be naturalized. (Op. No. 79, s. 1940) Inasmuch as the race qualification has been removed by the Revised Naturalization Law, it results that any woman who married a citizen of the Philippines prior to or after June 17, 1939, and the marriage not having been dissolved, and on the assumption that she possesses none of the disqualifications mentioned in Section 4 of Commonwealth Act No. 473, follows the citizenship of her husband. (Op. No. 176, s. 1940 of Justice Sec. Jose Abad Santos.) From the foregoing narration of facts, it would seem that the only material point of inquiry is as to the citizenship of Arce Machura. If he shall be found to be a citizen of the Philippines, his wife, Mrs. Lily James Machura, shall likewise be deemed a citizen of the Philippines pursuant to the provision of Section 15, Commonwealth Act No. 473, which reads in part as follows:

Any woman who is now or may hereafter be married to a citizen of the Philippines, and who might herself be lawfully naturalized shall be deemed a citizen of the Philippines. The phrase "who might herself be lawfully naturalized", as contained in the above provision, means that the woman who is married to a Filipino citizen must not belong to any of the disqualified classes enumerated in Section 4 of the Naturalization Law (Ops., Sec. of Jus., No. 28, s. 1950; No. 43, s. 1948, No. 95, s. 1941; Nos. 79 and 168, s. 1940). Under the facts stated in the within papers, Mrs. Machura does not appear to be among the disqualified classes mentioned in the law. It having been shown that Arce Machura or Arsenio Guevara was born as an illegitimate of a Filipino mother, he should be considered as a citizen of the Philippines in consonance with the well-settled rule that an illegitimate child follows the citizenship of his only legally recognized parent, the mother (Op., Sec. of Jus., Nos. 58, 98 & 281, s. 1948; No. 96, s. 1949). Her husband being a Filipino, Mrs. Machura must necessarily be deemed as a citizen of the Philippines by marriage (Sec. 15, Com. Act No. 473.) (Op. No. 52, s. 1950 of Justice Sec. Ricardo Nepomuceno.) The logic and authority of these opinions, compelling as they are, must have so appealed to this Court that five days later, on May 22, 1957, in Ricardo Cua v. The Board of Commissioners, 101 Phil. 521, Mr. Justice J.B.L. Reyes, reiterated the same ruling on the basis of the following facts: Tjioe Wu Suan, an Indonesian, arrived in Manila on November 1, 1952, but it turned out that her passport was forged. On December 10, 1953, a warrant was issued for her arrest for purpose of deportation. Later, on December 20, 1953, she married Ricardo Cua, a Filipino, and because of said marriage, the Board of Special Inquiry considered her a Filipina. Upon a review of the case, however, the Board of Immigration Commissioners insisted on continuing with the deportation proceedings and so, the husband filed prohibition and mandamus proceedings. The lower court denied the petition. Although this Court affirmed said decision, it held, on the other hand, that: Granting the validity of marriage, this Court has ruled in the recent case of Ly Giok Ha v. Galang, supra, p. 459, that the bare fact of a valid marriage to a citizen does not suffice to confer his citizenship upon the wife. Section 15 of the Naturalization Law requires that the alien woman who marries a Filipino must show, in addition, that she "might herself be lawfully naturalized" as a Filipino citizen. As construed in the decision cited, this last condition requires proof that the woman who married a Filipino is herself not disqualified under section 4 of the Naturalization Law. No such evidence appearing on record, the claim of assumption of Filipino citizenship by Tjioe Wu Suan, upon her marriage to petitioner, is untenable. The lower court, therefore, committed no error in refusing to interfere with the deportation proceedings, where she can anyway establish the requisites indispensable for her acquisition of Filipino citizenship, as well as the alleged validity of her Indonesian passport. (Ricardo Cua v. The Board of Immigration Commissioners, G. R. No. L-9997, May 22, 1957, 101 Phil. 521, 523.) [Emphasis supplied] . For emphasis, it is reiterated that in the above two cases, this Court expressly gave the parties concerned opportunity to prove the fact that they were not suffering from any of the disqualifications of the law without the need of undergoing any judicial naturalization proceeding. It may be stated, therefore, that according to the above decisions, the law in this country, on the matter of the effect of marriage of an alien woman to a Filipino is that she thereby becomes a Filipina, if it can be proven that at the time of such marriage, she does not possess any of the disqualifications enumerated in Section 4 of the Naturalization Law, without the need of submitting to any naturalization proceedings under said law. It is to be admitted that both of the above decisions made no reference to qualifications, that is, as to whether or not they need also to be proved, but, in any event, it is a fact that the Secretary of Justice understood them to mean that such qualifications need not be possessed nor proven. Then Secretary of Justice Jesus Barrera, who later became a distinguished member of this Court, 6 so ruled in opinions rendered by him subsequent to Ly Giok Ha, the most illustrative of which held: . At the outset it is important to note that an alien woman married to a Filipino citizen needs only to show that she "might herself be lawfully naturalized" in order to acquire Philippine citizenship. Compliance with other conditions of the statute, such as those relating to the qualifications of an applicant for naturalization through judicial proceedings, is not necessary. (See: Leonard v. Grant, 5 Fed. 11; 27 Ops. Atty. Gen [U.S.] 507; Ops. Sec. of Justice, No. 776, s. 1940, and No. 111, s. 1953. This view finds support in the case of Ly Giok Ha et al. v. Galang et al., G.R. No. L-10760, promulgated May 17, 1957, where the Supreme Court, construing the abovequoted section of the Naturalization Law, held that "marriage to a male Filipino does not vest Philippine citizenship to his foreign wife," unless she "herself may be lawfully naturalized," and that "this limitation of Section 15 excludes, from the benefits of naturalization by marriage, those disqualified from being naturalized as citizens of the Philippines under Section 4 of said Commonwealth Act No. 473." In other words, disqualification for any of the causes enumerated in Section 4 of the Act is the decisive factor that defeats the right of the foreign wife of a Philippine citizen to acquire Philippine citizenship. xxx xxx xxx Does petitioner, Lim King Bian, belong to any of these groups The Commissioner of Immigration does not say so but merely predicates his negative action on the ground that a warrant of deportation for "overstaying" is pending against the petitioner. We do not believe the position is well taken. Since the grounds for disqualification for naturalization are expressly enumerated in the law, a warrant of deportation not based on a finding of unfitness to become naturalized for any of those specified causes may not be invoked to negate acquisition of Philippine citizenship by a foreign wife of a Philippine

citizen under Section 15 of the Naturalization Law. (Inclusio unius est exclusio alterius) (Op. No. 12, s. 1958 of Justice Undersec. Jesus G. Barrera.) Regarding the steps that should be taken by an alien woman married to a Filipino citizen in order to acquire Philippine citizenship, the procedure followed in the Bureau of Immigration is as follows: The alien woman must file a petition for the cancellation of her alien certificate of registration alleging, among other things, that she is married to a Filipino citizen and that she is not disqualified from acquiring her husband's citizenship pursuant to section 4 of Commonwealth Act No. 473, as amended. Upon the filing of said petition, which should be accompanied or supported by the joint affidavit of the petitioner and her Filipino husband to the effect that the petitioner does not belong to any of the groups disqualified by the cited section from becoming naturalized Filipino citizen (please see attached CEB Form 1), the Bureau of Immigration conducts an investigation and thereafter promulgates its order or decision granting or denying the petition. (Op. No. 38, s. 19058 of Justice Sec. Jesus G. Barrera.) This view finds support in the case of Ly Giok Ha et al., v. Galang et al. (G.R. No. L-10760, promulgated May 17, 1957), where the Supreme Court, construing the above-quoted section in the Revised Naturalization Law, held that "marriage to a male Filipino does not vest Philippine citizenship to his foreign wife, unless she herself may be lawfully naturalized," and that "this limitation of Section 15 excludes, from the benefits of naturalization by marriage, those disqualified from being naturalized as citizens of the Philippines under Section 4 of said Commonwealth Act No. 473." In other words, disqualification for any of the causes enumerated in section 4 of the Act is the decisive factor that defeats the right of an alien woman married to a Filipino citizen to acquire Philippine citizenship. (Op. 57, s. 1958 of Justice Sec. Jesus G. Barrera.) The contention is untenable. The doctrine enunciated in the Ly Giok Ha case is not a new one. In that case, the Supreme Court held that under paragraph I of Section 15 Of Commonwealth Act No. 473, 'marriage to a male Filipino does not vest Philippine citizenship to his foreign wife unless she "herself may be lawfully naturalized"', and, quoting several earlier opinions of the Secretary of Justice, namely: No. 52, s. 1950; No. 168, s. 1940; No. 95, s. 1941; No. 63, s. 1948; No. 28. s. 1950, "this limitation of section 15 excludes from the benefits of naturalization by marriage, those disqualified from being naturalized as citizens of the Philippines under section 4 of said Commonwealth Act No. 473." (Op. 134, s. 1962 of Justice Undersec. Magno S. Gatmaitan.) It was not until more than two years later that, in one respect, the above construction of the law was importantly modified by this Court in Lee Suan Ay, supra, in which the facts were as follows: Upon expiration of the appellant Lee Suan Ay's authorized period of temporary stay in the Philippines (25 March 1955), on 26 March 1955 the Commissioner of Immigration asked the bondsman to present her to the Bureau of Immigration within 24 hours from receipt of notice, otherwise the bond will be confiscated(Annex 1). For failure of the bondsman to comply with the foregoing order, on 1 April 1955. the Commissioner of Immigration ordered the cash bond confiscated (Annex E). Therefore, there was an order issued by the Commissioner of Immigration confiscating or forfeiting the cash bond. Unlike in forfeiture of bail bonds in criminal proceedings, where the Court must enter an order forfeiting the bail bond and the bondsman must be given an opportunity to present his principal or give a satisfactory reason for his inability to do so, before final judgment may be entered against the bondsman,(section 15, Rule 110; U.S. v. Bonoan, 22 Phil. 1.) in forfeiture of bonds posted for the temporary stay of an alien in the Philippines, no court proceeding is necessary. Once a breach of the terms and conditions of the undertaking in the bond is committed, the Commissioner of Immigration may, under the terms and conditions thereof, declare it forfeited in favor of the Government. (In the meanwhile, on April 1, 1955, Lee Suan Ay and Alberto Tan, a Filipino, were joined in marriage by the Justice of the Peace of Las Pias, Rizal.) Mr. Justice Sabino Padilla speaking for a unanimous court which included Justices Concepcion and Reyes who had penned Ly Giok Ha, and Ricardo Cua, ruled thus: The fact that Lee Suan Ay (a Chinese) was married to a Filipino citizen does not relieve the bondsman from his liability on the bond. The marriage took place on 1 April 1955, and the violation of the terms and conditions of the undertaking in the bond failure to depart from the Philippines upon expiration of her authorized period of temporary stay in the Philippines (25 March 1955) and failure to report to the Commissioner of Immigration within 24 hours from receipt of notice were committed before the marriage. Moreover, the marriage of a Filipino citizen to an alien does not automatically confer Philippine citizenship upon the latter. She must possess the qualifications required by law to become a Filipino citizen by naturalization.* There is no showing that the appellant Lee Suan Ay possesses all the qualifications and none of the disqualifications provided for by law to become a Filipino citizen by naturalization. Pertinently to be noted at once in this ruling, which, to be sure, is the one relied upon in the appealed decision now before Us, is the fact that the footnote of the statement therein that the alien wife "must possess the qualifications required by law to become a Filipino citizen by naturalization" makes reference to Section 15, Commonwealth Act 473 and precisely, also to Ly Giok Ha v. Galang, supra. As will be recalled, on the other hand, in the opinions of the Secretary of Justice explicitly adopted by the Court in Ly Giok Ha, among them, Opinion No. 176, Series of 1940, above-quoted, it was clearly held that "(I)n a previous opinion rendered for your Office, I stated that the clause "who might herself be lawfully naturalized", should be construed as not requiring the woman to have the qualifications of residence, good character, etc., as in cases of naturalization by judicial proceedingsbut merely that she is of the race by persons who may be naturalized. (Op. No. 79, s. 1940) Since Justice Padilla gave no reason at all for the obviously significant modification of the construction of the law, it could be said that there was need for clarification of the seemingly new posture of the Court. The occasion for such clarification should have been in Kua Suy, etc., et al. vs. The Commissioner of Immigration, G.R. No. L-13790, October 31, 1963, penned by Mr. Justice J.B.L. Reyes, who had rendered the opinion in Ricardo Cua,supra, which followed that in Ly Giok Ha, supra, but apparently seeing no immediate relevancy in the case on hand then of the particular point in issue now, since it was not squarely raised therein similarly as in Lee Suan Ay, hence, anything said on the said matter would at best be no more than obiter dictum, Justice Reyes limited himself to holding that "Under Section 15 of the Naturalization Act, the wife is deemed a citizen of the Philippines only if she "might herself be lawfully naturalized," so that the fact of marriage to a citizen, by itself

alone, does not suffice to confer citizenship, as this Court has previously ruled in Ly Giok Ha v. Galang, 54 O.G. 356, and in Cua v. Board of Immigration Commissioners, 53 O.G. 8567; and there is here no evidence of record as to the qualifications or absence of disqualifications of appellee Kua Suy", without explaining the apparent departure already pointed out from Ly Giok Ha and Ricardo Cua. Even Justice Makalintal, who wrote a separate concurring and dissenting opinion merely lumped together Ly Giok Ha, Ricardo Cua and Lee Suan Ay and opined that both qualifications and non-disqualifications have to be shown without elucidating on what seemed to be departure from the said first two decisions. It was only on November 30, 1963 that to Mr. Justice Roberto Regala fell the task of rationalizing the Court's position. In Lo San Tuang v. Galang, G.R. No. L-18775, November 30, 1963, 9 SCRA 638, the facts were simply these: Lo San Tuang, a Chinese woman, arrived in the Philippines on July 1, 1960 as a temporary visitor with authority to stay up to June 30, 1961. She married a Filipino on January 7, 1961, almost six months before the expiry date of her permit, and when she was requested to leave after her authority to stay had expired, she refused to do so, claiming she had become a Filipina by marriage, and to bolster her position, she submitted an affidavit stating explicitly that she does not possess any of the disqualifications enumerated in the Naturalization Law, Commonwealth Act 473. When the case reached the court, the trial judge held for the government that in addition to not having any of the disqualifications referred to, there was need that Lo San Tuang should have also possessed all the qualifications of residence, moral character, knowledge of a native principal dialect, etc., provided by the law. Recognizing that the issue squarely to be passed upon was whether or not the possession of all the qualifications were indeed needed to be shown apart from non-disqualification, Justice Regala held affirmatively for the Court, reasoning out thus: . It is to be noted that the petitioner has anchored her claim for citizenship on the basis of the decision laid down in the case of Leonard v. Grant, 5 Swy. 603, 5 F 11, where the Circuit Court of Oregon held that it was only necessary that the woman "should be a person of the class or race permitted to be naturalized by existing laws, and that in respect of the qualifications arising out of her conduct or opinions, being the wife of a citizen, she is to be regarded as qualified for citizenship, and therefore considered a citizen." (In explanation of its conclusion, the Court said: "If, whenever during the life of the woman or afterwards, the question of her citizenship arises in a legal proceeding, the party asserting her citizenship by reason of her marriage with a citizen must not only prove such marriage, but also that the woman then possessed all the further qualifications necessary to her becoming naturalized under existing laws, the statute will be practically nugatory, if not a delusion and a share. The proof of the facts may have existed at the time of the marriage, but years after, when a controversy arises upon the subject, it may be lost or difficult to find.") In other words, all that she was required to prove was that she was a free white woman or a woman of African descent or nativity, in order to be deemed an American citizen, because, with respect to the rest of the qualifications on residence, moral character, etc., she was presumed to be qualified. Like the law in the United States, our former Naturalization Law (Act No. 2927, as amended by Act No. 3448) specified the classes of persons who alone might become citizens of the Philippines, even as it provided who were disqualified. Thus, the pertinent provisions of that law provided: Section 1. Who may become Philippine citizens Philippine citizenship may be acquired by (a) natives of the Philippines who are not citizens thereof under the Jones Law; (b) natives of the Insular possessions of the United States; (c) citizens of the United States, or foreigners who under the laws of the United States may become citizens of said country if residing therein. Section 2. Who are disqualified. The following cannot be naturalized as Philippine citizens: (a) Persons opposed to organized government or affiliated with any association or group of persons who uphold and teach doctrines opposing all organized government; (b) persons defending or teaching the necessity or propriety of violence, personal assault or assassination for the success and predominance of their ideas; (c) polygamists or believers in the practice of polygamy; (d) persons convicted of crimes involving moral turpitude; (e) persons suffering from mental alienation or incurable contagious diseases; (f) citizens or subjects of nations with whom the United States and the Philippines are at war, during the period of such war. Section 3. Qualifications. The persons comprised in subsection (a) of section one of this Act, in order to be able to acquire Philippine citizenship, must be not less than twenty-one years of age on the day of the hearing of their petition. The persons comprised in subsections (b) and (c) of said section one shall, in addition to being not less than twenty-one years of age on the day of the hearing of the petition, have all and each of the following qualifications: First. Residence in the Philippine Islands for a continuous period of not less than five years, except as provided in the next following section; Second. To have conducted themselves in a proper and irreproachable manner during the entire period of their residence in the Philippine Islands, in their relation with the constituted government as well as with the community in which they are living; Third. To hold in the Philippine Islands real estate worth not less than one thousand pesos, Philippine currency, or have some known trade or profession; and Fourth. To speak and write English, Spanish, or some native tongue.

In case the petitioner is a foreign subject, he shall, besides, declare in writing and under oath his intention of renouncing absolutely and perpetually all faith and allegiance to the foreign authority, state or sovereignty of which he was a native, citizen or subject. Applying the interpretation given by Leonard v. Grant supra, to our law as it then stood, alien women married to citizens of the Philippines must, in order to be deemed citizens of the Philippines, be either (1) natives of the Philippines who were not citizens thereof under the Jones Law, or (2) natives of other Insular possessions of the United States, or (3) citizens of the United States or foreigners who under the laws of the United States might become citizens of that country if residing therein. With respect to the qualifications set forth in Section 3 of the former law, they were deemed to have the same for all intents and purposes. But, with the approval of the Revised Naturalization Law (Commonwealth Act No. 473) on June 17, 1939, Congress has since discarded class or racial consideration from the qualifications of applicants for naturalization (according to its proponent, the purpose in eliminating this consideration was, first, to remove the features of the existing naturalization act which discriminated in favor of the Caucasians and against Asiatics who are our neighbors, and are related to us by racial affinity and, second, to foster amity with all nations [Sinco, Phil. Political Law 502 11 ed.]), even as it retained in Section 15 the phrase in question. The result is that the phrase "who might herself be lawfully naturalized" must be understood in the context in which it is now found, in a setting so different from that in which it was found by the Court in Leonard v. Grant. The only logical deduction from the elimination of class or racial consideration is that, as the Solicitor General points out, the phrase "who might herself be lawfully naturalized" must now be understood as referring to those who under Section 2 of the law are qualified to become citizens of the Philippines. There is simply no support for the view that the phrase "who might herself be lawfully naturalized" must now be understood as requiring merely that the alien woman must not belong to the class of disqualified persons under Section 4 of the Revised Naturalization Law. Such a proposition misreads the ruling laid down in Leonard v. Grant. A person who is not disqualified is not necessarily qualified to become a citizen of the Philippines, because the law treats "qualifications" and "disqualifications" in separate sections. And then it must not be lost sight of that even under the interpretation given to the former law, it was to be understood that the alien woman was not disqualified under Section 2 of that law. Leonard v. Grant did not rule that it was enough if the alien woman does not belong to the class of disqualified persons in order that she may be deemed to follow the citizenship of her husband: What that case held was that the phrase "who might herself be lawfully naturalized, merely means that she belongs to the class or race of persons qualified to become citizens by naturalization the assumption being always that she is not otherwise disqualified. We therefore hold that under the first paragraph of Section 15 of the Naturalization Law, an alien woman, who is married to a citizen of the Philippines, acquires the citizenship of her husband only if she has all the qualifications and none of the disqualifications provided by law. Since there is no proof in this case that petitioner has all the qualifications and is not in any way disqualified, her marriage to a Filipino citizen does not automatically make her a Filipino citizen. Her affidavit to the effect that she is not in any way disqualified to become a citizen of this country was correctly disregarded by the trial court, the same being self-serving. Naturally, almost a month later in Sun Peck Yong v. Commissioner of Immigration, G.R. No. L-20784, December 27, 1963, 9 SCRA 875, wherein the Secretary of Foreign Affairs reversed a previous resolution of the preceding administration to allow Sun Peck Yong and her minor son to await the taking of the oath of Filipino citizenship of her husband two years after the decision granting him nationalization and required her to leave and this order was contested in court, Justice Barrera held: In the case of Lo San Tuang v. Commissioner of Immigration (G.R. No. L-18775, promulgated November 30, 1963; Kua Suy vs. Commissioner of Immigration, L-13790, promulgated October 31, 1963), we held that the fact that the husband became a naturalized citizen does not automatically make the wife a citizen of the Philippines. It must also be shown that she herself possesses all the qualifications, and none of the disqualifications, to become a citizen. In this case, there is no allegation, much less showing, that petitioner-wife is qualified to become a Filipino citizen herself. Furthermore, the fact that a decision was favorably made on the naturalization petition of her husband is no assurance that he (the husband) would become a citizen, as to make a basis for the extension of her temporary stay. On the same day, in Tong Siok Sy v. Vivo, G.R. No. L-21136, December 27, 1963, 9 SCRA 876, Justice Barrera reiterated the same ruling and citing particularly Lo San Tuang and Kua Suy, held that the marriage of Tong Siok Sy to a Filipino on November 12, 1960 at Taichung, Taiwan and her taking oath of Filipino citizenship before the Philippine Vice-Consul at Taipeh, Taiwan on January 6, 1961 did not make her a Filipino citizen, since she came here only in 1961 and obviously, she had not had the necessary ten-year residence in the Philippines required by the law. Such then was the status of the jurisprudential law on the matter under discussion when Justice Makalintal sought a reexamination thereof in Choy King Tee v. Galang, G.R. No. L-18351, March 26, 1965, 13 SCRA 402. Choy King Tee's husband was granted Philippine citizenship on January 13, 1959 and took the oath on January 31 of the same year. Choy King Tee first came to the Philippines in 1955 and kept commuting between Manila and Hongkong since then, her last visa before the case being due to expire on February 14, 1961. On January 27, 1961, her husband asked the Commissioner of Immigration to cancel her alien certificate of registration, as well as their child's, for the reason that they were Filipinos, and when the request was denied as to the wife, a mandamus was sought, which the trial court granted. Discussing anew the issue of the need for qualifications, Justice Makalintal not only reiterated the arguments of Justice Regala in Lo San Tuang but added further that the ruling is believed to be in line with the national policy of selective admission to Philippine citizenship. 7 No wonder, upon this authority, in Austria v. Conchu, G.R. No. L-20716, June 22, 1965, 14 SCRA 336, Justice J.P. Bengzon readily reversed the decision of the lower court granting the writs of mandamus and prohibition against the Commissioner of Immigration, considering that Austria's wife, while admitting she did not possess all the qualifications for naturalization, had submitted only an affidavit that she had none of

the disqualifications therefor. So also did Justice Dizon similarly hold eight days later in Brito v. Commissioner, G.R. No. L-16829, June 30, 1965, 14 SCRA 539. Then came the second Ly Giok Ha case 8 wherein Justice J. B. L. Reyes took occasion to expand on the reasoning of Choy King Tee by illustrating with examples "the danger of relying exclusively on the absence of disqualifications, without taking into account the other affirmative requirements of the law." 9 Lastly, in Go Im Ty v. Republic, G.R. No. L-17919, decided on July 30, 1966, 10 Justice Zaldivar held for the Court that an alien woman who is widowed during the dependency of the naturalization proceedings of her husband, in order that she may be allowed to take the oath as Filipino, must, aside from proving compliance with the requirements of Republic Act 530, show that she possesses all the qualifications and does not suffer from any of the disqualifications under the Naturalization Law, citing in the process the decision to such effect discussed above, 11 even as he impliedly reversed pro tanto the ruling in Tan Lin v. Republic, G.R. No. L-13786, May 31, 1961, 2 SCRA 383. Accordingly, in Burca, Justice Sanchez premised his opinion on the assumption that the point now under discussion is settled law. In the case now at bar, the Court is again called upon to rule on the same issue. Under Section 15 of the Naturalization Law, Commonwealth Act 473, providing that: SEC. 15. Effect of the naturalization on wife and children. Any woman, who is now or may hereafter be married to a citizen of the Philippines, and who might herself be lawfully naturalized shall be deemed a citizen of the Philippines. Minor children of persons naturalized under this law who have been born in the Philippines shall be considered citizens thereof. A foreign-born minor child, if dwelling in the Philippines at the time of the naturalization of the parent, shall automatically become a Philippine citizen, and a foreign-born child, who is not in the Philippines at the time the parent is naturalized, shall be deemed a Philippine citizen only during his minority, unless he begins to reside permanently in the Philippines when still a minor, in which case, he will continue to be a Philippine citizen even after becoming of age. A child born outside of the Philippines after the naturalization of his parent, shall be considered a Philippine citizen unless within one year after reaching the age of majority he fails to register himself as a Philippine citizen at the American Consulate of the country where he resides, and to take the necessary oath of allegiance. is it necessary, in order that an alien woman who marries a Filipino or who is married to a man who subsequently becomes a Filipino, may become a Filipino citizen herself, that, aside from not suffering from any of the disqualifications enumerated in the law, she must also possess all the qualifications required by said law? if nothing but the unbroken line from Lee Suan Ay to Go Im Ty, as recounted above, were to be considered, it is obvious that an affirmative answer to the question would be inevitable, specially, if it is noted that the present case was actually submitted for decision on January 21, 1964 yet, shortly after Lo San Tuang, Tong Siok Sy and Sun Peck Yong, all supra, and even before Choy King Tee, supra, were decided. There are other circumstances, however, which make it desirable, if not necessary, that the Court take up the matter anew. There has been a substantial change in the membership of the Court since Go Im Ty, and of those who were in the Court already when Burca was decided, two members, Justice Makalintal and Castro concurred only in the result, precisely, according to them, because (they wanted to leave the point now under discussion open in so far as they are concerned. 12 Truth to tell, the views and arguments discussed at length with copious relevant authorities, in the motion for reconsideration as well as in the memorandum of the amici curae 13 in the Burca case cannot just be taken lightly and summarily ignored, since they project in the most forceful manner, not only the legal and logical angles of the issue, but also the imperative practical aspects thereof in the light of the actual situation of the thousands of alien wives of Filipinos who have so long, even decades, considered themselves as Filipinas and have always lived and acted as such, officially or otherwise, relying on the long standing continuous recognition of their status as such by the administrative authorities in charge of the matter, as well as by the courts. Under these circumstances, and if only to afford the Court an opportunity to consider the views of the five justices who took no part in Go Im Ty (including the writer of this opinion), the Court decided to further reexamine the matter. After all, the ruling first laid in Lee Suan Ay, and later in Lo San Tuang, Choy King Tee stand the second (1966) Ly Giok Ha, did not categorically repudiate the opinions of the Secretary of Justice relied upon by the first (1959) Ly Giok Ha. Besides, some points brought to light during the deliberations in this case would seem to indicate that the premises of the later cases can still bear further consideration. Whether We like it or not, it is undeniably factual that the legal provision We are construing, Section 15, aforequoted, of the Naturalization Law has been taken directly, copied and adopted from its American counterpart. To be more accurate, said provision is nothing less than a reenactment of the American provision. A brief review of its history proves this beyond per adventure of doubt. The first Naturalization Law of the Philippines approved by the Philippine Legislature under American sovereignty was that of March 26, 1920, Act No. 2927. Before then, as a consequence of the Treaty of Paris, our citizenship laws were found only in the Organic Laws, the Philippine Bill of 1902, the Act of the United States Congress of March 23, 1912 and later the Jones Law of 1916. In fact, Act No. 2927 was enacted pursuant to express authority granted by the Jones Law. For obvious reasons, the Philippines gained autonomy on the subjects of citizenship and immigration only after the effectivity of the Philippine Independence Act. This made it practically impossible for our laws on said subject to have any perspective or orientation of our own; everything was American. The Philippine Bill of 1902 provided pertinently: . SECTION 4. That all inhabitants of the Philippine Islands continuing to reside herein who were Spanish subjects on the eleventh day of April, eighteen-hundred and ninety-nine, and then resided in said Islands, and their children born subsequent thereto, shall be deemed and held to be citizens of the Philippine Islands and as such entitled to the protection of the United States, except such as shall have elected to preserve their allegiance to the Crown of Spain in accordance with the provisions of the treaty of peace between the United States and Spain signed at Paris December tenth, eighteen hundred and ninety-eight.

This Section 4 of the Philippine Bill of 1902 was amended by Act of Congress of March 23, 1912, by adding a provision as follows: Provided, That the Philippine Legislature is hereby authorized to provide by law for the acquisition of Philippine citizenship by those natives of the Philippine Islands who do not come within the foregoing provisions, the natives of other insular possessions of the United States, and such other persons residing in the Philippine Islands who would become citizens of the United States, under the laws of the United States, if residing therein. The Jones Law reenacted these provisions substantially: . SECTION 2. That all inhabitants of the Philippine Islands who were Spanish subjects on the eleventh day of April, eighteen hundred and ninety-nine, and then resided in said islands, and their children born subsequent thereto, shall be deemed and held to be citizens of the Philippine Islands, except such as shall have elected to preserve their allegiance to the Crown of Spain in accordance with the provisions of the treaty of peace between the United States and Spain, signed at Paris December tenth, eighteen hundred and ninety-eight and except such others as have since become citizens of some other country: Provided, That the Philippine Legislature, herein provided for, is hereby authorized to provide by law for the acquisition of Philippine citizenship by those natives of the Philippine Islands who do not come within the foregoing provisions, the natives of the insular possessions of the United States, and such other persons residing in the Philippine Islands who are citizens of the United States under the laws of the United States if residing therein. For aught that appears, there was nothing in any of the said organic laws regarding the effect of marriage to a Filipino upon the nationality of an alien woman, albeit under the Spanish Civil Code provisions on citizenship, Articles 17 to 27, which were, however, abrogated upon the change of sovereignty, it was unquestionable that the citizenship of the wife always followed that of the husband. Not even Act 2927 contained any provision regarding the effect of naturalization of an alien, upon the citizenship of his alien wife, nor of the marriage of such alien woman with a native born Filipino or one who had become a Filipino before the marriage, although Section 13 thereof provided thus: . SEC. 13. Right of widow and children of petitioners who have died. In case a petitioner should die before the final decision has been rendered, his widow and minor children may continue the proceedings. The decision rendered in the case shall, so far as the widow and minor children are concerned, produce the same legal effect as if it had been rendered during the life of the petitioner. It was not until November 30, 1928, upon the approval of Act 3448, amending Act 2977, that the following provisions were added to the above Section 13: SECTION 1. The following new sections are hereby inserted between sections thirteen and fourteen of Act Numbered Twenty-nine hundred and Twenty-seven: SEC. 13(a). Any woman who is now or may hereafter be married to a citizen of the Philippine Islands and who might herself be lawfully naturalized, shall be deemed a citizen of the Philippine Islands. SEC. 13(b). Children of persons who have been duly naturalized under this law, being under the age of twenty-one years at the time of the naturalization of their parents, shall, if dwelling in the Philippine Islands, be considered citizens thereof. SEC. 13(c). Children of persons naturalized under this law who have been born in the Philippine Islands after the naturalization of their parents shall be considered citizens thereof. When Commonwealth Act 473, the current naturalization law, was enacted on June 17, 1939, the above Section 13 became its Section 15 which has already been quoted earlier in this decision. As can be seen, Section 13 (a) abovequoted was re-enacted practically word for word in the first paragraph of this Section 15 except for the change of Philippine Islands to Philippines. And it could not have been on any other basis than this legislative history of our naturalization law that each and everyone of the decisions of this Court from the first Ly Giok Ha to Go Im Ty, discussed above, were rendered. As stated earlier, in the opinion of Chief Justice Concepcion in the first Ly Giok Ha, it was quite clear that for an alien woman who marries a Filipino to become herself a Filipino citizen, there is no need for any naturalization proceeding because she becomes a Filipina ipso facto from the time of such marriage, provided she does not suffer any of the disqualifications enumerated in Section 4 of Commonwealth Act 473, with no mention being made of whether or not the qualifications enumerated in Section 2 thereof need be shown. It was only in Lee Suan Ay in 1959 that the possession of qualifications were specifically required, but it was not until 1963, in Lo San Tuang, that Justice Regala reasoned out why the possession of the qualifications provided by the law should also be shown to be possessed by the alien wife of a Filipino, for her to become a Filipina by marriage. As may be recalled, the basic argument advanced by Justice Regala was briefly as follows: That "like the law in the United States, our Naturalization Law specified the classes of persons who alone might become citizens, even as it provided who were disqualified," and inasmuch as Commonwealth Act 473, our Naturalization Law since 1939 did not reenact the section providing who might become citizens, allegedly in order to remove racial discrimination in favor of Caucasians and against Asiatics, "the only logical deduction ... is that the phrase "who might herself be lawfully naturalized" must now be understood as referring to those who under Section 2 of the law are qualified to become citizens of the Philippines" and "there is simply no support for the view that the phrase "who might herself be lawfully naturalized" must now be understood as requiring merely that the alien woman must not belong to the class of disqualified persons under Section 4 of the Revised Naturalization Law." 14 A similar line of reasoning was followed in Choy King Tee, which for ready reference may be qouted:

The question has been settled by the uniform ruling of this Court in a number of cases. The alien wife of a Filipino citizen must first prove that she has all the qualifications required by Section 2 and none of the disqualifications enumerated in Section 4 of the Naturalization Law before she may be deemed a Philippine citizen (Lao Chay v. Galang, L-190977, Oct. 30, 1964, citing Lo San Tuang v. Galang, L-18775, Nov. 30, 1963; Sun Peck Yong v. Commissioner of Immigration, L20784, December 27, 1963; Tong Siok Sy v. Vivo, L-21136, December 27, 1963). The writer of this opinion has submitted the question anew to the court for a possible reexamination of the said ruling in the light of the interpretation of a similar law in the United States after which Section 15 of our Naturalization Law was patterned. That law was section 2 of the Act of February 10, 1855 (Section 1994 of the Revised Statutes of the U.S.). The local law, Act No. 3448, was passed on November 30, 1928 as an amendment to the former Philippine Naturalization Law, Act No. 2927, which was approved on March 26, 1920. Under this Naturalization Law, acquisition of Philippine citizenship was limited to three classes of persons, (a) Natives of the Philippines who were not citizens thereof; (b) natives of the other insular possessions of the United States; and (c) citizens of the United States, or foreigners who, under the laws of the United States, may become citizens of the latter country if residing therein. The reference in subdivision (c) to foreigners who may become American Citizens is restrictive in character, for only persons of certain specified races were qualified thereunder. In other words, in so far as racial restrictions were concerned there was at the time a similarity between the naturalization laws of the two countries and hence there was reason to accord here persuasive force to the interpretation given in the United States to the statutory provision concerning the citizenship of alien women marrying American citizens. This Court, however, believes that such reason has ceased to exist since the enactment of the Revised Naturalization Law, (Commonwealth Act No. 473) on June 17, 1939. The racial restrictions have been eliminated in this Act, but the provision found in Act No. 3448 has been maintained. It is logical to presume that when Congress chose to retain the said provision that to be deemed a Philippine citizen upon marriage the alien wife must be one "who might herself be lawfully naturalized," the reference is no longer to the class or race to which the woman belongs, for class or race has become immaterial, but to the qualifications and disqualifications for naturalization as enumerated in Sections 2 and 4 of the statute. Otherwise the requirement that the woman "might herself be lawfully naturalized" would be meaningless surplusage, contrary to settled norms of statutory construction. The rule laid down by this Court in this and in other cases heretofore decided is believed to be in line with the national policy of selective admission to Philippine citizenship, which after all is a privilege granted only to those who are found worthy thereof, and not indiscriminately to anybody at all on the basis alone of marriage to a man who is a citizen of the Philippines, irrespective of moral character, ideological beliefs, and identification with Filipino ideals, customs and traditions. Appellee here having failed to prove that she has all the qualifications for naturalization, even, indeed, that she has none of the disqualifications, she is not entitled to recognition as a Philippine citizen. In the second Ly Giok Ha, the Court further fortified the arguments in favor of the same conclusion thus: On cross-examination, she (Ly Giok Ha) failed to establish that: (1) she has been residing in the Philippines for a continuous period of at least (10) years (p. 27, t.s.n., id.); (2) she has a lucrative trade, profession, or lawful occupation (p. 13, t.s.n., id.); and (3) she can speak and write English, or any of the principal Philippine languages (pp. 12, 13, t.s.n., id.). While the appellant Immigration Commissioner contends that the words emphasized indicate that the present Naturalization Law requires that an alien woman who marries a Filipino husband must possess the qualifications prescribed by section 2 in addition to not being disqualified under any of the eight ("a" to "h") subheadings of section 4 of Commonwealth Act No. 473, in order to claim our citizenship by marriage, both the appellee and the court below (in its second decision) sustain the view that all that the law demands is that the woman be not disqualified under section 4. At the time the present case was remanded to the court of origin (1960) the question at issue could be regarded as not conclusively settled, there being only the concise pronouncement in Lee Suan Ay, et al. v. Galang, G. R. No. L-11855, Dec. 23, 1959, to the effect that: The marriage of a Filipino citizen to an alien does not automatically confer Philippine citizenship upon the latter. She must possess the qualifications required by law to become a Filipino citizen by naturalization. Since that time, however, a long line of decisions of this Court has firmly established the rule that the requirement of section 15 of Commonwealth Act 473 (the Naturalization Act), that an alien woman married to a citizen should be one who "might herself be lawfully naturalized," means not only woman free from the disqualifications enumerated in section 4 of the Act but also one who possesses the qualifications prescribed by section 2 of Commonwealth Act 473 (San Tuan v. Galang, L-18775, Nov. 30, 1963; Sun Peck Yong v. Com. of Immigration, L-20784, Dee. 27, 1963; Tong Siok Sy v. Vivo, L-21136, Dec. 27, 1963; Austria v. Conchu, L-20716, June 22, 1965; Choy King Tee v. Galang, L-18351, March 26, 1965; Brito v. Com. of Immigration, L-16829, June 30, 1965). Reflection will reveal why this must be so. The qualifications prescribed under section 2 of the Naturalization Act, and the disqualifications enumerated in its section 4 are not mutually exclusive; and if all that were to be required is that the wife of a Filipino be not disqualified under section 4, the result might well be that citizenship would be conferred upon persons in violation of the policy of the statute. For example, section 4 disqualifies only (c) Polygamists or believers in the practice of polygamy; and

(d) Persons convicted of crimes involving moral turpitude, so that a blackmailer, or a maintainer of gambling or bawdy houses, not previously convicted by a competent court would not be thereby disqualified; still, it is certain that the law did not intend such person to be admitted as a citizen in view of the requirement of section 2 that an applicant for citizenship "must be of good moral character." Similarly, the citizen's wife might be a convinced believer in racial supremacy, in government by certain selected classes, in the right to vote exclusively by certain "herrenvolk", and thus disbelieve in the principles underlying the Philippine Constitution; yet she would not be disqualified under section 4, as long as she is not "opposed to organized government," nor affiliated to groups "upholding or teaching doctrines opposing all organized governments", nor "defending or teaching the necessity or propriety of violence, personal assault or assassination for the success or predominance of their ideas." Et sic de caeteris. The foregoing instances should suffice to illustrate the danger of relying exclusively on the absence of disqualifications, without taking into account the other affirmative requirements of the law, which, in the case at bar, the appellee Ly Giok Ha admittedly does not possess. As to the argument that the phrase "might herself be lawfully naturalized" was derived from the U.S. Revised Statutes (section 1994) and should be given the same territorial and racial significance given to it by American courts, this Court has rejected the same in Lon San Tuang v. Galang, L-18775, November 30, 1963; and in Choy King Tee v. Galang, L18351, March 26, 1965. It is difficult to minimize the persuasive force of the foregoing rationalizations, but a closer study thereof cannot bat reveal certain relevant considerations which adversely affect the premises on which they are predicated, thus rendering the conclusions arrived thereby not entirely unassailable. 1. The main proposition, for instance, that in eliminating Section 1 of Act 2927 providing who are eligible for Philippine citizenship, the purpose of Commonwealth Act 473, the Revised Naturalization Law, was to remove the racial requirements for naturalization, thereby opening the door of Filipino nationality to Asiatics instead of allowing the admission thereto of Caucasians only, suffers from lack of exact accuracy. It is important to note, to start with, that Commonwealth Act 473 did away with the whole Section 1 of Act 2927 which reads, thus: SECTION 1. Who may become Philippine citizens. Philippine citizenship may be acquired by: (a) natives of the Philippines who are not citizens thereof under the Jones Law; (b) natives of the other Insular possessions of the United States; (c) citizens of the United States, or foreigners who under the laws of the United States may become citizens of said country if residing therein. and not only subdivision (c) thereof. Nowhere in this whole provision was there any mention of race or color of the persons who were then eligible for Philippine citizenship. What is more evident from said provision is that it reflected the inevitable subordination of our legislation during the pre-Commonwealth American regime to the understandable stations flowing from our staffs as a territory of the United States by virtue of the Treaty of Paris. In fact, Section 1 of Act 2927 was precisely approved pursuant to express authority without which it could not have been done, granted by an amendment to Section 4 of the Philippine Bill of 1902 introduced by the Act of the United States Congress of March 23, 1912 and which was reenacted as part of the Jones Law of 1916, the pertinent provisions of which have already been footed earlier. In truth, therefore, it was because of the establishment of the Philippine Commonwealth and in the exercise of our legislative autonomy on citizenship matters under the Philippine Independence Act that Section 1 of Act 2927 was eliminated, 15 and not purposely to eliminate any racial discrimination contained in our Naturalization Law. The Philippine Legislature naturally wished to free our Naturalization Law from the impositions of American legislation. In other words, the fact that such discrimination was removed was one of the effects rather than the intended purpose of the amendment. 2. Again, the statement in Choy King Tee to the effect that "the reference in subdivision (c) (of Section 1 of Act 2927) to foreigners who may become American citizens is restrictive in character, for only persons of certain specified races were qualified thereunder" fails to consider the exact import of the said subdivision. Explicitly, the thrust of the said subdivision was to confine the grant under it of Philippine citizenship only to the three classes of persons therein mentioned, the third of which were citizens of the United States and, corollarily, persons who could be American citizens under her laws. The words used in the provision do not convey any idea of favoring aliens of any particular race or color and of excluding others, but more accurately, they refer to all the disqualifications of foreigners for American citizenship under the laws of the United States. The fact is that even as of 1906, or long before 1920, when our Act 2927 became a law, the naturalization, laws of the United States already provided for the following disqualifications in the Act of the Congress of June 29, 1906: SEC. 7. That no person who disbelieves in or who is opposed to organized government, or who is a member of or affiliated with any organization entertaining and teaching such disbelief in or opposition to organized government, or who advocates or teaches the duty, necessity, or propriety of the unlawful assaulting or killing of any officer or officers, either of specific individuals or of officers generally, of the Government of the United States, or of any other organized government, because of his or their official character, or who is a polygamist, shall be naturalized or be made a citizen of the United States. and all these disqualified persons were, therefore, ineligible for Philippine citizenship under Section 1 of Act 2927 even if they happened to be Caucasians. More importantly, as a matter of fact, said American law, which was the first "Act to Establish a Bureau of Immigration and Naturalization and to provide for a Uniform Rule for Naturalization of Aliens throughout the United States" contained no racial disqualification requirement, except as to Chinese, the Act of May 6, 1882 not being among the expressly repealed by this law, hence it is clear that when Act 2927 was enacted, subdivision (e) of its Section 1 could not have had any connotation of racial exclusion necessarily, even if it were traced back to its origin in the Act of the United States Congress of 1912 already mentioned above. 16 Thus, it would seem that the rationalization in the qouted decisions predicated on the theory that the elimination of Section 1 of Act 2927 by Commonwealth Act 473 was purposely for no other end than the abolition of racial discrimination in our naturalization law has no clear factual basis. 17

3. In view of these considerations, there appears to be no cogent reason why the construction adopted in the opinions of the Secretary of Justice referred to in the first Ly Giok Ha decision of the Chief Justice should not prevail. It is beyond dispute that the first paragraph of Section 15 of Commonwealth Act 473 is a reenactment of Section 13(a) of Act 2927, as amended by Act 3448, and that the latter is nothing but an exact copy, deliberately made, of Section 1994 of the Raised Statutes of the United States as it stood before its repeal in 1922. 18 Before such repeal, the phrase "who might herself be lawfully naturalized" found in said Section 15 had a definite unmistakable construction uniformly foIlowed in all courts of the United States that had occasion to apply the same and which, therefore, must be considered, as if it were written in the statute itself. It is almost trite to say that when our legislators enacted said section, they knew of its unvarying construction in the United States and that, therefore, in adopting verbatim the American statute, they have in effect incorporated into the provision, as thus enacted, the construction given to it by the American courts as well as the Attorney General of the United States and all administrative authorities, charged with the implementation of the naturalization and immigration laws of that country. (Lo Cham v. Ocampo, 77 Phil., 635 [1946]; Laxamana v. Baltazar, 92 Phil., 32 [1952]; Hartley v. Commissioner, 295 U.S. 216, 79 L. ed. 1399, 55 S Ct. 756 [19353; Helvering v. Winmill, 305 U.S. 79, 83 L ed. 52, 59 S Ct. 45 [1938]; Helvering v. R. J. Reynolds Tobacco Co., 306 U.S. 110, 83 L ed. 536, 59 S Ct. 423 [1939]. [p. 32, Memo of Amicus Curiae]). A fairly comprehensive summary of the said construction by the American courts and administrative authorities is contained in United States of America ex rel. Dora Sejnensky v. Robert E. Tod, Commissioner of Immigration, Appt., 295 Fed. 523, decided November 14, 1922, 26 A. L. R. 1316 as follows: Section 1994 of the Revised Statutes (Comp. Stat. 3948, 2 Fed. Sta. Anno. 2d ed. p. 117) provides as follows: "Any woman who is now or may hereafter be married to a citizen of the United States, and who might herself be lawfully naturalized, shall be deemed a citizen." Section 1944 of the Revised Stat. is said to originate in the Act of Congress of February 10, 1855 (10 Stat. at L. 604, chap. 71), which in its second section provided "that any woman, who might lawfully be naturalized under the existing laws, married, or who shall be married to a citizen of the United States, shall be deemed and taken to be a citizen." And the American Statute of 1855 is substantially a copy of the earlier British Statute 7 & 8 Vict. chap. 66, s 16, 1844, which provided that "any woman married, or who shall be married, to a natural-born subject or person naturalized, shall be deemed and taken to be herself naturalized, and have all the rights and privileges of a natural born subject." The Act of Congress of September 22, 1922 (42 Stat. at L. 1021, chap. 411, Comp. Stat. 4358b, Fed. Stat. Anno. Supp. 1922, p. 255), being "An Act Relative to the Naturalization and Citizenship of Married Women," in 2, provides "that any woman who marries a citizen of the United States after the passage of this Act, ... shall not become a citizen of the United States by reason of such marriage ..." Section 6 of the act also provides "that 1994 of the Revised Statutes ... are repealed." Section 6 also provides that `such repeal shall not terminate citizenship acquired or retained under either of such sections, ..." meaning 2 and 6. So that this Act of September 22, 1922, has no application to the facts of the present case, as the marriage of the relator took place prior to its passage. This case, therefore, depends upon the meaning to be attached to 1994 of the Revised Statutes. In 1868 the Supreme Court, in Kelly v. Owen, 7 Wall. 496, 498, 19 L. ed. 283, 284, construed this provision as found in the Act of 1855 as follows: "The term, "who might lawfully be naturalized under the existing laws," only limits the application of the law to free white women. The previous Naturalization Act, existing at the time, only required that the person applying for its benefits should be "a free white person," and not an alien enemy." This construction limited the effect of the statute to those aliens who belonged to the class or race which might be lawfully naturalized, and did not refer to any of the other provisions of the naturalization laws as to residence or moral character, or to any of the provisions of the immigration laws relating to the exclusion or deportation of aliens. In 1880, in Leonard v. Grant (C. C.) 5 Fed. 11, District Judge Deady also construed the Act of 1855, declaring that "any woman who is now or may hereafter be married to a citizen of the United States, and might herself be lawfully naturalized, shall be deemed a citizen." He held that "upon the authorities, and the reason, if not the necessity, of the case," the statute must be construed as in effect declaring that an alien woman, who is of the class or race that may be lawfully naturalized under the existing laws, and who marries a citizen of the United States, is such a citizen also, and it was not necessary that it should appear affirmatively that she possessed the other qualifications at the time of her marriage to entitle her to naturalization. In 1882, the Act of 1855 came before Mr. Justice Harlan, sitting in the circuit court, in United States v. Kellar, 13 Fed. 82. An alien woman, a subject of Prussia came to the United States and married here a naturalized citizen. Mr. Justice Harlan, with the concurrence of Judge Treat, held that upon her marriage she became ipso facto a citizen of the United States as fully as if she had complied with all of the provisions of the statutes upon the subject of naturalization. He added: "There can be no doubt of this, in view of the decision of the Supreme Court of the United, States in Kelly v. Owen, 7 Wall. 496, 19 L. ed. 283." The alien "belonged to the class of persons" who might be lawfully naturalized. In 1904, in Hopkins v. Fachant, 65 C. C. A. 1, 130 Fed. 839, an alien woman came to the United States from France and entered the country contrary to the immigration laws. The immigration authorities took her into custody at the port of New York, with the view of deporting her. She applied for her release under a writ of habeas corpus, and pending the disposition of the matter she married a naturalized American citizen. The circuit court of appeals for the ninth Circuit held, affirming the court below, that she was entitled to be discharged from custody. The court declared: "The rule is well

settled that her marriage to a naturalized citizen of the United States entitled her to be discharged. The status of the wife follows that of her husband, ... and by virtue of her marriage her husband's domicil became her domicil." . In 1908, the circuit court for the district of Rhode Island in Re Rustigian, 165. Fed. 980, had before it the application of a husband for his final decree of naturalization. It appeared that at that time his wife was held by the immigration authorities at New York on the ground that she was afflicted with a dangerous and contagious disease. Counsel on both sides agreed that the effect of the husband's naturalization would be to confer citizenship upon the wife. In view of that contingency District Judge Brown declined to pass upon the husband's application for naturalization, and thought it best to wait until it was determined whether the wife's disease was curable. He placed his failure to act on the express ground that the effect of naturalizing the husband might naturalize her. At the same time he express his opinion that the husband's naturalization would not effect her naturalization, as she was not one who could become lawfully naturalized. "Her own capacity (to become naturalized)," the court stated "is a prerequisite to her attaining citizenship. If herself lacking in that capacity, the married status cannot confer it upon her." Nothing, however, was actually decided in that case, and the views expressed therein are really nothing more than mere dicta. But, if they can be regarded as something more than that, we find ourselves, with all due respect for the learned judge, unable to accept them. In 1909, in United States ex rel. Nicola v. Williams, 173 Fed. 626, District Judge Learned Hand held that an alien woman, a subject of the Turkish Empire, who married an American citizen while visiting Turkey, and then came to the United States, could not be excluded, although she had, at the time of her entry, a disease which under the immigration laws would have been sufficient ground for her exclusion, if she bad not had the status of a citizen. The case was brought into this court on appeal, and in 1911 was affirmed, in 106 C. C. A. 464, 184 Fed. 322. In that case, however at the time the relators married, they might have been lawfully naturalized, and we said: "Even if we assume the contention of the district attorney to be correct that marriage will not make a citizen of a woman who would be excluded under our immigration laws, it does not affect these relators." We held that, being citizens, they could not be excluded as aliens; and it was also said to be inconsistent with the policy of our law that the husband should be a citizen and the wife an alien. The distinction between that case and the one now before the court is that, in the former case, the marriage took place before any order of exclusion had been made, while in this the marriage was celebrated after such an order was made. But such an order is a mere administrative provision, and has not the force of a judgment of a court, and works no estoppel. The administrative order is based on the circumstances that existed at the time the order of exclusion was made. If the circumstances change prior to the order being carried into effect, it cannot be executed. For example, if an order of exclusion should be based on the ground that the alien was at the time afflicted with a contagious disease, and it should be made satisfactorily to appear, prior to actual deportation, that the alien had entirely recovered from the disease, we think it plain that the order could not be carried into effect. So, in this case, if, after the making of the order of exclusion and while she is permitted temporarily to remain, she in good faith marries an American citizen, we cannot doubt the validity of her marriage, and that she thereby acquired, under international law and under 1994 of the Revised Statutes, American citizenship, and ceased to be an alien. There upon, the immigration authorities lost their jurisdiction over her, as that jurisdiction applies only to aliens, and not to citizens. In 1910, District Judge Dodge, in Ex parte Kaprielian, 188 Fed. 694, sustained the right of the officials to deport a woman under the following circumstances: She entered this country in July, 1910, being an alien and having been born in Turkey. She was taken into custody by the immigration authorities in the following September, and in October a warrant for her deportation was issued. Pending hearings as to the validity of that order, she was paroled in the custody of her counsel. The ground alleged for her deportation was that she was afflicted with a dangerous and contagious disease at the time of her entry. One of the reasons assigned to defeat deportation was that the woman had married a citizen of the United States pending the proceedings for her deportation. Judge Dodge declared himself unable to believe that a marriage under such circumstances "is capable of having the effect claimed, in view of the facts shown." He held that it was no part of the intended policy of 1994 to annul or override the immigration laws, so as to authorize the admission into the country of the wife of a naturalized alien not otherwise entitled to enter, and that an alien woman, who is of a class of persons excluded by law from admission to the United States does not come within the provisions of that section. The court relied wholly upon the dicta contained in the Rustigian Case. No other authorities were cited. In 1914, District Judge Neterer, in Ex parte Grayson, 215 Fed. 449, construed 1994 and held that where, pending proceedings to deport an alien native of France as an alien prostitute, she was married to a citizen of the United States, she thereby became a citizen, and was not subject to deportation until her citizenship was revoked by due process of law. It was his opinion that if, as was contended, her marriage was conceived in fraud, and was entered into for the purpose of evading the immigration laws and preventing her deportation, such fact should be established in a court of competent jurisdiction in an action commenced for the purpose. The case was appealed and the appeal was dismissed. 134 C. C. A. 666, 219 Fed. 1022. It is interesting also to observe the construction placed upon the language of the statute by the Department of Justice. In 1874, Attorney General Williams, 14 Ops. Atty. Gen. 402, passing upon the Act of February 10, 1855, held that residence within the United States for the period required by the naturalization laws was riot necessary in order to constitute an alien woman a citizen, she having married a citizen of the United States abroad, although she never resided in the United States, she and her husband having continued to reside abroad after the marriage. In 1909, a similar construction was given to the Immigration Act of May 5, 1907, in an opinion rendered by Attorney General Wickersham. It appeared an unmarried woman, twenty-eight years of age and a native of Belgium, arrived in New York and went at once to a town in Nebraska, where she continued to reside. About fifteen months after her arrival she was taken before a United States commissioner by way of instituting proceedings under the Immigration Act (34 Stat. at L. 898, chap. 1134, Comp. Stat. 4242, 3 Fed. Stat. Anno. 2d ed. p. 637) for her deportation, on the ground that she had entered this country for the purpose of prostitution, and had been found an inmate of a house of prostitution and practicing the same within three years after landing. It appeared, however, that after she was taken before the United

States commissioner, but prior to her arrest under a warrant by the Department of Justice, she was lawfully married to a native-born citizen of the United States. The woman professed at the time of her marriage an intention to abandon her previous mode of life and to remove with her husband to his home in Pennsylvania. He knew what her mode of life had been, but professed to believe in her good intentions. The question was raised as to the right to deport her, the claim being advance that by her marriage she bad become an American citizen and therefore could not be deported. The Attorney General ruled against the right to deport her as she had become an American citizen. He held that the words, "who might herself be lawfully naturalized," refer to a class or race who might be lawfully naturalized, and that compliance with the other conditions of the naturalization laws was not required. 27 Ops. Atty. Gen. 507. Before concluding this opinion, we may add that it has not escaped our observation that Congress, in enacting the Immigration Act of 1917, so as to provide, in 19, "that the marriage to an American citizen of a female of the sexually immoral classes ... shall not invest such female with United States citizenship if the marriage of such alien female shall be solemnized after her arrest or after the commission of acts which make her liable to deportation under this act." Two conclusions seem irresistibly to follow from the above change in the law: (1) Congress deemed legislation essential to prevent women of the immoral class avoiding deportation through the device of marrying an American citizen. (2) If Congress intended that the marriage of an American citizen with an alien woman of any other of the excluded classes, either before or after her detention, should not confer upon her American citizenship, thereby entitling her to enter the country, its intention would have been expressed, and 19 would not have been confined solely to women of the immoral class. Indeed, We have examined all the leading American decisions on the subject and We have found no warrant for the proposition that the phrase "who might herself be lawfully naturalized" in Section 1994 of the Revised Statutes was meant solely as a racial bar, even if loose statements in some decisions and other treaties and other writings on the subject would seem to give such impression. The case of Kelley v. Owen, supra, which appears to be the most cited among the first of the decisions 19 simply held: As we construe this Act, it confers the privileges of citizenship upon women married to citizens of the United States, if they are of the class of persons for whose naturalization the previous Acts of Congress provide. The terms "married" or "who shall be married," do not refer in our judgment, to the time when the ceremony of marriage is celebrated, but to a state of marriage. They mean that, whenever a woman, who under previous Acts might be naturalized, is in a state of marriage to a citizen, whether his citizenship existed at the passage of the Act or subsequently, or before or after the marriage, she becomes, by that fact, a citizen also. His citizenship, whenever it exists, confers, under the Act, citizenship upon her. The construction which would restrict the Act to women whose husbands, at the time of marriage, are citizens, would exclude far the greater number, for whose benefit, as we think, the Act was intended. Its object, in our opinion, was to allow her citizenship to follow that of her husband, without the necessity of any application for naturalization on her part; and, if this was the object, there is no reason for the restriction suggested. The terms, "who might lawfully be naturalized under the existing laws," only limit the application of the law to free white women. The previous Naturalization Act, existing at the time only required that the person applying for its benefits should be "a free white person," and not an alien enemy. Act of April 14th, 1802, 2 Stat. at L. 153. A similar construction was given to the Act by the Court of Appeals of New York, in Burton v. Burton, 40 N. Y. 373; and is the one which gives the widest extension to its provisions. Note that write the court did say that "the terms, "who might lawfully be naturalized under existing laws" only limit the application to free white women" 20 it hastened to add that "the previous Naturalization Act, existing at the time, ... required that the person applying for its benefits should be (not only) a "free white person" (but also) ... not an alien enemy." This is simply because under the Naturalization Law of the United States at the time the case was decided, the disqualification of enemy aliens had already been removed by the Act of July 30, 1813, as may be seen in the corresponding footnote hereof anon. In other words, if in the case of Kelly v. Owen only the race requirement was mentioned, the reason was that there was no other non-racial requirement or no more alien enemy disqualification at the time; and this is demonstrated by the fact that the court took care to make it clear that under the previous naturalization law, there was also such requirement in addition to race. This is impotent, since as stated in re Rustigian, 165 Fed. Rep. 980, "The expression used by Mr. Justice Field, (in Kelly v. Owen) the terms "who might lawfully be naturalized under existing laws" only limit the application of the law to free white women, must be interpreted in the application to the special facts and to the incapacities under the then existing laws," (at p. 982) meaning that whether or not an alien wife marrying a citizen would be a citizen was dependent, not only on her race and nothing more necessarily, but on whether or not there were other disqualifications under the law in force at the time of her marriage or the naturalization of her husband. 4. As already stated, in Lo San Tuang, Choy King Tee and the second Ly Giok Ha, the Court drew the evidence that because Section 1 of Act 2927 was eliminated by Commonwealth Act 473, it follows that in place of the said eliminated section particularly its subdivision (c), being the criterion of whether or not an alien wife "may be lawfully naturalized," what should be required is not only that she must not be disqualified under Section 4 but that she must also possess the qualifications enumerated in Section 2, such as those of age, residence, good moral character, adherence to the underlying principles of the Philippine Constitution, irreproachable conduct, lucrative employment or ownership of real estate, capacity to speak and write English or Spanish and one of the principal local languages, education of children in certain schools, etc., thereby implying that, in effect, sails Section 2 has been purposely intended to take the place of Section 1 of Act 2927. Upon further consideration of the proper premises, We have come, to the conclusion that such inference is not sufficiently justified. To begin with, nothing extant in the legislative history, which We have already explained above of the mentioned provisions has been shown or can be shown to indicate that such was the clear intent of the legislature. Rather, what is definite is that Section 15 is, an exact copy of Section

1994 of the Revised Statutes of the United States, which, at the time of the approval of Commonwealth Act 473 had already a settled construction by American courts and administrative authorities. Secondly, as may be gleaned from the summary of pertinent American decisions quoted above, there can be no doubt that in the construction of the identically worded provision in the Revised Statutes of the United States, (Section 1994, which was taken, from the Act of February 10, 1855) all authorities in the United States are unanimously agreed that the qualifications of residence, good moral character, adherence to the Constitution, etc. are not supposed to be considered, and that the only eligibility to be taken into account is that of the race or class to which the subject belongs, the conceptual scope of which, We have just discussed. 21 In the very case ofLeonard v. Grant, supra, discussed by Justice Regala in Lo San Tuang, the explanation for such posture of the American authorities was made thus: The phrase, "shall be deemed a citizen" in section 1994 Rev. St., or as it was in the Act of 1855,supra, "shall be deemed and taken to be a citizen" while it may imply that the person to whom it relates has not actually become a citizen by ordinary means or in the usual way, as by the judgment of a competent court, upon a proper application and proof, yet it does not follow that such person is on that account practically any the less a citizen. The word "deemed" is the equivalent of "considered" or "judged"; and, therefore, whatever an act of Congress requires to be "deemed" or "taken" as true of any person or thing, must, in law, be considered as having been duly adjudged or established concerning "such person or thing, and have force and effect accordingly. When, therefore, Congress declares that an alien woman shall, under certain circumstances, be "deemed' an American citizen, the effect when the contingency occurs, is equivalent to her being naturalized directly by an act of Congress, or in the usual mode thereby prescribed. Unless We disregard now the long settled familiar rule of statutory construction that in a situation like this wherein our legislature has copied an American statute word for word, it is understood that the construction already given to such statute before its being copied constitute part of our own law, there seems to be no reason how We can give a different connotation or meaning to the provision in question. At least, We have already seen that the views sustaining the contrary conclusion appear to be based on in accurate factual premises related to the real legislative background of the framing of our naturalization law in its present form. Thirdly, the idea of equating the qualifications enumerated in Section 2 of Commonwealth Act 473 with the eligibility requirements of Section 1 of Act 2927 cannot bear close scrutiny from any point of view. There is no question that Section 2 of Commonwealth Act 473 is more or less substantially the same as Section 3 of Act 2927. In other words, Section 1 of Act 2927 co-existed already with practically the same provision as Section 2 of Commonwealth Act 473. If it were true that the phrase "who may be lawfully naturalized" in Section 13 (a) of Act 2927, as amended by Act 3448, referred to the so-called racial requirement in Section 1 of the same Act, without regard to the provisions of Section 3 thereof, how could the elimination of Section 1 have the effect of shifting the reference to Section 3, when precisely, according to the American jurisprudence, which was prevailing at the time Commonwealth Act 473 was approved, such qualifications as were embodied in said Section 3, which had their counterpart in the corresponding American statutes, are not supposed to be taken into account and that what should be considered only are the requirements similar to those provided for in said Section 1 together with the disqualifications enumerated in Section 4? Fourthly, it is difficult to conceive that the phrase "who might be lawfully naturalized" in Section 15 could have been intended to convey a meaning different than that given to it by the American courts and administrative authorities. As already stated, Act 3448 which contained said phrase and from which it was taken by Commonwealth Act 473, was enacted in 1928. By that, time, Section 1994 of the Revised Statutes of the United States was no longer in force because it had been repealed expressly the Act of September 22, 1922 which did away with the automatic naturalization of alien wives of American citizens and required, instead, that they submit to regular naturalization proceedings, albeit under more liberal terms than those of other applicants. In other words, when our legislature adopted the phrase in question, which, as already demonstrated, had a definite construction in American law, the Americans had already abandoned said phraseology in favor of a categorical compulsion for alien wives to be natural judicially. Simple logic would seem to dictate that, since our lawmakers, at the time of the approval of Act 3448, had two choices, one to adopt the phraseology of Section 1994 with its settled construction and the other to follow the new posture of the Americans of requiring judicial naturalization and it appears that they have opted for the first, We have no alternative but to conclude that our law still follows the old or previous American Law On the subject. Indeed, when Commonwealth Act 473 was approved in 1939, the Philippine Legislature, already autonomous then from the American Congress, had a clearer chance to disregard the old American law and make one of our own, or, at least, follow the trend of the Act of the U.S. Congress of 1922, but still, our legislators chose to maintain the language of the old law. What then is significantly important is not that the legislature maintained said phraseology after Section 1 of Act 2927 was eliminated, but that it continued insisting on using it even after the Americans had amended their law in order to provide for what is now contended to be the construction that should be given to the phrase in question. Stated differently, had our legislature adopted a phrase from an American statute before the American courts had given it a construction which was acquiesced to by those given upon to apply the same, it would be possible for Us to adopt a construction here different from that of the Americans, but as things stand, the fact is that our legislature borrowed the phrase when there was already a settled construction thereof, and what is more, it appears that our legislators even ignored the modification of the American law and persisted in maintaining the old phraseology. Under these circumstances, it would be in defiance of reason and the principles of Statutory construction to say that Section 15 has a nationalistic and selective orientation and that it should be construed independently of the previous American posture because of the difference of circumstances here and in the United States. It is always safe to say that in the construction of a statute, We cannot fall on possible judicial fiat or perspective when the demonstrated legislative point of view seems to indicate otherwise. 5. Viewing the matter from another angle, there is need to emphasize that in reality and in effect, the so called racial requirements, whether under the American laws or the Philippine laws, have hardly been considered as qualifications in the same sense as those enumerated in Section 3 of Act 2927 and later in Section 2 of Commonwealth Act 473. More accurately, they have always been considered as disqualifications, in the sense that those who did not possess them were the ones who could not "be lawfully naturalized," just as if they were suffering from any of the disqualifications under Section 2 of Act 2927 and later those under Section 4 of Commonwealth Act 473, which, incidentally, are practically identical to those in the former law, except those in paragraphs (f) and (h) of the latter. 22 Indeed, such is the clear impression anyone will surely get after going over all the American decisions and opinions quoted and/or cited in the latest USCA (1970), Title 8, section 1430, pp. 598-602, and the first decisions of this Court on the matter, Ly Giok Ha (1959) and Ricardo Cua, citing with approval the opinions of the secretary of Justice. 23 Such being the case, that is, that the so-called racial requirements were always treated as disqualifications in the same light as the other disqualifications under the law, why should their elimination not be viewed or understood as a subtraction from or a lessening of the disqualifications? Why should such elimination have instead the meaning that what were previously considered as irrelevant qualifications have become disqualifications, as seems to be the import of the holding in Choy King Tee to the effect that the retention in Section 15 of Commonwealth Act 473 of the same language of what used to be Section 13 (a) of Act 2927 (as amended by

Act 3448), notwithstanding the elimination of Section 1 of the latter, necessarily indicates that the legislature had in mind making the phrase in question "who may be lawfully naturalized" refer no longer to any racial disqualification but to the qualification under Section 2 of Commonwealth Act 473? Otherwise stated, under Act 2927, there were two groups of persons that could not be naturalized, namely, those falling under Section 1 and those falling under Section 2, and surely, the elimination of one group, i.e. those belonging to Section 1, could not have had, by any process of reasoning, the effect of increasing, rather than decreasing, the disqualifications that used to be before such elimination. We cannot see by what alchemy of logic such elimination could have convicted qualifications into disqualifications specially in the light of the fact that, after all, these are disqualifications clearly set out as such in the law distinctly and separately from qualifications and, as already demonstrated, in American jurisprudence, qualifications had never been considered to be of any relevance in determining "who might be lawfully naturalized," as such phrase is used in the statute governing the status of alien wives of American citizens, and our law on the matter was merely copied verbatim from the American statutes. 6. In addition to these arguments based on the applicable legal provisions and judicial opinions, whether here or in the United States, there are practical considerations that militate towards the same conclusions. As aptly stated in the motion for reconsideration of counsel for petitionerappellee dated February 23, 1967, filed in the case of Zita Ngo Burca v. Republic, supra: Unreasonableness of requiring alien wife to prove "qualifications" There is one practical consideration that strongly militates against a construction that Section 15 of the law requires that an alien wife of a Filipino must affirmatively prove that she possesses the qualifications prescribed under Section 2, before she may be deemed a citizen. Such condition, if imposed upon an alien wife, becomes unreasonably onerous and compliance therewith manifestly difficult. The unreasonableness of such requirement is shown by the following: 1. One of the qualifications required of an Applicant for naturalization under Section 2 of the law is that the applicant "must have resided in the Philippines for a continuous period of not less than ten years." If this requirement is applied to an alien wife married to a Filipino citizen, this means that for a period of ten years at least, she cannot hope to acquire the citizenship of her husband. If the wife happens to be a citizen of a country whose law declares that upon her marriage to a foreigner she automatically loses her citizenship and acquires the citizenship of her husband, this could mean that for a period of ten years at least, she would be stateless. And even after having acquired continuous residence in the Philippines for ten years, there is no guarantee that her petition for naturalization will be granted, in which case she would remain stateless for an indefinite period of time. 2. Section 2 of the law likewise requires of the applicant for naturalization that he "must own real estate in the Philippines worth not less than five thousand pesos, Philippine currency, or must have some known lucrative trade, profession, or lawful occupation." Considering the constitutional prohibition against acquisition by an alien of real estate except in cases of hereditary succession (Art. XIII, Sec. 5, Constitution), an alien wife desiring to acquire the citizenship of her husband must have to prove that she has a lucrative income derived from a lawful trade, profession or occupation. The income requirement has been interpreted to mean that the petitioner herself must be the one to possess the said income. (Uy v. Republic, L-19578, Oct. 27, 1964; Tanpa Ong vs. Republic, L-20605, June 30, 1965; Li Tong Pek v. Republic, L-20912, November 29, 1965). In other words, the wife must prove that she has a lucrative income derived from sources other than her husband's trade, profession or calling. It is of common knowledge, and judicial notice may be taken of the fact that most wives in the Philippines do not have gainful occupations of their own. Indeed, Philippine law, recognizing the dependence of the wife upon the husband, imposes upon the latter the duty of supporting the former. (Art. 291, Civil Code). It should be borne in mind that universally, it is an accepted concept that when a woman marries, her primary duty is to be a wife, mother and housekeeper. If an alien wife is not to be remiss in this duty, how can she hope to acquire a lucrative income of her own to qualify her for citizenship? 3. Under Section 2 of the law, the applicant for naturalization "must have enrolled his minor children of school age, in any of the public schools or private schools recognized by the Office of the Private Education of the Philippines, where Philippine history, government and civics are taught or prescribed as part of the school curriculum during the entire period of residence in the Philippines required of him prior to the hearing of his petition for naturalization as Philippine citizen." If an alien woman has minor children by a previous marriage to another alien before she marries a Filipino, and such minor children had not been enrolled in Philippine schools during her period of residence in the country, she cannot qualify for naturalization under the interpretation of this Court. The reason behind the requirement that children should be enrolled in recognized educational institutions is that they follow the citizenship of their father. (Chan Ho Lay v. Republic, L-5666, March 30, 1954; Tan Hi v. Republic, 88 Phil. 117 [1951]; Hao Lian Chu v. Republic, 87 Phil. 668 [1950]; Yap Chin v. Republic, L-4177, May 29, 1953; Lim Lian Hong v. Republic, L-3575, Dec. 26, 1950). Considering that said minor children by her first husband generally follow the citizenship of their alien father, the basis for such requirement as applied to her does not exist. Cessante ratione legis cessat ipsa lex. 4. Under Section 3 of the law, the 10-year continuous residence prescribed by Section 2 "shall be understood as reduced to five years for any petitioner (who is) married to a Filipino woman." It is absurd that an alien male married to a Filipino wife should be required to reside only for five years in the Philippines to qualify for citizenship, whereas an alien woman married to a Filipino husband must reside for ten years.

Thus under the interpretation given by this Court, it is more difficult for an alien wife related by marriage to a Filipino citizen to become such citizen, than for a foreigner who is not so related. And yet, it seems more than clear that the general purpose of the first paragraph of Section 15 was obviously to accord to an alien woman, by reason of her marriage to a Filipino, a privilege not similarly granted to other aliens. It will be recalled that prior to the enactment of Act No. 3448 in 1928, amending Act No. 2927 (the old Naturalization Law), there was no law granting any special privilege to alien wives of Filipinos. They were treated as any other foreigner. It was precisely to remedy this situation that the Philippine legislature enacted Act No. 3448. On this point, the observation made by the Secretary of Justice in 1941 is enlightening: It is true that under, Article 22 of the (Spanish) Civil Code, the wife follows the nationality of the husband; but the Department of State of the United States on October 31, 1921, ruled that the alien wife of a Filipino citizen is not a Filipino citizen, pointing out that our Supreme Court in the leading case of Roa v. Collector of Customs (23 Phil. 315) held that Articles 17 to 27 of the Civil Code being political have been abrogated upon the cession of the Philippine Islands to the United States. Accordingly, the stated taken by the Attorney-General prior to the envictment of Act No. 3448, was that marriage of alien women to Philippine citizens did not make the former citizens of this counting. (Op. Atty. Gen., March 16, 1928) . To remedy this anomalous condition, Act No. 3448 was enacted in 1928 adding section 13(a) to Act No. 2927 which provides that "any woman who is now or may hereafter be married to a citizen of the Philippine Islands, and who might herself be lawfully naturalized, shall be deemed a citizen of the Philippine Islands. (Op. No. 22, s. 1941; emphasis ours). If Section 15 of the, Revised Naturalization Law were to be interpreted, as this Court did, in such a way as to require that the alien wife must prove the qualifications prescribed in Section 2, the privilege granted to alien wives would become illusory. It is submitted that such a construction, being contrary to the manifested object of the statute must be rejected. A statute is to be construed with reference to its manifest object, and if the language is susceptible of two constructions, one which will carry out and the other defeat such manifest object, it should receive the former construction. (In re National Guard, 71 Vt. 493, 45 A. 1051; Singer v. United States, 323 U.S. 338, 89 L. ed. 285. See also, U.S. v. Navarro, 19 Phil. 134 [1911]; U. S. v. Toribio, 15 Phil. 85 [1910). ... A construction which will cause objectionable results should be avoided and the court will, if possible, place on the statute a construction which will not result in injustice, and in accordance with the decisions construing statutes, a construction which will result in oppression, hardship, or inconveniences will also be avoided, as will a construction which will prejudice public interest, or construction resulting in unreasonableness, as well as a construction which will result in absurd consequences. So a construction should, if possible, be avoided if the result would be an apparent inconsistency in legislative intent, as has been determined by the judicial decisions, or which would result in futility, redundancy, or a conclusion not contemplated by the legislature; and the court should adopt that construction which will be the least likely to produce mischief. Unless plainly shown to have been the intention of the legislature, an interpretation which would render the requirements of the statute uncertain and vague is to be avoided, and the court will not ascribe to the legislature an intent to confer an illusory right. ... (82 C.J.S., Statutes, sec. 326, pp. 623-632). 7. In Choy King Tee and the second Ly Giok Ha, emphasis was laid on the need for aligning the construction of Section 15 with "the national policy of selective admission to Philippine citizenship." But the question may be asked, is it reasonable to suppose that in the pursuit of such policy, the legislature contemplated to make it more difficult if not practically impossible in some instances, for an alien woman marrying a Filipino to become a Filipina than any ordinary applicant for naturalization, as has just been demonstrated above? It seems but natural and logical to assume that Section 15 was intended to extend special treatment to alien women who by marrying a Filipino irrevocably deliver themselves, their possessions, their fate and fortunes and all that marriage implies to a citizen of this country, "for better or for worse." Perhaps there can and will be cases wherein the personal conveniences and benefits arising from Philippine citizenship may motivate such marriage, but must the minority, as such cases are bound to be, serve as the criterion for the construction of law? Moreover, it is not farfetched to believe that in joining a Filipino family the alien woman is somehow disposed to assimilate the customs, beliefs and ideals of Filipinos among whom, after all, she has to live and associate, but surely, no one should expect her to do so even before marriage. Besides, it may be considered that in reality the extension of citizenship to her is made by the law not so much for her sake as for the husband. Indeed, We find the following observations anent the national policy rationalization in Choy King Tee and Ly Giok Ha (the second) to be quite persuasive: We respectfully suggest that this articulation of the national policy begs the question. The avowed policy of "selectives admission" more particularly refers to a case where citizenship is sought to be acquired in a judicial proceeding for naturalization. In such a case, the courts should no doubt apply the national policy of selecting only those who are worthy to become citizens. There is here a choice between accepting or rejecting the application for citizenship. But this policy finds no application in cases where citizenship is conferred by operation of law. In such cases, the courts have no choice to accept or reject. If the individual claiming citizenship by operation of law proves in legal proceedings that he satisfies the statutory requirements, the courts cannot do otherwise than to declare that he is a citizen of the Philippines. Thus, an individual who is able to prove that his father is a Philippine citizen, is a citizen of the Philippines, "irrespective of his moral character, ideological beliefs, and identification with Filipino ideals, customs, and traditions." A minor child of a person naturalized under the law, who is able to prove the fact of his birth in the Philippines, is likewise a citizen, regardless of whether he has lucrative income, or he adheres to the principles of the Constitution. So it is with an alien

wife of a Philippine citizen. She is required to prove only that she may herself be lawfully naturalized, i.e., that she is not one of the disqualified persons enumerated in Section 4 of the law, in order to establish her citizenship status as a fact. A paramount policy consideration of graver import should not be overlooked in this regard, for it explains and justifies the obviously deliberate choice of words. It is universally accepted that a State, in extending the privilege of citizenship to an alien wife of one of its citizens could have had no other objective than to maintain a unity of allegiance among the members of the family. (Nelson v. Nelson, 113 Neb. 453, 203 N. W. 640 [1925]; see also "Convention on the Nationality of Married Women: Historical Background and Commentary." UNITED NATIONS, Department of Economic and Social Affairs E/CN, 6/399, pp. 8 et seq.). Such objective can only be satisfactorily achieved by allowing the wife to acquire citizenship derivatively through the husband. This is particularly true in the Philippines where tradition and law has placed the husband as head of the family, whose personal status and decisions govern the life of the family group. Corollary to this, our laws look with favor on the unity and solidarity of the family (Art. 220, Civil Code), in whose preservation of State as a vital and enduring interest. (See Art. 216, Civil Code). Thus, it has been said that by tradition in our country, there is a theoretic identity of person and interest between husband and wife, and from the nature of the relation, the home of one is that of the other. (See De la Via v. Villareal, 41 Phil. 13). It should likewise be said that because of the theoretic identity of husband and wife, and the primacy of the husband, the nationality of husband should be the nationality of the wife, and the laws upon one should be the law upon the other. For as the court, in Hopkins v. Fachant (9th Cir., 1904) 65 C.C.A., 1, 130 Fed. 839, held: "The status of the wife follows that of the husband, ... and by virtue of her marriage her husband's domicile became her domicile." And the presumption under Philippine law being that the property relations of husband and wife are under the regime of conjugal partnership (Art. 119, Civil Code), the income of one is also that of the other. It is, therefore, not congruent with our cherished traditions of family unity and identity that a husband should be a citizen and the wife an alien, and that the national treatment of one should be different from that of the other. Thus, it cannot be that the husband's interests in property and business activities reserved by law to citizens should not form part of the conjugal partnership and be denied to the wife, nor that she herself cannot, through her own efforts but for the benefit of the partnership, acquire such interests. Only in rare instances should the identity of husband and wife be refused recognition, and we submit that in respect of our citizenship laws, it should only be in the instances where the wife suffers from the disqualifications stated in Section 4 of the Revised Naturalization Law. (Motion for Reconsideration, Burca vs. Republic, supra.) With all these considerations in mind, We are persuaded that it is in the best interest of all concerned that Section 15 of the Naturalization Law be given effect in the same way as it was understood and construed when the phrase "who may be lawfully naturalized," found in the American statute from which it was borrowed and copied verbatim, was applied by the American courts and administrative authorities. There is merit, of course in the view that Philippine statutes should be construed in the light of Philippine circumstances, and with particular reference to our naturalization laws. We should realize the disparity in the circumstances between the United States, as the so-called "melting pot" of peoples from all over the world, and the Philippines as a developing country whose Constitution is nationalistic almost in the come. Certainly, the writer of this opinion cannot be the last in rather passionately insisting that our jurisprudence should speak our own concepts and resort to American authorities, to be sure, entitled to admiration, and respect, should not be regarded as source of pride and indisputable authority. Still, We cannot close our eyes to the undeniable fact that the provision of law now under scrutiny has no local origin and orientation; it is purely American, factually taken bodily from American law when the Philippines was under the dominating influence of statutes of the United States Congress. It is indeed a sad commentary on the work of our own legislature of the late 1920's and 1930's that given the opportunity to break away from the old American pattern, it took no step in that direction. Indeed, even after America made it patently clear in the Act of Congress of September 22, 1922 that alien women marrying Americans cannot be citizens of the United States without undergoing naturalization proceedings, our legislators still chose to adopt the previous American law of August 10, 1855 as embodied later in Section 1994 of the Revised Statutes of 1874, Which, it is worth reiterating, was consistently and uniformly understood as conferring American citizenship to alien women marrying Americansipso facto, without having to submit to any naturalization proceeding and without having to prove that they possess the special qualifications of residence, moral character, adherence to American ideals and American constitution, provided they show they did not suffer from any of the disqualifications enumerated in the American Naturalization Law. Accordingly, We now hold, all previous decisions of this Court indicating otherwise notwithstanding, that under Section 15 of Commonwealth Act 473, an alien woman marrying a Filipino, native born or naturalized, becomes ipso facto a Filipina provided she is not disqualified to be a citizen of the Philippines under Section 4 of the same law. Likewise, an alien woman married to an alien who is subsequently naturalized here follows the Philippine citizenship of her husband the moment he takes his oath as Filipino citizen, provided that she does not suffer from any of the disqualifications under said Section 4. As under any other law rich in benefits for those coming under it, doubtless there will be instances where unscrupulous persons will attempt to take advantage of this provision of law by entering into fake and fictitious marriages or mala fide matrimonies. We cannot as a matter of law hold that just because of these possibilities, the construction of the provision should be otherwise than as dictated inexorably by more ponderous relevant considerations, legal, juridical and practical. There can always be means of discovering such undesirable practice and every case can be dealt with accordingly as it arises. III. The third aspect of this case requires necessarily a re-examination of the ruling of this Court in Burca, supra, regarding the need of judicial naturalization proceedings before the alien wife of a Filipino may herself be considered or deemed a Filipino. If this case which, as already noted, was submitted for decision in 1964 yet, had only been decided earlier, before Go Im Ty, the foregoing discussions would have been sufficient to dispose of it. The Court could have held that despite her apparent lack of qualifications, her marriage to her co-petitioner made her a Filipina, without her undergoing any naturalization proceedings, provided she could sustain, her claim that she is not disqualified under Section 4 of the law. But as things stand now, with the Burca ruling, the question We have still to decide is, may she be deemed a Filipina without submitting to a naturalization proceeding? Naturally, if Burca is to be followed, it is clear that the answer to this question must necessarily be in the affirmative. As already stated, however, the decision in Burca has not yet become final because there is still pending with Us a motion for its reconsideration which vigorously submits grounds worthy of serious consideration by this Court. On this account, and for the reasons expounded earlier in this opinion, this case is as good an occasion as any other to re-examine the issue.

In the said decision, Justice Sanchez held for the Court: We accordingly rule that: (1) An alien woman married to a Filipino who desires to be a citizen of this country must apply therefore by filing a petition for citizenship reciting that she possesses all the qualifications set forth in Section 2 and none of the disqualifications under Section 4, both of the Revised Naturalization Law; (2) Said petition must be filed in the Court of First Instance where petitioner has resided at least one year immediately preceding the filing of the petition; and (3) Any action by any other office, agency, board or official, administrative or otherwise other than the judgment of a competent court of justice certifying or declaring that an alien wife of the Filipino citizen is also a Filipino citizen, is hereby declared null and void. 3. We treat the present petition as one for naturalization. Or, in the words of law, a "petition for citizenship". This is as it should be. Because a reading of the petition will reveal at once that efforts were made to set forth therein, and to prove afterwards, compliance with Sections 2 and 4 of the Revised Naturalization law. The trial court itself apparently considered the petition as one for naturalization, and, in fact, declared petitioner "a citizen of the Philippines." In other words, under this holding, in order for an alien woman marrying a Filipino to be vested with Filipino citizenship, it is not enough that she possesses the qualifications prescribed by Section 2 of the law and none of the disqualifications enumerated in its Section 4. Over and above all these, she has to pass thru the whole process of judicial naturalization apparently from declaration of intention to oathtaking, before she can become a Filipina. In plain words, her marriage to a Filipino is absolutely of no consequence to her nationality vis-a-vis that of her Filipino husband; she remains to be the national of the country to which she owed allegiance before her marriage, and if she desires to be of one nationality with her husband, she has to wait for the same time that any other applicant for naturalization needs to complete, the required period of ten year residence, gain the knowledge of English or Spanish and one of the principle local languages, make her children study in Filipino schools, acquire real property or engage in some lawful occupation of her own independently of her husband, file her declaration of intention and after one year her application for naturalization, with the affidavits of two credible witnesses of her good moral character and other qualifications, etc., etc., until a decision is ordered in her favor, after which, she has to undergo the two years of probation, and only then, but not before she takes her oath as citizen, will she begin to be considered and deemed to be a citizen of the Philippines. Briefly, she can become a Filipino citizen only by judicial declaration. Such being the import of the Court's ruling, and it being quite obvious, on the other hand, upon a cursory reading of the provision, in question, that the law intends by it to spell out what is the "effect of naturalization on (the) wife and children" of an alien, as plainly indicated by its title, and inasmuch as the language of the provision itself clearly conveys the thought that some effect beneficial to the wife is intended by it, rather than that she is not in any manner to be benefited thereby, it behooves Us to take a second hard look at the ruling, if only to see whether or not the Court might have overlooked any relevant consideration warranting a conclusion different from that complained therein. It is undeniable that the issue before Us is of grave importance, considering its consequences upon tens of thousands of persons affected by the ruling therein made by the Court, and surely, it is for Us to avoid, whenever possible, that Our decision in any case should produce any adverse effect upon them not contemplated either by the law or by the national policy it seeks to endorse. AMICI CURIAE in the Burca case, respectable and impressive by their number and standing in the Bar and well known for their reputation for intellectual integrity, legal acumen and incisive and comprehensive resourcefulness in research, truly evident in the quality of the memorandum they have submitted in said case, invite Our attention to the impact of the decision therein thus: The doctrine announced by this Honorable Court for the first time in the present case -- that an alien woman who marries a Philippine citizen not only does not ipso facto herself become a citizen but can acquire such citizenship only through ordinary naturalization proceedings under the Revised Naturalization Law, and that all administrative actions "certifying or declaring such woman to be a Philippine citizen are null and void" has consequences that reach far beyond the confines of the present case. Considerably more people are affected, and affected deeply, than simply Mrs. Zita N. Burca. The newspapers report that as many as 15 thousand women married to Philippine citizens are affected by this decision of the Court. These are women of many and diverse nationalities, including Chinese, Spanish, British, American, Columbian, Finnish, Japanese, Chilean, and so on. These members of the community, some of whom have been married to citizens for two or three decades, have all exercised rights and privileges reserved by law to Philippine citizens. They will have acquired, separately or in conjugal partnership with their citizen husbands, real property, and they will have sold and transferred such property. Many of these women may be in professions membership in which is limited to citizens. Others are doubtless stockholders or officers or employees in companies engaged in business activities for which a certain percentage of Filipino equity content is prescribed by law. All these married women are now faced with possible divestment of personal status and of rights acquired and privileges exercised in reliance, in complete good faith, upon a reading of the law that has been accepted as correct for more than two decades by the very agencies of government charged with the administration of that law. We must respectfully suggest that judicial doctrines which would visit such comprehensive and far-reaching injury upon the wives and mothers of Philippine citizens deserve intensive scrutiny and reexamination. To be sure, this appeal can be no less than what this Court attended to in Gan Tsitung vs. Republic, G.R. No. L-20819, Feb. 21, 1967, 19 SCRA 401 when Chief Justice Concepcion observed: The Court realizes, however, that the rulings in the Barretto and Delgado cases although referring to situations the equities of which are not identical to those obtaining in the case at bar may have contributed materially to the irregularities committed therein and in other analogous cases, and induced the parties concerned to believe, although erroneously, that the procedure followed was valid under the law. Accordingly, and in view of the implications of the issue under consideration, the Solicitor General was required, not only, to comment thereon, but, also, to state "how many cases there are, like the one at bar, in which certificates of naturalization have been issued after notice of the filing of the petition for naturalization had been published in the Official Gazette only once, within the periods (a) from January 28, 1950" (when the decision in Delgado v. Republic was

promulgated) "to May 29, 1957" (when the Ong Son Cui was decided) "and (b) from May 29, 1957 to November 29, 1965" (when the decision in the present case was rendered). After mature deliberation, and in the light of the reasons adduced in appellant's motion for reconsideration and in the reply thereto of the Government, as well as of the data contained in the latter, the Court holds that the doctrine laid down in the Ong Son Cui case shall apply and affect the validity of certificates of naturalization issued after, not on or before May 29, 1957. Here We are met again by the same problem. In Gan Tsitung, the Court had to expressly enjoin the prospective application of its construction of the law made in a previous decision, 24 which had already become final, to serve the ends of justice and equity. In the case at bar, We do not have to go that far. As already observed, the decision in Burca still under reconsideration, while the ruling in Lee Suan Ay, Lo San Tuang, Choy King Tee and others that followed them have at the most become the law of the case only for the parties thereto. If there are good grounds therefor, all We have to do now is to reexamine the said rulings and clarify or modify them. For ready reference, We requote Section 15: Sec. 15. Effect of the naturalization on wife and children. Any woman who is now or may hereafter be married to a citizen of the Philippines, and who might herself be lawfully naturalized shall be deemed a citizen of the Philippines. Minor children of persons naturalized under this law who have been born in the Philippines shall be considered citizens thereof. A foreign-born minor child, if dwelling in the Philippines at the time of naturalization of the parents, shall automatically become a Philippine citizen, and a foreign-born minor child, who is not in the Philippines at the time the parent is naturalized, shall be deemed a Philippine citizen only during his minority, unless he begins to reside permanently in the Philippines when still a minor, in which case, he will continue to be a Philippine citizen even after becoming of age. A child born outside of the Philippines after the naturalization of his parent, shall be considered a Philippine citizen, unless within one year after reaching the age of majority, he fails to register himself as a Philippine citizen at the American Consulate of the country where he resides, and to take the necessary oath of allegiance. It is obvious that the main subject-matter and purpose of the statute, the Revised Naturalization Law or Commonwealth Act 473, as a whole, is to establish a complete procedure for the judicial conferment of the status of citizenship upon qualified aliens. After laying out such a procedure, remarkable for its elaborate and careful inclusion of all safeguards against the possibility of any undesirable persons becoming a part of our citizenry, it carefully but categorically states the consequence of the naturalization of an alien undergoing such procedure it prescribes upon the members of his immediate family, his wife and children, 25 and, to that end, in no uncertain terms it ordains that: (a) all his minor children who have been born in the Philippines shall be "considered citizens" also; (b) all such minor children, if born outside the Philippines but dwelling here at the time of such naturalization "shall automatically become" Filipinos also, but those not born in the Philippines and not in the Philippines at the time of such naturalization, are also redeemed citizens of this country provided that they shall lose said status if they transfer their permanent residence to a foreign country before becoming of age; (c) all such minor children, if born outside of the Philippines after such naturalization, shall also be "considered" Filipino citizens, unless they expatriate themselves by failing to register as Filipinos at the Philippine (American) Consulate of the country where they reside and take the necessary oath of allegiance; and (d) as to the wife, she "shall be deemed a citizen of the Philippines" if she is one "who might herself be lawfully naturalized". 26 No doubt whatever is entertained, so Burca holds very correctly, as to the point that the minor children, falling within the conditions of place and time of birth and residence prescribed in the provision, are vested with Philippine citizenship directly by legislative fiat or by force of the law itself and without the need for any judicial proceeding or declaration. (At p. 192, 19 SCRA). Indeed, the language of the provision, is not susceptible of any other interpretation. But it is claimed that the same expression "shall be deemed a citizen of the Philippines" in reference to the wife, does not necessarily connote the vesting of citizenship status upon her by legislative fiat because the antecedent phrase requiring that she must be one "who might herself be lawfully naturalized" implies that such status is intended to attach only after she has undergone the whole process of judicial naturalization required of any person desiring to become a Filipino. Stated otherwise, the ruling in Burca is that while Section 15 envisages and intends legislative naturalization as to the minor children, the same section deliberately treats the wife differently and leaves her out for the ordinary judicial naturalization. Of course, it goes without saying that it is perfectly within the constitutional authority of the Congress of the Philippines to confer or vest citizenship status by legislative fiat. (U.S. v. Wong Kim Ark, 169 U.S. 649, 42 L ed. 890 [1898]; See, 1 Taada & Carreon, Political Law of the Philippines 152 [1961 ed.]) In fact, it has done so for particular individuals, like two foreign religious prelates, 27 hence there is no reason it cannot do it for classes or groups of persons under general conditions applicable to all of the members of such class or group, like women who marry Filipinos, whether native-born or naturalized. The issue before Us in this case is whether or not the legislature hag done so in the disputed provisions of Section 15 of the Naturalization Law. And Dr. Vicente G. Sinco, one of the most respect authorities on political law in the Philippines 28 observes in this connection thus: "A special form of naturalization is often observed by some states with respect to women. Thus in the Philippines a foreign woman married to a Filipino citizen becomes ipso facto naturalized, if she belongs to any of the classeswho may apply for naturalization under the Philippine Laws." (Sinco, Phil. Political Law 498-499 [10th ed. 1954]; emphasis ours; this comment is substantially reiterated in the 1962 edition, citing Ly Giok Ha and Ricardo Cua,supra.) More importantly, it may be stated, at this juncture, that in construing the provision of the United States statutes from which our law has been copied, 28a the American courts have held that the alien wife does not acquire American citizenship by choice but by operation of law. "In the Revised Statutes the words "and taken" are omitted. The effect of this statute is that every alien woman who marries a citizen of the United States becomes perforce a citizen herself, without the formality of naturalization, and regardless of her wish in that respect." (USCA 8, p. 601 [1970 ed.], citing Mackenzie v. Hare, 1913, 134 P. 713, 165 Cal. 766, affirmed 36 S. Ct. 106, 239 U.S. 299, 60 L ed. 297.) .

We need not recount here again how this provision in question was first enacted as paragraph (a) of Section 13, by way of an insertion into Act 2927 by Act 3448 of November 30, 1928, and that, in turn, and paragraph was copied verbatim from Section 1994 of the Revised Statutes of the United States, which by that time already had a long accepted construction among the courts and administrative authorities in that country holding that under such provision an alien woman who married a citizen became, upon such marriage, likewise a citizen by force of law and as a consequence of the marriage itself without having to undergo any naturalization proceedings, provided that, it could be shown that at the time of such marriage, she was not disqualified to be naturalized under the laws then in force. To repeat the discussion We already made of these undeniable facts would unnecessarily make this decision doubly extensive. The only point which might be reiterated for emphasis at this juncture is that whereas in the United States, the American Congress, recognizing the construction, of Section 1994 of the Revised Statutes to be as stated above, and finding it desirable to avoid the effects of such construction, approved the Act of September 22, 1922 Explicitly requiring all such alien wives to submit to judicial naturalization albeit under more liberal terms than those for other applicants for citizenship, on the other hand, the Philippine Legislature, instead of following suit and adopting such a requirement, enacted Act 3448 on November 30, 1928 which copied verbatim the aforementioned Section 1994 of the Revised Statutes, thereby indicating its preference to adopt the latter law and its settled construction rather than the reform introduced by the Act of 1922. Obviously, these considerations leave Us no choice. Much as this Court may feel that as the United States herself has evidently found it to be an improvement of her national policy vis-a-vis the alien wives of her citizens to discontinue their automatic incorporation into the body of her citizenry without passing through the judicial scrutiny of a naturalization proceeding, as it used to be before 1922, it seems but proper, without evidencing any bit of colonial mentality, that as a developing country, the Philippines adopt a similar policy, unfortunately, the manner in which our own legislature has enacted our laws on the subject, as recounted above, provides no basis for Us to construe said law along the line of the 1922 modification of the American Law. For Us to do so would be to indulge in judicial legislation which it is not institutionally permissible for this Court to do. Worse, this court would be going precisely against the grain of the implicit Legislative intent. There is at least one decision of this Court before Burca wherein it seems it is quite clearly implied that this Court is of the view that under Section 16 of the Naturalization Law, the widow and children of an applicant for naturalization who dies during the proceedings do not have to submit themselves to another naturalization proceeding in order to avail of the benefits of the proceedings involving the husband. Section 16 provides: . SEC. 16. Right of widow and children of petitioners who have died. In case a petitioner should die before the final decision has been rendered, his widow and minor children may continue the proceedings. The decision rendered in the case shall, so far as the widow and minor children are concerned, produce the same legal effect as if it had been rendered during the life of the petitioner. In Tan Lin v. Republic, G.R. No. L-13706, May 31, 1961, 2 SCRA 383, this Court held: Invoking the above provisions in their favor, petitioners-appellants argue (1) that under said Sec. 16, the widow and minor children are allowed to continue the same proceedings and are not substituted for the original petitioner; (2) that the qualifications of the original petitioner remain to be in issue and not those of the widow and minor children, and (3) that said Section 16 applies whether the petitioner dies before or after final decision is rendered, but before the judgment becomes executory. There is force in the first and second arguments. Even the second sentence of said Section 16 contemplate the fact that the qualifications of the original petitioner remains the subject of inquiry, for the simple reason that it states that "The decision rendered in the case shall, so far as the widow and minor children are concerned, produce the same legal effect as if it had been rendered during the life of the petitioner." This phraseology emphasizes the intent of the law to continue the proceedings with the deceased as the theoretical petitioner, for if it were otherwise, it would have been unnecessary to consider the decision rendered, as far as it affected the widow and the minor children. xxx xxx xxx The Chua Chian case (supra), cited by the appellee, declared that a dead person can not be bound to do things stipulated in the oath of allegiance, because an oath is a personal matter. Therein, the widow prayed that she be allowed to take the oath of allegiance for the deceased. In the case at bar, petitioner Tan Lin merely asked that she be allowed to take the oath of allegiance and the proper certificate of naturalization, once the naturalization proceedings of her deceased husband, shall have been completed, not on behalf of the deceased but on her own behalf and of her children, as recipients of the benefits of his naturalization. In other words, the herein petitioner proposed to take the oath of allegiance, as a citizen of the Philippines, by virtue of the legal provision that "any woman who is now or may hereafter be married to a citizen of the Philippines and who might herself be lawfully naturalized shall be deemed a citizen of the Philippines. Minor children of persons naturalized under this law who have been born in the Philippines shall be considered citizens thereof." (Section 15, Commonwealth Act No. 473). The decision granting citizenship to Lee Pa and the record of the case at bar, do not show that the petitioning widow could not have been lawfully naturalized, at the time Lee Pa filed his petition, apart from the fact that his 9 minor children were all born in the Philippines. (Decision, In the Matter of the Petition of Lee Pa to be admitted a citizen of the Philippines, Civil Case No. 16287, CFI, Manila, Annex A; Record on Appeal, pp. 8-11). The reference to Chua Chian case is, therefore, premature. Section 16, as may be seen, is a parallel provision to Section 15. If the widow of an applicant for naturalization as Filipino, who dies during the proceedings, is not required to go through a naturalization preceeding, in order to be considered as a Filipino citizen hereof, it should follow that the wife of a living Filipino cannot be denied the same privilege. This is plain common sense and there is absolutely no evidence that the Legislature intended to treat them differently. Additionally, We have carefully considered the arguments advanced in the motion for reconsideration in Burca, and We see no reason to disagree with the following views of counsel: .

It is obvious that the provision itself is a legislative declaration of who may be considered citizens of the Philippines. It is a proposition too plain to be disputed that Congress has the power not only to prescribe the mode or manner under which foreigners may acquire citizenship, but also the very power of conferring citizenship by legislative fiat. (U. S. v. Wong Kim Ark, 169 U. S. 649, 42 L. Ed. 890 [1898] ; see 1 Taada and Carreon, Political Law of the Philippines 152 [1961 ed.]) The Constitution itself recognizes as Philippine citizens "Those who are naturalized in accordance with law" (Section 1[5], Article IV, Philippine Constitution). Citizens by naturalization, under this provision, include not only those who are naturalized in accordance with legal proceedings for the acquisition of citizenship, but also those who acquire citizenship by "derivative naturalization" or by operation of law, as, for example, the "naturalization" of an alien wife through the naturalization of her husband, or by marriage of an alien woman to a citizen. (See Taada & Carreon, op. cit. supra, at 152, 172; Velayo, Philippine Citizenship and Naturalization 2 [1965 ed.]; 1 Paras, Civil Code 186 [1967 ed.]; see also 3 Hackworth, Digest of International Law 3). The phrase "shall be deemed a citizen of the Philippines" found in Section 14 of the Revised Naturalization Law clearly manifests an intent to confer citizenship. Construing a similar phrase found in the old U.S. naturalization law (Revised Statutes, 1994), American courts have uniformly taken it to mean that upon her marriage, the alien woman becomes by operation of law a citizen of the United States as fully as if she had complied with all the provisions of the statutes upon the subject of naturalization. (U.S. v. Keller, 13 F. 82; U.S. Opinions of the US Attorney General dated June 4, 1874 [14 Op. 4021, July 20, 1909 [27 Op. 507], December 1, 1910 [28 Op. 508], Jan. 15, 1920 [32 Op. 2091 and Jan. 12, 1923 [23 398]). The phrase "shall be deemed a citizen," in Section 1994 Revised Statute (U.S. Comp. Stat. 1091, 1268) or as it was in the Act of 1855 (10 Stat. at L. 604, Chapt. 71, Sec. 2), "shall be deemed and taken to be a citizens" while it may imply that the person to whom it relates has not actually become a citizen by the ordinary means or in the usual way, as by the judgment of a competent court, upon a proper application and proof, yet it does not follow that such person is on that account practically any the less a citizen. The word "deemed" is the equivalent of "considered" or "judged," and therefore, whatever an Act of Congress requires to be "deemed" or "taken" as true of any person or thing must, in law, be considered as having been duly adjudged or established concerning such person or thing, and have force and effect accordingly. When, therefore, Congress declares that an alien woman shall, under certain circumstances, be "deemed" an American citizen, the effect when the contingency occurs, is equivalent to her being naturalized directly by an Act of Congress or in the usual mode thereby prescribed. (Van Dyne, Citizenship of the United States 239, cited in Velayo, Philippine Citizenship and Naturalization 146-147 [1965 ed.]; emphasis ours). That this was likewise the intent of the Philippine legislature when it enacted the first paragraph of Section 15 of the Revised Naturalization Law is shown by a textual analysis of the entire statutory provision. In its entirety, Section 15 reads: (See supra). The phrases "shall be deemed" "shall be considered," and "shall automatically become" as used in the above provision, are undoubtedly synonymous. The leading idea or purpose of the provision was to confer Philippine citizenship by operation of law upon certain classes of aliens as a legal consequence of their relationship, by blood or by affinity, to persons who are already citizens of the Philippines. Whenever the fact of relationship of the persons enumerated in the provision concurs with the fact of citizenship of the person to whom they are related, the effect is for said persons to become ipso facto citizens of the Philippines. "Ipso facto" as here used does not mean that all alien wives and all minor children of Philippine citizens, from the mere fact of relationship, necessarily become such citizens also. Those who do not meet the statutory requirements do not ipso factobecome citizens; they must apply for naturalization in order to acquire such status. What it does mean, however, is that in respect of those persons enumerated in Section 15, the relationship to a citizen of the Philippines is the operative fact which establishes the acquisition of Philippine citizenship by them. Necessarily, it also determines the point of time at which such citizenship commences. Thus, under the second paragraph of Section 15, a minor child of a Filipino naturalized under the law, who was born in the Philippines, becomes ipso facto a citizen of the Philippines from the time the fact of relationship concurs with the fact of citizenship of his parent, and the time when the child became a citizen does not depend upon the time that he is able to prove that he was born in the Philippines. The child may prove some 25 years after the naturalization of his father that he was born in the Philippines and should, therefore, be "considered" a citizen thereof. It does not mean that he became a Philippine citizen only at that later time. Similarly, an alien woman who married a Philippine citizen may be able to prove only some 25 years after her marriage (perhaps, because it was only 25 years after the marriage that her citizenship status became in question), that she is one who might herself be lawfully naturalized." It is not reasonable to conclude that she acquired Philippine citizenship only after she had proven that she "might herself be lawfully naturalized." It is not reasonable to conclude that she acquired Philippine citizenship only after she had proven that she "might herself be lawfully naturalized." The point that bears emphasis in this regard is that in adopting the very phraseology of the law, the legislature could not have intended that an alien wife should not be deemed a Philippine citizenunless and until she proves that she might herself be lawfully naturalized. Far from it, the law states in plain terms that she shall be deemed a citizen of the Philippines if she is one "who might herself be lawfully naturalized." The proviso that she must be one "who might herself be lawfully naturalized" is not a condition precedent to the vesting or acquisition of citizenship; it is only a condition or a state of fact necessary to establish her citizenship as a factum probandum, i.e., as a fact established and proved in evidence. The word "might," as used in that phrase, precisely replies that at the time of her marriage to a Philippine citizen, the alien woman "had (the) power" to become such a citizen herself under the laws then in force. (Owen v. Kelly, 6 DC 191 [1867], aff'd Kelly v. Owen, 76 US 496, 19 L ed 283 [1869). That she establishes such power long after her marriage does not alter the fact that at her marriage, she became a citizen.

(This Court has held) that "an alien wife of a Filipino citizen may not acquire the status of a citizen of the Philippines unless there is proof that she herself may be lawfully naturalized" (Decision, pp. 3-4). Under this view, the "acquisition" of citizenship by the alien wife depends on her having proven her qualifications for citizenship, that is, she is not a citizen unless and until she proves that she may herself be lawfully naturalized. It is clear from the words of the law that the proviso does not mean that she must first prove that she "might herself be lawfully naturalized" before she shall be deemed (by Congress, not by the courts) a citizen. Even the "uniform" decisions cited by this Court (at fn. 2) to support its holding did not rule that the alien wife becomes a citizen only after she has proven her qualifications for citizenship. What those decisions ruled was that the alien wives in those cases failed to prove their qualifications and therefore they failed to establish their claim to citizenship. Thus in Ly Giok Ha v. Galang, 101 Phil. 459 [l957], the case was remanded to the lower court for determination of whether petitioner, whose claim to citizenship by marriage to a Filipino was disputed by the Government, "might herself be lawfully naturalized," for the purpose of " proving her alleged change of political status from alien to citizen" (at 464). In Cua v. Board, 101 Phil. 521 [1957], the alien wife who was being deported, claimed she was a Philippine citizen by marriage to a Filipino. This Court finding that there was no proof that she was not disqualified under Section 4 of the Revised Naturalization Law, ruled that: "No such evidence appearing on record, the claim of assumption of Philippine citizenship by Tijoe Wu Suan, upon her marriage to petitioner, is untenable." (at 523) It will be observed that in these decisions cited by this Court, the lack of proof that the alien wives "might (themselves) be lawfully naturalized" did not necessarily imply that they did not become, in truth and in fact, citizens upon their marriage to Filipinos. What the decisions merely held was that these wives failed to establish their claim to that status as a proven fact. In all instances where citizenship is conferred by operation of law, the time when citizenship is conferred should not be confused with the time when citizenship status is established as a proven fact. Thus, even a natural-born citizen of the Philippines, whose citizenship status is put in issue in any proceeding would be required to prove, for instance, that his father is a citizen of the Philippines in order to factually establish his claim to citizenship. * His citizenship status commences from the time of birth, although his claim thereto is established as a fact only at a subsequent time. Likewise, an alien woman who might herself be lawfully naturalized becomes a Philippine citizen at the time of her marriage to a Filipino husband, not at the time she is able to establish that status as a proven fact by showing that she might herself be lawfully naturalized. Indeed, there is no difference between a statutory declaration that a person is deemed a citizen of the Philippines provided his father is such citizen from a declaration that an alien woman married to a Filipino citizen of the Philippines provided she might herself be lawfully naturalized. Both become citizens by operation of law; the former becomes a citizen ipso facto upon birth; the later ipso facto upon marriage. It is true that unless and until the alien wife proves that she might herself be lawfully naturalized, it cannot be said that she has established her status as a proven fact. But neither can it be said that on that account, she did not become a citizen of the Philippines. If her citizenship status is not questioned in any legal proceeding, she obviously has no obligation to establish her status as a fact. In such a case, the presumption of law should be that she is what she claims to be. (U.S. v. Roxas, 5 Phil. 375 [1905]; Hilado v. Assad, 51 O.G. 4527 [1955]). There is a presumption that a representation shown to have been made is true. (Aetna Indemnity Co. v. George A. Fuller, Co., 73 A. 738, 74 A. 369, 111 ME. 321). The question that keeps bouncing back as a consequence of the foregoing views is, what substitute is them for naturalization proceedings to enable the alien wife of a Philippine citizen to have the matter of her own citizenship settled and established so that she may not have to be called upon to prove it everytime she has to perform an act or enter in to a transaction or business or exercise a right reserved only to Filipinos? The ready answer to such question is that as the laws of our country, both substantive and procedural, stand today, there is no such procedure, but such paucity is no proof that the citizenship under discussion is not vested as of the date of marriage or the husband's acquisition of citizenship, as the case may be, for the truth is that the same situation objections even as to native-born Filipinos. Everytime the citizenship of a person is material or indispensable in a judicial or administrative case, whatever the corresponding court or administrative authority decides therein as to such citizenship is generally not considered as res adjudicata, hence it has to be threshed out again and again as the occasion may demand. This, as We view it, is the sense in which Justice Dizon referred to "appropriate proceeding" in Brito v. Commissioner, supra. Indeed, only the good sense and judgment of those subsequently inquiring into the matter may make the effort easier or simpler for the persons concerned by relying somehow on the antecedent official findings, even if these are not really binding. It may not be amiss to suggest, however, that in order to have a good starting point and so that the most immediate relevant public records may be kept in order, the following observations in Opinion No. 38, series of 1958, of then Acting Secretary of Justice Jesus G. Barrera, may be considered as the most appropriate initial step by the interested parties: Regarding the steps that should be taken by an alien woman married to a Filipino citizen in order to acquire Philippine citizenship, the procedure followed in the Bureau of Immigration is as follows: The alien woman must file a petition for the cancellation of her alien certificate of registration alleging, among other things, that she is married to a Filipino, citizen and that she is not disqualified from acquiring her husband's citizenship pursuant to section 4 of Commonwealth Act No. 473, as amended. Upon the filing of said petition, which should be accompanied or supported by the joint affidavit of the petitioner and her Filipino husband to the effect that the petitioner does not belong to any of the groups disqualified by the cited section from becoming naturalized Filipino citizen (please see attached CEB Form 1), the Bureau of Immigration conducts an investigation and thereafter promulgates its order or decision granting or denying the petition. Once the Commissioner of Immigration cancels the subject's registration as an alien, there will probably be less difficulty in establishing her Filipino citizenship in any other proceeding, depending naturally on the substance and vigor of the opposition. Before closing, it is perhaps best to clarify that this third issue We have passed upon was not touched by the trial court, but as the point is decisive in this case, the Court prefers that the matter be settled once and for all now. IN VIEW OF ALL THE FOREGOING, the judgment of the Court a quo dismissing appellants' petition for injunction is hereby reversed and the Commissioner of Immigration and/or his authorized representative is permanently enjoined from causing the arrest and deportation and the

confiscation of the bond of appellant Lau Yuen Yeung, who is hereby declared to have become a Filipino citizen from and by virtue of her marriage to her co-appellant Moy Ya Lim Yao alias Edilberto Aguinaldo Lim, a Filipino citizen on January 25, 1962. No costs. Dizon, Castro, Teehankee and Villamor, JJ., concur

Mo Ya Lim Yao vs. Commissioner of Immigration GR L-21289, 4 October 1971 Fact of the case: On 8 February 1961, Lau Yuen Yeung applied for a passport visa to

enter the Philippines as a non-immigrant, for a temporary visitor's visa to enter thePhilippines. She was permitted to come into the Philippines on 13 March 1961. On thedate of her arrival, Asher Y, Cheng filed a bond in the amount of P1,000.00 to undertake,among others, that said Lau Yuen Yeung would actually depart from the Philippines onor before the expiration of her authorized period of stay in this country or within theperiod as in his discretion the Commissioner of Immigration. After repeated extensions,she was allowed to stay in the Philippines up to 13 February 1962. On 25 January 1962,she contracted marriage with Moy Ya Lim Yao alias Edilberto Aguinaldo Lim an allegedFilipino citizen. Because of the contemplated action of the Commissioner of Immigrationto confiscate her bond and order her arrest and immediate deportation, after the expirationof her authorized stay, she brought an action for injunction with preliminary injunction.The Court of First Instance of Manila (Civil Case 49705) denied the prayer forpreliminary injunction. Moya Lim Yao and Lau Yuen Yeung appealed. Issue: Whether Lau Yuen Yeung ipso facto became a Filipino citizen upon her marriage to a Filipino citizen. Held: Under Section 15 of Commonwealth Act 473, an alien woman marrying aFilipino, native born or naturalized, becomes ipso facto a Filipina provided she is notdisqualified to be a citizen of the Philippines under Section 4 of the same law. Likewise,an alien woman married to an alien who is subsequently naturalized here follows thePhilippine citizenship of her husband the moment he takes his oath as Filipino citizen,provided that she does not suffer from any of the disqualifications under said Section 4.Whether the alien woman requires to undergo the naturalization proceedings, Section 15is a parallel provision to Section 16. Thus, if the widow of an applicant for naturalizationas Filipino, who dies during the proceedings, is not required to go through anaturalization proceedings, in order to be considered as a Filipino citizen hereof, it shouldfollow that the wife of a living Filipino cannot be denied the same privilege. Everytimethe citizenship of a person is material or indispensible in a judicial or administrative case,Whatever the corresponding court or administrative authority decides therein as to suchcitizenship is generally not considered as res adjudicata, hence it has to be threshed outagain and again as the occasion may demand. Lau Yuen Yeung, was declared to havebecome a Filipino citizen from and by virtue of her marriage to Moy Ya Lim Yao al asEdilberto Aguinaldo Lim, a Filipino citizen of 25 January 1962. Download

Republic of the Philippines SUPREME COURT Manila EN BANC G.R. No. 83820 May 25, 1990 JOSE B. AZNAR (as Provincial Chairman of PDP Laban in Cebu), petitioner, vs. COMMISSION ON ELECTIONS and EMILIO MARIO RENNER OSMEA, respondents. Rufino B. Requina for petitioner.

Angara, Abello, Concepcion, Regala & Cruz for private respondent.

PARAS, J.: Before Us is a petition for certiorari assailing the Resolution of the Commission on Elections (COMELEC) dated June 11, 1988, which dismissed the petition for the disqualification of private respondent Emilio "Lito" Osmea as candidate for Provincial Governor of Cebu Province. The facts of the case are briefly as follows: On November 19, 1987, private respondent Emilio "Lito" Osmea filed his certificate of candidacy with the COMELEC for the position of Provincial Governor of Cebu Province in the January 18, 1988 local elections. On January 22, 1988, the Cebu PDP-Laban Provincial Council (Cebu-PDP Laban, for short), as represented by petitioner Jose B. Aznar in his capacity as its incumbent Provincial Chairman, filed with the COMELEC a petition for the disqualification of private respondent on the ground that he is allegedly not a Filipino citizen, being a citizen of the United States of America. On January 27, 1988, petitioner filed a Formal Manifestation submitting a Certificate issued by the then Immigration and Deportation Commissioner Miriam Defensor Santiago certifying that private respondent is an American and is a holder of Alien Certificate of Registration (ACR) No. B-21448 and Immigrant Certificate of Residence (ICR) No. 133911, issued at Manila on March 27 and 28, 1958, respectively. (Annex "B-1"). The petitioner also filed a Supplemental Urgent Ex-Parte Motion for the Issuance of a Temporary Restraining Order to temporarily enjoin the Cebu Provincial Board of Canvassers from tabulating/canvassing the votes cast in favor of private respondent and proclaiming him until the final resolution of the main petition. Thus, on January 28, 1988, the COMELEC en banc resolved to order the Board to continue canvassing but to suspend the proclamation. At the hearing before the COMELEC (First Division), the petitioner presented the following exhibits tending to show that private respondent is an American citizen: Application for Alien Registration Form No. 1 of the Bureau of Immigration signed by private respondent dated November 21, 1979 (Exh. "B"); Alien Certificate of Registration No. 015356 in the name of private respondent dated November 21, 1979 (Exh. "C"); Permit to Re-enter the Philippines dated November 21, 1979 (Exh. "D"); Immigration Certificate of Clearance dated January 3, 1980 (Exh. "E"). (pp. 117-118, Rollo) Private respondent, on the other hand, maintained that he is a Filipino citizen, alleging: that he is the legitimate child of Dr. Emilio D. Osmea, a Filipino and son of the late President Sergio Osmea, Sr.; that he is a holder of a valid and subsisting Philippine Passport No. 0855103 issued on March 25, 1987; that he has been continuously residing in the Philippines since birth and has not gone out of the country for more than six months; and that he has been a registered voter in the Philippines since 1965. (pp. 107-108, Rollo) On March 3, 1988, COMELEC (First Division) directed the Board of Canvassers to proclaim the winning candidates. Having obtained the highest number of votes, private respondent was proclaimed the Provincial Governor of Cebu. Thereafter, on June 11, 1988, COMELEC (First Division) dismissed the petition for disqualification for not having been timely filed and for lack of sufficient proof that private respondent is not a Filipino citizen. Hence, the present petition. The petition is not meritorious. There are two instances where a petition questioning the qualifications of a registered candidate to run for the office for which his certificate of candidacy was filed can be raised under the Omnibus Election Code (B.P. Blg. 881), to wit: (1) Before election, pursuant to Section 78 thereof which provides that: 'Section 78. Petition to deny due course or to cancel a certificate of candidacy. A verified petition seeking to deny due course or to cancel a certificate of candidacy may be filed by any person exclusively on the ground that any material representation contained therein as required under Section 74 hereof is false. The petition may be filed at any time not later than twenty-five days from the time of the filing of the certificate of candidacy and shall be decided, after the notice and hearing, not later than fifteen days before the election. and (2) After election, pursuant to Section 253 thereof, viz: 'Sec. 253. Petition for quo warranto. Any voter contesting the election of any Member of the Batasang Pambansa, regional, provincial, or city officer on the ground of ineligibility or of disloyalty to the Republic of the Philippines shall file a sworn petition for quo warranto with the Commission within ten days after the proclamation of the results of the election.

The records show that private respondent filed his certificate of candidacy on November 19, 1987 and that the petitioner filed its petition for disqualification of said private respondent on January 22, 1988. Since the petition for disqualification was filed beyond the twenty five-day period required in Section 78 of the Omnibus Election Code, it is clear that said petition was filed out of time. The petition for the disqualification of private respondent cannot also be treated as a petition for quo warrantounder Section 253 of the same Code as it is unquestionably premature, considering that private respondent was proclaimed Provincial Governor of Cebu only on March 3, 1988. However, We deem it is a matter of public interest to ascertain the respondent's citizenship and qualification to hold the public office to which he has been proclaimed elected. There is enough basis for us to rule directly on the merits of the case, as the COMELEC did below. Petitioner's contention that private respondent is not a Filipino citizen and, therefore, disqualified from running for and being elected to the office of Provincial Governor of Cebu, is not supported by substantial and convincing evidence. In the proceedings before the COMELEC, the petitioner failed to present direct proof that private respondent had lost his Filipino citizenship by any of the modes provided for under C.A. No. 63. Among others, these are: (1) by naturalization in a foreign country; (2) by express renunciation of citizenship; and (3) by subscribing to an oath of allegiance to support the Constitution or laws of a foreign country. From the evidence, it is clear that private respondent Osmea did not lose his Philippine citizenship by any of the three mentioned hereinabove or by any other mode of losing Philippine citizenship. In concluding that private respondent had been naturalized as a citizen of the United States of America, the petitioner merely relied on the fact that private respondent was issued alien certificate of registration and was given clearance and permit to re-enter the Philippines by the Commission on Immigration and Deportation. Petitioner assumed that because of the foregoing, the respondent is an American and "being an American", private respondent "must have taken and sworn to the Oath of Allegiance required by the U.S. Naturalization Laws." (p. 81, Rollo) Philippine courts are only allowed to determine who are Filipino citizens and who are not. Whether or not a person is considered an American under the laws of the United States does not concern Us here. By virtue of his being the son of a Filipino father, the presumption that private respondent is a Filipino remains. It was incumbent upon the petitioner to prove that private respondent had lost his Philippine citizenship. As earlier stated, however, the petitioner failed to positively establish this fact. The cases of Juan Gallanosa Frivaldo v. COMELEC et al, (G.R. No. 87193, June 21, 1989) and Ramon L. Labo v. COMELEC et al (G.R. No. 86564, August 1, 1989) are not applicable to the case at bar. In the Frivaldo case, evidence shows that he was naturalized as a citizen of the United States in 1983 per certification from the United States District Court, Northern District of California, as duly authenticated by Vice Consul Amado P. Cortez of the Philippine Consulate General in San Francisco, California, U.S.A. Frivaldo expressly admitted in his answer that he was naturalized in the United States but claimed that he was forced to embrace American citizenship to protect himself from the persecution of the Marcos government. The Court, however, found this suggestion of involuntariness unacceptable, pointing out that there were many other Filipinos in the United States similarly situated as Frivaldo who did not find it necessary to abandon their status as Filipinos. Likewise, in the case of Labo, records show that Labo was married to an Australian citizen and that he was naturalized as an Australian citizen in 1976, per certification from the Australian Government through its Consul in the Philippines. This was later affirmed by the Department of Foreign Affairs. The authenticity of the above evidence was not disputed by Labo. In fact, in a number of sworn statements, Labo categorically declared that he was a citizen of Australia. In declaring both Frivaldo and Labo not citizens of the Philippines, therefore, disqualified from serving as Governor of the Province of Sorsogon and Mayor of Baguio City, respectively, the Court considered the fact that by their own admissions, they are indubitably aliens, no longer owing any allegiance to the Republic of the Philippines since they have sworn their total allegiance to a foreign state. In the instant case, private respondent vehemently denies having taken the oath of allegiance of the United States (p. 81, Rollo). He is a holder of a valid and subsisting Philippine passport and has continuously participated in the electoral process in this country since 1963 up to the present, both as a voter and as a candidate (pp. 107-108, Rollo). Thus, private respondent remains a Filipino and the loss of his Philippine citizenship cannot be presumed. In the learned dissent of Mr. Justice Teodoro Padilla, he stresses the fact that because Osmea obtained Certificates of Alien Registration as an American citizen, the first in 1958 when he was 24 years old and the second in 1979, he, Osmea should be regarded as having expressly renounced Philippine citizenship. To Our mind, this is a case of non sequitur (It does not follow). Considering the fact that admittedly Osmea was both a Filipino and an American, the mere fact that he has a Certificate stating he is an American does not mean that he is not still a Filipino. Thus, by way of analogy, if a person who has two brothers named Jose and Mario states or certifies that he has a brother named Jose, this does not mean that he does not have a brother named Mario; or if a person is enrolled as student simultaneously in two universities, namely University X and University Y, presents a Certification that he is a student of University X, this does not necessarily mean that he is not still a student of University Y. In the case of Osmea, the Certification that he is an American does not mean that he is not still a Filipino, possessed as he is, of both nationalities or citizenships. Indeed, there is no express renunciation here of Philippine citizenship; truth to tell, there is even no implied renunciation of said citizenship. When We consider that the renunciation needed to lose Philippine citizenship must be

"express", it stands to reason that there can be no such loss of Philippine 'citizenship when there is no renunciation either "'express" or "implied". Parenthetically, the statement in the 1987 Constitution that "dual allegiance of citizens is inimical to the national interest and shall be dealt with by law"(Art. IV, Sec. 5) has no retroactive effect. And while it is true that even before the 1987 Constitution, Our country had already frowned upon the concept of dual citizenship or allegiance, the fact is it actually existed. Be it noted further that under the aforecited proviso, the effect of such dual citizenship or allegiance shall be dealt with by a future law. Said law has not yet been enacted. WHEREFORE, the petition for certiorari is hereby DISMISSED and the Resolution of the COMELEC is hereby AFFIRMED. SO ORDERED. Narvasa, Bidin, Grio-Aquino, Medialdea and Regalado, JJ., concur. Feliciano, J., I concur. I also join in the concurring opinion of Justice Sarmiento. Cortes, J., concur in the result. Fernan, C.J., took no part. Gancayco, J., is on leave.

Separate Opinions

SARMIENTO, J., concurring: The majority seems agreed that the private respondent has acquired American citizenship, only that he did not necessarily lose his Filipino citizenship. The important question, however, inheres in how he obtained American citizenship. I find that there is a dearth of facts here. For, if the private respondent became an American by naturalization, he has lost Filipino citizenship (Com. Act No. 63; Frivaldo v. COMELEC, G.R. No. 87193, June 21, 1989; Labo v. COMELEC, G.R. No. 86564, August 1, 1989). If he, however, became one by the application of the principle of jus soli it is by force of circumstances rather than choice. But he does not lose his Filipino citizenship, if he were otherwise born of Filipino parents. In the absence of evidence, we can not presume that he had ceased to be a citizen of the Philippines, simply because he is, at the same time. a citizen of the United States. There must be a clear showing that he lost his Filipino citizenship by any of the means enumerated by Commonwealth Act No. 63. The fact that he had obtained an alien certificate of registration, standing alone, does not amount to "express renunciation."

MELENCIO-HERRERA, J., dissenting: I join the dissent of Messrs. Justices Isagani A. Cruz and Teodoro R. Padilla. While it may be that dual citizenship usually results from accident of birth, a choice will have to be made by the individual concerned at some point in time in his life, involving as it does the priceless heritage of citizenship. That election was made by private respondent when, in 1958, at the age of 24, and in 1979, at 45, he obtained Alien Certificates of Registration. Registration as an alien is a clear and unambiguous act or declaration that one is not a citizen. If, in fact, private respondent was merely compelled to so register because of the "uncooperativeness" of the past regime, he could have, under the new dispensation, asked for the cancellation of those Alien Certificates and abandoned his alienage, specially before he ran for public office in 1988. The 1987 Constitution declares in no uncertain terms that "dual allegiance of citizens is inimical to the national interest and shall be dealt with by law" (Article IV, Section 5). That statement is but a reaffirmation of an innate conviction shared by every Filipino. The law referred to need not be awaited for one to consider giving up the legal convenience of dual citizenship. Accordingly, I vote to grant the Petition.

CRUZ, J., dissenting: I join Mr. Justice Padilla in his dissent. It seems to me that when a person voluntarily registers as an alien, he is in effect affirming that he is not a citizen. The terms "citizen" and "alien" are mutually exclusive from the viewpoint of municipal law, which is what really matters in the case at bar. Under this discipline, one is either a citizen of the local state or he is not; and the question is resolved on the basis of its own laws alone and not those of any other state. One of the several modes of losing Philippine citizenship under C.A. No. 63 is by "express renunciation" thereof. In the case of Frivaldo v. Commission on Elections, G.R. No. 87193, June 23,1989, there was such renunciation when the petitioner took an oath as a naturalized citizen of the United States in which he renounced all allegiance to all other states. In the case of Labo v. Commission on Elections, G.R. No. 86546, August 1, 1989, the petitioner not only took a similar oath after his naturalization in Australia but also executed other documents in which he stated that he was not a Filipino. The fact that his naturalization was later revoked did not also invalidate his disavowal of Philippine citizenship. "Express renunciation" is a separate mode of losing Philippine citizenship and is not necessarily dependent on "naturalization in a foreign country," which is another and different mode. When a person rejects and divorces his wife to enter into a second marriage, he cannot say he still loves her despite his desertion. The undeniable fact is that he has left her for another woman to whom he has totally and solemnly transferred his troth. It does him no credit when he protests he married a second time simply for material convenience and that his heart still belongs to the wife he has abandoned. At worst, it would reveal his sordid and deceitful character. By the same token, professing continued allegiance to the Philippines after renouncing it because of its meager resources, or for other ulterior and equally base reasons, is to me a paltry form of patriotism. It is a sop to the repudiated state and a slight to the adopted state. No matter how noble this attitude may appear to others, it is to me nothing less than plain and simple hypocrisy that we should not condone, let alone extol. Coming now to the case at bar, I note first of all that no naturalization is involved here as the private respondent claims to be a citizen both of the Philippines and of the United States. The question I think we must answer is: Was there an express renunciation of Philippine citizenship by the private respondent when he knowingly and voluntarily registered as an alien with the Commission of Immigration and Deportation in 1958 and in 1979? In Yu v. Commission of Immigration and Deportation, G.R. No. 83882, January 24, 1989, I made the following observations in a separate opinion: Regretfully, I cannot agree with the finding that the petitioner has expressly renounced his Philippine citizenship. The evidence on this point is in my view rather meager. Express renunciation of citizenship as a made of losing citizenship under Com. Act No. 63 is an unequivocal and deliberate act with full awareness of its significance and consequences. I do not think the "commercial documents he signed" suggest such categorical disclaimer. That case is distinguished from the one before us now in that Yu did not ask the Philippine government to register him as an alien. Gov. Osmea did. It is my opinion that if the governor had confined himself to simply seeking and using an American passport, these acts could not have by themselves alone constituted a repudiation of Philippine citizenship. The problem, though, is that he did more than enjoy this legal convenience. What he actually did was register with the Philippine government as an alien within its own territory, presumably so he could be insulated from the jurisdiction it exercises over its nationals. This was a voluntary act. As a citizen of the Philippines, he was not required to register as an alien. Nevertheless, he chose to do so of his own free will. By this decision, he categorically asked the Republic of the Philippines to treat him as an American and not a Filipino, choosing to be an alien in this land that was willing to consider him its own. C.A. No. 63 does not necessarily require that the express renunciation of Philippine citizenship be made in connection with the naturalization of the erstwhile Filipino in a foreign country. Renunciation may be made independently of naturalization proceedings. Moreover, no sacramental words are prescribed by the statute for the express renunciation of Philippine citizenship. As long as the repudiation is categorical enough and the preference for the foreign state is unmistakable, as in the case at bar, Philippine citizenship is lost. The private respondent would have his cake and eat it too, but this can never be allowed where Philippine citizenship is involved. It is a gift that must be deserved to be retained. The Philippines for all her modest resources compared to those of other states, is a jealous and possessive mother demanding total love and loyalty from her children. It is bad enough that the love of the dual national is shared with another state; what is worse is where he formally rejects the Philippines, and in its own territory at that, and offers his total devotion to the other state. I am aware of the praiseworthy efforts of Gov. Osmea to improve the province of Cebu, and also, I should add, of the commendable record of Gov. Frivaldo and Mayor Labo in the administration of their respective jurisdictions. But that is not the point. The point is that it is not lawful to maintain in public office any person who, although supported by the electorate, is not a Filipino citizen. This is a relentless restriction we cannot ignore. Regretfully, therefore, I must vote to GRANT the petition.

PADILLA, J., dissenting: I am constrained to dissent. I start from the premise that the private respondent Emilio Mario Renner Osmea enjoyed at one time dual citizenship, i.e,, Philippine and U.S. citizenships. He was born in the Philippines of a Filipino father and an American (U.S.) mother. However, his sworn application for alien registration dated 21 November 1979 (Exh. B) filed with the Philippine immigration authorities was, in my view, an express renunciation of his Philippine citizenship. As held in Board of Immigration Commissioners vs. Go Callano 1 express renunciation means a renunciation that is made known distinctly and explicitly and not left to inference or implication. Nothing can be more distinct and explicit than when a dual citizenship holder-like the private respondent of age, and with full legal capacity to act, voluntarily and under oath applies with the Philippine Government for registration as an alien, insofar as his intention not to remain a Filipino citizen is concerned. And because of that distinct and explicit manifestation of desire to be considered an alien in the Philippines, the Philippine immigration authorities issued to private respondent Alien Certificate of Registration No. 015356 dated 21 November 1979 (Exh. C), Permit to Re- enter the Philippines No. 122018 dated 21 November 1979 (Exh. D) and Immigration Certificate of Clearance No. D-146483 dated 3 January 1980 (Exh E) 2 All the foregoing documents issued by the Philippine immigration authorities to the private respondent at his request are predicated on the proposition that private respondent is an alien under Philippine laws. It should also be mentioned that, while not marked as exhibit in the case at bar, private respondent was likewise issued in Cebu City Native Born Certificate of Residence No. 115883 on 21 November 1979 (as verified from Immigration records). This document, copy of which is attached hereto as Annex A, is again predicated on the proposition that private respondent is a duly-registered align (American) residing in the Philippines. Another relevant document that merits attention is the Application for Re-entry Permit executed and signed by private respondent on 3 January 1980, again under oath, and verified from the records at the CID wherein private respondent expressly stated that he is a U.S. national. The importance of this document cannot be underestimated For, if private respondent believed that he is a Filipino citizen, he would not have executed said Application for Re-entry Permit, since it is the right of every Filipino citizen to return to his country (the Philippines). The fact, therefore, that private respondent executed said sworn Application for Re-entry Permit, copy of which is attached hereto as Annex B, is again an abundant proof that he himself, no less, believed that he was, as he continuous to be, a resident alien (American) in the Philippines. It will further be noted that earlier, or in 1958, private respondent had already registered as an alien with the Bureau of Immigration under the Alien Registration Act of 1950 RA 562). Section 1 of said Act provides: SECTION 1. Aliens residing in the Philippines shall, within thirty days after the approval of this Act, apply for registration, in the case of those residing in the City of Manila, at the Bureau of Immigration and in the case of those residing in other localities at the office of the city or municipal treasurers, or at any other office designated by the President. ... . 3 (Emphasis supplied) Accordingly, per certification of the Commissioner of Immigration and Deportation Miriam Defensor Santiago (Exh. A), issued on 26 January 1988, private respondent had been issued ACR No. B-21-448 and ICR No. 13391 on 27 and 28 March 1958 respectively. He, therefore, registered himself in the Philippines as an alien twice; first, in the year 1958, when he was 24 years old and again in 1979, when he was 45 years old. By twice registering under oath as an alien with the Bureau of Immigration, private respondent thereby clearly, distinctly and explicitly manifested and declared that he was an alien (and, therefore, not a Filipino citizen) residing in the Philippines and under its laws. At this point, and to be objectively fair to the private respondent, a clarification should be made. In his Comment on the Petition at bar (Rollo, p. 81), it is stated by his counsel that he (private respondent) was born in 1934 hence, our mathematical conclusion that when he first registered as an alien in 1958, he was 24 years old and in 1979 when he re-registered as an alien, he was 45 years old. However, private respondent's immigration records disclose that he was born in 1938 (not in 1934). On the assumption that the year 1938 is the correct year of birth of private respondent (and that his alleged year of birth, 1934, as stated in his Comment at bar is erroneous), then in 1958, when he first registered as an alien, he was 20 years old, while in 1979 when he re-registered as an alien, he was 41 years old. Still, his first registration as an alien (at age 20) has to be taken, in my view, as an express renunciation of his Philippine citizenship, because (1) at that time, he was almost 21 years old the age of majority, and (2) more importantly, under the applicable Alien Registration Act RA 562), an alien 14 years or over has to register in person (and not through his parents or guardian). It provides: The parent or legal guardian of an alien who is less than fourteen years of age, shall have the duty of registering such alien: Provided, That whenever any such alien attains his fourteenth birthday in the Philippines he shall, within fifteen days thereafter, apply in person for registration. (Sec. 1, par. 2) I take the above provision to mean that the choice by a dual nationality holder on whether to remain a Filipino citizen or an alien has to be made at age 14, and private respondent (although a bit late) made the notice in 1958 (at age 20) in favor of his U.S. citizenship. If all the foregoing acts of express renunciation of Philippine citizenship had been made or filed by private respondent elsewhere (not with the Philippine Government), there could perhaps be some room for contention that vis-a- vis the Philippine Government, private respondent had not renounced his Philippine citizenship. But said acts of express renunciation were filed with the Philippine Government and done right in the Philippines. In turn the Philippine Government, through the immigration authorities, accepted and acted on private respondent's aforesaid representations, and registered and documented him TWICE as an alien under Philippine law. The policy of our laws has been, and with laudable reason, to discourage dual citizenship, because this condition or status assumes as a necessary complement thereof dual allegiance at the same time to two (2) different countries. As early as 16 September 1947, a unanimous

Supreme Court, speaking thru Mr. Justice Sabino Padilla in the celebrated case of Tan Chong vs. Secretary of Labor, rejected the principle of jus soli as determinative of Philippine citizenship, for the following reason, among others: ... . Citizenship, the main integrate element of which is allegiance, must not be taken lightly. Dual allegiance must be discouraged and prevented. But the application of the principle jus soli to persons born in this country of alien parentage would encourage dual allegiance which in the long run would be detrimental to both countries of which such persons might claim to be citizens. 4 This policy found later expression in the 1987 Constitution which now provides Sec. 5. Dual allegiance of citizen is inimical to the national interest and shall be dealt with by law. (Article IV) Dual citizenship, in my considered opinion, must be eschewed. While having the "best of two (2) words" maybe the result of birth or other factors accidentally brought about, the "dual citizen" has to make a choice at one time or another. Having two (2) citizenships is, as I see it, similar in many ways to having two (2) legal spouses, when as a matter of principle and sound public policy, fealty to only one (1) spouse is both compelling and certainly desirable. Gordon and Rosenfield in their book on Immigration Law and Procedure state: Dual nationality is universally recognized as an undesirable phenomenon. It inevitably results in questionable loyalties and leads to international conflicts. Dual nationality also makes possible the use of citizenship as a badge of convenience rather than of undivided loyalty. And it impairs the singleness of commitment which is the hallmark of citizenship and allegiance. A person should have a right to choose his own nationality, and this choice should be honored by all countries. However, he should not be entitled to claim more than one nationality. 5 (Emphasis supplied) Private respondent made a deliberate and decisive choice when he asked the Philippine Government which, like many other countries, considers dual allegiance as against national or public interest to register him at least twice (and, therefore, unmistakably) as an alien in this country. That choice pro tanto was a renunciation of his Philippine citizenship. The choice must be respected as a conscious and knowledgeable act of a discerning, distinguished and respected person who must be presumed to have known the full import of his acts. Finally, the last thing that should be said against the Court is that it is inconsistent in its rulings. In the light of its recent decision in G.R. No. 86565 (Ramon L. Labo, Jr. vs. The Commission on Elections, et al.) I see no valid justification for holding Mr. Labo an alien upper Philippine law while holding private respondent herein a Filipino citizen. For, as the majority states: "In fact, in a number of sworn statements, Labo categorically declared that he was a citizen of Australia" (p. 7, Decision). And is exactly what private respondent did. In a number of sworn statements, he declared that he was a citizen of the United States. To Mr. Labo, the Court said, "so be it, you are an Australian," yet to the private respondent, despite such sworn statements that he is a U.S. citizen, the Court says, "never mind those sworn statements, you are still a Filipino." Sauce for the goose, as the saying goes, is sauce for the gander. The doctrinal basis of the Court's decisions should be built on the merits, not on distinctions that really make no difference. ACCORDINGLY, I vote to GRANT the petition and to declare the private respondent not a Filipino citizen by his own acts of express renunciation of such citizenship.

GUTIERREZ, JR., J., dissenting: My stand in the cases of Willie Yu vs. Miriam Defensor Santiago, et al. (G.R. No, 83882, January 24, 1989) andRamon Labo, Jr, v. Commission on Elections (G.R. 86564, August 2, 1989) is clear. I regret, however, that I cannot participate in this case because one of the principal counsel is my relative by affinity, within the fourth civil degree.

Separate Opinions

SARMIENTO, J., concurring: The majority seems agreed that the private respondent has acquired American citizenship, offly that he did not necessarily lose his Filipino citizenship. The important question, however, inheres in how he obtained American citizenship. I find that there is a dearth of facts here. For, if the private respondent became an American by naturalization, he has lost Filipino citizenship (Com. Act No. 63; Frivaldo v. COMELEC, G.R. No. 87193, June 21, 1989; Labo v. COMELEC, G.R. No. 86564, August 1, 1989). If he, however, became one by the application of the

principle of jus soli it is by force of circumstances rather than choice. But he does not lose his Filipino citizenship, if he were otherwise born of Filipino parents. In the absence of evidence, we can not presume that he had ceased to be a citizen of the Philippines, simply because he is, at the same time. a citizen of the United States. There must be a clear showing that he lost his Filipino citizenship by any of the means enumerated by Commonwealth Act No. 63. The fact that he had obtained an alien certificate of registration, standing alone, does not amount to "express renunciation."

MELENCIO-HERRERA, J., dissenting: I join the dissent of Messrs. Justices Isagani A. Cruz and Teodoro R. Padilla. While it may be that dual citizenship usually results from accident of birth, a choice will have to be made by the individual concerned at some point in time in his life, involving as it does the priceless heritage of citizenship. That election was made by private respondent when, in 1958, at the age of 24, and in 1979, at 45, he obtained Alien Certificates of Registration. Registration as an alien is a clear and unambiguous act or declaration that one is not a citizen. If, in fact, private respondent was merely compelled to so register because of the "uncooperativeness" of the past regime, he could have, under the new dispensation, asked for the cancellation of those Alien Certificates and abandoned his alienage, specially before he ran for public office in 1988. The 1987 Constitution declares in no uncertain terms that "dual allegiance of citizens is inimical to the national interest and shall be dealt with by law" (Article IV, Section 5). That statement is but a reaffirmation of an innate conviction shared by every Filipino. The law referred to need not be awaited for one to consider giving up the legal convenience of dual citizenship. Accordingly, I vote to grant the Petition.

CRUZ, J., dissenting: I join Mr. Justice Padilla in his dissent. It seems to me that when a person voluntarily registers as an alien, he is in effect affirming that he is not a citizen. The terms "citizen" and "alien" are mutually exclusive from the viewpoint of municipal law, which is what really matters in the case at bar. Under this discipline, one is either a citizen of the local state or he is not; and the question is resolved on the basis of its own laws alone and not those of any other state. One of the several modes of losing Philippine citizenship under C.A. No. 63 is by "express renunciation" thereof. In the case of Frivaldo v. Commission on Elections, G.R. No. 87193, June 23,1989, there was such renunciation when the petitioner took an oath as a naturalized citizen of the United States in which he renounced all allegiance to all other states. In the case of Labo v. Commission on Elections, G.R. No. 86546, August 1, 1989, the petitioner not only took a similar oath after his naturalization in Australia but also executed other documents in which he stated that he was not a Filipino. The fact that his naturalization was later revoked did not also invalidate his disavowal of Philippine citizenship. "Express renunciation" is a separate mode of losing Philippine citizenship and is not necessarily dependent on "naturalization in a foreign country," which is another and different mode. When a person rejects and divorces his wife to enter into a second marriage, he cannot say he still loves her despite his desertion. The undeniable fact is that he has left her for another woman to whom he has totally and solemnly transferred his troth. It does him no credit when he protests he married a second time simply for material convenience and that his heart still belongs to the wife he has abandoned. At worst, it would reveal his sordid and deceitful character. By the same token, professing continued allegiance to the Philippines after renouncing it because of its meager resources, or for other ulterior and equally base reasons, is to me a paltry form of patriotism. It is a sop to the repudiated state and a slight to the adopted state. No matter how noble this attitude may appear to others, it is to me nothing less than plain and simple hypocrisy that we should not condone, let alone extol. Coming now to the case at bar, I note first of all that no naturalization is involved here as the private respondent claims to be a citizen both of the Philippines and of the United States. The question I think we must answer is: Was there an express renunciation of Philippine citizenship by the private respondent when he knowingly and voluntarily registered as an alien with the Commission of Immigration and Deportation in 1958 and in 1979? In Yu v. Commission of Immigration and Deportation, G.R. No. 83882, January 24, 1989, I made the following observations in a separate opinion: Regretfully, I cannot agree with the finding that the petitioner has expressly renounced his Philippine citizenship. The evidence on this point is in my view rather meager. Express renunciation of citizenship as a made of losing citizenship under Com. Act No. 63 is an unequivocal and

deliberate act with full awareness of its significance and consequences. I do not think the "commercial documents he signed" suggest such categorical disclaimer. That case is distinguished from the one before us now in that Yu did not ask the Philippine government to register him as an alien. Gov. Osmea did. It is my opinion that if the governor had confined himself to simply seeking and using an American passport, these acts could not have by themselves alone constituted a repudiation of Philippine citizenship. The problem, though, is that he did more than enjoy this legal convenience. What he actually did was register with the Philippine government as an alien within its own territory, presumably so he could be insulated from the jurisdiction it exercises over its nationals. This was a voluntary act. As a citizen of the Philippines, he was not required to register as an alien. Nevertheless, he chose to do so of his own free will. By this decision, he categorically asked the Republic of the Philippines to treat him as an American and not a Filipino, choosing to be an alien in this land that was willing to consider him its own. C.A. No. 63 does not necessarily require that the express renunciation of Philippine citizenship be made in connection with the naturalization of the erstwhile Filipino in a foreign country. Renunciation may be made independently of naturalization proceedings. Moreover, no sacramental words are prescribed by the statute for the express renunciation of Philippine citizenship. As long as the repudiation is categorical enough and the preference for the foreign state is unmistakable, as in the case at bar, Philippine citizenship is lost. The private respondent would have his cake and eat it too, but this can never be allowed where Philippine citizenship is involved. It is a gift that must be deserved to be retained. The Philippines for all her modest resources compared to those of other states, is a jealous and possessive mother demanding total love and loyalty from her children. It is bad enough that the love of the dual national is shared with another state; what is worse is where he formally rejects the Philippines, and in its own territory at that, and offers his total devotion to the other state. I am aware of the praiseworthy efforts of Gov. Osmea to improve the province of Cebu, and also, I should add, of the commendable record of Gov. Frivaldo and Mayor Labo in the administration of their respective jurisdictions. But that is not the point. The point is that it is not lawful to maintain in public office any person who, although supported by the electorate, is not a Filipino citizen. This is a relentless restriction we cannot ignore. Regretfully, therefore, I must vote to GRANT the petition.

PADILLA, J., dissenting: I am constrained to dissent. I start from the premise that the private respondent Emilio Mario Renner Osmea enjoyed at one time dual citizenship, i.e,, Philippine and U.S. citizenships. He was born in the Philippines of a Filipino father and an American (U.S.) mother. However, his sworn application for alien registration dated 21 November 1979 (Exh. B) filed with the Philippine immigration authorities was, in my view, an express renunciation of his Philippine citizenship. As held in Board of Immigration Commissioners vs. Go Callano 1 express renunciation means a renunciation that is made known distinctly and explicitly and not left to inference or implication. Nothing can be more distinct and explicit than when a dual citizenship holder-like the private respondent of age, and with full legal capacity to act, voluntarily and under oath applies with the Philippine Government for registration as an alien, insofar as his intention not to remain a Filipino citizen is concerned. And because of that distinct and explicit manifestation of desire to be considered an alien in the Philippines, the Philippine immigration authorities issued to private respondent Alien Certificate of Registration No. 015356 dated 21 November 1979 (Exh. C), Permit to Re- enter the Philippines No. 122018 dated 21 November 1979 (Exh. D) and Immigration Certificate of Clearance No. D-146483 dated 3 January 1980 (Exh E) 2 All the foregoing documents issued by the Philippine immigration authorities to the private respondent at his request are predicated on the proposition that private respondent is an alien under Philippine laws. It should also be mentioned that, while not marked as exhibit in the case at bar, private respondent was likewise issued in Cebu City Native Born Certificate of Residence No. 115883 on 21 November 1979 (as verified from Immigration records). This document, copy of which is attached hereto as Annex A, is again predicated on the proposition that private respondent is a duly-registered alien (American) residing in the Philippines. Another relevant document that merits attention is the Application for Re-entry Permit executed and signed by private respondent on 3 January 1980, again under oath, and verified from the records at the CID wherein private respondent expressly stated that he is a U.S. national. The importance of this document cannot be underestimated. For, if private respondent believed that he is a Filipino citizen, he would not have executed said Application for Re-entry Permit, since it is the right of every Filipino citizen to return to his country (the Philippines). The fact, therefore, that private respondent executed said sworn Application for Re-entry Permit, copy of which is attached hereto as Annex B, is again an abundant proof that he himself, no less, believed that he was, as he continuous to be, a resident alien (American) in the Philippines. It will further be noted that earlier, or in 1958, private respondent had already registered as an alien with the Bureau of Immigration under the Alien Registration Act of 1950 RA 562). Section 1 of said Act provides: SECTION 1. Aliens residing in the Philippines shall, within thirty days after the approval of this Act, apply for registration, in the case of those residing in the City of Manila, at the Bureau of Immigration and in the case of those residing in other localities at the office of the city or municipal treasurers, or at any other office designated by the President. ... . 3 (Emphasis supplied)

Accordingly, per certification of the Commissioner of Immigration and Deportation Miriam Defensor Santiago (Exh. A), issued on 26 January 1988, private respondent had been issued ACR No. B-21-448 and ICR No. 13391 on 27 and 28 March 1958 respectively. He, therefore, registered himself in the Philippines as an alien twice; first, in the year 1958, when he was 24 years old and again in 1979, when he was 45 years old. By twice registering under oath as an alien with the Bureau of Immigration, private respondent thereby clearly, distinctly and explicitly manifested and declared that he was an alien (and, therefore, not a Filipino citizen) residing in the Philippines and under its laws. At this point, and to be objectively fair to the private respondent, a clarification should be made. In his Comment on the Petition at bar (Rollo, p. 81), it is stated by his counsel that he (private respondent) was born in 1934-hence, our mathematical conclusion that when he first registered as an alien in 1958, he was 24 years old and in 1979 when he re-registered as an alien, he was 45 years old. However, private respondent's immigration records disclose that he was born in 1938 (not in 1934). On the assumption that the year 1938 is the correct year of birth of private respondent (and that his alleged year of birth, 1934, as stated in his Comment at bar is erroneous), then in 1958, when he first registered as an alien, he was 20 years old, while in 1979 when he re-registered as an alien, he was 41 years old. Still, his first registration as an alien (at age 20) has to be taken, in my view, as an express renunciation of his Philippine citizenship, because (1) at that time, he was almost 21 years old the age of majority, and (2) more importantly, under the applicable Alien Registration Act RA 562), an alien 14 years or over has to register in person (and not through his parents or guardian). It provides: The parent or legal guardian of an alien who is less than fourteen years of age, shall have the duty of registering such alien: Provided, That whenever any such alien attains his fourteenth birthday in the Philippines he shall, within fifteen days thereafter, apply in person for registration. (Sec. 1, par. 2) I take the above provision to mean that the choice by a dual nationality holder on whether to remain a Filipino citizen or an alien has to be made at age 14, and private respondent (although a bit late) made the notice in 1958 (at age 20) in favor of his U.S. citizenship. If all the foregoing acts of express renunciation of Philippine citizenship had been made or filed by private respondent elsewhere (not with the Philippine Government), there could perhaps be some room for contention that vis-a- vis the Philippine Government, private respondent had not renounced his Philippine citizenship. But said acts of express renunciation were filed with the Philippine Government and done right in the Philippines. In turn the Philippine Government, through the immigration authorities, accepted and acted on private respondent's aforesaid representations, and registered and documented him TWICE as an alien under Philippine law. The policy of our laws has been, and with laudable reason, to discourage dual citizenship, because this condition or status assumes as a necessary complement thereof dual allegiance at the same time to two (2) different countries. As early as 16 September 1947, a unanimous Supreme Court, speaking thru Mr. Justice Sabino Padilla in the celebrated case of Tan Chong vs. Secretary of Labor, rejected the principle of jus soli as determinative of Philippine citizenship, for the following reason, among others: ... . Citizenship, the main integrate element of which is allegiance, must not be taken lightly. Dual allegiance must be discouraged and prevented. But the application of the principle jus soli to persons born in this country of alien parentage would encourage dual allegiance which in the long run would be detrimental to both countries of which such persons might claim to be citizens. 4 This policy found later expression in the 1987 Constitution which now providesSec. 5. Dual allegiance of citizen is inimical to the national interest and shall be dealt with by law. (Article IV) Dual citizenship, in my considered opinion, must be eschewed. While having the "best of two (2) words" maybe the result of birth or other factors accidentally brought about, the "dual citizen" has to make a choice at one time or another. Having two (2) citizenships is, as I see it, similar in many ways to having two (2) legal spouses, when as a matter of principle and sound public policy, fealty to only one (1) spouse is both compelling and certainly desirable. Gordon and Rosenfield in their book on Immigration Law and Procedure state: Dual nationality is universally recognized as an undesirable phenomenon. It inevitably results in questionable loyalties and leads to international conflicts. Dual nationality also makes possible the use of citizenship as a badge of convenience rather than of undivided loyalty. And it impairs the singleness of commitment which is the hallmark of citizenship and allegiance. A person should have a right to choose his own nationality, and this choice should be honored by all countries. However, he should not be entitled to claim more than one nationality. 5 (Emphasis supplied) Private respondent made a deliberate and decisive choice when he asked the Philippine Government which, like many other countries, considers dual allegiance as against national or public interest to register him at least twice (and, therefore, unmistakably) as an alien in this country. That choice pro tanto was a renunciation of his Philippine citizenship. The choice must be respected as a conscious and knowledgeable act of a discerning, distinguished and respected person who must be presumed to have known the full import of his acts. Finally, the last thing that should be said against the Court is that it is inconsistent in its rulings. In the light of its recent decision in G.R. No. 86565 (Ramon L. Labo, Jr. vs. The Commission on Elections, et al.), I see no valid justification for holding Mr. Labo an alien upper Philippine law while holding private respondent herein a Filipino citizen. For, as the majority states: "In fact, in a number of sworn statements, Labo categorically declared that he was a citizen of Australia" (p. 7, Decision). And is exactly what private respondent did. In a number of sworn statements, he declared that he was a citizen of the United States. To Mr. Labo, the Court said, "so be it, you are an Australian," yet to the private respondent, despite such sworn statements that he is a U.S. citizen, the Court says, "never mind those sworn statements, you are still a Filipino." Sauce for the goose, as the saying goes, is sauce for the gander The doctrinal basis of the Court's decisions should be built on the merits, not on distinctions that really make no difference.

ACCORDINGLY, I vote to GRANT the petition and to declare the private respondent not a Filipino citizen by his own acts of express renunciation of such citizenship.

GUTIERREZ, JR., J., separate opinion: My stand in the cases of Willie Yu vs. Miriam Defensor Santiago, et al. (G.R. No, 83882, January 24, 1989) andRamon Labo, Jr, v. Commission on Elections (G.R. 86564, August 2, 1989) is clear. I regret, however, that I cannot participate in this case because one of the principal counsel is my relative by affinity, within the fourth civil degree.

Separate Opinions SARMIENTO, J., concurring: The majority seems agreed that the private respondent has acquired American citizenship, offly that he did not necessarily lose his Filipino citizenship. The important question, however, inheres in how he obtained American citizenship. I find that there is a dearth of facts here. For, if the private respondent became an American by naturalization, he has lost Filipino citizenship (Com. Act No. 63; Frivaldo v. COMELEC, G.R. No. 87193, June 21, 1989; Labo v. COMELEC, G.R. No. 86564, August 1, 1989). If he, however, became one by the application of the principle of jus soli it is by force of circumstances rather than choice. But he does not lose his Filipino citizenship, if he were otherwise born of Filipino parents. In the absence of evidence, we can not presume that he had ceased to be a citizen of the Philippines, simply because he is, at the same time. a citizen of the United States. There must be a clear showing that he lost his Filipino citizenship by any of the means enumerated by Commonwealth Act No. 63. The fact that he had obtained an alien certificate of registration, standing alone, does not amount to "express renunciation."

MELENCIO-HERRERA, J., dissenting: I join the dissent of Messrs. Justices Isagani A. Cruz and Teodoro R. Padilla. While it may be that dual citizenship usually results from accident of birth, a choice will have to be made by the individual concerned at some point in time in his life, involving as it does the priceless heritage of citizenship. That election was made by private respondent when, in 1958, at the age of 24, and in 1979, at 45, he obtained Alien Certificates of Registration. Registration as an alien is a clear and unambiguous act or declaration that one is not a citizen. If, in fact, private respondent was merely compelled to so register because of the "uncooperativeness" of the past regime, he could have, under the new dispensation, asked for the cancellation of those Alien Certificates and abandoned his alienage, specially before he ran for public office in 1988. The 1987 Constitution declares in no uncertain terms that "dual allegiance of citizens is inimical to the national interest and shall be dealt with by law" (Article IV, Section 5). That statement is but a reaffirmation of an innate conviction shared by every Filipino. The law referred to need not be awaited for one to consider giving up the legal convenience of dual citizenship. Accordingly, I vote to grant the Petition.

CRUZ, J., dissenting: I join Mr. Justice Padilla in his dissent. It seems to me that when a person voluntarily registers as an alien, he is in effect affirming that he is not a citizen. The terms "citizen" and "alien" are mutually exclusive from the viewpoint of municipal law, which is what really matters in the case at bar. Under this discipline, one is either a citizen of the local state or he is not; and the question is resolved on the basis of its own laws alone and not those of any other state. One of the several modes of losing Philippine citizenship under C.A. No. 63 is by "express renunciation" thereof. In the case of Frivaldo v. Commission on Elections, G.R. No. 87193, June 23,1989, there was such renunciation when the petitioner took an oath as a naturalized citizen of the United States in which he renounced all allegiance to all other states. In the case of Labo v. Commission on Elections, G.R. No. 86546, August 1, 1989, the petitioner not only took a similar oath after his naturalization in Australia but also executed other documents in which he stated that he was not a Filipino.

The fact that his naturalization was later revoked did not also invalidate his disavowal of Philippine citizenship. "Express renunciation" is a separate mode of losing Philippine citizenship and is not necessarily dependent on "naturalization in a foreign country," which is another and different mode. When a person rejects and divorces his wife to enter into a second marriage, he cannot say he still loves her despite his desertion. The undeniable fact is that he has left her for another woman to whom he has totally and solemnly transferred his troth It does him no credit when he protests he married a second time simply for material convenience and that his heart still belongs to the wife he has abandoned. At worst, it would reveal his sordid and deceitful character. By the same token, professing continued allegiance to the Philippines after renouncing it because of its meager resources, or for other ulterior and equally base reasons, is to me a paltry form of patriotism. It is a sop to the repudiated state and a slight to the adopted state. No matter how noble this attitude may appear to others, it is to me nothing less than plain and simple hypocrisy that we should not condone, let alone extol. Coming now to the case at bar, I note first of all that no naturalization is involved here as the private respondent claims to be a citizen both of the Philippines and of the United States. The question I think we must answer is: Was there an express renunciation of Philippine citizenship by the private respondent when he knowingly and voluntarily registered as an alien with the Commission of Immigration and Deportation in 1958 and in 1979? In Yu v. Commission of Immigration and Deportation, G.R. No. 83882, January 24, 1989, I made the following observations in a separate opinion: Regretfully, I cannot agree with the finding that the petitioner has expressly renounced his Philippine citizenship. The evidence on this point is in my view rather meager. Express renunciation of citizenship as a made of losing citizenship under Com. Act No. 63 is an unequivocal and deliberate act with full awareness of its significance and consequences. I do not think the "commercial documents he signed" suggest such categorical disclaimer. That case is distinguished from the one before us now in that Yu did not ask the Philippine government to register him as an alien. Gov. Osmea did. It is my opinion that if the governor had confined himself to simply seeking and using an American passport, these acts could not have by themselves alone constituted a repudiation of Philippine citizenship. The problem, though, is that he did more than enjoy this legal convenience. What he actually did was register with the Philippine government as an alien within its own territory, presumably so he could be insulated from the jurisdiction it exercises over its nationals. This was a voluntary act. As a citizen of the Philippines, he was not required to register as an alien. Nevertheless, he chose to do so of his own free will. By this decision, he categorically asked the Republic of the Philippines to treat him as an American and not a Filipino, choosing to be an alien in this land that was willing to consider him its own. C.A. No. 63 does not necessarily require that the express renunciation of Philippine citizenship be made in connection with the naturalization of the erstwhile Filipino in a foreign country. Renunciation may be made independently of naturalization proceedings. Moreover, no sacramental words are prescribed by the statute for the express renunciation of Philippine citizenship. As long as the repudiation is categorical enough and the preference for the foreign state is unmistakable, as in the case at bar, Philippine citizenship is lost. The private respondent would have his cake and eat it too, but this can never be allowed where Philippine citizenship is involved. It is a gift that must be deserved to be retained. The Philippines for all her modest resources compared to those of other states, is a jealous and possessive mother demanding total love and loyalty from her children. It is bad enough that the love of the dual national is shared with another state; what is worse is where he formally rejects the Philippines, and in its own territory at that, and offers his total devotion to the other state. I am aware of the praiseworthy efforts of Gov. Osmea to improve the province of Cebu, and also, I should add, of the commendable record of Gov. Frivaldo and Mayor Labo in the administration of their respective jurisdictions. But that is not the point. The point is that it is not lawful to maintain in public office any person who, although supported by the electorate, is not a Filipino citizen. This is a relentless restriction we cannot ignore. Regretfully, therefore, I must vote to GRANT the petition.

PADILLA, J., dissenting: I am constrained to dissent. I start from the premise that the private respondent Emilio Mario Renner Osmea enjoyed at one time dual citizenship, i.e,, Philippine and U.S. citizenships. He was born in the Philippines of a Filipino father and an American (U.S.) mother. However, his sworn application for alien registration dated 21 November 1979 (Exh. B) filed with the Philippine immigration authorities was, in my view, an express renunciation of his Philippine citizenship. As held in Board of Immigration Commissioners vs. Go Callano 1 express renunciation means a renunciation that is made known distinctly and explicitly and not left to inference or implication. Nothing can be more distinct and explicit than when a dual citizenship holder-like the private respondent of age, and with full legal capacity to act, voluntarily and under oath applies with the Philippine Government for registration as an alien, insofar as his intention not to remain a Filipino citizen is concerned. And because of that distinct and explicit manifestation of desire to be considered an alien in the Philippines, the Philippine immigration authorities issued to private respondent Alien Certificate of Registration No. 015356 dated 21 November 1979 (Exh. C),

Permit to Re- enter the Philippines No. 122018 dated 21 November 1979 (Exh. D) and Immigration Certificate of Clearance No. D-146483 dated 3 January 1980 (Exh E) 2 All the foregoing documents issued by the Philippine immigration authorities to the private respondent at his request are predicated on the proposition that private respondent is an alien under Philippine laws. It should also be mentioned that, while not marked as exhibit in the case at bar, private respondent was likewise issued in Cebu City Native Born Certificate of Residence No. 115883 on 21 November 1979 (as verified from Immigration records). This document, copy of which is attached hereto as Annex A, is again predicated on the proposition that private respondent is a duly-registered align (American) residing in the Philippines. Another relevant document that merits attention is the Application for Re-entry Permit executed and signed by private respondent on 3 January 1980, again under oath, and verified from the records at the CID wherein private respondent expressly stated that he is a U.S. national. The importance of this document cannot be underestimated For, if private respondent believed that he is a Filipino citizen, he would not have executed said Application for Re-entry Permit, since it is the right of every Filipino citizen to return to his country (the Philippines). The fact, therefore, that private respondent executed said sworn Application for Re-entry Permit, copy of which is attached hereto as Annex B, is again an abundant proof that he himself, no less, believed that he was, as he continuous to be, a resident alien (American) in the Philippines. It will further be noted that earlier, or in 1958, private respondent had already registered as an alien with the Bureau of Immigration under the Alien Registration Act of 1950 RA 562). Section 1 of said Act provides: SECTION 1. Aliens residing in the Philippines shall, within thirty days after the approval of this Act, apply for registration, in the case of those residing in the City of Manila, at the Bureau of Immigration and in the case of those residing in other localities at the office of the city or municipal treasurers, or at any other office designated by the President. ... . 3 (Emphasis supplied) Accordingly, per certification of the Commissioner of Immigration and Deportation Miriam Defensor Santiago (Exh. A), issued on 26 January 1988, private respondent had been issued ACR No. B-21-448 and ICR No. 13391 on 27 and 28 March 1958 respectively. He, therefore, registered himself in the Philippines as an alien twice; first, in the year 1958, when he was 24 years old and again in 1979, when he was 45 years old. By twice registering under oath as an alien with the Bureau of Immigration, private respondent thereby clearly, distinctly and explicitly manifested and declared that he was an alien (and, therefore, not a Filipino citizen) residing in the Philippines and under its laws. At this point, and to be objectively fair to the private respondent, a clarification should be made. In his Comment on the Petition at bar (Rollo, p. 81), it is stated by his counsel that he (private respondent) was born in 1934-hence, our mathematical conclusion that when he first registered as an alien in 1958, he was 24 years old and in 1979 when he re-registered as an alien, he was 45 years old. However, private respondent's immigration records disclose that he was born in 1938 (not in 1934). On the assumption that the year 1938 is the correct year of birth of private respondent (and that his alleged year of birth, 1934, as stated in his Comment at bar is erroneous), then in 1958, when he first registered as an alien, he was 20 years old, while in 1979 when he re-registered as an alien, he was 41 years old. Still, his first registration as an alien (at age 20) has to be taken, in my view, as an express renunciation of his Philippine citizenship, because (1) at that time, he was almost 21 years old the age of majority, and (2) more importantly, under the applicable Alien Registration Act RA 562), an alien 14 years or over has to register in person (and not through his parents or guardian). It provides: The parent or legal guardian of an alien who is less than fourteen years of age, shall have the duty of registering such alien: Provided, That whenever any such alien attains his fourteenth birthday in the Philippines he shall, within fifteen days thereafter, apply in person for registration. (Sec. 1, par. 2) I take the above provision to mean that the choice by a dual nationality holder on whether to remain a Filipino citizen or an alien has to be made at age 14, and private respondent (although a bit late) made the notice in 1958 (at age 20) in favor of his U.S. citizenship. If all the foregoing acts of express renunciation of Philippine citizenship had been made or filed by private respondent elsewhere (not with the Philippine Government), there could perhaps be some room for contention that vis-a- vis the Philippine Government, private respondent had not renounced his Philippine citizenship. But said acts of express renunciation were filed with the Philippine Government and done right in the Philippines. In turn the Philippine Government, through the immigration authorities, accepted and acted on private respondent's aforesaid representations, and registered and documented him TWICE as an alien under Philippine law. The policy of our laws has been, and with laudable reason, to discourage dual citizenship, because this condition or status assumes as a necessary complement thereof dual allegiance at the same time to two (2) different countries. As early as 16 September 1947, a unanimous Supreme Court, speaking thru Mr. Justice Sabino Padilla in the celebrated case of Tan Chong vs. Secretary of Labor, rejected the principle of jus soli as determinative of Philippine citizenship, for the following reason, among others: ... . Citizenship, the main integrate element of which is allegiance, must not be taken lightly. Dual allegiance must be discouraged and prevented. But the application of the principle jus soli to persons born in this country of alien parentage would encourage dual allegiance which in the long run would be detrimental to both countries of which such persons might claim to be citizens. 4 This policy found later expression in the 1987 Constitution which now providesSec. 5. Dual allegiance of citizen is inimical to the national interest and shall be dealt with by law. (Article IV) Dual citizenship, in my considered opinion, must be eschewed. While having the "best of two (2) words" maybe the result of birth or other factors accidentally brought about, the "dual citizen" has to make a choice at one time or another. Having two (2) citizenships is, as I see it,

similar in many ways to having two (2) legal spouses, when as a matter of principle and sound public policy, fealty to only one (1) spouse is both compelling and certainly desirable. Gordon and Rosenfield in their book on Immigration Law and Procedure state: Dual nationality is universally recognized as an undesirable phenomenon. It inevitably results in questionable loyalties and leads to international conflicts. Dual nationality also makes possible the use of citizenship as a badge of convenience rather than of undivided loyalty. And it impairs the singleness of commitment which is the hallmark of citizenship and allegiance. A person should have a right to choose his own nationality, and this choice should be honored by all countries. However, he should not be entitled to claim more than one nationality. 5 (Emphasis supplied) Private respondent made a deliberate and decisive choice when he asked the Philippine Government which, like many other countries, considers dual allegiance as against national or public interest to register him at least twice (and, therefore, unmistakably) as an alien in this country. That choice pro tanto was a renunciation of his Philippine citizenship. The choice must be respected as a conscious and knowledgeable act of a discerning, distinguished and respected person who must be presumed to have known the full import of his acts. Finally, the last thing that should be said against the Court is that it is inconsistent in its rulings. In the light of its recent decision in G.R. No. 86565 (Ramon L. Labo, Jr. vs. The Commission on Elections, et al., I see no valid justification for holding Mr. Labo an alien upper Ph. Philippine law while holding private respondent herein a Filipino citizen. For, as the majority states: "In fact,, in a number of sworn statements, Labo categorically declared that he was a citizen of Australia"(p. 7, Decision). And is exactly what private respondent did. In a number of sworn statements, he declared that he was a citizen of the United States. To Mr. Labo, the Court said, "so be it, you are an Australian," yet to the private respondent, despite such sworn statements that he is a U.S. citizen, the Court says, "never mind those sworn statements, you are still a Filipino." Sauce for the goose, as the saying goes, is sauce for the gender The doctrinal basis of the Court's decisions should be built on the merits, not on distinctions that really make no difference. ACCORDINGLY, I vote to GRANT the petition and to declare the private respondent not a Filipino citizen by his own acts of express renunciation of such citizenship.

GUTIERREZ, JR., J., separate opinion: My stand in the cases of Willie Yu vs. Miriam Defensor Santiago, et al. (G.R. No, 83882, January 24, 1989) andRamon Labo, Jr, v. Commission on Elections (G.R. 86564, August 2, 1989) is clear. I regret, however, that I cannot participate in this case because one of the principal counsel is my relative by affinity, within the fourth civil degree.

G.R. No. 83820 May 25, 1990 JOSE B. AZNAR, petitioner,vs.COMMISSION ON ELECTIONS and EMILIO MARIO RENNER OSMEA, respondents.Ponente: PARAS, J.:

Petitioner: Aznar provincial chairman of PDP Laban in Cebu Respondent: COMELEC and Osmena candidate for provincial governor of CebuFACTS:1)

On November 19, 1987, private respondent Emilio "Lito" Osmea filed his certificate of candidacy with theCOMELEC for the position of Provincial Governor of Cebu Province in the January 18, 1988 local elections.2)

On January 22, 1988, petition er Jose B. Aznar in his capacity as its incumbent Provincial Chairman filed with theCOMELEC a petition for the disqualificati on of private respondent on the ground that he is allegedly not a Filipinocitizen, being a citizen of the United States of America.3)

On January 27, 1988, petitioner filed a Formal Manifestation submitting a Certificate issued by the thenImmigration and Deportation Commissioner Miriam Defensor Santiago certifying that private respondent is anAmerican and is a holder of Alien Certificate of Registration (ACR) No. B-21448 and Immigrant Certificate of Residence (ICR) No. 133911, issued at Manila on March 27 and 28, 1958, respectively. (Annex "B-1").4)

During the hearing at the COMELEC Private respondent, maintained that he is a Filipino citizen, alleging: that heis the legitimate child of Dr. Emilio D. Osmea, a Filipino and son of the late President Sergio Osmea, Sr.; that heis a holder of a valid and subsisting Philippine Passport No. 0855103 issued on March 25, 1987; that he has beencontinuously residing in the Philippines since birth and has not gone out of the country for more than six months;and that he has been a registered voter in the Philippines since 19 65.5)

Thereafter, on June 11, 1988, COMELEC (First Division) dismissed the petition for disqualification for not havingbeen timely filed and for lack of sufficient proof that private respondent is not a Filipino citizen. Hence, thepetition for Certiorari.ISSUE:Whether or not respondent Osmena is no longer a Filipino citizen by acquiring dualcitizenship? HELD:SC dismissed petition for certiorari upholding COMELEC s decision. The petitioner failed to present direct proof that privaterespondent had lost his Filipino citizenship by any of the modes provided for under C.A. No. 63. these are: (1) bynaturalization in a foreign country; (2) by express renunciation of citizenship; and (3) by subscribing to an oath of allegia nceto support the Constitution or laws of a foreign country. From the evi dence, it is clear that private respondent Osmea didnot lose his Philippine citizenship by any of the three mentioned hereinabove or by any other mode of losing Philippinecitizenship.In the instant case, private respondent vehemently denies having taken the oath of allegiance of the United States. He is aholder of a valid and subsisting Philippine passport and has continuously participated in the electoral process in thiscountry since 1963 up to the present, both as a voter and as a candidate. Thus, private respondent remains a Filipino andthe loss of his Philippine citizenship cannot be presumed.Considering the fact that admittedly Osmea was both a Filipino and an American, the mere fact that he has a Certificatestating he is an American does not mean that he is not still a Filipino. In the case of Osmea, the Certification that he is anAmerican does not mean that he is not still a Filipino, possessed as he is, of both nationalities or citizenships. Indeed, th ereis no express renunciation here of Philippi ne citizenship; truth to tell, there is even no implied renunciation of saidcitizenship. When we consider that the renunciation needed to lose Philippine citizenship must be "express", it stands toreason that there can be no such loss of Philippine 'citizenship when there is no renunciation either "'express" or "implied" .

FRIVALDO vs. COMELEC Case Digest FRIVALDO vs. COMELEC 174 SCRA 245 G.R. No. 87193 June 23, 1989 Facts: Petitioner Juan G. Frivaldo was proclaimed governor-elect and assume office in due time. The League of Municipalities filed with the COMELEC a petition for annulment of Frivaldos election and proclamation on the ground that he was not a Fil ipino citizen, having been naturalized in the United States. Frivaldo admitted the allegation but pleaded the special and affirmative defenses that his naturalization was merely forced upon himself as a means of survival against the unrelenting prosecution by the Martial Law Dictators agent abroad. Issue: Whether or not Frivaldo was a citizen of the Philippines at the time of his election.

Held: No. Section 117 of the Omnibus Election Code provides that a qualified voter must be, among other qualifications, a citizen of the Philippines, this being an indispensable requirement for suffrage under Article V, Section 1, of the Constitution. Even if he did lose his naturalized American citizenship, such forfeiture did not and could not have the effect of automatically restoring his citizenship in the Philippines that he had earlier renounced. Qualifications for public office are continuing requirements and must be possessed not only at the time of appointment or election or assumption of office but during the officers entire tenure. Frivaldo declared not a citizen of the Philippines and therefore disqualified from serving as a Governor of the Province of Sorsogon.

EN BANC

[G.R. No. 120295. June 28, 1996]

JUAN G. FRIVALDO, petitioner, vs. COMMISSION ON ELECTIONS, and RAUL R. LEE, respondents.

[G.R. No. 123755. June 28, 1996]

RAUL R. LEE, petitioner, vs. COMMISSION ON ELECTIONS and JUAN G. FRIVALDO, respondents. DECISION PANGANIBAN, J.: The ultimate question posed before this Court in these twin cases is: Who should be declared the rightful governor of Sorsogon (i) Juan G. Frivaldo, who unquestionably obtained the highest number of votes in three successive elections but who was twice declared by this Court to be disqualified to hold such office due to his alien citizenship, and who now claims to have re-assumed his lost Philippine citizenship thru repatriation; (ii) Raul R. Lee, who was the second placer in the canvass, but who claims that the votes cast in favor of Frivaldo should be considered void; that the electorate should be deemed to have intentionally thrown away their ballots; and that legally, he secured the most number of valid votes; or (iii) The incumbent Vice-Governor, Oscar G. Deri, who obviously was not voted directly to the position of governor, but who according to prevailing jurisprudence should take over the said post inasmuch as, by the ineligibility of Frivaldo, a "permanent vacancy in the contested office has occurred"? In ruling for Frivaldo, the Court lays down new doctrines on repatriation, clarifies/reiterates/amplifies existing jurisprudence on citizenship and elections, and upholds the superiority of substantial justice over pure legalisms. G.R. No. 123755. This is a special civil action under Rules 65 and 58 of the Rules of Court for certiorari and preliminary injunction to review and annul a Resolution of the respondent Commission on Elections (Comelec), First Division,1 promulgated on December 19,19952 and another Resolution of the Comelec en bane promulgated February 23, 19963 denying petitioner's motion for reconsideration.

The Facts On March 20, 1995, private respondent Juan G. Frivaldo filed his Certificate of Candidacy for the office of Governor of Sorsogon in the May 8, 1995 elections. On March 23, 1995, petitioner Raul R. Lee, another candidate, filed a petition4 with the Comelec docketed as SPA

No. 95-028 praying that Frivaldo "be disqualified from seeking or holding any public office or position by reason of not yet being a citizen of the Philippines," and that his Certificate of Candidacy be cancelled. On May 1, 1995, the Second Division of the Comelec promulgated a Resolution5 granting the petition with the following disposition:6 "WHEREFORE, this Division resolves to GRANT the petition and declares that respondent is DISQUALIFIED to run for the Office of Governor of Sorsogon on the ground that he is NOT a citizen of thePhilippines. Accordingly, respondent's certificate of candidacy is cancelled." The Motion for Reconsideration filed by Frivaldo remained unacted upon until after the May 8, 1995 elections. So, his candidacy continued and he was voted for during the elections held on said date. On May 11, 1995, the Comelec en banc7 affirmed the aforementioned Resolution of the Second Division. The Provincial Board of Canvassers completed the canvass of the election returns and a Certificate of Votes 8.dated May 27, 1995 was issued showing the following votes obtained by the candidates for the position of Governor of Sorsogon: Antonio H. Escudero, Jr. Juan G. Frivaldo RaulR.Lee Isagani P. Ocampo 51,060 73,440 53,304 1,925

On June 9, 1995, Lee filed in said SPA No. 95-028, a (supplemental) petition9 praying for his proclamation as the duly-elected Governor of Sorsogon. In an order10 dated June 21, 1995, but promulgated according to the petition "only on June 29, 1995," the Comelec en bane directed "the Provincial Board of Canvassers of Sorsogon to reconvene for the purpose of proclaiming candidate Raul Lee as the winning gubernatorial candidate in the province of Sorsogon on June 29,1995 x x x." Accordingly, at8:30 in the evening of June 30,1995, Lee was proclaimed governor of Sorsogon. On July 6, 1995, Frivaldo filed with the Comelec a new petition,11 docketed as SPC No. 95-317, praying for the annulment of the June 30, 1995 proclamation of Lee and for his own proclamation. He alleged that on June 30, 1995, at 2:00 in the afternoon, he took his oath of allegiance as a citizen of the Philippines after "his petition for repatriation under P.D. 725 which he filed with the Special Committee on Naturalization in September 1994 had been granted." As such, when "the said order (dated June 21, 1995) (of the Comelec) x x x was released and received by Frivaldo on June 30, 1995 at 5:30 o'clock in the evening, there was no more legal impediment to the proclamation (of Frivaldo) as governor x x x." In the alternative, he averred that pursuant to the two cases of Labo vs. Comelec,12 the Vice-Governor not Lee should occupy said position of governor. On December 19, 1995, the Comelec First Division promulgated the herein assailed Resolution13 holding that Lee, "not having garnered the highest number of votes," was not legally entitled to be proclaimed as duly-elected governor; and that Frivaldo, "having garnered the highest number of votes, and xxx having reacquired his Filipino citizenship by repatriation on June 30, 1995 under the provisions of Presidential Decree No. 725 xxx (is) qualified to hold the office of governor of Sorsogon"; thus: "PREMISES CONSIDERED, the Commission (First Division), therefore RESOLVES to GRANT the Petition. Consistent with the decisions of the Supreme Court, the proclamation of Raul R. Lee as Governor of Sorsogon is hereby ordered annulled, being contrary to law, he not having garnered the highest number of votes to warrant his proclamation. Upon the finality of the annulment of the proclamation of Raul R. Lee, the Provincial Board of Canvassers is directed to immediately reconvene and, on the basis of the completed canvass, proclaim petitioner Juan G. Frivaldo as the duly elected Governor of Sorsogon having garnered the highest number of votes, and he having reacquired his Filipino citizenship by repatriation on June 30,1995 under the provisions of Presidential Decree No. 725 and, thus, qualified to hold the office of Governor of Sorsogon. Conformably with Section 260 of the Omnibus Election Code (B.P. Blg. 881), the Clerk of the Commission is directed to notify His Excellency the President of the Philippines, and the Secretary of the Sangguniang Panlalawigan of the Province of Sorsogon of this resolution immediately upon the due implementation thereof." On December 26,1995, Lee filed a motion for reconsideration which was denied by the Comelec en banc in its Resolution14 promulgated on February 23, 1996. On February 26, 1996, the present petition was filed. Acting on the prayer for a temporary restraining order, this Court issued on February 27, 1996 a Resolution which inter alia directed the parties "to maintain the status quo prevailing prior to the filing of this petition."

The Issues in G.R. No. 123755 Petitioner Lee's "position on the matter at hand briefly be capsulized in the following propositions": 15 "First - The initiatory petition below was so far insufficient in form and substance to warrant the exercise by the COMELEC of its jurisdiction with the result that, in effect, the COMELEC acted without jurisdiction in taking cognizance of and deciding said petition;

Second- The judicially declared disqualification of respondent was a continuing condition and rendered him ineligible to run for, to be elected to and to hold the Office of Governor; Third - The alleged repatriation of respondent was neither valid nor is the effect thereof retroactive as to cure his ineligibility and qualify him to hold the Office of Governor; and Fourth - Correctly read and applied, the Labo Doctrine fully supports the validity of petitioner's proclamation as duly elected Governor of Sorsogon."

G.R. No. 120295 This is a petition to annul three Resolutions of the respondent Comelec, the first two of which are also at issue in G.R. No. 123755, as follows: 1. Resolution16 of the Second Division, promulgated on May 1, 1995, disqualifying Frivaldo from running for governor of Sorsogon in the May 8, 1995 elections "on the ground that he is not a citizen of thePhilippines"; 2. Resolution17 of the Comelec en bane, promulgated on May 11, 1995; and 3. Resolution18 of the Comelec en bane, promulgated also on May 11, 1995 suspending the proclamation of, among others, Frivaldo.

The Facts and the Issue The facts of this case are essentially the same as those in G.R. No. 123755. However, Frivaldo assails the above-mentioned resolutions on a different ground: that under Section 78 of the Omnibus Election Code, which is reproduced hereinunder: "Section 78. Petition to deny due course or to cancel a certificate of candidacy. A verified petition seeking to deny due course or to cancel a certificate of candidacy may be filed by any person exclusively on the ground that any material representation contained therein as required under Section 74 hereof is false. The petition may be filed at any time not later than twenty-five days from the time of the filing of the certificate of candidacy and shall be decided, after notice and hearing, not later than fifteen days before the election." (Italics supplied.) the Comelec had no jurisdiction to issue said Resolutions because they were not rendered "within the period allowed by law," i.e., "not later than fifteen days before the election." Otherwise stated, Frivaldo contends that the failure of the Comelec to act on the petition for disqualification within the period of fifteen days prior to the election as provided by law is a jurisdictional defect which renders the said Resolutions null and void. By Resolution on March 12, 1996, the Court consolidated G.R. Nos. 120295 and 123755 since they are intimately related in their factual environment and are identical in the ultimate question raised, viz., who should occupy the position of governor of the province of Sorsogon. On March 19, 1995, the Court heard oral argument from the parties and required them thereafter to file simultaneously their respective memoranda.

The Consolidated Issues From the foregoing submissions, the consolidated issues may be restated as follows: 1. Was the repatriation of Frivaldo valid and legal? If so, did it seasonably cure his lack of citizenship as to qualify him to be proclaimed and to hold the Office of Governor? If not, may it be given retroactive effect? If so, from when? 2. Is Frivaldo's "judicially declared" disqualification for lack of Filipino citizenship a continuing bar to his eligibility to run for, be elected to or hold the governorship of Sorsogon? 3. Did the respondent Comelec have jurisdiction over the initiatory petition in SPC No. 95-317 considering that : said petition is not "a preproclamation case, an election protest or a quo warranto case"? 4. Was the proclamation of Lee, a runner-up in the election, valid and legal in light of existing jurisprudence? 5. Did the respondent Commission on Elections exceed its jurisdiction in promulgating the assailed Resolutions, all of which prevented Frivaldo from assuming the governorship of Sorsogon, considering that they were not rendered within ( the period referred to in Section 78 of the Omnibus Election Code, viz., "not later than fifteen days before the elections"?

The First Issue: Frivaldo's Repatriation The validity and effectivity of Frivaldo's repatriation is the lis mota, the threshold legal issue in this case. All the other matters raised are secondary to this. The Local Government Code of 199119 expressly requires Philippine citizenship as a qualification for elective local officials, including that of provincial governor, thus: "Sec. 39. Qualifications. (a) An elective local official must be a citizen of the Philippines; a registered voter in the barangay, municipality, city, or province or, in the case of a member of the sangguniang panlalawigan, sangguniang panlungsod, or sangguniang bayan, the district where he intends to be elected; a resident therein for at least one (1) year immediately preceding the day of the election; and able to read and write Filipino or any other local language or dialect. (b) Candidates for the position of governor, vice governor or member of the sangguniang panlalawigan, or mayor, vice mayor or member of the sangguniang panlungsod of highly urbanized cities must be at least twenty-three (23) years of age on election day. xxx Court20 xxx xxx

Inasmuch as Frivaldo had been declared by this as a non-citizen, it is therefore incumbent upon him to show that he has reacquired citizenship; in fine, that he possesses the qualifications prescribed under the said statute (R. A. 7160). Under Philippine law,21 citizenship may be reacquired by direct act of Congress, by naturalization or by repatriation. Frivaldo told this Court in G.R. No. 10465422 and during the oral argument in this case that he tried to resume his citizenship by direct act of Congress, but that the bill allowing him to do so "failed to materialize, notwithstanding the endorsement of several members of the House of Representatives" due, according to him, to the "maneuvers of his political rivals." In the same case, his attempt at naturalizationwas rejected by this Court because of jurisdictional, substantial and procedural defects. Despite his lack of Philippine citizenship, Frivaldo was overwhelmingly elected governor by the electorate of Sorsogon, with a margin of 27,000 votes in the 1988 elections, 57,000 in 1992, and 20,000 in 1995 over the same opponent Raul Lee. Twice, he was judicially declared a non-Filipino and thus twice disqualified from holding and discharging his popular mandate. Now, he comes to us a third time, with a fresh vote from the people of Sorsogon and a favorable decision from the Commission on Elections to boot. Moreover, he now boasts of having successfully passed through the third and last mode of reacquiring citizenship: by repatriation under P.D. No. 725, with no less than the Solicitor General himself, who was the prime opposing counsel in the previous cases he lost, this time, as counsel for co-respondent Comelec, arguing the validity of his cause (in addition to his able private counsel Sixto S. Brillantes, Jr.). That he took his oath of allegiance under the provisions of said Decree at 2:00 p.m. on June 30, 1995 is not disputed. Hence, he insists that henot Leeshould have been proclaimed as the duly-elected governor of Sorsogon when the Provincial Board of Canvassers met at 8:30 p.m. on the said date since, clearly and unquestionably, he garnered the highest number of votes in the elections and since at that time, he already reacquired his citizenship. En contrario, Lee argues that Frivaldo's repatriation is tainted ; with serious defects, which we shall now discuss in seriatim. First, Lee tells us that P.D. No. 725 had "been effectively repealed," asserting that "then President Corazon Aquino exercising legislative powers under the Transitory Provisions of the 1987 Constitution, forbade the grant of citizenship by Presidential Decree or Executive Issuances as the same poses a serious and contentious issue of policy which the present government, in the exercise of prudence and sound discretion, should best leave to the judgment of the first Congress under the 1987 Constitution," adding that in her memorandum dated March 27,1987 to the members of the Special Committee on Naturalization constituted for purposes of Presidential Decree No. 725, President Aquino directed them "to cease and desist from undertaking any and all proceedings within your functional area of responsibility as defined under Letter of Instructions (LOI) No. 270 dated April 11, 1975, as amended."23 This memorandum dated March 27, 198724 cannot by any stretch of legal hermeneutics be construed as a law sanctioning or authorizing a repeal of P.D. No. 725. Laws are repealed only by subsequent ones25 and a repeal may be express or implied. It is obvious that no express repeal was made because then President Aquino in her memorandum based on the copy furnished us by Lee did not categorically and/or impliedly state that P.D. 725 was being repealed or was being rendered without any legal effect. In fact, she did not even mention it specifically by its number or text. On the other hand, it is a basic rule of statutory construction that repeals by implication are not favored. An implied repeal will not be allowed "unless it is convincingly and unambiguously demonstrated that the two laws are clearly repugnant and patently inconsistent that they cannot co-exist."26 The memorandum of then President Aquino cannot even be regarded as a legislative enactment, for not every pronouncement of the Chief Executive even under the Transitory Provisions of the 1987 Constitution can nor should be regarded as an exercise of her law-making powers. At best, it could be treated as an executive policy addressed to the Special Committee to halt the acceptance and processing of applications for repatriation pending whatever "judgment the first Congress under the 1987 Constitution" might make. In other words, the former President did not repeal P.D. 725 but left it to the first Congress once createdto deal with the matter. If she had intended to repeal such law, she should have unequivocally said so instead of referring the matter to Congress. The fact is she carefully couched her presidential issuance in terms that clearly indicated the intention of "the present government, in the exercise of prudence and sound discretion" to leave the matter of repeal to the new Congress. Any other interpretation of the said Presidential Memorandum, such as is now being proffered to the Court by Lee, would visit unmitigated violence not only upon statutory construction but on common sense as well. Second. Lee also argues that "serious congenital irregularities flawed the repatriation proceedings," asserting that Frivaldo's application therefor was "filed on June 29, 1995 x x x (and) was approved in just one day or on June 30, 1995 x x x," which "prevented a judicious review and evaluation of the merits thereof." Frivaldo counters that he filed his application for repatriation with the Office of the President in Malacanang Palace on August 17, 1994. This is confirmed by the Solicitor General. However, the Special Committee was reactivated only on June 8, 1995, when presumably the said Committee started processing his application. On June 29, 1995, he filled up and re-submitted the FORM that the Committee required. Under these circumstances, it could not be said that there was "indecent haste" in the processing of his application. Anent Lee's charge that the "sudden reconstitution of the Special Committee on Naturalization was intended solely for the personal interest of respondent,"27 the Solicitor General explained during the oral argument on March 19, 1996 that such allegation is simply baseless as

there were many others who applied and were considered for repatriation, a list of whom was submitted by him to this Court, through a Manifestation28 filed on April 3, 1996. On the basis of the parties' submissions, we are convinced that the presumption of regularity in the performance of official duty and the presumption of legality in the repatriation of Frivaldo have not been successfully rebutted by Lee. The mere fact that the proceedings were speeded up is by itself not a ground to conclude that such proceedings were necessarily tainted. After all, the requirements of repatriation under P.D. No. 725 are not difficult to comply with, nor are they tedious and cumbersome. In fact, P.D. 72529 itself requires very little of an applicant, and even the rules and regulations to implement the said decree were left to the Special Committee to promulgate. This is not unusual since, unlike in naturalization where an alien covets a first-time entry into Philippine political life, in repatriation the applicant is a former natural-born Filipino who is merely seeking to reacquire his previous citizenship. In the case of Frivaldo, he was undoubtedly a natural-born citizen who openly and faithfully served his country and his province prior to his naturalization in the United States a naturalization he insists was made necessary only to escape the iron clutches of a dictatorship he abhorred and could not in conscience embrace and who, after the fall of the dictator and the re-establishment of democratic space, wasted no time in returning to his country of birth to offer once more his talent and services to his people. So too, the fact that ten other persons, as certified to by the Solicitor General, were granted repatriation argues convincingly and conclusively against the existence of favoritism vehemently posited by Raul Lee. At any rate, any contest on the legality of Frivaldo's repatriation should have been pursued before the Committee itself, and, failing there, in the Office of the President, pursuant to the doctrine of exhaustion of administrative remedies. Third. Lee further contends that assuming the assailed repatriation to be valid, nevertheless it could only be effective as at 2:00 p.m. of June 30, 1995 whereas the citizenship qualification prescribed by the Local Government Code "must exist on the date of his election, if not when the certificate of candidacy is filed," citing our decision in G.R. 104654 30which held that "both the Local Government Code and the Constitution require that only Philippine citizens can run and be elected to Public office" Obviously, however, this was a mere obiter as the only issue in said case was whether Frivaldo's naturalization was valid or not and NOT the effective date thereof. Since the Court held his naturalization to be invalid, then the issue of when an aspirant for public office should be a citizen was NOT resolved at all by the Court. Which question we shall now directly rule on. Under Sec. 39 of the Local Government Code, "(a)n elective local official must be: * a citizen of the Philippines; * a registered voter in the barangay, municipality, city, or province x x x where he intends to be elected; * a resident therein for at least one (1) year immediately preceding the day of the election; * able to read and write Filipino or any other local language or dialect." * In addition, "candidates for the position of governor x x x must be at least twenty-three (23) years of age on election day." From the above, it will be noted that the law does not specify any particular date or time when the candidate must possess citizenship, unlike that for residence (which must consist of at least one year's residency immediately preceding the day of election) and age (at least twenty three years of age on election day). Philippine citizenship is an indispensable requirement for holding an elective public office, 31 and the purpose of the citizenship qualification is none other than to ensure that no alien, i.e., no person owing allegiance to another nation, shall govern our people and our country or a unit of territory thereof. Now, an official begins to govern or to discharge his functions only upon his proclamation and on the day the law mandates his term of office to begin. Since Frivaldo re-assumed his citizenship on June 30, 1995the very day32 the term of office of governor (and other elective officials) beganhe was therefore already qualified to be proclaimed, to hold such office and to discharge the functions and responsibilities thereof as of said date. In short, at that time, he was already qualified to govern his native Sorsogon. This is the liberal interpretation that should give spirit, life and meaning to our law on qualifications consistent with the purpose for which such law was enacted. So too, even from a literal (as distinguished from liberal) construction, it should be noted that Section 39 of the Local Government Code speaks of "Qualifications" of "ELECTIVE OFFICIALS," not of candidates. Why then should such qualification be required at the time of election or at the time of the filing of the certificates of candidacies, as Lee insists? Literally, such qualifications unless otherwise expressly conditioned, as in the case of age and residence should thus be possessed when the "elective [or elected] official" begins to govern, i.e., at the time he is proclaimed and at the start of his term in this case, on June 30, 1995. Paraphrasing this Court's ruling in Vasquez vs. Giapand Li Seng Giap & Sons,33 if the purpose of the citizenship requirement is to ensure that our people and country do not end up being governed by aliens, i.e., persons owing allegiance to another nation, that aim or purpose would not be thwarted but instead achieved by construing the citizenship qualification as applying to the time of proclamation of the elected official and at the start of his term. But perhaps the more difficult objection was the one raised during the oral argument34 to the effect that the citizenship qualification should be possessed at the time the candidate (or for that matter the elected official) registered as a voter. After all, Section 39, apart from requiring the official to be a citizen, also specifies as another item of qualification, that he be a "registered voter." And, under the law35 a "voter" must be a citizen of the Philippines. So therefore, Frivaldo could not have been a voter-much less a validly registered one if he was not a citizen at the time of such registration. The answer to this problem again lies in discerning the purpose of the requirement. If the law intended the citizenship qualification to be possessed prior to election consistent with the requirement of being a registered voter, then it would not have made citizenship a SEPARATE qualification. The law abhors a redundancy. It therefore stands to reason that the law intended CITIZENSHIP to be a qualification distinct from being a VOTER, even if being a voter presumes being a citizen first. It also stands to reason that the voter requirement was included as another qualification (aside from "citizenship"), not to reiterate the need for nationality but to require that the official be registered as a voter IN THE AREA OR TERRITORY he seeks to govern, i.e., the law states: "a registered voter in the barangay, municipality, city, or province x x x where he intends to be elected." It should be emphasized that the Local Government Code requires an elective official to be a registered voter. It does not require him to vote actually. Hence, registrationnot the actual votingis the core of this "qualification." In other words, the law's purpose in this second requirement is to ensure that the prospective official is actually registered in the area he seeks to govern and not anywhere else. Before this Court, Frivaldo has repeatedly emphasizedand Lee has not disputed that he "was and is a registered voter of Sorsogon, and his registration as a voter has been sustained as valid by judicial declaration x x x In fact, he cast his vote in his precinct on May 8, 1995."36

So too, during the oral argument, his counsel stead-fastly maintained that "Mr. Frivaldo has always been a registered voter of Sorsogon. He has voted in 1987,1988,1992, then he voted again in 1995. In fact, his eligibility as a voter was questioned, but the court dismissed (sic) his eligibility as a voter and he was allowed to vote as in fact, he voted in all the previous elections including on May 8,1995.37 It is thus clear that Frivaldo is a registered voter in the province where he intended to be elected. There is yet another reason why the prime issue of citizenship should be reckoned from the date of proclamation, not necessarily the date of election or date of filing of the certificate of candidacy. Section 253 of the Omnibus Election Code 38 gives any voter, presumably including the defeated candidate, the opportunity to question the ELIGIBILITY (or the disloyalty) of a candidate. This is the only provision of the Code that authorizes a remedy on how to contest before the Comelec an incumbent's ineligibility arising from failure to meet the qualifications enumerated under Sec. 39 of the Local Government Code. Such remedy of Quo Warranto can be availed of "within ten days after proclamation" of the winning candidate. Hence, it is only at such time that the issue of ineligibility may be taken cognizance of by the Commission. And since, at the very moment of Lee's proclamation (8:30 p.m., June 30, 1995), Juan G. Frivaldo was already and indubitably a citizen, having taken his oath of allegiance earlier in the afternoon of the same day, then he should have been the candidate proclaimed as he unquestionably garnered the highest number of votes in the immediately preceding elections and such oath had already cured his previous "judicially-declared" alienage. Hence, at such time, he was no longer ineligible. But to remove all doubts on this important issue, we also hold that the repatriation of Frivaldo RETRO ACTED to the date of the filing of his application on August 17,1994. It is true that under the Civil Code of the Philippines,39 "(l)aws shall have no retroactive effect, unless the contrary is provided." But there are settled exceptions40 to this general rule, such as when the statute is CURATIVE or REMEDIAL in nature or when it CREATES NEW RIGHTS. According to Tolentino,41 curative statutes are those which undertake to cure errors and irregularities, thereby validating judicial or administrative proceedings, acts of public officers, or private deeds and contracts which otherwise would not produce their intended consequences by reason of some statutory disability or failure to comply with some technical requirement. They operate on conditions already existing, and are necessarily retroactive in operation. Agpalo,42 on the other hand, says that curative statutes are "healing acts x x x curing defects and adding to the means of enforcing existing obligations x x x (and) are intended to supply defects, abridge superfluities in existing laws, and curb certain evils x x x By their very nature, curative statutes are retroactive xxx (and) reach back to past events to correct errors or irregularities and to render valid and effective attempted acts which would be otherwise ineffective for the purpose the parties intended." On the other hand, remedial or procedural laws, i.e., those statutes relating to remedies or modes of procedure, which do not create new or take away vested rights, but only operate in furtherance of the remedy or confirmation of such rights, ordinarily do not come within the legal meaning of a retrospective law, nor within the general rule against the retrospective operation of statutes.43 A reading of P.D. 725 immediately shows that it creates a new right, and also provides for a new remedy, thereby filling certain voids in our laws. Thus, in its preamble, P.D. 725 expressly recognizes the plight of "many Filipino women (who) had lost their Philippine citizenship by marriage to aliens" and who could not, under the existing law (C. A. No. 63, as amended) avail of repatriation until "after the death of their husbands or the termination of their marital status" and who could neither be benefitted by the 1973 Constitution's new provision allowing "a Filipino woman who marries an alien to retain her Philippine citizenship xxx" because "such provision of the new Constitution does not apply to Filipino women who had married aliens before said constitution took effect." Thus, P.D. 725 granted a new right to these womenthe right to re-acquire Filipino citizenship even during their marital coverture, which right did not exist prior to P.D. 725. On the other hand, said statute also provided a new remedy and a new right in favor of other "natural born Filipinos who (had) lost their Philippine citizenship but now desire to reacquire Philippine citizenship," because prior to the promulgation of P.D. 725 such former Filipinos would have had to undergo the tedious and cumbersome process of naturalization, but with the advent of P.D. 725 they could now re-acquire their Philippine citizenship under the simplified procedure of repatriation. The Solicitor General44 argues: "By their very nature, curative statutes are retroactive, (DBP vs. CA, 96 SCRA 342), since they are intended to supply defects, abridge superfluities in existing laws (Del Castillo vs. Securities and Exchange Commission, 96 Phil. 119) and curb certain evils (Santos vs. Duata, 14 SCRA 1041). In this case, P.D. No. 725 was enacted to cure the defect in the existing naturalization law, specifically C. A. No. 63 wherein married Filipino women are allowed to repatriate only upon the death of their husbands, and natural-born Filipinos who lost their citizenship by naturalization and other causes faced the difficulty of undergoing the rigid procedures of C.A. 63 for reacquisition of Filipino citizenship by naturalization. Presidential Decree No. 725 provided a remedy for the aforementioned legal aberrations and thus its provisions are considered essentially remedial and curative." In light of the foregoing, and prescinding from the wording of the preamble, it is unarguable that the legislative intent was precisely to give the statute retroactive operation. "(A) retrospective operation is given to a statute or amendment where the intent that it should so operate clearly appears from a consideration of the act as a whole, or from the terms thereof." 45 It is obvious to the Court that the statute was meant to "reach back" to those persons, events and transactions not otherwise covered by prevailing law and jurisprudence. And inasmuch as it has been held that citizenship is a political and civil right equally as important as the freedom of speech, liberty of abode, the right against unreasonable searches and seizures and other guarantees enshrined in the Bill of Rights, therefore the legislative intent to give retrospective operation to P.D. 725 must be given the fullest effect possible. "(I)t has been said that a remedial statute must be so construed as to make it effect the evident purpose for -which it was enacted, so that if the reason of the statute extends to past transactions, as well as to those in the future, then it will be so applied although the statute does not in terms so direct, unless to do so would impair some vested right or violate some constitutional guaranty."46 This is all the more true of P.D. 725, which did not specify any restrictions on or delimit or qualify the right of repatriation granted therein. At this point, a valid question may be raised: How can the retroactivity of P.D. 725 benefit Frivaldo considering that said law was enacted on June 5,1975, while Frivaldo lost his Filipino citizenship much later, on January 20, 1983, and applied for repatriation even later, on August 17, 1994?

While it is true that the law was already in effect at the time that Frivaldo became an American citizen, nevertheless, it is not only the law itself (P.D. 725) which is tobe given retroactive effect, but even the repatriation granted under said law to Frivaldo on June 30, 1995 is to be deemed to have retroacted to the date of his application therefor, August 17, 1994. The reason for this is simply that if, as in this case, it was the intent of the legislative authority that the law should apply to past events i.e., situations and transactions existing even before the law came into being in order to benefit the greatest number of former Filipinos possible thereby enabling them to enjoy and exercise the constitutionally guaranteed right of citizenship, and such legislative intention is to be given the fullest effect and expression, then there is all the more reason to have the law apply in a retroactive or retrospective manner to situations, events and transactions subsequent to the passage of such law. That is, the repatriation granted to Frivaldo on June 30, 1995 can and should be made to take effect as of date of his application. As earlier mentioned, there is nothing in the law that would bar this or would show a contrary intention on the part of the legislative authority; and there is no showing that damage or prejudice to anyone, or anything unjust or injurious would result from giving retroactivity to his repatriation. Neither has Lee shown that there will result the impairment of any contractual obligation, disturbance of any vested right or breach of some constitutional guaranty. Being a former Filipino who has served the people repeatedly, Frivaldo deserves a liberal interpretation of Philippine laws and whatever defects there were in his nationality should now be deemed mooted by his repatriation. Another argument for retroactivity to the date of filing is that it would prevent prejudice to applicants. If P.D. 725 were not to be given retroactive effect, and the Special Committee decides not to act, i.e., to delay the processing of applications for any substantial length of time, then the former Filipinos who may be stateless, as Frivaldohaving already renounced his American citizenship was, may be prejudiced for causes outside their control. This should not be. In case of doubt in the interpretation or application of laws, it is to be presumed that the lawmaking body intended right and justice to prevail.47 And as experience will show, the Special Committee was able to process, act upon and grant applications for repatriation within relatively short spans of time after the same were filed.48 The fact that such interregna were relatively insignificant minimizes the likelihood of prejudice to the government as a result of giving retroactivity to repatriation. Besides, to the mind of the Court, direct prejudice to the government is possible only where a person's repatriation has the effect of wiping out a liability of his to the government arising in connection with or as a result of his being an alien, and accruing only during the interregnum between application and approval, a situation that is not present in the instant case. And it is but right and just that the mandate of the people, already twice frustrated, should now prevail. Under the circumstances, there is nothing unjust or iniquitous in treating Frivaldo's repatriation as having become effective as of the date of his application, i.e., on August 17, 1994. This being so, all questions about his possession of the nationality qualification whether at the date of proclamation (June 30, 1995) or the date of election (May 8, 1995) or date of filing his certificate of candidacy (March 20, 1995) would become moot. Based on the foregoing, any question regarding Frivaldo's status as a registered voter would also be deemed settled. Inasmuch as he is considered as having been repatriatedi.e., his Filipino citizenship restored as of August 17, 1994, his previous registration as a voter is likewise deemed validated as of said date. It is not disputed that on January 20, 1983 Frivaldo became an American. Would the retroactivity of his repatriation not effectively give him dual citizenship, which under Sec. 40 of the Local Government Code would disqualify him "from running for any elective local position?"49 We answer this question in the negative, as there is cogent reason to hold that Frivaldo was really STATELESS at the time he took said oath of allegiance and even before that, when he ran for governor in 1988. In his Comment, Frivaldo wrote that he "had long renounced and had long abandoned his American citizenshiplong beforeMay 8, 1995. At best, Frivaldo was stateless in the interim when he abandoned and renounced his US citizenship but before he was repatriated to his Filipino citizenship."50 On this point, we quote from the assailed Resolution dated December 19, 1995:51 "By the laws of the United States, petitioner Frivaldo lost his American citizenship when he took his oath of allegiance to the Philippine Government when he ran for Governor in 1988, in 1992, and in 1995. Every certificate of candidacy contains an oath of allegiance to the Philippine Government." These factual findings that Frivaldo has lost his foreign nationality long before the elections of 1995 have not been effectively rebutted by Lee. Furthermore, it is basic that such findings of the Commission are conclusive upon this Court, absent any showing of capriciousness or arbitrariness or abuse.52

The Second Issue: Is Lack of Citizenship a Continuing Disqualification? Lee contends that the May 1,1995 Resolution53 of the Comelec Second Division in SPA No. 95-028 as affirmed in toto by Comelec En Banc in its Resolution of May 11, 1995 "became final and executory after five (5) days or on May 17,1995, no restraining order having been issued by this Honorable Court."54 Hence, before Lee "was proclaimed as the elected governor on June 30, 1995, there was already a final and executory judgment disqualifying" Frivaldo. Lee adds that this Court's two rulings (which Frivaldo now concedes were legally "correct") declaring Frivaldo an alien have also become final and executory way before the 1995 elections, and these "judicial pronouncements of his political status as an American citizen absolutely and for all time disqualified (him) from running for, and holding any public office in the Philippines." We do not agree. It should be noted that our first ruling in G.R. No. 87193 disqualifying Frivaldo was rendered in connection with the 1988 elections while that in G.R. No. 104654 was in connection with the 1992 elections. That he was disqualified for such elections is final and can no longer be changed. In the words of the respondent Commission (Second Division) in its assailed Resolution:55 "The records show that the Honorable Supreme Court had decided that Frivaldo was not a Filipino citizen and thus disqualified for the purpose of the 1988 and 1992 elections. However, there is no record of any 'final judgment' of the disqualification of Frivaldo as a candidate for the May 8, 1995 elections. What the Commission said in its Order of June 21, 1995 (implemented on June 30, 1995), directing the proclamation of Raul

R. Lee, was that Frivaldo was not a Filipino citizen 'having been declared by the Supreme Court in its Order dated March 25, 1995, not a citizen of the Philippines.' This declaration of the Supreme Court, however, was in connection with the 1992 elections." Indeed, decisions declaring the acquisition or denial of citizenship cannot govern a person's future status with finality. This is because a person may subsequently reacquire, or for that matter lose, his citizenship under any of the modes recognized by law for the purpose. Hence, in Lee vs. Commissioner of Immigration,56 we held: "Everytime the citizenship of a person is material or indispensable in a judicial or administrative case, whatever the corresponding court or administrative authority decides therein as to such citizenship is generally not considered res judicata, hence it has to be threshed out again and again, as the occasion demands." The Third Issue: Comelec's Jurisdiction Over The Petition in SPC No. 95-317 Lee also avers that respondent Comelec had no jurisdiction to entertain the petition in SPC No. 95-317 because the only "possible types of proceedings that may be entertained by the Comelec are a pre-proclamation case, an election protest or a quo warranto case." Again, Lee reminds us that he was proclaimed on June 30, 1995 but that Frivaldo filed SPC No. 95-317 questioning his (Lee's) proclamation only on July 6, 1995 "beyond the 5-day reglementary period." Hence, according to him, Frivaldo's "recourse was to file either an election protest or a quo warranto action." This argument is not meritorious. The Constitution57 has given the Comelec ample power to "exercise exclusive original jurisdiction over all contests relating to the elections, returns and qualifications of all elective x x x provincial x x x officials." Instead of dwelling at length on the various petitions that Comelec, in the exercise of its constitutional prerogatives, may entertain, suffice it to say that this Court has invariably recognized the Commission's authority to hear and decide petitions for annulment of proclamations of which SPC No. 95-317 obviously is one.58 Thus, in Mentang vs. COMELEC,59 we ruled: "The petitioner argues that after proclamation and assumption of office, a pre-proclamation controversy is no longer viable. Indeed, we are aware of cases holding that pre-proclamation controversies may no longer be entertained by the COMELEC after the winning candidate has been proclaimed, (citing Gallardo vs. Rimando, 187 SCRA 463; Salvacion vs. COMELEC, 170 SCRA 513; Casimiro vs.COMELEC, 171 SCRA 468.) This rule, however, is premised on an assumption that the proclamation is no proclamation at all and the proclaimed candidate's assumption of office cannot deprive the COMELEC of the power to make such declaration of nullity. (citing Aguam vs. COMELEC, 23 SCRA 883; Agbayani vs. COMELEC, 186 SCRA 484.)" The Court however cautioned that such power to annul a proclamation must "be done within ten (10) days following the proclamation." Inasmuch as Frivaldo's petition was filed only six (6) days after Lee's proclamation, there is no question that the Comelec correctly acquired jurisdiction over the same.

The Fourth Issue: Was Lee's Proclamation Valid Frivaldo assails the validity of the Lee proclamation. We uphold him for the following reasons: First. To paraphrase this Court in Labo vs. COMELEC,60 "the fact remains that he (Lee) was not the choice of the sovereign will," and in Aquino vs. COMELEC,61 Lee is "a second placer, xxx just that, a second placer." In spite of this, Lee anchors his claim to the governorship on the pronouncement of this Court in the aforesaid Labo62 case, as follows: "The rule would have been different if the electorate fully aware in fact and in law of a candidate's disqualification so as to bring such awareness within the realm of notoriety, would nonetheless cast their votes in favor of the ineligible candidate. In such case, the electorate may be said to have waived the validity and efficacy of their votes by notoriously misapplying their franchise or throwing away their votes, in which case, the eligible candidate obtaining the next higher number of votes may be deemed elected." But such holding is qualified by the next paragraph, thus: "But this is not the situation obtaining in the instant dispute. It has not been shown, and none was alleged, that petitioner Labo was notoriously known as an ineligible candidate, much less the electorate as having known of such fact. On the contrary, petitioner Labo was even allowed by no less than the Comelec itself in its resolution dated May 10, 1992 to be voted for the office of the city mayor as its resolution dated May 9,1992 denying due course to petitioner Labo's certificate of candidacy had not yet become final and subject to the final outcome of this case." The last-quoted paragraph in Labo, unfortunately for Lee, is the ruling appropriate in this case because Frivaldo was in 1995 in an identical situation as Labo was in 1992 when the Comelec's cancellation of his certificate of candidacy was not yet final on election day as there was in both cases a pending motion for reconsideration, for which reason Comelec issued an (omnibus) resolution declaring that Frivaldo (like Labo in 1992) and several others can still be voted for in the May 8, 1995 election, as in fact, he was. Furthermore, there has been no sufficient evidence presented to show that the electorate of Sorsogon was "fully aware in fact and in law" of Frivaldo's alleged disqualification as to "bring such awareness within the realm of notoriety", in other words, that the voters intentionally wasted their ballots knowing that, in spite of their voting for him, he was ineligible. If Labo has any relevance at all, it is that the vice-governor and not Leeshould be proclaimed, since in losing the election, Lee was, to paraphrase Labo again, "obviously not the choice of the people" of Sorsogon. This is the emphatic teaching of Labo:

"The rule, therefore, is: the ineligibility of a candidate receiving majority votes does not entitle the eligible candidate receiving the next highest number of votes to be declared elected. A minority or defeated candidate cannot be deemed elected to the office." Second. As we have earlier declared Frivaldo to have seasonably re-acquired his citizenship and inasmuch as he obtained the highest number of votes in the 1995 elections, henot Lee should be proclaimed. Hence, Lee's proclamation was patently erroneous and should now be corrected.

The Fifth Issue: Is Section 78 of the Election Code Mandatory? In G.R. No. 120295, Frivaldo claims that the assailed Resolution of the Comelec (Second Division) dated May 1, 1995 and the confirmatory en banc Resolution of May 11, 1995 disqualifying him for want of citizenship should be annulled because they were rendered beyond the fifteen (15) day period prescribed by Section 78 of the Omnibus Election Code which reads as follows: "Section 78. Petition to deny due course or to cancel a certificate of candidacy. A verified petition seeking to deny due course or to cancel a certificate of candidacy may be filed by any person exclusively on the ground that any material representation contained therein as required under Section 74 hereof is false. The petition may be filed at any time not later than twenty-five days from the time of the filing of the certificate of candidacy and shall be decided after notice and hearing, not later than fifteen days before the election" (italics supplied.) This claim is now moot and academic inasmuch as these resolutions are deemed superseded by the subsequent ones issued by the Commission (First Division) on December 19, 1995, affirmed en banc63 on February 23, 1996, which both upheld his election. At any rate, it is obvious that Section 78 is merely directory as Section 6 of R.A. No. 6646 authorizes the Commission to try and decide petitions for disqualifications even after the elections, thus: "SEC. 6. Effect of Disqualification Case. Any candidate who has been declared by final judgment to be disqualified shall not be voted for, and the votes cast for him shall not be counted. If for any reason a candidate is not declared by final judgment before an election to be disqualified and he is voted for and receives the -winning number of votes in such election, the Court or Commission shall continue with the trial and hearing of the action, inquiry or protest and, upon motion of the complainant or any intervenor, may during the pendency thereof order the suspension of the proclamation of such candidate whenever the evidence of his guilt is strong." (Italics supplied)

Refutation of Mr. Justice Davide's Dissent In his dissenting opinion, the esteemed Mr. Justice Hilario G. Davide, Jr. argues that President Aquino's memorandum dated March 27, 1987 should be viewed as a suspension (not a repeal, as urged by Lee) of P.D. 725. But whether it decrees a suspension or a repeal is a purely academic distinction because the said issuance is not a statute that can amend or abrogate an existing law. The existence and subsistence of P.D. 725 were recognized in the first Frivaldo case;64 viz, "(u)nder CA No. 63 as amended by CA No. 473 and P.D. No. 725, Philippine citizenship maybe reacquired by xxx repatriation" He also contends that by allowing Frivaldo to register and to remain as a registered voter, the Comelec and in effect this Court abetted a "mockery" of our two previous judgments declaring him a non-citizen. We do not see such abetting or mockery. The retroactivity of his repatriation, as discussed earlier, legally cured whatever defects there may have been in his registration as a voter for the purpose of the 1995 elections. Such retroactivity did not change his disqualifications in 1988 and 1992, which were the subjects of such previous rulings. Mr. Justice Davide also believes that Quo Warranto is not the sole remedy to question the ineligibility of a candidate, citing the Comelec's authority under Section 78 of the Omnibus Election Code allowing the denial of a certificate of candidacy on the ground of a false material representation therein as required by Section 74. Citing Loong, he then states his disagreement with our holding that Section 78 is merely directory. We really have no quarrel. Our point is that Frivaldo was in error in his claim in G.R. No. 120295 that the Comelec Resolutions promulgated on May 1, 1995 and May 11, 1995 were invalid because they were issued "not later than fifteen days before the election" as prescribed by Section 78. In dismissing the petition in G.R. No. 120295, we hold that the Comelec did not commit grave abuse of discretion because "Section 6 of R. A. 6646 authorizes the Comelec to try and decide disqualifications even after the elections." In spite of his disagreement with us on this point, i.e., that Section 78 "is merely directory," we note that just like us, Mr. Justice Davide nonetheless votes to "DISMISS G.R. No. 120295." One other point. Loong, as quoted in the dissent, teaches that a petition to deny due course under Section 78 must be filedwithin the 25-day period prescribed therein. The present case however deals with the period during which the Comelec may decide such petition. And we hold that it may be decided even after the fifteen day period mentioned in Section 78. Here, we rule that a decision promulgated by the Comelec even after the elections is valid but Loong held that a petitionfiled beyond the 25-day period is out of time. There is no inconsistency nor conflict. Mr. Justice Davide also disagrees with the Court's holding that, given the unique factual circumstances of Frivaldo, repatriation may be given retroactive effect. He argues that such retroactivity "dilutes" our holding in the first Frivaldo case. But the first (and even the second Frivaldo) decision did not directly involve repatriation as a mode of acquiring citizenship. If we may repeat, there is no question that Frivaldo was not a Filipino for purposes of determining his qualifications in the 1988 and 1992 elections. That is settled. But his supervening repatriation has changed his political status not in 1988 or 1992, but only in the 1995 elections. Our learned colleague also disputes our holding that Frivaldo was stateless prior to his repatriation, saying that "informal renunciation or abandonment is not a ground to lose American citizenship." Since our courts are charged only with the duty of the determining who are Philippine nationals, we cannot rule on the legal question of who are or who are not Americans. It is basic in international law that a State determines ONLY those who are its own citizens not who are the citizens of other countries.65 The issue here is: the Comelec made a finding of fact that Frivaldo was stateless and such finding has not been shown by Lee to be arbitrary or whimsical. Thus, following settled case law, such finding is binding and final. The dissenting opinion also submits that Lee who lost by chasmic margins to Frivaldo in all three previous elections, should be declared winner because "Frivaldo's ineligibility for being an American was publicly known." First, there is absolutely no empirical evidence for such

"public" knowledge. Second, even if there is, such knowledge can be true post factoonly of the last two previous elections. Third, even the Comelec and now this Court were/are still deliberating on his nationality before, during and after the 1995 elections. How then can there be such "public" knowledge? Mr. Justice Davide submits that Section 39 of the Local Government Code refers to the qualifications of elective local officials, i.e., candidates, and not elected officials, and that the citizenship qualification [under par. (a) of that section] must be possessed by candidates, not merely at the commencement of the term, but by election day at the latest. We see it differently. Section 39, par. (a) thereof speaks of "elective local official" while par. (b) to (f) refer to "candidates." If the qualifications under par. (a) were intended to apply to "candidates" and not elected officials, the legislature would have said so, instead of differentiating par. (a) from the rest of the paragraphs. Secondly, if Congress had meant that the citizenship qualification should be possessed at election day or prior thereto, it would have specifically stated such detail, the same way it did in pars. (b) to (f) for other qualifications of candidates for governor, mayor, etc. Mr. Justice Davide also questions the giving of retroactive effect to Frivaldo's repatriation on the ground, among others, that the law specifically provides that it is only after taking the oath of allegiance that applicants shall be deemed to have reacquired Philippine citizenship. We do not question what the provision states. We hold however that the provision should be understood thus: that after taking the oath of allegiance the applicant is deemed to have reacquired Philippine citizenship, which reacquisition (or repatriation) is deemed for all purposes and intents to have retroacted to the date of his application therefor. In any event, our "so too" argument regarding the literal meaning of the word "elective" in reference to Section 39 of the Local Government Code, as well as regarding Mr. Justice Davide's thesis that the very wordings of P.D. 725 suggest non-retroactivity, were already taken up rather extensively earlier in this Decision. Mr. Justice Davide caps his paper with a clarion call: "This Court must be the first to uphold the Rule of Law." We agree we must all follow the rule of law. But that is NOT the issue here. The issue is how should the law be interpreted and applied in this case so it can be followed, so it can rule! At balance, the question really boils down to a choice of philosophy and perception of how to interpret and apply laws relating to elections: literal or liberal; the letter or the spirit; the naked provision or its ultimate purpose; legal syllogism or substantial justice; in isolation or in the context of social conditions; harshly against or gently in favor of the voters' obvious choice. In applying election laws, it would be far better to err in favor of popular sovereignty than to be right in complex but little understood legalisms. Indeed, to inflict a thrice rejected candidate upon the electorate of Sorsogon would constitute unmitigated judicial tyranny and an unacceptable assault upon this Court's conscience.

EPILOGUE In sum, we rule that the citizenship requirement in the Local Government Code is to be possessed by an elective official at the latest as of the time he is proclaimed and at the start of the term of office to which he has been elected. We further hold P.D. No. 725 to be in full force and effect up to the present, not having been suspended or repealed expressly nor impliedly at any time, and Frivaldo's repatriation by virtue thereof to have been properly granted and thus valid and effective. Moreover, by reason of the remedial or curative nature of the law granting him a new right to resume his political status and the legislative intent behind it, as well as his unique situation of having been forced to give up his citizenship and political aspiration as his means of escaping a regime he abhorred, his repatriation is to be given retroactive effect as of the date of his application therefor, during the pendency of which he was stateless, he having given ' up his U. S. nationality. Thus, in contemplation of law, he possessed the vital requirement of Filipino citizenship as of the start of the term of office of governor, and should have been proclaimed instead of Lee. Furthermore, since his reacquisition of citizenship retroacted to August 17, 1994, his registration as a voter of Sorsogon is deemed to have been validated as of said date as well. The foregoing, of course, are precisely consistent with our holding that lack of the citizenship requirement is not a continuing disability or disqualification to run for and hold public office. And once again, we emphasize herein our previous rulings recognizing the Comelec's authority and jurisdiction to hear and decide petitions for annulment of proclamations. This Court has time and again liberally and equitably construed the electoral laws of our country to give fullest effect to the manifest will of our people,66 for in case of doubt, political laws must be interpreted to give life and spirit to the popular mandate freely expressed through the ballot. Otherwise stated, legal niceties and technicalities cannot stand in the way of the sovereign will. Consistently, we have held: "x x x (L)aws governing election contests must be liberally construed to the end that the will of the people in the choice of public officials may not be defeated by mere technical objections (citations omitted)."67 The law and the courts must accord Frivaldo every possible protection, defense and refuge, in deference to the popular will. Indeed, this Court has repeatedly stressed the importance of giving effect to the sovereign will in order to ensure the survival of our democracy. In any action involving the possibility of a reversal of the popular electoral choice, this Court must exert utmost effort to resolve the issues in a manner that would give effect to the will of the majority, for it is merely sound public policy to cause elective offices to be filled by those who are the choice of the majority. To successfully challenge a winning candidate's qualifications, the petitioner must clearly demonstrate that the ineligibility is so patently antagonistic68 to constitutional and legal principles that overriding such ineligibility and thereby giving effect to the apparent will of the people, would ultimately create greater prejudice to the very democratic institutions and juristic traditions that our Constitution and laws so zealously protect and promote. In this undertaking, Lee has miserably failed. In Frivaldo's case, it would have been technically easy to find fault with his cause. The Court could have refused to grant retroactivity to the effects of his repatriation and hold him still ineligible due to his failure to show his citizenship at the time he registered as a voter before the 1995 elections. Or, it could have disputed the factual findings of the Comelec that he was stateless at the time of repatriation and thus hold his consequent dual citizenship as a disqualification "from running for any elective local position." But the real essence of justice does not emanate from quibblings over patchwork legal technicality. It proceeds from the spirit's gut consciousness of the dynamic role of law as a brick in the ultimate development of the social edifice. Thus, the Court struggled against and eschewed the easy, legalistic, technical and sometimes harsh anachronisms of the law in order to evoke substantial justice in the larger social context consistent with Frivaldo's unique situation approximating venerability in Philippine political life. Concededly, he sought American citizenship only to escape the clutches of the dictatorship. At this stage, we cannot seriously entertain any doubt about his loyalty and dedication to this country. At the first opportunity, he

returned to this land, and sought to serve his people once more. The people of Sorsogon overwhelmingly voted for him three times. He took an oath of allegiance to this Republic every time he filed his certificate of candidacy and during his failed naturalization bid. And let it not be overlooked, his demonstrated tenacity and sheer determination to re-assume his nationality of birth despite several legal set-backs speak more loudly, in spirit, in fact and in truth than any legal technicality, of his consuming intention and burning desire to re-embrace his native Philippines even now at the ripe old age of 81 years. Such loyalty to and love of country as well as nobility of purpose cannot be lost on this Court of justice and equity. Mortals of lesser mettle would have given up. After all, Frivaldo was assured of a life of ease and plenty as a citizen of the most powerful country in the world. But he opted, nay, single-mindedly insisted on returning to and serving once more his struggling but beloved land of birth. He therefore deserves every liberal interpretation of the law which can be applied in his favor. And in the final analysis, over and above Frivaldo himself, the indomitable people of Sorsogon most certainly deserve to be governed by a leader of their overwhelming choice. WHEREFORE, in consideration of the foregoing: (1) The petition in G.R. No. 123755 is hereby DISMISSED. The assailed Resolutions of the respondent Commission are AFFIRMED. (2) The petition in G.R. No. 120295 is also DISMISSED for being moot and academic. In any event, it has no merit. No costs. SO ORDERED.

Bengzon v. Cruz: On Repatriation

In the case of Bengson III v. House of Representatives Electoral Tribunal [1], the major issue was whether or not repatriation causes a person, formerly a natural-born Filipino, to regain his citizenship in its original state. Cruz was born in Tarlac in 1960 of Filipino parents, but lost his citizenship after enlisting in the US Marine Corps and later naturalized as a US citizen. He reacquired his citizenship through repatriation and ran for Congress. When he won, his opponent claimed that he was no longer a natural-born Filipino and therefore ineligible as a member of Congress. The Supreme Court ruled in favor of Cruz, stating that repatriation allows one to recover, or return to, his original status before he lost his Philippine citizenship. The 1987 Constitution defines natural-born citizens as those citizens of the Philippines without having to perform any act to acquire or perfect his Philippine citizenship. Naturalization is a mode for both acquisition and reacquisition of Philippine citizenship. Commonwealth Act No. 473 or the Revised Naturalization Law governs the process of naturalization to acquire citizenship. For reacquisition of citizenship, CA No. 63 is applicable. The applicant, under both laws, has to prove that he possesses all the qualifications and none of the disqualifications provided by law to become a Filipino citizen.[2] Repatriation may be used to reacquire citizenship for those who have lost it through (1) desertion of the armed forces ; (2) service in the Armed Forces of the United States at any other time; (3) marriage of a Filipino woman to an alien; and (4) political and economic necessity.[3] Aside from different requirements, naturalization and repatriation also have different processes. As distinguished from the lengthy process of naturalization, repatriation simply consists of the taking of an oath of allegiance to the Republic of the Philippines and registering said oath in the Local Civil Registry of the place where the person concerned resides or last resided.[4]Furthermore, repatriation results in the recovery of the original nationality.[5] Since citizenship may be classified as either natural-born or naturalized, and those who repatriated clearly do not fall under the process of naturalization, Cruz was considered a natural-born citizen. Though both naturalization and repatriation are ways by which a former Filipino

citizen may reacquire citizenship, repatriation is granted to a more limited number of people. More importance was given to those who have lost their citizenship through the desertion of the armed forces, service in the Armed Forces of the United States, marriage of a Filipino woman to a foreigner, and political or economic necessity. Due to this distinction made, it is understandable that these people have been given the privilege to regain their original citizenship without going through the more tedious process of naturalization. However, there is still some ambiguity, since the Constitution itself states that natural-born citizens are those who do not need to perform any other act to perfect or obtain their citizenship. Though reacquisition is not mentioned, it could be said that their citizenship, if not totally lost, was burdened by some imperfection, since they somehow gained allegiance to another country. Since they need to perform an act to perfect their citizenship once again, they should be considered naturalized and not natural born citizens. Justice Guttierez, in his dissent, said that the provision on natural born citizenship is precise, clear and definite, and should not be construed in any other way aside from what its plain meaning conveys. He said that it was an issue that should be left to Constitutional amendment. [6]

Republic of the Philippines SUPREME COURT Manila EN BANC

G.R. Nos. 92191-92 July 30, 1991 ANTONIO Y. CO, petitioner, vs. ELECTORAL TRIBUNAL OF THE HOUSE OF REPRESENTATIVES AND JOSE ONG, JR., respondents. G.R. Nos. 92202-03 July 30, 1991 SIXTO T. BALANQUIT, JR., petitioner, vs. ELECTORAL TRIBUNAL OF THE HOUSE OF REPRESENTATIVES AND JOSE ONG, JR., respondents. Hechanova & Associates for petitioner Co. Brillantes, Nachura, Navarro and Arcilla Law Offices for respondent Ong, Jr.

GUTIERREZ, JR., J.:p

The petitioners come to this Court asking for the setting aside and reversal of a decision of the House of Representatives Electoral Tribunal (HRET). The HRET declared that respondent Jose Ong, Jr. is a natural born Filipino citizen and a resident of Laoang, Northern Samar for voting purposes. The sole issue before us is whether or not, in making that determination, the HRET acted with grave abuse of discretion. On May 11, 1987, the congressional election for the second district of Northern Samar was held. Among the candidates who vied for the position of representative in the second legislative district of Northern Samar are the petitioners, Sixto Balinquit and Antonio Co and the private respondent, Jose Ong, Jr. Respondent Ong was proclaimed the duly elected representative of the second district of Northern Samar. The petitioners filed election protests against the private respondent premised on the following grounds: 1) Jose Ong, Jr. is not a natural born citizen of the Philippines; and 2) Jose Ong, Jr. is not a resident of the second district of Northern Samar. The HRET in its decision dated November 6, 1989, found for the private respondent. A motion for reconsideration was filed by the petitioners on November 12, 1989. This was, however, denied by the HRET in its resolution dated February 22, 1989. Hence, these petitions for certiorari. We treat the comments as answers and decide the issues raised in the petitions. ON THE ISSUE OF JURISDICTION The first question which arises refers to our jurisdiction. The Constitution explicitly provides that the House of Representatives Electoral Tribunal (HRET) and the Senate Electoral Tribunal (SET) shall be the sole judges of all contests relating to the election, returns, and qualificationsof their respective members. (See Article VI, Section 17, Constitution) The authority conferred upon the Electoral Tribunal is full, clear and complete. The use of the word soleemphasizes the exclusivity of the jurisdiction of these Tribunals. The Supreme Court in the case of Lazatin v. HRET (168 SCRA 391 [1988]) stated that under the 1987 Constitution, the jurisdiction of the Electoral Tribunal is original and exclusive, viz: The use of the word "sole" emphasizes the exclusive character of the jurisdiction conferred (Angara v. Electoral Commission, supra at p. 162). The exercise of power by the Electoral Commission under the 1935 Constitution has been described as "intended to be as complete and unimpaired as if it had originally remained in the legislature." (id., at p. 175) Earlier this grant of power to the legislature was characterized by Justice Malcolm as "full, clear and complete; (Veloso v. Board of Canvassers of Leyte and Samar, 39 Phil. 886 [1919]) Under the amended 1935 Constitution, the power was unqualifiedly reposed upon the Electoral Tribunal and it remained as full, clear and complete as that previously granted the Legislature and the Electoral Commission, (Lachica v. Yap, 25 SCRA 140 [1968]) The same may be said with regard to the jurisdiction of the Electoral Tribunal under the 1987 Constitution. (p. 401) The Court continued further, ". . . so long as the Constitution grants the HRET the power to be the sole judge of all contests relating to election, returns and qualifications of members of the House of Representatives, any final action taken by the HRET on a matter within its jurisdiction shall, as a rule, not be reviewed by this Court . . . the power granted to the Electoral Tribunal is full, clear and complete and excludes the exercise of any authority on the part of this Court that would in any wise restrict it or curtail it or even affect the same." (pp. 403-404) When may the Court inquire into acts of the Electoral Tribunals under our constitutional grants of power? In the later case of Robles v. HRET (181 SCRA 780 [1990]) the Supreme Court stated that the judgments of the Tribunal are beyond judicial interference save only "in the exercise of this Court's so-called extraordinary jurisdiction, . . . upon a determination that the Tribunal's decision or resolution was rendered without or in excess of its jurisdiction, or with grave abuse of discretion or paraphrasing Morrero, upon a clear showing of such arbitrary and improvident use by the Tribunal of its power as constitutes a denial of due process of law, or upon a demonstration of a very clear unmitigated ERROR, manifestly constituting such GRAVE ABUSE OF DISCRETION that there has to be a remedy for such abuse." (at pp. 785-786) In the leading case of Morrero v. Bocar (66 Phil. 429 [1938]) the Court ruled that the power of the Electoral Commission "is beyond judicial interference except, in any event, upon a clear showing of such arbitrary and improvident use of power as will constitute a denial of due process." The Court does not venture into the perilous area of trying to correct perceived errors of independent branches of the Government, It

comes in only when it has to vindicate a denial of due process or correct an abuse of discretion so grave or glaring that no less than the Constitution calls for remedial action. The Supreme Court under the 1987 Constitution, has been given an expanded jurisdiction, so to speak, to review the decisions of the other branches and agencies of the government to determine whether or not they have acted within the bounds of the Constitution. (See Article VIII, Section 1, Constitution) Yet, in the exercise thereof, the Court is to merely check whether or not the governmental branch or agency has gone beyond the Constitutional limits of its jurisdiction, not that it erred or has a different view. In the absence of a showing that the HRET has committed grave abuse of discretion amounting to lack of jurisdiction, there is no occasion for the Court to exercise its corrective power; it will not decide a matter which by its nature is for the HRET alone to decide. (See Marcos v. Manglapus, 177 SCRA 668 [1989]) It has no power to look into what it thinks is apparent error. As constitutional creations invested with necessary power, the Electoral Tribunals, although not powers in the tripartite scheme of the government, are, in the exercise of their functions independent organs independent of Congress and the Supreme Court. The power granted to HRET by the Constitution is intended to be as complete and unimpaired as if it had remained originally in the legislature. (Angara v. Electoral Commission, 63 Phil. 139 [1936]) In passing upon petitions, the Court with its traditional and careful regard for the balance of powers, must permit this exclusive privilege of the Tribunals to remain where the Sovereign authority has place it. (See Veloso v. Boards of Canvassers of Leyte and Samar, 39 Phil. 886 [1919]) It has been argued that under Article VI, Section 17 of the present Constitution, the situation may exist as it exists today where there is an unhealthy one-sided political composition of the two Electoral Tribunals. There is nothing in the Constitution, however, that makes the HRET because of its composition any less independent from the Court or its constitutional functions any less exclusive. The degree of judicial intervention should not be made to depend on how many legislative members of the HRET belong to this party or that party. The test remains the same-manifest grave abuse of discretion. In the case at bar, the Court finds no improvident use of power, no denial of due process on the part of the HRET which will necessitate the exercise of the power of judicial review by the Supreme Court. ON THE ISSUE OF CITIZENSHIP The records show that in the year 1895, the private respondent's grandfather, Ong Te, arrived in the Philippines from China. Ong Te established his residence in the municipality of Laoang, Samar on land which he bought from the fruits of hard work. As a resident of Laoang, Ong Te was able to obtain a certificate of residence from the then Spanish colonial administration. The father of the private respondent, Jose Ong Chuan was born in China in 1905. He was brought by Ong Te to Samar in the year 1915. Jose Ong Chuan spent his childhood in the province of Samar. In Laoang, he was able to establish an enduring relationship with his neighbors, resulting in his easy assimilation into the community. As Jose Ong Chuan grew older in the rural and seaside community of Laoang, he absorbed Filipino cultural values and practices. He was baptized into Christianity. As the years passed, Jose Ong Chuan met a natural born-Filipino, Agripina Lao. The two fell in love and, thereafter, got married in 1932 according to Catholic faith and practice. The couple bore eight children, one of whom is the private respondent who was born in 1948. The private respondent's father never emigrated from this country. He decided to put up a hardware store and shared and survived the vicissitudes of life in Samar. The business prospered. Expansion became inevitable. As a result, a branch was set-up in Binondo, Manila. In the meantime, the father of the private respondent, unsure of his legal status and in an unequivocal affirmation of where he cast his life and family, filed with the Court of First Instance of Samar an application for naturalization on February 15, 1954. On April 28, 1955, the CFI of Samar, after trial, declared Jose Ong Chuan a Filipino citizen. On May 15, 1957, the Court of First Instance of Samar issued an order declaring the decision of April 28, 1955 as final and executory and that Jose Ong Chuan may already take his Oath of Allegiance. Pursuant to said order, Jose Ong Chuan took his Oath of Allegiance; correspondingly, a certificate of naturalization was issued to him. At the time Jose Ong Chuan took his oath, the private respondent then a minor of nine years was finishing his elementary education in the province of Samar. There is nothing in the records to differentiate him from other Filipinos insofar as the customs and practices of the local populace were concerned. Fortunes changed. The house of the family of the private respondent in Laoang, Samar was burned to the ground.

Undaunted by the catastrophe, the private respondent's family constructed another one in place of their ruined house. Again, there is no showing other than that Laoang was their abode and home. After completing his elementary education, the private respondent, in search for better education, went to Manila in order to acquire his secondary and college education. In the meantime, another misfortune was suffered by the family in 1975 when a fire gutted their second house in Laoang, Samar. The respondent's family constructed still another house, this time a 16-door apartment building, two doors of which were reserved for the family. The private respondent graduated from college, and thereafter took and passed the CPA Board Examinations. Since employment opportunities were better in Manila, the respondent looked for work here. He found a job in the Central Bank of the Philippines as an examiner. Later, however, he worked in the hardware business of his family in Manila. In 1971, his elder brother, Emil, was elected as a delegate to the 1971 Constitutional Convention. His status as a natural born citizen was challenged. Parenthetically, the Convention which in drafting the Constitution removed the unequal treatment given to derived citizenship on the basis of the mother's citizenship formally and solemnly declared Emil Ong, respondent's full brother, as a natural born Filipino. The Constitutional Convention had to be aware of the meaning of natural born citizenship since it was precisely amending the article on this subject. The private respondent frequently went home to Laoang, Samar, where he grew up and spent his childhood days. In 1984, the private respondent married a Filipina named Desiree Lim. For the elections of 1984 and 1986, Jose Ong, Jr. registered himself as a voter of Laoang, Samar, and correspondingly, voted there during those elections. The private respondent after being engaged for several years in the management of their family business decided to be of greater service to his province and ran for public office. Hence, when the opportunity came in 1987, he ran in the elections for representative in the second district of Northern Samar. Mr. Ong was overwhelmingly voted by the people of Northern Samar as their representative in Congress. Even if the total votes of the two petitioners are combined, Ong would still lead the two by more than 7,000 votes. The pertinent portions of the Constitution found in Article IV read: SECTION 1, the following are citizens of the Philippines: 1. Those who are citizens of the Philippines at the time of the adoption of the Constitution; 2. Those whose fathers or mothers are citizens of the Philippines; 3. Those born before January 17, 1973, of Filipino mothers, who elect Philippine citizenship upon reaching the age of majority; and 4. Those who are naturalized in accordance with law. SECTION 2, Natural-born Citizens are those who are citizens of the Philippines from birth without having to perform any act to acquire or perfect their citizenship. Those who elect Philippine citizenship in accordance with paragraph 3 hereof shall be deemed natural-born citizens. The Court interprets Section 1, Paragraph 3 above as applying not only to those who elect Philippine citizenship after February 2, 1987 but also to those who, having been born of Filipino mothers, elected citizenship before that date. The provision in Paragraph 3 was intended to correct an unfair position which discriminates against Filipino women. There is no ambiguity in the deliberations of the Constitutional Commission, viz: Mr. Azcuna: With respect to the provision of section 4, would this refer only to those who elect Philippine citizenship after the effectivity of the 1973 Constitution or would it also cover those who elected it under the 1973 Constitution? Fr. Bernas: It would apply to anybody who elected Philippine citizenship by virtue of the provision of the 1935 Constitution whether the election was done before or after January 17, 1973. (Records of the Constitutional Commission, Vol. 1, p. 228; Emphasis supplied) xxx xxx xxx Mr. Trenas: The Committee on Citizenship, Bill of Rights, Political Rights and Obligations and Human Rights has more or less decided to extend the interpretation of who is a natural-born citizen as provided in section 4 of the 1973 Constitution by adding that persons who have elected

Philippine Citizenship under the 1935 Constitution shall be natural-born? Am I right Mr. Presiding Officer? Fr. Bernas: yes. xxx xxx xxx Mr. Nolledo: And I remember very well that in the Reverend Father Bernas' well written book, he said that the decision was designed merely to accommodate former delegate Ernesto Ang and that the definition on natural-born has no retroactive effect. Now it seems that the Reverend Father Bernas is going against this intention by supporting the amendment? Fr. Bernas: As the Commissioner can see, there has been an evolution in my thinking. (Records of the Constitutional Commission, Vol. 1, p. 189) xxx xxx xxx Mr. Rodrigo: But this provision becomes very important because his election of Philippine citizenship makes him not only a Filipino citizen but a natural-born Filipino citizen entitling him to run for Congress. . . Fr. Bernas: Correct. We are quite aware of that and for that reason we will leave it to the body to approve that provision of section 4. Mr. Rodrigo: I think there is a good basis for the provision because it strikes me as unfair that the Filipino citizen who was born a day before January 17, 1973 cannot be a Filipino citizen or a natural-born citizen. (Records of the Constitutional Commission, Vol. 1, p. 231) xxx xxx xxx Mr. Rodrigo: The purpose of that provision is to remedy an inequitable situation. Between 1935 and 1973 when we were under the 1935 Constitution, those born of Filipino fathers but alien mothers were natural-born Filipinos. However, those born of Filipino mothers but alien fathers would have to elect Philippine citizenship upon reaching the age of majority; and if they do elect, they become Filipino citizens but not natural-born Filipino citizens. (Records of the Constitutional Commission, Vol. 1, p. 356) The foregoing significantly reveals the intent of the framers. To make the provision prospective from February 3, 1987 is to give a narrow interpretation resulting in an inequitable situation. It must also be retroactive. It should be noted that in construing the law, the Courts are not always to be hedged in by the literal meaning of its language. The spirit and intendment thereof, must prevail over the letter, especially where adherence to the latter would result in absurdity and injustice. (Casela v. Court of Appeals, 35 SCRA 279 [1970]) A Constitutional provision should be construed so as to give it effective operation and suppress the mischief at which it is aimed, hence, it is the spirit of the provision which should prevail over the letter thereof. (Jarrolt v. Mabberly, 103 U.S. 580) In the words of the Court in the case of J.M. Tuason v. LTA (31 SCRA 413 [1970]: To that primordial intent, all else is subordinated. Our Constitution, any constitution is not to be construed narrowly or pedantically for the prescriptions therein contained, to paraphrase Justice Holmes, are not mathematical formulas having their essence in their form but are organic living institutions, the significance of which is vital not formal. . . . (p. 427) The provision in question was enacted to correct the anomalous situation where one born of a Filipino father and an alien mother was automatically granted the status of a natural-born citizen while one born of a Filipino mother and an alien father would still have to elect Philippine citizenship. If one so elected, he was not, under earlier laws, conferred the status of a natural-born. Under the 1973 Constitution, those born of Filipino fathers and those born of Filipino mothers with an alien father were placed on equal footing. They were both considered as natural-born citizens. Hence, the bestowment of the status of "natural-born" cannot be made to depend on the fleeting accident of time or result in two kinds of citizens made up of essentially the same similarly situated members. It is for this reason that the amendments were enacted, that is, in order to remedy this accidental anomaly, and, therefore, treat equally all those born before the 1973 Constitution and who elected Philippine citizenship either before or after the effectivity of that Constitution. The Constitutional provision in question is, therefore curative in nature. The enactment was meant to correct the inequitable and absurd situation which then prevailed, and thus, render those acts valid which would have been nil at the time had it not been for the curative provisions. (See Development Bank of the Philippines v. Court of Appeals, 96 SCRA 342 [1980])

There is no dispute that the respondent's mother was a natural born Filipina at the time of her marriage. Crucial to this case is the issue of whether or not the respondent elected or chose to be a Filipino citizen. Election becomes material because Section 2 of Article IV of the Constitution accords natural born status to children born of Filipino mothers before January 17, 1973, if they elect citizenship upon reaching the age of majority. To expect the respondent to have formally or in writing elected citizenship when he came of age is to ask for the unnatural and unnecessary. The reason is obvious. He was already a citizen. Not only was his mother a natural born citizen but his father had been naturalized when the respondent was only nine (9) years old. He could not have divined when he came of age that in 1973 and 1987 the Constitution would be amended to require him to have filed a sworn statement in 1969 electing citizenship inspite of his already having been a citizen since 1957. In 1969, election through a sworn statement would have been an unusual and unnecessary procedure for one who had been a citizen since he was nine years old. We have jurisprudence that defines "election" as both a formal and an informal process. In the case of In Re: Florencio Mallare (59 SCRA 45 [1974]), the Court held that the exercise of the right of suffrage and the participation in election exercises constitute a positive act of election of Philippine citizenship. In the exact pronouncement of the Court, we held: Esteban's exercise of the right of suffrage when he came of age, constitutes a positive act of election of Philippine citizenship (p. 52; emphasis supplied) The private respondent did more than merely exercise his right of suffrage. He has established his life here in the Philippines. For those in the peculiar situation of the respondent who cannot be expected to have elected citizenship as they were already citizens, we apply the In Re Mallare rule. The respondent was born in an outlying rural town of Samar where there are no alien enclaves and no racial distinctions. The respondent has lived the life of a Filipino since birth. His father applied for naturalization when the child was still a small boy. He is a Roman Catholic. He has worked for a sensitive government agency. His profession requires citizenship for taking the examinations and getting a license. He has participated in political exercises as a Filipino and has always considered himself a Filipino citizen. There is nothing in the records to show that he does not embrace Philippine customs and values, nothing to indicate any tinge of alien-ness no acts to show that this country is not his natural homeland. The mass of voters of Northern Samar are frilly aware of Mr. Ong's parentage. They should know him better than any member of this Court will ever know him. They voted by overwhelming numbers to have him represent them in Congress. Because of his acts since childhood, they have considered him as a Filipino. The filing of sworn statement or formal declaration is a requirement for those who still have to elect citizenship. For those already Filipinos when the time to elect came up, there are acts of deliberate choice which cannot be less binding. Entering a profession open only to Filipinos, serving in public office where citizenship is a qualification, voting during election time, running for public office, and other categorical acts of similar nature are themselves formal manifestations of choice for these persons. An election of Philippine citizenship presupposes that the person electing is an alien. Or his status is doubtful because he is a national of two countries. There is no doubt in this case about Mr. Ong's being a Filipino when he turned twenty-one (21). We repeat that any election of Philippine citizenship on the part of the private respondent would not only have been superfluous but it would also have resulted in an absurdity. How can a Filipino citizen elect Philippine citizenship? The respondent HRET has an interesting view as to how Mr. Ong elected citizenship. It observed that "when protestee was only nine years of age, his father, Jose Ong Chuan became a naturalized Filipino. Section 15 of the Revised Naturalization Act squarely applies its benefit to him for he was then a minor residing in this country. Concededly, it was the law itself that had already elected Philippine citizenship for protestee by declaring him as such." (Emphasis supplied) The petitioners argue that the respondent's father was not, validly, a naturalized citizen because of his premature taking of the oath of citizenship. The Court cannot go into the collateral procedure of stripping Mr. Ong's father of his citizenship after his death and at this very late date just so we can go after the son. The petitioners question the citizenship of the father through a collateral approach. This can not be done. In our jurisdiction, an attack on a person's citizenship may only be done through a direct action for its nullity. (See Queto v. Catolico, 31 SCRA 52 [1970]) To ask the Court to declare the grant of Philippine citizenship to Jose Ong Chuan as null and void would run against the principle of due process. Jose Ong Chuan has already been laid to rest. How can he be given a fair opportunity to defend himself. A dead man cannot speak. To quote the words of the HRET "Ong Chuan's lips have long been muted to perpetuity by his demise and obviously he could not use beyond where his mortal remains now lie to defend himself were this matter to be made a central issue in this case." The issue before us is not the nullification of the grant of citizenship to Jose Ong Chuan. Our function is to determine whether or not the HRET committed abuse of authority in the exercise of its powers. Moreover, the respondent traces his natural born citizenship through his mother, not through the citizenship of his father. The citizenship of the father is relevant only to determine whether or not the respondent "chose" to be a Filipino when he came of age. At that time and up to the present, both mother and father were Filipinos. Respondent Ong could not have

elected any other citizenship unless he first formally renounced Philippine citizenship in favor of a foreign nationality. Unlike other persons faced with a problem of election, there was no foreign nationality of his father which he could possibly have chosen. There is another reason why we cannot declare the HRET as having committed manifest grave abuse of discretion. The same issue of naturalborn citizenship has already been decided by the Constitutional Convention of 1971 and by the Batasang Pambansa convened by authority of the Constitution drafted by that Convention. Emil Ong, full blood brother of the respondent, was declared and accepted as a natural born citizen by both bodies. Assuming that our opinion is different from that of the Constitutional Convention, the Batasang Pambansa, and the respondent HRET, such a difference could only be characterized as error. There would be no basis to call the HRET decision so arbitrary and whimsical as to amount to grave abuse of discretion. What was the basis for the Constitutional Convention's declaring Emil Ong a natural born citizen? Under the Philippine Bill of 1902, inhabitants of the Philippines who were Spanish subjects on the 11th day of April 1899 and then residing in said islands and their children born subsequent thereto were conferred the status of a Filipino citizen. Was the grandfather of the private respondent a Spanish subject? Article 17 of the Civil Code of Spain enumerates those who were considered Spanish Subjects, viz: ARTICLE 17. The following are Spaniards: 1. Persons born in Spanish territory. 2. Children born of a Spanish father or mother, even though they were born out of Spain. 3. Foreigners who may have obtained naturalization papers. 4. Those without such papers, who may have acquired domicile in any town in the Monarchy. (Emphasis supplied) The domicile of a natural person is the place of his habitual residence. This domicile, once established is considered to continue and will not be deemed lost until a new one is established. (Article 50, NCC; Article 40, Civil Code of Spain; Zuellig v. Republic, 83 Phil. 768 [1949]) As earlier stated, Ong Te became a permanent resident of Laoang, Samar around 1895. Correspondingly, a certificate of residence was then issued to him by virtue of his being a resident of Laoang, Samar. (Report of the Committee on Election Protests and Credentials of the 1971 Constitutional Convention, September 7, 1972, p. 3) The domicile that Ong Te established in 1895 continued until April 11, 1899; it even went beyond the turn of the 19th century. It is also in this place were Ong Te set-up his business and acquired his real property. As concluded by the Constitutional Convention, Ong Te falls within the meaning of sub-paragraph 4 of Article 17 of the Civil Code of Spain. Although Ong Te made brief visits to China, he, nevertheless, always returned to the Philippines. The fact that he died in China, during one of his visits in said country, was of no moment. This will not change the fact that he already had his domicile fixed in the Philippines and pursuant to the Civil Code of Spain, he had become a Spanish subject. If Ong Te became a Spanish subject by virtue of having established his domicile in a town under the Monarchy of Spain, necessarily, Ong Te was also an inhabitant of the Philippines for an inhabitant has been defined as one who has actual fixed residence in a place; one who has a domicile in a place. (Bouvier's Law Dictionary, Vol. II) Apriori, there can be no other logical conclusion but to educe that Ong Te qualified as a Filipino citizen under the provisions of section 4 of the Philippine Bill of 1902. The HRET itself found this fact of absolute verity in concluding that the private respondent was a natural-born Filipino. The petitioners' sole ground in disputing this fact is that document presented to prove it were not in compliance with the best the evidence rule. The petitioners allege that the private respondent failed to present the original of the documentary evidence, testimonial evidence and of the transcript of the proceedings of the body which the aforesaid resolution of the 1971 Constitutional Convention was predicated. On the contrary, the documents presented by the private respondent fall under the exceptions to the best evidence rule. It was established in the proceedings before the HRET that the originals of the Committee Report No. 12, the minutes of the plenary session of 1971 Constitutional Convention held on November 28, 1972 cannot be found. This was affirmed by Atty. Ricafrente, Assistant Secretary of the 1971 Constitutional Convention; by Atty. Nolledo, Delegate to the 1971 Constitutional Convention; and by Atty. Antonio Santos, Chief Librarian of the U.P Law Center, in their respective testimonies given before the HRET to the effect that there is no governmental agency which is the official custodian of the records of the 1971 Constitutional Convention. (TSN, December 12, 1988, pp. 30-31; TSN, January 17, 1989, pp. 34-35; TSN, February 1, 1989, p. 44; TSN, February 6, 1989, pp. 28-29)

The execution of the originals was established by Atty. Ricafrente, who as the Assistant Secretary of the 1971 Constitutional Convention was the proper party to testify to such execution. (TSN, December 12, 1989, pp. 11-24) The inability to produce the originals before the HRET was also testified to as aforestated by Atty. Ricafrente, Atty. Nolledo, and Atty. Santos. In proving the inability to produce, the law does not require the degree of proof to be of sufficient certainty; it is enough that it be shown that after a bona fide diligent search, the same cannot be found. (see Government of P.I. v. Martinez, 44 Phil. 817 [1918]) Since the execution of the document and the inability to produce were adequately established, the contents of the questioned documents can be proven by a copy thereof or by the recollection of witnesses. Moreover, to erase all doubts as to the authenticity of the documentary evidence cited in the Committee Report, the former member of the 1971 Constitutional Convention, Atty. Nolledo, when he was presented as a witness in the hearing of the protest against the private respondent, categorically stated that he saw the disputed documents presented during the hearing of the election protest against the brother of the private respondent. (TSN, February 1, 1989, pp. 8-9) In his concurring opinion, Mr. Justice Sarmiento, a vice-president of the Constitutional Convention, states that he was presiding officer of the plenary session which deliberated on the report on the election protest against Delegate Emil Ong. He cites a long list of names of delegates present. Among them are Mr. Chief Justice Fernan, and Mr. Justice Davide, Jr. The petitioners could have presented any one of the long list of delegates to refute Mr. Ong's having been declared a natural-born citizen. They did not do so. Nor did they demur to the contents of the documents presented by the private respondent. They merely relied on the procedural objections respecting the admissibility of the evidence presented. The Constitutional Convention was the sole judge of the qualifications of Emil Ong to be a member of that body. The HRET by explicit mandate of the Constitution, is the sole judge of the qualifications of Jose Ong, Jr. to be a member of Congress. Both bodies deliberated at length on the controversies over which they were sole judges. Decisions were arrived at only after a full presentation of all relevant factors which the parties wished to present. Even assuming that we disagree with their conclusions, we cannot declare their acts as committed with grave abuse of discretion. We have to keep clear the line between error and grave abuse. ON THE ISSUE OF RESIDENCE The petitioners question the residence qualification of respondent Ong. The petitioners lose sight of the meaning of "residence" under the Constitution. The term "residence" has been understood as synonymous with domicile not only under the previous Constitutions but also under the 1987 Constitution. The deliberations of the Constitutional Commission reveal that the meaning of residence vis-a-vis the qualifications of a candidate for Congress continues to remain the same as that of domicile, to wit: Mr. Nolledo: With respect to Section 5, I remember that in the 1971 Constitutional Convention, there was an attempt to require residence in the place not less than one year immediately preceding the day of the elections. So my question is: What is the Committee's concept of residence of a candidate for the legislature? Is it actual residence or is it the concept of domicile or constructive residence? Mr. Davide: Madame President, in so far as the regular members of the National Assembly are concerned, the proposed section merely provides, among others, and a resident thereof, that is, in the district, for a period of not less than one year preceding the day of the election. This was in effect lifted from the 1973 Constitution, the interpretation given to it was domicile. (Records of the 1987 Constitutional Convention, Vol. 11, July 22, 1986. p. 87) xxx xxx xxx Mrs. Rosario Braid: The next question is on Section 7, page 2. I think Commissioner Nolledo has raised the same point that "resident" has been interpreted at times as a matter of intention rather than actual residence. Mr. De los Reyes: Domicile. Ms. Rosario Braid: Yes, So, would the gentlemen consider at the proper time to go back to actual residence rather than mere intention to reside? Mr. De los Reyes: But we might encounter some difficulty especially considering that a provision in the Constitution in the Article on Suffrage says that Filipinos living abroad may vote as enacted by law. So, we have to stick to the original concept that it should be by domicile and not physical and actual residence. (Records of the 1987 Constitutional Commission, Vol. 11, July 22, 1986, p. 110) The framers of the Constitution adhered to the earlier definition given to the word "residence" which regarded it as having the same meaning as domicile.

The term "domicile" denotes a fixed permanent residence to which when absent for business or pleasure, one intends to return. (Ong Huan Tin v. Republic, 19 SCRA 966 [1967]) The absence of a person from said permanent residence, no matter how long, notwithstanding, it continues to be the domicile of that person. In other words, domicile is characterized by animus revertendi (Ujano v. Republic, 17 SCRA 147 [1966]) The domicile of origin of the private respondent, which was the domicile of his parents, is fixed at Laoang, Samar. Contrary to the petitioners' imputation, Jose Ong, Jr. never abandoned said domicile; it remained fixed therein even up to the present. The private respondent, in the proceedings before the HRET sufficiently established that after the fire that gutted their house in 1961, another one was constructed. Likewise, after the second fire which again destroyed their house in 1975, a sixteen-door apartment was built by their family, two doors of which were reserved as their family residence. (TSN, Jose Ong, Jr., November 18,1988, p. 8) The petitioners' allegation that since the private respondent owns no property in Laoang, Samar, he cannot, therefore, be a resident of said place is misplaced. The properties owned by the Ong Family are in the name of the private respondent's parents. Upon the demise of his parents, necessarily, the private respondent, pursuant to the laws of succession, became the co-owner thereof (as a co- heir), notwithstanding the fact that these were still in the names of his parents. Even assuming that the private respondent does not own any property in Samar, the Supreme Court in the case ofDe los Reyes v. Solidum (61 Phil. 893 [1935]) held that it is not required that a person should have a house in order to establish his residence and domicile. It is enough that he should live in the municipality or in a rented house or in that of a friend or relative. (Emphasis supplied) To require the private respondent to own property in order to be eligible to run for Congress would be tantamount to a property qualification. The Constitution only requires that the candidate meet the age, citizenship, voting and residence requirements. Nowhere is it required by the Constitution that the candidate should also own property in order to be qualified to run. (see Maquera v. Borra, 122 Phil. 412 [1965]) It has also been settled that absence from residence to pursue studies or practice a profession or registration as a voter other than in the place where one is elected, does not constitute loss of residence. (Faypon v. Quirino, 96 Phil. 294 [1954]) As previously stated, the private respondent stayed in Manila for the purpose of finishing his studies and later to practice his profession, There was no intention to abandon the residence in Laoang, Samar. On the contrary, the periodical journeys made to his home province reveal that he always had the animus revertendi. The Philippines is made up not only of a single race; it has, rather, undergone an interracial evolution. Throughout our history, there has been a continuing influx of Malays, Chinese, Americans, Japanese, Spaniards and other nationalities. This racial diversity gives strength to our country. Many great Filipinos have not been whole-blooded nationals, if there is such a person, for there is none. To mention a few, the great Jose Rizal was part Chinese, the late Chief Justice Claudio Teehankee was part Chinese, and of course our own President, Corazon Aquino is also part Chinese. Verily, some Filipinos of whom we are proud were ethnically more Chinese than the private respondent. Our citizens no doubt constitute the country's greatest wealth. Citizenship is a special privilege which one must forever cherish. However, in order to truly revere this treasure of citizenship, we do not, on the basis of too harsh an interpretation, have to unreasonably deny it to those who qualify to share in its richness. Under the overly strict jurisprudence surrounding our antiquated naturalization laws only the very affluent backed by influential patrons, who were willing to suffer the indignities of a lengthy, sometimes humiliating, and often corrupt process of clearances by minor bureaucrats and whose lawyers knew how to overcome so many technical traps of the judicial process were able to acquire citizenship. It is time for the naturalization law to be revised to enable a more positive, affirmative, and meaningful examination of an applicant's suitability to be a Filipino. A more humane, more indubitable and less technical approach to citizenship problems is essential. WHEREFORE, the petitions are hereby DISMISSED. The questioned decision of the House of Representatives Electoral Tribunal is AFFIRMED. Respondent Jose Ong, Jr. is declared a natural-born citizen of the Philippines and a resident of Laoang, Northern Samar. SO ORDERED. Bidin, Grio-Aquino, Medialdea and Davide, Jr., JJ., concur. Fernan, C.J., Melencio-Herrera, Cruz, Feliciano and Gancayco, JJ., took no part.

Separate Opinions

PADILLA, J., dissenting: I dissent. These separate petitions for certiorari and mandamus seek to annul the decision * of respondent House of Representatives Electoral Tribunal (hereinafter referred to as the tribunal) dated 6 November 1989 which declared private respondent Jose L. Ong, a natural-born citizen of the Philippines and a legal resident of Laoang, Northern Samar, and the resolution of the tribunal dated 22 February 1990 denying petitioners' motions for reconsideration. In G.R. Nos. 92191-92, petitioner Co also prays that the Court declare private respondent Ong not qualified to be a Member of the House of Representatives and to declare him (petitioner Co) who allegedly obtained the highest number of votes among the qualified candidates, the duly elected representative of the second legislative district of Northern Samar. In G.R. Nos. 92202-03, petitioner Balanquit prays that the Court declare private respondent Ong and Co (petitioner in G.R. Nos. 92191-92) not qualified for membership in the House of Representatives and to proclaim him (Balanguit) as the duly elected representative of said district. Petitioners Antonio Y. Co, Sixto T. Balanquit, Jr. and private respondent Jose Ong Chuan, Jr. were among the candidates for the position of Representative or Congressman for the second district of Northern Samar during the 11 May 1987 congressional elections. Private respondent was proclaimed duly-elected on 18 May 1987 with a plurality of some sixteen thousand (16,000) votes over petitioner Co who obtained the next highest number of votes. Petitioners Co and Balanquit then filed separate election protests against private respondent with the tribunal, docketed as HRET Cases Nos. 13 and 15 respectively. Both protests raised almost the same issues and were thus considered and decided jointly by the tribunal. The issues raised before the tribunal were the following: 1. Whether or not protestee (meaning, Ong) is a natural-born citizen of the Philippines in contemplation of Section 6, Article VI of the 1987 Constitution in relation to Sections 2 and 1(3), Article IV thereof; and 2. Whether or not protestee was a resident of Laoang, Northern Samar, in contemplation of Section 6, Article VI of the same Constitution, for a period of not less than one year immediately preceding the congressional elections of May 1987. The respondent tribunal in its decision dated 6 November 1989 held that respondent Jose L. Ong is a natural-born citizen of the Philippines and was a legal resident of Laoang, Northern Samar for the required period prior to the May 1987 congressional elections. He was, therefore, declared qualified to continue in office as Member of the House of Representatives, Congress of the Philippines, representing the second legislative district of Northern Samar. The factual antecedents taken from the consolidated proceedings in the tribunal are the following: 1. The Protestee (Ong) was born on June 19, 1948 to the legal spouses Ong Chuan also known as Jose Ong Chuan and Agrifina E. Lao. His place of birth is Laoang which is now one of the municipalities comprising the province of Northern Samar (Republic Act No. 6132 approved on August 24, 1970 and the Ordinance appended to the 1987 Constitution). 2. On the other hand, Jose Ong Chuan was born in China and arrived in Manila on December 16, 1915. (Exhibit zz) Subsequently thereafter, he took up residence in Laoang, Samar. 3. On February 4, 1932, he married Agrifina E. Lao. Their wedding was celebrated according to the rites and practices of the Roman Catholic Church in the Municipality of Laoang (Exh. E). 4. At the time of her marriage to Jose Ong Chuan, Agrifina E. Lao was a natural-born Filipino citizen, both her parents at the time of her birth being Filipino citizens. (Exhibits E & I) 5. On February 15, 1954, Jose Ong Chuan, desiring to acquire Philippine citizenship, filed his petition for naturalization with the Court of First Instance of Samar, pursuant to Commonwealth Act No. 473, otherwise known as the Revised Naturalization Law. 6. On April 28, 1955, the Court of First Instance of Samar rendered a decision approving the application of Jose Ong Chuan for naturalization and declaring said petitioner a Filipino citizen "with all the rights and privileges and duties, liabilities and obligations inherent to Filipino citizens. (Exh. E) 7. On May 15, 1957, the same Court issued an order: (1) declaring the decision of this Court of April 28, 1955 final and executory; (2) directing the clerk of court to issue the corresponding Certificate of Naturalization in favor of the applicant Ong Chuan who prefers to take his oath and register his name as Jose Ong Chuan. Petitioner may take his oath as Filipino citizen under Ms new christian name, Jose Ong Chuan. (Exh. F)

8. On the same day, Jose Ong Chuan having taken the corresponding oath of allegiance to the Constitution and the Government of the Philippines as prescribed by Section 12 of Commonwealth Act No. 473, was issued the corresponding Certificate of Naturalization. (Exh. G) 9. On November 10, 1970, Emil L. Ong, a full-brother of the protestee and a son born on July 25, 1937 at Laoang, Samar to the spouses Jose Ong Chuan and Agrifina E. Lao, was elected delegate from Northern Samar to the 1971 Constitutional Convention. 10. By protestee's own -testimony, it was established that he had attended grade school in Laoang. Thereafter, he went to Manila where he finished his secondary as well as his college education. While later employed in Manila, protestee however went home to Laoang whenever he had the opportunity to do so, which invariably would be as frequent as twice to four times a year. 11. Protestee also showed that being a native and legal resident of Laoang, he registered as a voter therein and correspondingly voted in said municipality in the 1984 and 1986 elections. 12. Again in December 1986, during the general registration of all voters in the country, Protestee re-registered as a voter in Precinct No. 4 of Barangay Tumaguinting in Laoang. In his voter's affidavit, Protestee indicated that he is a resident of Laoang since birth. (Exh. 7) 1 Petitioners' motions for reconsideration of the tribunal's decision having been denied, petitioners filed the present petitions. In their comments, the respondents first raise the issue of the Court's jurisdiction to review the decision of the House Electoral Tribunal, considering the constitutional provision vesting upon said tribunal the power and authority to act as the sole judge of all contests relating to the qualifications of the Members of the House of Representatives. 2 On the question of this Court's jurisdiction over the present controversy, I believe that, contrary to the respondents' contentions, the Court has the jurisdiction and competence to review the questioned decision of the tribunal and to decide the present controversy. Article VIII, Section I of the 1987 Constitution provides that: Judicial power includes the duty of the courts of justice to settle actual controversies involving rights which are legally demandable and enforceable, and to determine whether or not there has been a grave abuse of discretion amounting to lack or excess of jurisdiction on the part of any branch or instrumentality of the Government. The Constitution, it is true, constitutes the tribunal as the sole judge of all contests relating to the election, returns, and qualifications of Members of the House of Representatives. But as early as 1938, it was held in Morrero vs.Bocar, 3 construing Section 4, Article VI of the 1935 Constitution which provided that ". . . The Electoral Commission shall be the sole judge of all contests relating to the election, returns and qualifications of the Members of the National Assembly," that: The judgment rendered by the (electoral) commission in the exercise of such an acknowledged power is beyond judicial interference, except, in any event, "upon a clear showing of such arbitrary and improvident use of the power as will constitute a denial of due process of law." (Barry vs. US ex rel. Cunningham, 279 US 597; 73 Law. ed., 867; Angara vs. Electoral Commission, 35 Off. Gaz., 23.) And then under the afore-quoted provisions of Article VIII, Section 1 of the 1987 Constitution, this Court is duty-bound to determine whether or not, in an actual controversy, there has been a grave abuse of discretion amounting to lack or excess of jurisdiction on the part of any branch or instrumentality of the Government. The present controversy, it will be observed, involves more than perceived irregularities in the conduct of a congressional election or a disputed appreciation of ballots, in which cases, it may be contended with great legal force and persuasion that the decision of the electoral tribunal should be final and conclusive, for it is, by constitutional directive, made the sole judge of contests relating to such matters. The present controversy, however, involves no less than a determination of whether the qualifications for membership in the House of Representatives, as prescribed by the Constitution, have been met. Indeed, this Court would be unforgivably remiss in the performance of its duties, as mandated by the Constitution, were it to allow a person, not a natural-born Filipino citizen, to continue to sit as a Member of the House of Representatives, solely because the House Electoral Tribunal has declared him to be so. In such a case, the tribunal would have acted with grave abuse of discretion amounting to lack or excess of jurisdiction as to require the exercise by this Court of its power of judicial review. Besides, the citizenship and residence qualifications of private respondent for the office of Member of the House of Representatives, are here controverted by petitioners who, at the same time, claim that they are entitled to the office illegally held by private respondent. From this additional direction, where one asserts an earnestly perceived right that in turn is vigorously resisted by another, there is clearly a justiciable controversy proper for this Court to consider and decide. Nor can it be said that the Court, in reviewing the decision of the tribunal, asserts supremacy over it in contravention of the time-honored principle of constitutional separation of powers. The Court in this instance simply performs a function entrusted and assigned to it by the Constitution of interpreting, in a justiciable controversy, the pertinent provisions of the Constitution with finality. It is the role of the Judiciary to refine and, when necessary, correct constitutional (and/or statutory) interpretation, in the context of the interactions of the three branches of the government, almost always in situations where some agency of the State has engaged in action that stems ultimately from some legitimate area of governmental power (the Supreme Court in Modern Role, C.B. Sevisher, 1958, p. 36). 4

Moreover, it is decidedly a matter of great public interest and concern to determine whether or not private respondent is qualified to hold so important and high a public office which is specifically reserved by the Constitution only to natural-born Filipino citizens. After a careful consideration of the issues and the evidence, it is my considered opinion that the respondent tribunal committed grave abuse of discretion amounting to lack or excess of jurisdiction in rendering its questioned decision and resolution, for reasons to be presently stated. The Constitution 5 requires that a Member of the House of Representatives must be a natural-born citizen of the Philippines and, on the day of the election, is at least twenty-five (25) years of age, able to read and write, and, except the party-list representatives, a registered voter in the district in which he shall be elected, and a resident thereof for a period of not less than one (1) year immediately preceding the day of the election. Article IV, Section 2 of the 1987 Constitution defines natural-born (Filipino) citizens as: Natural-born citizens are those who are citizens of the Philippines from birth without having to perform any act to acquire or perfect their Philippine citizenship. Those who elect Philippine citizenship in accordance with paragraph (3), Section I hereof shall be deemed natural-born citizen, Article IV, Section 1, paragraph (3) of the 1987 Constitution provides that: Section 1. The following are citizens of the Philippines: xxx xxx xxx (3) Those born before January 17, 1973, of Filipino mothers, who elect Philippine citizenship upon reaching the age of majority. The Court in this case is faced with the duty of interpreting the above-quoted constitutional provisions. The first sentence of Section 2 of Article IV states the basic definition of a natural-born Filipino citizen. Does private respondent fall within said definition? To the respondent tribunal, Protestee may even be declared a natural-born citizen of the Philippines under the first sentence of Sec. 2 of Article IV of the 1987 Constitution because he did not have "to perform any act to acquire or perfect his Philippine citizenship." It bears to repeat that on 15 May 1957, while still a minor of 9 years he already became a Filipino citizen by declaration of law. Since his mother was a natural-born citizen at the time of her marriage, protestee had an inchoate right to Philippine citizenship at the moment of his birth and, consequently the declaration by virtue of Sec. 15 of CA 473 that he was a Filipino citizen retroacted to the moment of his birth without his having to perform any act to acquire or perfect such Philippine citizenship. 6 I regret that I am neither convinced nor persuaded by such kaleidoscopic ratiocination. The records show that private respondent was born on 19 June 1948 to the spouses Jose Ong Chuan, a Chinese citizen, and Agrifina E. Lao, a natural-born Filipino citizen, in Laoang, Northern Samar. In other words, at birth, private respondent was a Chinese citizen (not a natural-born Filipino citizen) because his father was then a Chinese citizen (not a naturalized Filipino citizen). Under the 1935 Constitution which was enforced at the time of private respondent's birth on 19 June 1948, only those whose fathers were citizens of the Philippines were considered Filipino citizens. Those whose mothers were citizens of the Philippines had to elect Philippine citizenship upon reaching the age of majority, in order to be considered Filipino citizens. 7 Following the basic definition in the 1987 Constitution of a natural-born citizen, in relation to the 1935 Constitution, private respondent is not a natural-born Filipino citizen, having been born a Chinese citizen by virtue of the Chinese citizenship of his father at the time of his birth, although from birth, private respondent had the right to elect Philippine citizenship, the citizenship of his mother, but only upon his reaching the age of majority. While under Section 15 of the Revised Naturalization Law (C.A. 473) minor children of a naturalized citizen (father), who were born in the Philippines prior to the naturalization of the parent automatically become Filipino citizens, 8 this does not alter the fact that private respondent was not born to a Filipino father, and the operation of Section 15 of CA 473 did not confer upon him the status of a natural-born citizen merely because he did not have to perform any act to acquire or perfect his status as a Filipino citizen. But even assuming arguendo that private respondent could be considered a natural-born citizen by virtue of the operation of CA 473, petitioners however contend that the naturalization of private respondent's father was invalid and void from the beginning, and, therefore, private respondent is not even a Filipino citizen. Respondent tribunal in its questioned decision ruled that only a direct proceeding for nullity of naturalization as a Filipino citizen is permissible, and, therefore, a collateral attack on Ong Chuan's naturalization is barred in an electoral contest which does not even involve him (Ong Chuan). Private respondent, for his part, avers in his Comment that the challenge against Ong Chuan's naturalization must emanate from the Government and must be made in a proper/appropriate and direct proceeding for de-naturalization directed against the proper party, who in such case is Ong Chuan, and also during his lifetime.

A judgment in a naturalization proceeding is not, however, afforded the character of impregnability under the principle of res judicata. 9 Section 18 of CA 473 provides that a certificate of naturalization may be cancelled upon motion made in the proper proceeding by the Solicitor General or his representative, or by the proper provincial fiscal. In Republic vs. Go Bon Lee, 10 this Court held that: An alien friend is offered under certain conditions the privilege of citizenship. He may accept the offer and become a citizen upon compliance with the prescribed conditions, but not otherwise. His claim is of favor, not of right. He can only become a citizen upon and after a strict compliance with the acts of Congress. An applicant for this high privilege is bound, therefore, to conform to the terms upon which alone the right he seeks can be conferred. It is his province, and he is bound, to see that the jurisdictional facts upon which the grant is predicated actually exist and if they do not he takes nothing by this paper grant. xxx xxx xxx Congress having limited this privilege to a specified class of persons, no other person is entitled to such privilege, nor to a certificate purporting to grant it, and any such certificate issued to a person not so entitled to receive it must be treated as a mere nullity, which confers no legal rights as against the government, from which it has been obtained without warrant of law. "Naturalization is not a right, but a privilege of the most discriminating as well as delicate and exacting nature, affecting public interest of the highest order, and which may be enjoyed only under the precise conditions prescribed by law therefor." 11 Considering the legal implications of the allegation made by the petitioners that the naturalization of private respondent's father Ong Chuan, is a nullity, the Court should make a ruling on the validity of said naturalization proceedings. This course of action becomes all the more inevitable and justified in the present case where, to repeat for stress, it is claimed that a foreigner is holding a public office. 12 It cannot be overlooked, in this connection, that the citizenship of private respondent is derived from his father. If his father's Filipino citizenship is void from the beginning, then there is nothing from which private respondent can derive his own claimed Filipino citizenship. For a spring cannot rise higher than its source. And to allow private respondent to avail of the privileges of Filipino citizenship by virtue of a void naturalization of his father, would constitute or at least sanction a continuing offense against the Constitution. The records show that private respondent's father, Jose Ong Chuan, took the oath of allegiance to the Constitution and the Philippine Government, as prescribed by Section 12 of CA 473 on the same day (15 May 1957) that the CFI issued its order directing the clerk of court to issue the corresponding Certificate of Naturalization and for the applicant to take the oath of allegiance. However, it is settled that an order granting a petition to take the requisite oath of allegiance of one who has previously obtained a decision favorable to his application for naturalization, is appealable. It is, therefore, improper and illegal to authorize the taking of said oath upon the issuance of said order and before the expiration of the reglementary period to perfect any appeal from said order. 13 In Cua Sun Ke vs. Republic, 14 this Court held that: Administration of the oath of allegiance on the same day as issuance of order granting citizenship is irregular and makes the proceedings so taken null and void. (Republic vs. Guy, 115 SCRA 244 [1982]; citing the case of Ong So vs. Republic of the Philippines, 121 Phil. 1381). It would appear from the foregoing discussion that the naturalization of Jose Ong Chuan (private respondent's father) was null and void. It follows that the private respondent did not acquire any legal rights from the void naturalization of his father and thus he cannot himself be considered a Filipino citizen, more so, a natural-born Filipino citizen. But assuming that the CFI order of 15 May 1957 directing the clerk of court to issue the certificate of naturalization to Ong Chuan and for the latter to take the oath of allegiance was final and not appealable, the resulting naturalization of Ong Chuan effected, as previously stated, an automatic naturalization of private respondent, then a minor, as a Filipino citizen on 15 May 1957, but not his acquisition or perfection of the status of a natural-born Filipino citizen. Let us now look into the question of whether or not private respondent acquired the status of a natural-born Filipino citizen by reason of the undisputed fact that his mother was a natural-born Filipino citizen. This in turn leads us to an examination of the second sentence in Article IV, Section 2 of the 1987 Constitution. It expands, in a manner of speaking, in relation to Section 1, paragraph (3) of the same Article IV, the status of a natural-born Filipino citizen to those who elect Philippine citizenship upon reaching the age of majority. The right or privilege of election is available, however, only to those born to Filipino mothers under the 1935 Constitution, and before the 1973 Constitution took effect on 17 January 1973. The petitioners contend that the respondent tribunal acted in excess of its jurisdiction or gravely abused its discretion as to exceed its jurisdiction in "distorting" the conferment by the 1987 Constitution of the status of "natural-born" Filipino citizen on those who elect Philippine citizenship all in its strained effort, according to petitioners, to support private respondent's qualification to be a Member of the House of Representatives. 15 Petitioners argue that the clear, unambiguous wording of section 1(3) of Article IV of the 1987 Constitution contemplates that only the legitimate children of Filipino mothers with alien father, born before 17 January 1973 and who would reach the age of majority (and thus elect Philippine citizenship) after the effectivity of the 1987 Constitution are entitled to the status of natural-born Filipino citizen. 16

The respondent tribunal in resolving the issue of the constitutional provisions' interpretation, found reason to refer to the interpellations made during the 1986 Constitutional Commission. It said: That the benevolent provisions of Sections 2 and 1(3) of Article IV of the 1987 Constitution was (sic) intended by its (sic) framers to be endowed, without distinction, to all Filipinos by election pursuant to the 1935 Constitution is more than persuasively established by the extensive interpellations and debate on the issue as borne by the official records of the 1986 Constitutional Commission. 17 Although I find the distinction as to when election of Philippine citizenship was made irrelevant to the case at bar, since private respondent, contrary to the conclusion of the respondent tribunal, did not elect Philippine citizenship, as provided by law, I still consider it necessary to settle the controversy regarding the meaning of the constitutional provisions in question. I agree with respondent tribunal that the debates, interpellations petitions and opinions expressed in the 1986 Constitutional Commission may be resorted to in ascertaining the meaning of somewhat elusive and even nebulous constitutional provisions. Thus The ascertainment of that intent is but in keeping with the fundamental principle of constitutional construction that the intent of the framers of the organic law and of the people adopting it should be given effect. The primary task in constitutional construction is to ascertain and thereafter assure the realization of the purpose of the framers and of the people in the adoption of the Constitution. It may also be safely assumed that the people in ratifying the constitution were guided mainly by the explanation offered by the framers. 18 The deliberations of the 1986 Constitutional Commission relevant to Section 2, Article IV in relation to Section 1(3) of the same Article, appear to negate the contention of petitioners that only those born to Filipino mothers before 17 January 1973 and who would elect Philippine citizenship after the effectivity of the 1987 Constitution, are to be considered natural-born Filipino citizens. During the free-wheeling discussions on citizenship, Commissioner Treas specifically asked Commissioner Bernas regarding the provisions in question, thus: MR. TRENAS: The Committee on Citizenship, Bill of Rights, Political Rights and Obligations and Human Rights has more or less decided to extend the interpretation of who is a natural-born Filipino citizen as provided in Section 4 of the 1973 Constitution, by adding that persons who have elected Philippine citizenship under the 1935 Constitution shall be considered natural-born. Am I right, Mr. Presiding Officer? FR BERNAS: Yes. MR. TRENAS: And does the Commissioner think that tills addition to Section 4 of the 1973 Constitution would be contrary to the spirit of that section? FR BERNAS: Yes, we are quite aware that it is contrary to the letter really. But whether it is contrary to the spirit is something that has been debated before and is being debated even now. We will recall that during the 1971 Constitutional Convention, the status of natural-born citizenship of one of the delegates, Mr. Ang, was challenged precisely because he was a citizen by election. Finally, the 1971 Constitutional Convention considered him a natural-born citizen, one of the requirements to be a Member of the 1971 Constitutional Convention. The reason behind that decision was that a person under his circumstances already had the inchoate right to be a citizen by the fact that the mother was a Filipino. And as a matter of fact, the 1971 Constitutional Convention formalized that recognition by adopting paragraph 2 of Section 1 of the 1971 Constitution. So, the entire purpose of this proviso is simply to perhaps remedy whatever injustice there may be so that these people born before January 17, 1973 who are not naturalized and people who are not natural born but who are in the same situation as we are considered naturalborn citizens. So, the intention of the Committee in proposing this is to equalize their status. 19 When asked to clarify the provision on natural-born citizens, Commissioner Bernas replied to Commissioner Azcuna thus: MR. AZCUNA: With respect to the proviso in Section 4, would this refer only to those who elect Philippine citizenship after the effectivity of the 1973 Constitution or would it also cover those who elected it under the 1935 Constitution? FR BERNAS: It would apply to anybody who elected Philippine citizenship by virtue of the provision of the 1935 Constitution, whether the election was done before or after 17 January 1973.20 And during the period of amendments. Commissioner Rodrigo explained the purpose of what now appear as Section 2 and Section 1, paragraph (3) of Article IV of the 1987 Constitution, thus: MR. RODRIGO: The purpose of that proviso is to remedy an inequitable situation. Between 1935 and 1973, when we were under the 1935 Constitution, those born of Filipino fathers but alien mothers were natural-born Filipinos. However, those born of Filipino mothers but alien fathers would have to elect Philippine citizenship upon reaching the age of majority; and, if they do elect, they become Filipino citizens, yet, but not natural-born Filipino citizens.

The 1973 Constitution equalized the status of those born of Filipino mothers and those born of Filipino fathers. So that from January 17, 1973 when the 1973 Constitution took effect, those born of Filipino mothers but of alien fathers are natural-born Filipino citizens. Also, those who are born of Filipino fathers and alien mothers are natural-born Filipino citizens. If the 1973 Constitution equalized the status of a child born of a Filipino mother and that born of a Filipino father, why do we not give a chance to a child born before January 17, 1973, if and when he elects Philippine citizenship, to be in the same status as one born of a Filipino father namely, natural-born citizen. Another thing I stated is equalizing the status of a father and a mother vis-a-vis the child. I would like to state also that we showed equalize the status of a child born of a Filipino mother the day before January 17, 1973 and a child born also of a Filipino mother on January 17 or 24 hours later. A child born of a Filipino mother but an alien father one day before January 17, 1973 is a Filipino citizen, if he elects Philippine citizenship, but he is not a natural-born Filipino citizen. However, the other child who luckily was born 24 hours later maybe because of parto laborioso is a natural-born Filipino citizen. 21 It would appear then that the intent of the framers of the 1987 Constitution in defining a natural-born Filipino citizen was to equalize the position of Filipino fathers and Filipino mothers as to their children becoming natural-born Filipino citizens. In other words, after 17 January 1973, effectivity date of the 1973 Constitution, all those born of Filipino fathers (with alien spouse) or Filipino mothers (with alien spouse) are naturalborn Filipino citizens. But those born to Filipino mothers prior to 17 January 1973 must still elect Philippine citizenship upon their reaching the age of majority, in order to be deemed natural-born Filipino citizens. The election, which is related to the attainment of the age of majority, may be made before or after 17 January 1973. This interpretation appears to be in consonance with the fundamental purpose of the Constitution which is to protect and enhance the people's individual interests, 22 and to foster equality among them. Since private respondent was born on 19 June 1948 (or before 17 January 1973) to a Filipino mother (with an alien spouse) and should have elected Philippine citizenship on 19 June 1969 (when he attained the age of majority), or soon thereafter, in order to have the status of a natural-born Filipino citizen under the 1987 Constitution, the vital question is: did private respondent really elect Philippine citizenship? As earlier stated, I believe that private respondent did not elect Philippine citizenship, contrary to the ruling of the respondent tribunal. The respondent tribunal, on this issue, ruled as follows: Where a person born to a Filipino mother and an alien father had exercised the right of suffrage when he came of age, the same constitutes a positive act of election of Philippine citizenship. (Florencio vs. Mallare) [sic] The acts of the petitioner in registering as a voter, participating in elections and campaigning for certain candidates were held by the Supreme Court as sufficient to show his preference for Philippine citizenship. Accordingly, even without complying with the formal requisites for election, the petitioner's Filipino citizenship was judicially upheld. 23 I find the above ruling of the respondent tribunal to be patently erroneous and clearly untenable, as to amount to grave abuse of discretion. For it is settled doctrine in this jurisdiction that election of Philippine citizenship must be made in accordance with Commonwealth Act 625. Sections 1 and 2 24 of the Act mandate that the option to elect Philippine citizenship must be effected expressly not impliedly. The respondent tribunal cites In re: Florencio Mallare 25 which held that Esteban Mallare's exercise of the right of suffrage when he came of age, constituted a positive act of election of Philippine citizenship. Mallare, cited by respondent tribunal as authority for the doctrine of implied election of Philippine citizenship, is not applicable to the case at bar. The respondent tribunal failed to consider that Esteban Mallare reached the age of majority in 1924, or seventeen (17) years before CA 625 was approved and, more importantly, eleven (11) years before the 1935 Constitution (which granted the right of election) took effect. To quote Mr. Justice Fernandez in Mallare: Indeed, it would be unfair to expect the presentation of a formal deed to that effect considering that prior to the enactment of Commonwealth Act 625 on June 7, 1941, no particular proceeding was required to exercise the option to elect Philippine citizenship, granted to the proper party by Section 1, subsection 4, Article IV of the 1935 Philippine Constitution. 26 Moreover, Esteban Mallare was held to be a Filipino citizen because he was an illegitimate (natural) child of a Filipino mother and thus followed her citizenship. I therefore agree with the petitioners' submission that, inciting the Mallare case, the respondent tribunal had engaged in an obiter dictum. The respondent tribunal also erred in ruling that by operation of CA 473, the Revised Naturalization Law, providing for private respondent's acquisition of Filipino citizenship by reason of the naturalization of his father, the law itself had already elected Philippine citizenship for him. For, assuming arguendo that the naturalization of private respondent's father was valid, and that there was no further need for private respondent to elect Philippine citizenship (as he had automatically become a Filipino citizen) yet, this did not mean that the operation of the Revised Naturalization Law amounted to an election by him of Philippine citizenship as contemplated by the Constitution. Besides, election of Philippine citizenship derived from one's Filipino mother, is made upon reaching the age of majority, not during one's minority. There is no doubt in my mind, therefore, that private respondent did not elect Philippine citizenship upon reaching the age of majority in 1969 or within a reasonable time thereafter as required by CA 625. Consequently, he cannot be deemed a natural-born Filipino citizen under Sections 2 and 1(3), Article IV of the 1987 Constitution.

Based on all the foregoing considerations and premises, I am constrained to state that private respondent is not a natural-born citizen of the Philippines in contemplation of Section 6, Article VI of the 1987 Constitution in relation to Sections 2 and 1(3), Article IV thereof, and hence is disqualified or ineligible to be a Member of the House of Representatives. At this point, I find it no longer necessary to rule on the issue of required residence, inasmuch as the Constitution requires that a Member of the House of Representatives must be both a natural-born Filipino citizen and a resident for at least one (1) year in the district in which he shall be elected. The next question that comes up is whether or not either of the petitioners can replace private respondent as the Representative of the second legislative district of Northern Samar in the House of Representatives. I agree with respondent tribunal that neither of the petitioners may take the place of private respondent in the House of Representatives representing the second district of Northern Samar. The ruling of this Court in Ramon L.Labo, Jr. vs. The Commission on Elections (COMELEC) EN BANC and Luis L. Lardizabal, 27 is controlling. There we held that Luis L. Lardizabal, who filed the quo warranto petition, could not replace Ramon L. Labo, Jr. as mayor of Baguio City for the simple reason that as he obtained only the second highest number of votes in the election, he was obviously not the choice of the people of Baguio City for mayor of that City. A petition alleging that the candidate-elect is not qualified for the office is, in effect, a quo warranto proceeding even if it is labelled an election protest. 28 It is a proceeding to unseat the ineligible person from office but not necessarily to install the protestant in his place. 29 The general rule is that the fact that a plurality or a majority of the votes are cast for an ineligible candidate in an election does not entitle the candidate receiving the next highest number of votes to be declared elected. In such a case, the electors have failed to make a choice and the election is a nullity. 30 Sound policy dictates that public elective offices are filled by those who have the highest number of votes cast in the election for that office, and it is a fundamental idea in all republican forms of government that no one can be declared elected and no measure can be declared carried unless he or it receives a majority or plurality of the legal votes cast in the election. (20 Corpus Juris 2nd, S 243, p. 676). As early as 1912, this Court has already declared that the candidate who lost in an election cannot be proclaimed the winner in the event that the candidate who won is found ineligible for the office to which he was elected. This was the ruling in Topacio v. Paredes (23 Phil. 238) Again, the effect of a decision that a candidate is not entitled to the office because of fraud or irregularities in the election is quite different from that produced by declaring a person ineligible to hold such an office. . . . If it be found that the successful candidate (according to the board of canvassers) obtained a plurality in an illegal manner, and that another candidate was the real victor, the former must retire in favor of the latter. In the other case, there is not, strictly speaking, a contest, as the wreath of victory cannot be transferred from an ineligible to any other candidate when the sole question is the eligibility of the one receiving a plurality of the legally cast ballots. . . . 31 The recognition of Emil L. Ong by the 1971 Constitutional Convention as a natural-born Filipino citizen, in relation to the present case. Private respondent, as previously stated, is a full brother of Emil L. Ong, both of them having the same father and mother. Private respondent, relying on a resolution of the 1971 Constitutional Convention 32 to the effect that Emil L. Ong was a natural-born Filipino citizen, alleged before the House Electoral Tribunal that, by analogy, he is himself a natural-born Filipino citizen. This submission, while initially impressive, is, as will now be shown, flawed and not supported by the evidence. Not even the majority decision of the electoral tribunal adopted the same as the basis of its decision in favor of private respondent. The tribunal, in reference to this submission, said: Be that as it may and in the light of the Tribunal's disposition of protestee's citizenship based on an entirely different set of circumstances, apart from the indisputable fact that the matters attempted to be brought in issue in connection therewith are too far removed in point of time and relevance from the decisive events relied upon by the Tribunal, we view these two issues as being already inconsequential. 33 The electoral tribunal (majority) instead chose to predicate its decision on the alleged citizenship by naturalization of private respondent's father (Ong Chuan) and on the alleged election of Philippine citizenship by private respondent. Emil L. Ong, was elected delegate to the 1971 Constitutional Convention. Electoral protests, numbers EP-07 and EP-08, were filed by Leonardo D. Galing and Gualberto D. Luto against Emil L. Ong, contesting his citizenship qualification. The Committee on Election Protests Credentials of the 1971 Contitution Convention heard the protests and submitted to the Convention a report dated 4 September 1972, the dispositive portion of which stated: It appearing that protestee's grandfather was himself a Filipino citizen under the provisions of the Philippine Bill of 1902 and the Treaty of Paris of December 10, 1898, thus conferring upon protestee's own father, Ong Chuan, Philippine citizenship at birth, the conclusion is inescapable that protestee himself is a natural-born citizen, and is therefore qualified to hold the office of delegate to the Constitutional Convention. 34

On 28 November 1972, during a plenary session of the 1971 Constitutional Convention, the election protests filed against Emil L. Ong were dismissed, following the report of the Committee on Election Protests and Credentials. 35 It is evident, up to this point, that the action of the 1971 Constitutional Convention in the case of Emil L. Ong is, to say the least, inconclusive to the case at bar, because a) the 1971 Constitutional Convention decision in the Emil L. Ong case involved the 1935 Constitution; the present case, on the other hand involves the 1987 Constitution: b) the 1935 Constitution contained no specific definition of a "natural-born citizen" of the Philippines; the 1987 Constitution contains a precise and specific definition of a "natural-born citizen" of the Philippines in Sec. 2, Art. IV thereof and private respondent does not qualify under such definition in the 1987 Constitution; c) the decision of the 1971 Constitutional Convention in the case of Emil L. Ong was a decision of apolitical body, not a court of law. And, even if we have to take such a decision as a decision of aquasi-judicial body (i.e., a political body exercising quasi-judicial functions), said decision in the Emil L. Ong case can not have the category or character of res judicata in the present judicial controversy, because between the two (2) cases, there is no identity of parties (one involves Emil L. Ong, while the other involves private respondent) and, more importantly, there is no identity of causes of action because the first involves the 1935 Constitution while the second involves the 1987 Constitution. But even laying aside the foregoing reasons based on procedural rules and logic, the evidence submitted before the electoral tribunal and, therefore, also before this Court, does not support the allegations made by Emil L. Ong before the 1971 Constitutional Convention and inferentially adopted by private respondent in the present controversy. This leads us to an interesting inquiry and finding. The 1971 Constitutional Convention in holding that Emil L. Ong was a "natural-born citizen" of the Philippines under the 1935 Constitution laid stress on the "fact" and this appears crucial and central to its decision that Emil L. Ong's grandfather, Ong Te became a Filipino citizen under the Philippine Bill of 1902 and, therefore, his descendants like Emil L. Ong (and therefore, also private respondent) became natural-born Filipinos. The 1971 Constitutional Convention said: Ong Te Emil Ong's grandfather, was a Spanish subject residing in the Philippines on April 11, 1899 and was therefore one of the many who became ipso facto citizens of the Philippines under the provisions of the Philippine Bill of 1902. Said law expressly declared that all inhabitants of the Philippine Islands who continued to reside therein and who were Spanish subjects on April 11, 1899 as well as their children born subsequent thereto, "shall be deemed and held to be citizens of the Philippine Islands." (Section 4, Philippine Bill of 1902). 36 The "test" then, following the premises of the 1971 Constitutional Convention, is whether or not Ong Te private respondent's and Emil L. Ong's grandfather was "an inhabitant of the Philippines who continued to reside therein and was a Spanish subject on April 11, 1899." If he met these requirements of the Philippine Bill of 1902, then, Ong Te was a Filipino citizen; otherwise, he was not a Filipino citizen. Petitioners (protestants) submitted and offered in evidence before the House Electoral Tribunal exhibits W, X, Y, Z ,AA, BB, CC, DD and EE which are copies of entries in the "Registro de Chinos" from years 1896 to 1897 which show that Ong Te was not listed as an inhabitant of Samar where he is claimed to have been a resident. Petitioners (protestants) also submitted and offered in evidence before the House Electoral Tribunal exhibit V, a certification of the Chief of the Archives Division, Records and Management and Archives Office, stating that the name of Ong Te does not appear in the "Registro Central de Chinos" for the province of Samar for 1895. These exhibits prove or at least, as petitioners validly argue, tend to prove that Ong Te was NOT a resident of Samar close to 11 April 1899 and, therefore, could not continue residing in Samar, Philippines after 11 April 1899, contrary to private respondent's pretense. In the face of these proofs or evidence, private respondent FAILED TO PRESENT ANY REBUTTAL OR COUNTERVAILING EVIDENCE, except the decision of the 1971 Constitutional Convention in the case of Emil L. Ong, previously discussed. It is not surprising then that, as previously noted, the majority decision of the House Electoral Tribunal skirted any reliance on the alleged ipso facto Filipino citizenship of Ong Te under the Philippine Bill of 1902. It is equally not surprising that Ong Chuan, the son of Ong Te and father or private respondent, did not even attempt to claim Filipino citizenship by reason of Ong Te's alleged Filipino citizenship under the Philippine Bill of 1902 but instead applied for Philippine citizenship through naturalization. Nor can it be contended by the private respondent that the House Electoral Tribunal should no longer have reviewed the factual question or issue of Ong Te's citizenship in the light of the resolution of the 1971 Constitutional Convention finding him (Ong Te to have become a Filipino citizen under the Philippine Bill of 1902. The tribunal had to look into the question because the finding that Ong Te had become a Filipino citizen under the Philippine Bill of 1902 was the central core of said 1971 resolution but as held in Lee vs. Commissioners of Immigration: 37 . . . Everytime the citizenship of a person is material or indispensable in a judicial or administrative case, whatever the corresponding Court or administrative authority decides therein as to such citizenship is generally not considered as res adjudicata, hence it has to be threshed out again and again as the occasion may demand. Which finally brings us to the resolution of this Court in Emil L. Ong vs. COMELEC, et al., G.R. No. 67201, 8 May 1984. In connection with said resolution, it is contended by private respondent that the resolution of the 1971 Constitutional Convention in the Emil L. Ong case was elevated to this Court on a question involving Emil L. Ong's disqualification to run for membership in the Batasang Pambansa and that, according to private respondent, this Court allowed the use of the Committee Report to the 1971 Constitutional Convention.

To fully appreciate the implications of such contention, it would help to look into the circumstances of the case brought before this Court in relation to the Court's action or disposition. Emil L. Ong and Edilberto Del Valle were both candidates for the Batasang Pambansa in the 14 May 1984 election. Valle filed a petition for disqualification with the Commission on Election on 29 March 1984 docketed as SPC No. 84-69 contending that Ong is not a natural-born citizen. Ong filed a motion to dismiss the petition on the ground that the judgment of the 1971 Constitutional Convention on his status as a natural-born citizen of the Philippines bars the petitioner from raising the Identical issue before the COMELEC. (G.R. No. 67201, Rollo, p. 94) The motion was denied by the COMELEC, thus, prompting Emil L. Ong to file with this Court a petition for certiorari, prohibition and mandamus with preliminary injunction against the COMELEC, docketed as G.R. No. 67201. In a resolution dated 8 May 1984, this Court resolved to issue a writ of preliminary injunction enjoining respondent COMELEC from holding any further hearing on the disqualification case entitled "Edilberto Del Valle vs. Emil Ong(SPC No. 84-69) except to dismiss the same. (G.R. Nos. 92202-03, Rollo, p. 335) This Court, in explaining its action, held that: Acting on the prayer of the petitioner for the issuance of a Writ of Preliminary Injunction, and considering that at the hearing this morning, it was brought out that the 1971 Constitutional Convention, at its session of November 28, 1972, after considering the Report of its Committee on Election Protests and Credentials, found that the protest questioning the citizenship of the protestee (the petitioner herein) was groundless and dismissed Election Protests Nos. EP 07 and EP 08 filed against said petitioner (p. 237, Rollo), the authenticity of the Minutes of said session as well as of the said Committee's Report having been duly admitted in evidence without objection and bears out, for now, without need for a full hearing, that petitioner is a natural-born citizen, the Court Resolved to ISSUE, effective immediately, a Writ of Preliminary Injunction enjoining respondent COMELEC from holding any further hearing on the disqualification case entitled Edilberto Del Valle vs. Emil Ong (SPC No. 84-69) scheduled at 3:00 o'clock this afternoon, or any other day, except to dismiss the same.This is without prejudice to any appropriate action that private respondent may wish to take after the elections. (emphasis supplied) It is thus clear that the resolution of this Court in G.R. No. 67201 was rendered without the benefit of a hearing on the merits either by the Court or by the COMELEC and merely on the basis of a Committee's Report to the 1971 Constitutional Convention, and that this Court (and this is quite significant) did not foreclose any appropriate action that Del Valle (therein petitioner) may wish to take after the elections. It is thus abundantly clear also that to this Court, the resolution of the 1971 Constitutional Convention recognizing Emil L. Ong as a natural-born citizen under the 1935 Constitution did not foreclose a future or further proceeding in regard to the same question and that, consequently, there is no vested right of Emil L. Ong to such recognition. How much more when the Constitution involved is not the 1935 Constitution but the 1987 Constitution whose provisions were never considered in all such proceedings because the 1987 Constitution was still inexistent. A final word. It is regrettable that one (as private respondent) who unquestionably obtained the highest number of votes for the elective position of Representative (Congressman) to the House of Representatives for the second district of Northern Samar, would have had to cease in office by virtue of this Court's decision, if the full membership of the Court had participated in this case, with the result that the legislative district would cease to have, in the interim, a representative in the House of Representatives. But the fundamental consideration in cases of this nature is the Constitution and only the Constitution. It has to be assumed, therefore, that when the electorate in the second legislative district of Northern Samar cast the majority of their votes for private respondent, they assumed and believed that he was fully eligible and qualified for the office because he is a natural-born Filipino citizen. That erroneous assumption and belief can not prevail over, but must yield to the majesty of the Constitution. This is a sad day for the Constitution. As I see it, the Constitution mandates that members of the House of Representatives should be "naturalborn citizens of the Philippines". The voting majority of the present Court says, "Filipino citizens will do." This is bad enough. What is worse is, the same voting majority, in effect, says, "even aliens will do as well." WHEREFORE, my vote is clear: to declare private respondent Jose L. Ong Chua, Jr., as he clearly is, NOT a natural-born citizen of the Philippines and therefore NOT QUALIFIED to be a Member of the House of Representatives, Congress of the Philippines. Narvasa, J., Paras, J. and Regalado, J., dissenting. SARMIENTO, J., concurring: I concur with the majority. (1) I wish to point out first that the question of citizenship is a question of fact, and as a rule, the Supreme Court leaves facts to the tribunal that determined them. I am quite agreed that the Electoral Tribunal of the House of Representatives, as the "sole judge" of all contests relating to the membership in the House, as follows: Sec. 17. The Senate and the House of Representatives shall each have an Electoral Tribunal which shall be the sole judge of all contests relating to the election, returns, and qualifications of their respective Members. Each Electoral Tribunal shall be composed of nine Members, three of whom shall be Justices of the Supreme Court to be designated by the Chief Justice, and the remaining six shall be Members of the Senate or the House of Representatives, as the case may be, who shall be chosen on the basis of proportional representation from the political parties and the parties or organizations registered under the party-list system represented therein. The senior Justice in the Electoral Tribunal shall be its Chairman. 1

is the best judge of facts and this Court can not substitute its judgment because it thinks it knows better. In the case of Aratuc v. Commission on Elections, 2 it was held that this Court can not review the errors of the Commission on Elections (then the "sole judge" of all election contests) in the sense of reviewing facts and unearthing mistakes and that this Court's jurisdiction is to see simply whether or not it is guilty of a grave abuse of discretion. It is true that the new Constitution has conferred expanded powers on the Court, 3 but as the Charter states, our authority is "to determine whether or not there has been a grave abuse of discretion amounting to lack or excess of jurisdiction on the part of any branch or instrumentality of the Government." 4 It is not to review facts. "Grave abuse of discretion" has been defined as whimsical exercise of power amounting to excess of jurisdiction, or otherwise, to denial of due process of law. 5 I find none of that here. As the majority indicates, Jose Ong's citizenship is a matter of opinion with which men may differ, but certainly, it is quite another thing to say that the respondent Tribunal has gravely abused its discretion because the majority has begged to differ. It does not form part of the duty of the Court to remedy all imagined wrongs committed by the Government. The respondent Tribunal has spoken. According to the Tribunal, Jose Ong is a Filipino citizen and consequently, is possessed of the qualifications to be a member of the House. As the sole judge, precisely, of this question, the Court can not be more popish than the pope. (2) I can not say, in the second place, that the Decision in question stands exactly on indefensible grounds. It is to be noted that Jose Ong had relied on the Report dated September 4, 1972 of the 1971 Constitutional Convention Committee 6 on Election Protests and Credentials, in which the Committees upheld the citizenship, and sustained the qualification to sit as Delegate, of Emil Ong, Jose Ong's full blood brother. According to the Report, Ong Te the Ongs' grandfather, was already a Filipino citizen having complied with the requirements on Filipinization by existing laws for which his successors need not have elected Filipino citizenship. I quote: xxx xxx xxx There is merit in protestee's claim. There can hardly be any doubt that Ong Te protestees's grandfather, was a Spanish subject residing in the Philippines on April 11, 1899, and was therefore one of the many who becameipso facto citizens of the Philippines under the provisions of the Philippine Bill of 1902. Said law expressly declared that all inhabitants of the Philippine Islands who continued to reside therein and who were Spanish subjects on April 11, 1899, as well as their children born subsequent thereto, "shall be deemed and held to be citizens of the Philippine Islands" (Sec. 4, Philippine Bill of 1902). Excepted from the operation of this rule were Spanish subjects who shall have elected to preserve their allegiance to the Crown of Spain in accordance with the Treaty of Paris of December 10, 1898. But under the Treaty of Paris, only Spanish subjects who were natives of Peninsular Spain had the privilege of preserving their Spanish nationality. 7 xxx xxx xxx xxx xxx xxx As earlier noted, protestee's grandfather established residence in the Philippines in 1895, as shown by theRegistro Central de Chinos. He was also issued a certificate of registration. He established a business here, and later acquired real property. Although he went back to China for brief visits, he invariably came back. He even brought his eldest son, Ong Chuan, to live in the Philippines when the latter was only 10 years old. And Ong Chuan was admitted into the country because, as duly noted on his landing certificate, his father, Ong Te had been duly enrolled under CR 1600936755 i.e., as a permanent resident. Indeed, even when Ong Te went back to China in the 1920's for another visit, he left his son, Ong Chuan, who was then still a minor, in the Philippines obviously because he had long considered the Philippines his home. The domicile he established in 1895 is presumed to have continued up to, and beyond, April 11, 1899, for, as already adverted to, a domicile once acquired is not lost until a new one is gained. The only conclusion then can thus be drawn is that Ong Te was duly domiciled in the Philippines as of April 11, 1899, within the meaning of par. 4, Art. 17, of the Civil Code of 1889 and was, consequently, a Spanish subject, he qualified as a Filipino citizen under the provisions of Section 4 of the Philippine Bill of 1902. 8 It is true that Ong Chuan, the Ong brothers' father, subsequently sought naturalization in the belief that he was, all along, a Chinese citizen, but as the Report held: Protestants, however, make capital of the fact that both Ong Te and his son, Ong Chuan (protestee's father), appear to have been registered as Chinese citizens even long after the turn of the century. Worse, Ong Chuan himself believed the was alien, to the extent of having to seek admission as a Pilipino citizen through naturalization proceedings. The point, to our mind, is neither crucial nor substantial. Ong's status as a citizen is a matter of law, rather than of personal belief. It is what the law provides, and not what one thinks his status to be, which determines whether one is a citizen of a particular state or not. Mere mistake or misapprehension as to one's citizenship, it has been held, is not a sufficient cause or reason for forfeiture of Philippine citizenship; it does not even constitute estoppel (Palanca vs. Republic, 80 Phil. 578, 584). Too, estoppel applies only to questions of fact and not of law (Tanada v. Cuenco, L-10520, Feb. 28, 1957). 9 It is to be noted that the Report was unanimously approved by the Committee, and on November 28, 1972, approved without any objection by the Convention in plenary session. 10

I am not, of course, to be mistaken as acting as mouthpiece of Emil Ong, but in all candor, I speak from experience, because when the Convention approved the Report in question, I was one of its vice-presidents and the presiding officer. It is to be noted finally, that the matter was elevated to this Court (on a question involving Emil Ong's qualification to sit as member of the defunct Batasang Pambansa) 11 in which this Court allowed the use of the Committee Report. Faced with such positive acts of the Government, I submit that the question of the Ong's citizenship is a settled matter. Let it rest. It is true that Electoral Protest Nos. EP-07 and EP-08 of the Convention as well as G.R. No. 67201 of this Court, involved Emil Ong and not his brother; I submit, however, that what is sauce for the goose is sauce for the gander. I also submit that the fundamental question is whether or not we will overturn the unanimous ruling of 267 delegates, indeed, also of this Court.

Separate Opinions PADILLA, J., dissenting: I dissent. These separate petitions for certiorari and mandamus seek to annul the decision * of respondent House of Representatives Electoral Tribunal (hereinafter referred to as the tribunal) dated 6 November 1989 which declared private respondent Jose L. Ong, a natural-born citizen of the Philippines and a legal resident of Laoang, Northern Samar, and the resolution of the tribunal dated 22 February 1990 denying petitioners' motions for reconsideration. In G.R. Nos. 92191-92, petitioner Co also prays that the Court declare private respondent Ong not qualified to be a Member of the House of Representatives and to declare him (petitioner Co) who allegedly obtained the highest number of votes among the qualified candidates, the duly elected representative of the second legislative district of Northern Samar. In G.R. Nos. 92202-03, petitioner Balanquit prays that the Court declare private respondent Ong and Co (petitioner in G.R. Nos. 92191-92) not qualified for membership in the House of Representatives and to proclaim him (Balanguit) as the duly elected representative of said district. Petitioners Antonio Y. Co, Sixto T. Balanquit, Jr. and private respondent Jose Ong Chuan, Jr. were among the candidates for the position of Representative or Congressman for the second district of Northern Samar during the 11 May 1987 congressional elections. Private respondent was proclaimed duly-elected on 18 May 1987 with a plurality of some sixteen thousand (16,000) votes over petitioner Co who obtained the next highest number of votes. Petitioners Co and Balanquit then filed separate election protests against private respondent with the tribunal, docketed as HRET Cases Nos. 13 and 15 respectively. Both protests raised almost the same issues and were thus considered and decided jointly by the tribunal. The issues raised before the tribunal were the following: 1. Whether or not protestee (meaning, Ong) is a natural-born citizen of the Philippines in contemplation of Section 6, Article VI of the 1987 Constitution in relation to Sections 2 and 1(3), Article IV thereof; and 2. Whether or not protestee was a resident of Laoang, Northern Samar, in contemplation of Section 6, Article VI of the same Constitution, for a period of not less than one year immediately preceding the congressional elections of May 1987. The respondent tribunal in its decision dated 6 November 1989 held that respondent Jose L. Ong is a natural-born citizen of the Philippines and was a legal resident of Laoang, Northern Samar for the required period prior to the May 1987 congressional elections. He was, therefore, declared qualified to continue in office as Member of the House of Representatives, Congress of the Philippines, representing the second legislative district of Northern Samar. The factual antecedents taken from the consolidated proceedings in the tribunal are the following: 1. The Protestee (Ong) was born on June 19, 1948 to the legal spouses Ong Chuan also known as Jose Ong Chuan and Agrifina E. Lao. His place of birth is Laoang which is now one of the municipalities comprising the province of Northern Samar (Republic Act No. 6132 approved on August 24, 1970 and the Ordinance appended to the 1987 Constitution). 2. On the other hand, Jose Ong Chuan was born in China and arrived in Manila on December 16, 1915. (Exhibit zz) Subsequently thereafter, he took up residence in Laoang, Samar. 3. On February 4, 1932, he married Agrifina E. Lao. Their wedding was celebrated according to the rites and practices of the Roman Catholic Church in the Municipality of Laoang (Exh. E). 4. At the time of her marriage to Jose Ong Chuan, Agrifina E. Lao was a natural-born Filipino citizen, both her parents at the time of her birth being Filipino citizens. (Exhibits E & I)

5. On February 15, 1954, Jose Ong Chuan, desiring to acquire Philippine citizenship, filed his petition for naturalization with the Court of First Instance of Samar, pursuant to Commonwealth Act No. 473, otherwise known as the Revised Naturalization Law. 6. On April 28, 1955, the Court of First Instance of Samar rendered a decision approving the application of Jose Ong Chuan for naturalization and declaring said petitioner a Filipino citizen "with all the rights and privileges and duties, liabilities and obligations inherent to Filipino citizens. (Exh. E) 7. On May 15, 1957, the same Court issued an order: (1) declaring the decision of this Court of April 28, 1955 final and executory; (2) directing the clerk of court to issue the corresponding Certificate of Naturalization in favor of the applicant Ong Chuan who prefers to take his oath and register his name as Jose Ong Chuan. Petitioner may take his oath as Filipino citizen under Ms new christian name, Jose Ong Chuan. (Exh. F) 8. On the same day, Jose Ong Chuan having taken the corresponding oath of allegiance to the Constitution and the Government of the Philippines as prescribed by Section 12 of Commonwealth Act No. 473, was issued the corresponding Certificate of Naturalization. (Exh. G) 9. On November 10, 1970, Emil L. Ong, a full-brother of the protestee and a son born on July 25, 1937 at Laoang, Samar to the spouses Jose Ong Chuan and Agrifina E. Lao, was elected delegate from Northern Samar to the 1971 Constitutional Convention. 10. By protestee's own -testimony, it was established that he had attended grade school in Laoang. Thereafter, he went to Manila where he finished his secondary as well as his college education. While later employed in Manila, protestee however went home to Laoang whenever he had the opportunity to do so, which invariably would be as frequent as twice to four times a year. 11. Protestee also showed that being a native and legal resident of Laoang, he registered as a voter therein and correspondingly voted in said municipality in the 1984 and 1986 elections. 12. Again in December 1986, during the general registration of all voters in the country, Protestee re-registered as a voter in Precinct No. 4 of Barangay Tumaguinting in Laoang. In his voter's affidavit, Protestee indicated that he is a resident of Laoang since birth. (Exh. 7) 1 Petitioners' motions for reconsideration of the tribunal's decision having been denied, petitioners filed the present petitions. In their comments, the respondents first raise the issue of the Court's jurisdiction to review the decision of the House Electoral Tribunal, considering the constitutional provision vesting upon said tribunal the power and authority to act as the sole judge of all contests relating to the qualifications of the Members of the House of Representatives. 2 On the question of this Court's jurisdiction over the present controversy, I believe that, contrary to the respondents' contentions, the Court has the jurisdiction and competence to review the questioned decision of the tribunal and to decide the present controversy. Article VIII, Section I of the 1987 Constitution provides that: Judicial power includes the duty of the courts of justice to settle actual controversies involving rights which are legally demandable and enforceable, and to determine whether or not there has been a grave abuse of discretion amounting to lack or excess of jurisdiction on the part of any branch or instrumentality of the Government. The Constitution, it is true, constitutes the tribunal as the sole judge of all contests relating to the election, returns, and qualifications of Members of the House of Representatives. But as early as 1938, it was held in Morrero vs.Bocar, 3 construing Section 4, Article VI of the 1935 Constitution which provided that ". . . The Electoral Commission shall be the sole judge of all contests relating to the election, returns and qualifications of the Members of the National Assembly," that: The judgment rendered by the (electoral) commission in the exercise of such an acknowledged power is beyond judicial interference, except, in any event, "upon a clear showing of such arbitrary and improvident use of the power as will constitute a denial of due process of law." (Barry vs. US ex rel. Cunningham, 279 US 597; 73 Law. ed., 867; Angara vs. Electoral Commission, 35 Off. Gaz., 23.) And then under the afore-quoted provisions of Article VIII, Section 1 of the 1987 Constitution, this Court is duty-bound to determine whether or not, in an actual controversy, there has been a grave abuse of discretion amounting to lack or excess of jurisdiction on the part of any branch or instrumentality of the Government. The present controversy, it will be observed, involves more than perceived irregularities in the conduct of a congressional election or a disputed appreciation of ballots, in which cases, it may be contended with great legal force and persuasion that the decision of the electoral tribunal should be final and conclusive, for it is, by constitutional directive, made the sole judge of contests relating to such matters. The present controversy, however, involves no less than a determination of whether the qualifications for membership in the House of Representatives, as

prescribed by the Constitution, have been met. Indeed, this Court would be unforgivably remiss in the performance of its duties, as mandated by the Constitution, were it to allow a person, not a natural-born Filipino citizen, to continue to sit as a Member of the House of Representatives, solely because the House Electoral Tribunal has declared him to be so. In such a case, the tribunal would have acted with grave abuse of discretion amounting to lack or excess of jurisdiction as to require the exercise by this Court of its power of judicial review. Besides, the citizenship and residence qualifications of private respondent for the office of Member of the House of Representatives, are here controverted by petitioners who, at the same time, claim that they are entitled to the office illegally held by private respondent. From this additional direction, where one asserts an earnestly perceived right that in turn is vigorously resisted by another, there is clearly a justiciable controversy proper for this Court to consider and decide. Nor can it be said that the Court, in reviewing the decision of the tribunal, asserts supremacy over it in contravention of the time-honored principle of constitutional separation of powers. The Court in this instance simply performs a function entrusted and assigned to it by the Constitution of interpreting, in a justiciable controversy, the pertinent provisions of the Constitution with finality. It is the role of the Judiciary to refine and, when necessary, correct constitutional (and/or statutory) interpretation, in the context of the interactions of the three branches of the government, almost always in situations where some agency of the State has engaged in action that stems ultimately from some legitimate area of governmental power (the Supreme Court in Modern Role, C.B. Sevisher, 1958, p. 36). 4 Moreover, it is decidedly a matter of great public interest and concern to determine whether or not private respondent is qualified to hold so important and high a public office which is specifically reserved by the Constitution only to natural-born Filipino citizens. After a careful consideration of the issues and the evidence, it is my considered opinion that the respondent tribunal committed grave abuse of discretion amounting to lack or excess of jurisdiction in rendering its questioned decision and resolution, for reasons to be presently stated. The Constitution 5 requires that a Member of the House of Representatives must be a natural-born citizen of the Philippines and, on the day of the election, is at least twenty-five (25) years of age, able to read and write, and, except the party-list representatives, a registered voter in the district in which he shall be elected, and a resident thereof for a period of not less than one (1) year immediately preceding the day of the election. Article IV, Section 2 of the 1987 Constitution defines natural-born (Filipino) citizens as: Natural-born citizens are those who are citizens of the Philippines from birth without having to perform any act to acquire or perfect their Philippine citizenship. Those who elect Philippine citizenship in accordance with paragraph (3), Section I hereof shall be deemed natural-born citizen, Article IV, Section 1, paragraph (3) of the 1987 Constitution provides that: Section 1. The following are citizens of the Philippines: xxx xxx xxx (3) Those born before January 17, 1973, of Filipino mothers, who elect Philippine citizenship upon reaching the age of majority. The Court in this case is faced with the duty of interpreting the above-quoted constitutional provisions. The first sentence of Section 2 of Article IV states the basic definition of a natural-born Filipino citizen. Does private respondent fall within said definition? To the respondent tribunal, Protestee may even be declared a natural-born citizen of the Philippines under the first sentence of Sec. 2 of Article IV of the 1987 Constitution because he did not have "to perform any act to acquire or perfect his Philippine citizenship." It bears to repeat that on 15 May 1957, while still a minor of 9 years he already became a Filipino citizen by declaration of law. Since his mother was a natural-born citizen at the time of her marriage, protestee had an inchoate right to Philippine citizenship at the moment of his birth and, consequently the declaration by virtue of Sec. 15 of CA 473 that he was a Filipino citizen retroacted to the moment of his birth without his having to perform any act to acquire or perfect such Philippine citizenship. 6 I regret that I am neither convinced nor persuaded by such kaleidoscopic ratiocination. The records show that private respondent was born on 19 June 1948 to the spouses Jose Ong Chuan, a Chinese citizen, and Agrifina E. Lao, a natural-born Filipino citizen, in Laoang, Northern Samar. In other words, at birth, private respondent was a Chinese citizen (not a natural-born Filipino citizen) because his father was then a Chinese citizen (not a naturalized Filipino citizen). Under the 1935 Constitution which was enforced at the time of private respondent's birth on 19 June 1948, only those whose fathers were citizens of the Philippines were considered Filipino citizens. Those whose mothers were citizens of the Philippines had to elect Philippine citizenship upon reaching the age of majority, in order to be considered Filipino citizens. 7 Following the basic definition in the 1987 Constitution of a natural-born citizen, in relation to the 1935 Constitution, private respondent is not a natural-born Filipino citizen, having been born a Chinese citizen by virtue of the Chinese citizenship of his father at the time of his birth, although from birth, private respondent had the right to elect Philippine citizenship, the citizenship of his mother, but only upon his reaching the age of majority.

While under Section 15 of the Revised Naturalization Law (C.A. 473) minor children of a naturalized citizen (father), who were born in the Philippines prior to the naturalization of the parent automatically become Filipino citizens, 8 this does not alter the fact that private respondent was not born to a Filipino father, and the operation of Section 15 of CA 473 did not confer upon him the status of a natural-born citizen merely because he did not have to perform any act to acquire or perfect his status as a Filipino citizen. But even assuming arguendo that private respondent could be considered a natural-born citizen by virtue of the operation of CA 473, petitioners however contend that the naturalization of private respondent's father was invalid and void from the beginning, and, therefore, private respondent is not even a Filipino citizen. Respondent tribunal in its questioned decision ruled that only a direct proceeding for nullity of naturalization as a Filipino citizen is permissible, and, therefore, a collateral attack on Ong Chuan's naturalization is barred in an electoral contest which does not even involve him (Ong Chuan). Private respondent, for his part, avers in his Comment that the challenge against Ong Chuan's naturalization must emanate from the Government and must be made in a proper/appropriate and direct proceeding for de-naturalization directed against the proper party, who in such case is Ong Chuan, and also during his lifetime. A judgment in a naturalization proceeding is not, however, afforded the character of impregnability under the principle of res judicata. 9 Section 18 of CA 473 provides that a certificate of naturalization may be cancelled upon motion made in the proper proceeding by the Solicitor General or his representative, or by the proper provincial fiscal. In Republic vs. Go Bon Lee, 10 this Court held that: An alien friend is offered under certain conditions the privilege of citizenship. He may accept the offer and become a citizen upon compliance with the prescribed conditions, but not otherwise. His claim is of favor, not of right. He can only become a citizen upon and after a strict compliance with the acts of Congress. An applicant for this high privilege is bound, therefore, to conform to the terms upon which alone the right he seeks can be conferred. It is his province, and he is bound, to see that the jurisdictional facts upon which the grant is predicated actually exist and if they do not he takes nothing by this paper grant. xxx xxx xxx Congress having limited this privilege to a specified class of persons, no other person is entitled to such privilege, nor to a certificate purporting to grant it, and any such certificate issued to a person not so entitled to receive it must be treated as a mere nullity, which confers no legal rights as against the government, from which it has been obtained without warrant of law. "Naturalization is not a right, but a privilege of the most discriminating as well as delicate and exacting nature, affecting public interest of the highest order, and which may be enjoyed only under the precise conditions prescribed by law therefor." 11 Considering the legal implications of the allegation made by the petitioners that the naturalization of private respondent's father Ong Chuan, is a nullity, the Court should make a ruling on the validity of said naturalization proceedings. This course of action becomes all the more inevitable and justified in the present case where, to repeat for stress, it is claimed that a foreigner is holding a public office. 12 It cannot be overlooked, in this connection, that the citizenship of private respondent is derived from his father. If his father's Filipino citizenship is void from the beginning, then there is nothing from which private respondent can derive his own claimed Filipino citizenship. For a spring cannot rise higher than its source. And to allow private respondent to avail of the privileges of Filipino citizenship by virtue of a void naturalization of his father, would constitute or at least sanction a continuing offense against the Constitution. The records show that private respondent's father, Jose Ong Chuan, took the oath of allegiance to the Constitution and the Philippine Government, as prescribed by Section 12 of CA 473 on the same day (15 May 1957) that the CFI issued its order directing the clerk of court to issue the corresponding Certificate of Naturalization and for the applicant to take the oath of allegiance. However, it is settled that an order granting a petition to take the requisite oath of allegiance of one who has previously obtained a decision favorable to his application for naturalization, is appealable. It is, therefore, improper and illegal to authorize the taking of said oath upon the issuance of said order and before the expiration of the reglementary period to perfect any appeal from said order. 13 In Cua Sun Ke vs. Republic, 14 this Court held that: Administration of the oath of allegiance on the same day as issuance of order granting citizenship is irregular and makes the proceedings so taken null and void. (Republic vs. Guy, 115 SCRA 244 [1982]; citing the case of Ong So vs. Republic of the Philippines, 121 Phil. 1381). It would appear from the foregoing discussion that the naturalization of Jose Ong Chuan (private respondent's father) was null and void. It follows that the private respondent did not acquire any legal rights from the void naturalization of his father and thus he cannot himself be considered a Filipino citizen, more so, a natural-born Filipino citizen. But assuming that the CFI order of 15 May 1957 directing the clerk of court to issue the certificate of naturalization to Ong Chuan and for the latter to take the oath of allegiance was final and not appealable, the resulting naturalization of Ong Chuan effected, as previously stated, an

automatic naturalization of private respondent, then a minor, as a Filipino citizen on 15 May 1957, but not his acquisition or perfection of the status of a natural-born Filipino citizen. Let us now look into the question of whether or not private respondent acquired the status of a natural-born Filipino citizen by reason of the undisputed fact that his mother was a natural-born Filipino citizen. This in turn leads us to an examination of the second sentence in Article IV, Section 2 of the 1987 Constitution. It expands, in a manner of speaking, in relation to Section 1, paragraph (3) of the same Article IV, the status of a natural-born Filipino citizen to those who elect Philippine citizenship upon reaching the age of majority. The right or privilege of election is available, however, only to those born to Filipino mothers under the 1935 Constitution, and before the 1973 Constitution took effect on 17 January 1973. The petitioners contend that the respondent tribunal acted in excess of its jurisdiction or gravely abused its discretion as to exceed its jurisdiction in "distorting" the conferment by the 1987 Constitution of the status of "natural-born" Filipino citizen on those who elect Philippine citizenship all in its strained effort, according to petitioners, to support private respondent's qualification to be a Member of the House of Representatives. 15 Petitioners argue that the clear, unambiguous wording of section 1(3) of Article IV of the 1987 Constitution contemplates that only the legitimate children of Filipino mothers with alien father, born before 17 January 1973 and who would reach the age of majority (and thus elect Philippine citizenship) after the effectivity of the 1987 Constitution are entitled to the status of natural-born Filipino citizen. 16 The respondent tribunal in resolving the issue of the constitutional provisions' interpretation, found reason to refer to the interpellations made during the 1986 Constitutional Commission. It said: That the benevolent provisions of Sections 2 and 1(3) of Article IV of the 1987 Constitution was (sic) intended by its (sic) framers to be endowed, without distinction, to all Filipinos by election pursuant to the 1935 Constitution is more than persuasively established by the extensive interpellations and debate on the issue as borne by the official records of the 1986 Constitutional Commission. 17 Although I find the distinction as to when election of Philippine citizenship was made irrelevant to the case at bar, since private respondent, contrary to the conclusion of the respondent tribunal, did not elect Philippine citizenship, as provided by law, I still consider it necessary to settle the controversy regarding the meaning of the constitutional provisions in question. I agree with respondent tribunal that the debates, interpellations petitions and opinions expressed in the 1986 Constitutional Commission may be resorted to in ascertaining the meaning of somewhat elusive and even nebulous constitutional provisions. Thus The ascertainment of that intent is but in keeping with the fundamental principle of constitutional construction that the intent of the framers of the organic law and of the people adopting it should be given effect. The primary task in constitutional construction is to ascertain and thereafter assure the realization of the purpose of the framers and of the people in the adoption of the Constitution. It may also be safely assumed that the people in ratifying the constitution were guided mainly by the explanation offered by the framers. 18 The deliberations of the 1986 Constitutional Commission relevant to Section 2, Article IV in relation to Section 1(3) of the same Article, appear to negate the contention of petitioners that only those born to Filipino mothers before 17 January 1973 and who would elect Philippine citizenship after the effectivity of the 1987 Constitution, are to be considered natural-born Filipino citizens. During the free-wheeling discussions on citizenship, Commissioner Treas specifically asked Commissioner Bernas regarding the provisions in question, thus: MR. TRENAS: The Committee on Citizenship, Bill of Rights, Political Rights and Obligations and Human Rights has more or less decided to extend the interpretation of who is a natural-born Filipino citizen as provided in Section 4 of the 1973 Constitution, by adding that persons who have elected Philippine citizenship under the 1935 Constitution shall be considered natural-born. Am I right, Mr. Presiding Officer? FR BERNAS: Yes. MR. TRENAS: And does the Commissioner think that tills addition to Section 4 of the 1973 Constitution would be contrary to the spirit of that section? FR BERNAS: Yes, we are quite aware that it is contrary to the letter really. But whether it is contrary to the spirit is something that has been debated before and is being debated even now. We will recall that during the 1971 Constitutional Convention, the status of natural-born citizenship of one of the delegates, Mr. Ang, was challenged precisely because he was a citizen by election. Finally, the 1971 Constitutional Convention considered him a natural-born citizen, one of the requirements to be a Member of the 1971 Constitutional Convention. The reason behind that decision was that a person under his circumstances already had the inchoate right to be a citizen by the fact that the mother was a Filipino. And as a matter of fact, the 1971 Constitutional Convention formalized that recognition by adopting paragraph 2 of Section 1 of the 1971 Constitution. So, the entire purpose of this proviso is simply to perhaps remedy whatever injustice there may be so that these people born before January 17, 1973 who are not naturalized and people who are not natural born but who are in the same situation as we are considered naturalborn citizens. So, the intention of the Committee in proposing this is to equalize their status. 19

When asked to clarify the provision on natural-born citizens, Commissioner Bernas replied to Commissioner Azcuna thus: MR. AZCUNA: With respect to the proviso in Section 4, would this refer only to those who elect Philippine citizenship after the effectivity of the 1973 Constitution or would it also cover those who elected it under the 1935 Constitution? FR BERNAS: It would apply to anybody who elected Philippine citizenship by virtue of the provision of the 1935 Constitution, whether the election was done before or after 17 January 1973.20 And during the period of amendments. Commissioner Rodrigo explained the purpose of what now appear as Section 2 and Section 1, paragraph (3) of Article IV of the 1987 Constitution, thus: MR. RODRIGO: The purpose of that proviso is to remedy an inequitable situation. Between 1935 and 1973, when we were under the 1935 Constitution, those born of Filipino fathers but alien mothers were natural-born Filipinos. However, those born of Filipino mothers but alien fathers would have to elect Philippine citizenship upon reaching the age of majority; and, if they do elect, they become Filipino citizens, yet, but not natural-born Filipino citizens. The 1973 Constitution equalized the status of those born of Filipino mothers and those born of Filipino fathers. So that from January 17, 1973 when the 1973 Constitution took effect, those born of Filipino mothers but of alien fathers are natural-born Filipino citizens. Also, those who are born of Filipino fathers and alien mothers are natural-born Filipino citizens. If the 1973 Constitution equalized the status of a child born of a Filipino mother and that born of a Filipino father, why do we not give a chance to a child born before January 17, 1973, if and when he elects Philippine citizenship, to be in the same status as one born of a Filipino father namely, natural-born citizen. Another thing I stated is equalizing the status of a father and a mother vis-a-vis the child. I would like to state also that we showed equalize the status of a child born of a Filipino mother the day before January 17, 1973 and a child born also of a Filipino mother on January 17 or 24 hours later. A child born of a Filipino mother but an alien father one day before January 17, 1973 is a Filipino citizen, if he elects Philippine citizenship, but he is not a natural-born Filipino citizen. However, the other child who luckily was born 24 hours later maybe because of parto laborioso is a natural-born Filipino citizen. 21 It would appear then that the intent of the framers of the 1987 Constitution in defining a natural-born Filipino citizen was to equalize the position of Filipino fathers and Filipino mothers as to their children becoming natural-born Filipino citizens. In other words, after 17 January 1973, effectivity date of the 1973 Constitution, all those born of Filipino fathers (with alien spouse) or Filipino mothers (with alien spouse) are naturalborn Filipino citizens. But those born to Filipino mothers prior to 17 January 1973 must still elect Philippine citizenship upon their reaching the age of majority, in order to be deemed natural-born Filipino citizens. The election, which is related to the attainment of the age of majority, may be made before or after 17 January 1973. This interpretation appears to be in consonance with the fundamental purpose of the Constitution which is to protect and enhance the people's individual interests, 22 and to foster equality among them. Since private respondent was born on 19 June 1948 (or before 17 January 1973) to a Filipino mother (with an alien spouse) and should have elected Philippine citizenship on 19 June 1969 (when he attained the age of majority), or soon thereafter, in order to have the status of a natural-born Filipino citizen under the 1987 Constitution, the vital question is: did private respondent really elect Philippine citizenship? As earlier stated, I believe that private respondent did not elect Philippine citizenship, contrary to the ruling of the respondent tribunal. The respondent tribunal, on this issue, ruled as follows: Where a person born to a Filipino mother and an alien father had exercised the right of suffrage when he came of age, the same constitutes a positive act of election of Philippine citizenship. (Florencio vs. Mallare) [ sic] The acts of the petitioner in registering as a voter, participating in elections and campaigning for certain candidates were held by the Supreme Court as sufficient to show his preference for Philippine citizenship. Accordingly, even without complying with the formal requisites for election, the petitioner's Filipino citizenship was judicially upheld. 23 I find the above ruling of the respondent tribunal to be patently erroneous and clearly untenable, as to amount to grave abuse of discretion. For it is settled doctrine in this jurisdiction that election of Philippine citizenship must be made in accordance with Commonwealth Act 625. Sections 1 and 2 24 of the Act mandate that the option to elect Philippine citizenship must be effected expressly not impliedly. The respondent tribunal cites In re: Florencio Mallare 25 which held that Esteban Mallare's exercise of the right of suffrage when he came of age, constituted a positive act of election of Philippine citizenship. Mallare, cited by respondent tribunal as authority for the doctrine of implied election of Philippine citizenship, is not applicable to the case at bar. The respondent tribunal failed to consider that Esteban Mallare reached the age of majority in 1924, or seventeen (17) years before CA 625 was approved and, more importantly, eleven (11) years before the 1935 Constitution (which granted the right of election) took effect. To quote Mr. Justice Fernandez in Mallare: Indeed, it would be unfair to expect the presentation of a formal deed to that effect considering that prior to the enactment of Commonwealth Act 625 on June 7, 1941, no particular proceeding was required to exercise the option to elect

Philippine citizenship, granted to the proper party by Section 1, subsection 4, Article IV of the 1935 Philippine Constitution. 26 Moreover, Esteban Mallare was held to be a Filipino citizen because he was an illegitimate (natural) child of a Filipino mother and thus followed her citizenship. I therefore agree with the petitioners' submission that, inciting the Mallare case, the respondent tribunal had engaged in an obiter dictum. The respondent tribunal also erred in ruling that by operation of CA 473, the Revised Naturalization Law, providing for private respondent's acquisition of Filipino citizenship by reason of the naturalization of his father, the law itself had already elected Philippine citizenship for him. For, assuming arguendo that the naturalization of private respondent's father was valid, and that there was no further need for private respondent to elect Philippine citizenship (as he had automatically become a Filipino citizen) yet, this did not mean that the operation of the Revised Naturalization Law amounted to an election by him of Philippine citizenship as contemplated by the Constitution. Besides, election of Philippine citizenship derived from one's Filipino mother, is made upon reaching the age of majority, not during one's minority. There is no doubt in my mind, therefore, that private respondent did not elect Philippine citizenship upon reaching the age of majority in 1969 or within a reasonable time thereafter as required by CA 625. Consequently, he cannot be deemed a natural-born Filipino citizen under Sections 2 and 1(3), Article IV of the 1987 Constitution. Based on all the foregoing considerations and premises, I am constrained to state that private respondent is not a natural-born citizen of the Philippines in contemplation of Section 6, Article VI of the 1987 Constitution in relation to Sections 2 and 1(3), Article IV thereof, and hence is disqualified or ineligible to be a Member of the House of Representatives. At this point, I find it no longer necessary to rule on the issue of required residence, inasmuch as the Constitution requires that a Member of the House of Representatives must be both a natural-born Filipino citizen and a resident for at least one (1) year in the district in which he shall be elected. The next question that comes up is whether or not either of the petitioners can replace private respondent as the Representative of the second legislative district of Northern Samar in the House of Representatives. I agree with respondent tribunal that neither of the petitioners may take the place of private respondent in the House of Representatives representing the second district of Northern Samar. The ruling of this Court in Ramon L.Labo, Jr. vs. The Commission on Elections (COMELEC) EN BANC and Luis L. Lardizabal, 27 is controlling. There we held that Luis L. Lardizabal, who filed the quo warranto petition, could not replace Ramon L. Labo, Jr. as mayor of Baguio City for the simple reason that as he obtained only the second highest number of votes in the election, he was obviously not the choice of the people of Baguio City for mayor of that City. A petition alleging that the candidate-elect is not qualified for the office is, in effect, a quo warranto proceeding even if it is labelled an election protest. 28 It is a proceeding to unseat the ineligible person from office but not necessarily to install the protestant in his place. 29 The general rule is that the fact that a plurality or a majority of the votes are cast for an ineligible candidate in an election does not entitle the candidate receiving the next highest number of votes to be declared elected. In such a case, the electors have failed to make a choice and the election is a nullity. 30 Sound policy dictates that public elective offices are filled by those who have the highest number of votes cast in the election for that office, and it is a fundamental idea in all republican forms of government that no one can be declared elected and no measure can be declared carried unless he or it receives a majority or plurality of the legal votes cast in the election. (20 Corpus Juris 2nd, S 243, p. 676). As early as 1912, this Court has already declared that the candidate who lost in an election cannot be proclaimed the winner in the event that the candidate who won is found ineligible for the office to which he was elected. This was the ruling in Topacio v. Paredes (23 Phil. 238) Again, the effect of a decision that a candidate is not entitled to the office because of fraud or irregularities in the election is quite different from that produced by declaring a person ineligible to hold such an office. . . . If it be found that the successful candidate (according to the board of canvassers) obtained a plurality in an illegal manner, and that another candidate was the real victor, the former must retire in favor of the latter. In the other case, there is not, strictly speaking, a contest, as the wreath of victory cannot be transferred from an ineligible to any other candidate when the sole question is the eligibility of the one receiving a plurality of the legally cast ballots. . . . 31 The recognition of Emil L. Ong by the 1971 Constitutional Convention as a natural-born Filipino citizen, in relation to the present case. Private respondent, as previously stated, is a full brother of Emil L. Ong, both of them having the same father and mother. Private respondent, relying on a resolution of the 1971 Constitutional Convention 32 to the effect that Emil L. Ong was a natural-born Filipino citizen, alleged before the House Electoral Tribunal that, by analogy, he is himself a natural-born Filipino citizen. This submission, while initially impressive, is, as will now be shown, flawed and not supported by the evidence. Not even the majority decision of the electoral tribunal adopted the same as the basis of its decision in favor of private respondent. The tribunal, in reference to this submission, said:

Be that as it may and in the light of the Tribunal's disposition of protestee's citizenship based on an entirely different set of circumstances, apart from the indisputable fact that the matters attempted to be brought in issue in connection therewith are too far removed in point of time and relevance from the decisive events relied upon by the Tribunal, we view these two issues as being already inconsequential. 33 The electoral tribunal (majority) instead chose to predicate its decision on the alleged citizenship by naturalization of private respondent's father (Ong Chuan) and on the alleged election of Philippine citizenship by private respondent. Emil L. Ong, was elected delegate to the 1971 Constitutional Convention. Electoral protests, numbers EP-07 and EP-08, were filed by Leonardo D. Galing and Gualberto D. Luto against Emil L. Ong, contesting his citizenship qualification. The Committee on Election Protests Credentials of the 1971 Contitution Convention heard the protests and submitted to the Convention a report dated 4 September 1972, the dispositive portion of which stated: It appearing that protestee's grandfather was himself a Filipino citizen under the provisions of the Philippine Bill of 1902 and the Treaty of Paris of December 10, 1898, thus conferring upon protestee's own father, Ong Chuan, Philippine citizenship at birth, the conclusion is inescapable that protestee himself is a natural-born citizen, and is therefore qualified to hold the office of delegate to the Constitutional Convention. 34 On 28 November 1972, during a plenary session of the 1971 Constitutional Convention, the election protests filed against Emil L. Ong were dismissed, following the report of the Committee on Election Protests and Credentials. 35 It is evident, up to this point, that the action of the 1971 Constitutional Convention in the case of Emil L. Ong is, to say the least, inconclusive to the case at bar, because a) the 1971 Constitutional Convention decision in the Emil L. Ong case involved the 1935 Constitution; the present case, on the other hand involves the 1987 Constitution: b) the 1935 Constitution contained no specific definition of a "natural-born citizen" of the Philippines; the 1987 Constitution contains a precise and specific definition of a "natural-born citizen" of the Philippines in Sec. 2, Art. IV thereof and private respondent does not qualify under such definition in the 1987 Constitution; c) the decision of the 1971 Constitutional Convention in the case of Emil L. Ong was a decision of apolitical body, not a court of law. And, even if we have to take such a decision as a decision of aquasi-judicial body (i.e., a political body exercising quasi-judicial functions), said decision in the Emil L. Ong case can not have the category or character of res judicata in the present judicial controversy, because between the two (2) cases, there is no identity of parties (one involves Emil L. Ong, while the other involves private respondent) and, more importantly, there is no identity of causes of action because the first involves the 1935 Constitution while the second involves the 1987 Constitution. But even laying aside the foregoing reasons based on procedural rules and logic, the evidence submitted before the electoral tribunal and, therefore, also before this Court, does not support the allegations made by Emil L. Ong before the 1971 Constitutional Convention and inferentially adopted by private respondent in the present controversy. This leads us to an interesting inquiry and finding. The 1971 Constitutional Convention in holding that Emil L. Ong was a "natural-born citizen" of the Philippines under the 1935 Constitution laid stress on the "fact" and this appears crucial and central to its decision that Emil L. Ong's grandfather, Ong Te became a Filipino citizen under the Philippine Bill of 1902 and, therefore, his descendants like Emil L. Ong (and therefore, also private respondent) became natural-born Filipinos. The 1971 Constitutional Convention said: Ong Te Emil Ong's grandfather, was a Spanish subject residing in the Philippines on April 11, 1899 and was therefore one of the many who became ipso facto citizens of the Philippines under the provisions of the Philippine Bill of 1902. Said law expressly declared that all inhabitants of the Philippine Islands who continued to reside therein and who were Spanish subjects on April 11, 1899 as well as their children born subsequent thereto, "shall be deemed and held to be citizens of the Philippine Islands." (Section 4, Philippine Bill of 1902). 36 The "test" then, following the premises of the 1971 Constitutional Convention, is whether or not Ong Te private respondent's and Emil L. Ong's grandfather was "an inhabitant of the Philippines who continued to reside therein and was a Spanish subject on April 11, 1899." If he met these requirements of the Philippine Bill of 1902, then, Ong Te was a Filipino citizen; otherwise, he was not a Filipino citizen. Petitioners (protestants) submitted and offered in evidence before the House Electoral Tribunal exhibits W, X, Y, Z ,AA, BB, CC, DD and EE which are copies of entries in the "Registro de Chinos" from years 1896 to 1897 which show that Ong Te was not listed as an inhabitant of Samar where he is claimed to have been a resident. Petitioners (protestants) also submitted and offered in evidence before the House Electoral Tribunal exhibit V, a certification of the Chief of the Archives Division, Records and Management and Archives Office, stating that the name of Ong Te does not appear in the "Registro Central de Chinos" for the province of Samar for 1895. These exhibits prove or at least, as petitioners validly argue, tend to prove that Ong Te was NOT a resident of Samar close to 11 April 1899 and, therefore, could not continue residing in Samar, Philippines after 11 April 1899, contrary to private respondent's pretense. In the face of these proofs or evidence, private respondent FAILED TO PRESENT ANY REBUTTAL OR COUNTERVAILING EVIDENCE, except the decision of the 1971 Constitutional Convention in the case of Emil L. Ong, previously discussed. It is not surprising then that, as previously noted, the majority decision of the House Electoral Tribunal skirted any reliance on the alleged ipso facto Filipino citizenship of Ong Te under the Philippine Bill of 1902. It is equally not surprising that Ong Chuan, the son of Ong Te and father or

private respondent, did not even attempt to claim Filipino citizenship by reason of Ong Te's alleged Filipino citizenship under the Philippine Bill of 1902 but instead applied for Philippine citizenship through naturalization. Nor can it be contended by the private respondent that the House Electoral Tribunal should no longer have reviewed the factual question or issue of Ong Te's citizenship in the light of the resolution of the 1971 Constitutional Convention finding him (Ong Te to have become a Filipino citizen under the Philippine Bill of 1902. The tribunal had to look into the question because the finding that Ong Te had become a Filipino citizen under the Philippine Bill of 1902 was the central core of said 1971 resolution but as held in Lee vs. Commissioners of Immigration: 37 . . . Everytime the citizenship of a person is material or indispensable in a judicial or administrative case, whatever the corresponding Court or administrative authority decides therein as to such citizenship is generally not considered as res adjudicata, hence it has to be threshed out again and again as the occasion may demand. Which finally brings us to the resolution of this Court in Emil L. Ong vs. COMELEC, et al., G.R. No. 67201, 8 May 1984. In connection with said resolution, it is contended by private respondent that the resolution of the 1971 Constitutional Convention in the Emil L. Ong case was elevated to this Court on a question involving Emil L. Ong's disqualification to run for membership in the Batasang Pambansa and that, according to private respondent, this Court allowed the use of the Committee Report to the 1971 Constitutional Convention. To fully appreciate the implications of such contention, it would help to look into the circumstances of the case brought before this Court in relation to the Court's action or disposition. Emil L. Ong and Edilberto Del Valle were both candidates for the Batasang Pambansa in the 14 May 1984 election. Valle filed a petition for disqualification with the Commission on Election on 29 March 1984 docketed as SPC No. 84-69 contending that Ong is not a natural-born citizen. Ong filed a motion to dismiss the petition on the ground that the judgment of the 1971 Constitutional Convention on his status as a natural-born citizen of the Philippines bars the petitioner from raising the Identical issue before the COMELEC. (G.R. No. 67201, Rollo, p. 94) The motion was denied by the COMELEC, thus, prompting Emil L. Ong to file with this Court a petition for certiorari, prohibition and mandamus with preliminary injunction against the COMELEC, docketed as G.R. No. 67201. In a resolution dated 8 May 1984, this Court resolved to issue a writ of preliminary injunction enjoining respondent COMELEC from holding any further hearing on the disqualification case entitled "Edilberto Del Valle vs. Emil Ong(SPC No. 84-69) except to dismiss the same. (G.R. Nos. 92202-03, Rollo, p. 335) This Court, in explaining its action, held that: Acting on the prayer of the petitioner for the issuance of a Writ of Preliminary Injunction, and considering that at the hearing this morning, it was brought out that the 1971 Constitutional Convention, at its session of November 28, 1972, after considering the Report of its Committee on Election Protests and Credentials, found that the protest questioning the citizenship of the protestee (the petitioner herein) was groundless and dismissed Election Protests Nos. EP 07 and EP 08 filed against said petitioner (p. 237, Rollo), the authenticity of the Minutes of said session as well as of the said Committee's Report having been duly admitted in evidence without objection and bears out, for now, without need for a full hearing, that petitioner is a natural-born citizen, the Court Resolved to ISSUE, effective immediately, a Writ of Preliminary Injunction enjoining respondent COMELEC from holding any further hearing on the disqualification case entitled Edilberto Del Valle vs. Emil Ong (SPC No. 84-69) scheduled at 3:00 o'clock this afternoon, or any other day, except to dismiss the same.This is without prejudice to any appropriate action that private respondent may wish to take after the elections. (emphasis supplied) It is thus clear that the resolution of this Court in G.R. No. 67201 was rendered without the benefit of a hearing on the merits either by the Court or by the COMELEC and merely on the basis of a Committee's Report to the 1971 Constitutional Convention, and that this Court (and this is quite significant) did not foreclose any appropriate action that Del Valle (therein petitioner) may wish to take after the elections. It is thus abundantly clear also that to this Court, the resolution of the 1971 Constitutional Convention recognizing Emil L. Ong as a natural-born citizen under the 1935 Constitution did not foreclose a future or further proceeding in regard to the same question and that, consequently, there is no vested right of Emil L. Ong to such recognition. How much more when the Constitution involved is not the 1935 Constitution but the 1987 Constitution whose provisions were never considered in all such proceedings because the 1987 Constitution was still inexistent. A final word. It is regrettable that one (as private respondent) who unquestionably obtained the highest number of votes for the elective position of Representative (Congressman) to the House of Representatives for the second district of Northern Samar, would have had to cease in office by virtue of this Court's decision, if the full membership of the Court had participated in this case, with the result that the legislative district would cease to have, in the interim, a representative in the House of Representatives. But the fundamental consideration in cases of this nature is the Constitution and only the Constitution. It has to be assumed, therefore, that when the electorate in the second legislative district of Northern Samar cast the majority of their votes for private respondent, they assumed and believed that he was fully eligible and qualified for the office because he is a natural-born Filipino citizen. That erroneous assumption and belief can not prevail over, but must yield to the majesty of the Constitution. This is a sad day for the Constitution. As I see it, the Constitution mandates that members of the House of Representatives should be "naturalborn citizens of the Philippines". The voting majority of the present Court says, "Filipino citizens will do." This is bad enough. What is worse is, the same voting majority, in effect, says, "even aliens will do as well." WHEREFORE, my vote is clear: to declare private respondent Jose L. Ong Chua, Jr., as he clearly is, NOT a natural-born citizen of the Philippines and therefore NOT QUALIFIED to be a Member of the House of Representatives, Congress of the Philippines. Narvasa, J., Paras, J. and Regalado, J., dissenting.

SARMIENTO, J., concurring: I concur with the majority. (1) I wish to point out first that the question of citizenship is a question of fact, and as a rule, the Supreme Court leaves facts to the tribunal that determined them. I am quite agreed that the Electoral Tribunal of the House of Representatives, as the "sole judge" of all contests relating to the membership in the House, as follows: Sec. 17. The Senate and the House of Representatives shall each have an Electoral Tribunal which shall be the sole judge of all contests relating to the election, returns, and qualifications of their respective Members. Each Electoral Tribunal shall be composed of nine Members, three of whom shall be Justices of the Supreme Court to be designated by the Chief Justice, and the remaining six shall be Members of the Senate or the House of Representatives, as the case may be, who shall be chosen on the basis of proportional representation from the political parties and the parties or organizations registered under the party-list system represented therein. The senior Justice in the Electoral Tribunal shall be its Chairman. 1 is the best judge of facts and this Court can not substitute its judgment because it thinks it knows better. In the case of Aratuc v. Commission on Elections, 2 it was held that this Court can not review the errors of the Commission on Elections (then the "sole judge" of all election contests) in the sense of reviewing facts and unearthing mistakes and that this Court's jurisdiction is to see simply whether or not it is guilty of a grave abuse of discretion. It is true that the new Constitution has conferred expanded powers on the Court, 3 but as the Charter states, our authority is "to determine whether or not there has been a grave abuse of discretion amounting to lack or excess of jurisdiction on the part of any branch or instrumentality of the Government." 4 It is not to review facts. "Grave abuse of discretion" has been defined as whimsical exercise of power amounting to excess of jurisdiction, or otherwise, to denial of due process of law. 5 I find none of that here. As the majority indicates, Jose Ong's citizenship is a matter of opinion with which men may differ, but certainly, it is quite another thing to say that the respondent Tribunal has gravely abused its discretion because the majority has begged to differ. It does not form part of the duty of the Court to remedy all imagined wrongs committed by the Government. The respondent Tribunal has spoken. According to the Tribunal, Jose Ong is a Filipino citizen and consequently, is possessed of the qualifications to be a member of the House. As the sole judge, precisely, of this question, the Court can not be more popish than the pope. (2) I can not say, in the second place, that the Decision in question stands exactly on indefensible grounds. It is to be noted that Jose Ong had relied on the Report dated September 4, 1972 of the 1971 Constitutional Convention Committee 6 on Election Protests and Credentials, in which the Committees upheld the citizenship, and sustained the qualification to sit as Delegate, of Emil Ong, Jose Ong's full blood brother. According to the Report, Ong Te the Ongs' grandfather, was already a Filipino citizen having complied with the requirements on Filipinization by existing laws for which his successors need not have elected Filipino citizenship. I quote: xxx xxx xxx There is merit in protestee's claim. There can hardly be any doubt that Ong Te protestees's grandfather, was a Spanish subject residing in the Philippines on April 11, 1899, and was therefore one of the many who becameipso facto citizens of the Philippines under the provisions of the Philippine Bill of 1902. Said law expressly declared that all inhabitants of the Philippine Islands who continued to reside therein and who were Spanish subjects on April 11, 1899, as well as their children born subsequent thereto, "shall be deemed and held to be citizens of the Philippine Islands" (Sec. 4, Philippine Bill of 1902). Excepted from the operation of this rule were Spanish subjects who shall have elected to preserve their allegiance to the Crown of Spain in accordance with the Treaty of Paris of December 10, 1898. But under the Treaty of Paris, only Spanish subjects who were natives of Peninsular Spain had the privilege of preserving their Spanish nationality. 7 xxx xxx xxx xxx xxx xxx As earlier noted, protestee's grandfather established residence in the Philippines in 1895, as shown by theRegistro Central de Chinos. He was also issued a certificate of registration. He established a business here, and later acquired real property. Although he went back to China for brief visits, he invariably came back. He even brought his eldest son, Ong Chuan, to live in the Philippines when the latter was only 10 years old. And Ong Chuan was admitted into the country because, as duly noted on his landing certificate, his father, Ong Te had been duly enrolled under CR 1600936755 i.e., as a permanent resident. Indeed, even when Ong Te went back to China in the 1920's for another visit, he left his son, Ong Chuan, who was then still a minor, in the Philippines obviously because he had long considered the Philippines his home. The domicile he established in 1895 is presumed to have continued up to, and beyond, April 11,

1899, for, as already adverted to, a domicile once acquired is not lost until a new one is gained. The only conclusion then can thus be drawn is that Ong Te was duly domiciled in the Philippines as of April 11, 1899, within the meaning of par. 4, Art. 17, of the Civil Code of 1889 and was, consequently, a Spanish subject, he qualified as a Filipino citizen under the provisions of Section 4 of the Philippine Bill of 1902. 8 It is true that Ong Chuan, the Ong brothers' father, subsequently sought naturalization in the belief that he was, all along, a Chinese citizen, but as the Report held: Protestants, however, make capital of the fact that both Ong Te and his son, Ong Chuan (protestee's father), appear to have been registered as Chinese citizens even long after the turn of the century. Worse, Ong Chuan himself believed the was alien, to the extent of having to seek admission as a Pilipino citizen through naturalization proceedings. The point, to our mind, is neither crucial nor substantial. Ong's status as a citizen is a matter of law, rather than of personal belief. It is what the law provides, and not what one thinks his status to be, which determines whether one is a citizen of a particular state or not. Mere mistake or misapprehension as to one's citizenship, it has been held, is not a sufficient cause or reason for forfeiture of Philippine citizenship; it does not even constitute estoppel (Palanca vs. Republic, 80 Phil. 578, 584). Too, estoppel applies only to questions of fact and not of law (Tanada v. Cuenco, L-10520, Feb. 28, 1957). 9 It is to be noted that the Report was unanimously approved by the Committee, and on November 28, 1972, approved without any objection by the Convention in plenary session. 10 I am not, of course, to be mistaken as acting as mouthpiece of Emil Ong, but in all candor, I speak from experience, because when the Convention approved the Report in question, I was one of its vice-presidents and the presiding officer. It is to be noted finally, that the matter was elevated to this Court (on a question involving Emil Ong's qualification to sit as member of the defunct Batasang Pambansa) 11 in which this Court allowed the use of the Committee Report. Faced with such positive acts of the Government, I submit that the question of the Ong's citizenship is a settled matter. Let it rest. It is true that Electoral Protest Nos. EP-07 and EP-08 of the Convention as well as G.R. No. 67201 of this Court, involved Emil Ong and not his brother; I submit, however, that what is sauce for the goose is sauce for the gander. I also submit that the fundamental question is whether or not we will overturn the unanimous ruling of 267 delegates, indeed, also of this Court.

CO vs. HRETFacts: The HRET declared that respondent Jose Ong, Jr. is a natural born Filipino citizen and a resident of Laoang, Northern Samar for voting purposes. The congressional election for the second district of NorthernSamar was held. Among the candidates who vied for the position of representative in the second legislativedistrict are the petitioners, Sixto Balinquit and Antonio Co and the private respondent, Jose Ong, Jr. RespondentOng was proclaimed the duly elected representative of the second district of Northern Samar. The petitioners filed election protests on the grounds that Jose Ong, Jr. is not a natural born citizen of thePhilippines and not a resident of the second district of Northern Samar. Issue: Whether or not Jose Ong, Jr. is a citizen of the Philippines. Held: Yes. In the year 1895, the private res pondents grandfather, Ong Te, arrived in the Philippines fromChina and established his residence in the municipality of Laoang, Samar. The father of the private respondent, Jose Ong Chuan was born in China in 1905 but was brought by Ong Te to Samar in the year 1915, he filed withthe court an application for naturalization and was declared a Filipino citizen.In 1984, the private respondent married a Filipina named Desiree Lim. For the elections of 1984 and1986, Jose Ong, Jr. registered himself as a voter of Laoang, Samar, and voted there during those elections.Under the 1973 Constitution, those born of Filipino fathers and those born of Filipino mothers with analien father were placed on equal footing. They were both considered as natural born citizens. Besides,

privater e s p o n d e n t d i d m o r e t h a n m e r e l y e x e r c i s e h i s r i g h t o f s u f f r a g e . H e h a s e s t a b l i s h e d h i s l i f e h e r e i n t h e Philippines.On the issue of residence, it is not required that a person should have a house in order to establish hisresidence and domicile. It is enough that he should live in the municipality or in a rented house or in that of afriend or relative. To require him to own property in order to be eligible to run for Congress would be tantamountto a property qualification. The Constitution only requires that the candidate meet the age, citizenship, voting and residence requirements.

THIRD DIVISION

JOEVANIE ARELLANO TABASA, Petitioner,

G.R. No. 125793

Present:

- versus -

QUISUMBING, J., Chairperson, CARPIO, CARPIO MORALES, TINGA, and

HON. COURT OF APPEALS, BUREAU OF IMMIGRATION and DEPORTATION and WILSON SOLUREN, Respondents.

VELASCO, JR., JJ.

Promulgated:

August 29, 2006

x-----------------------------------------------------------------------------------------x

DECISION

VELASCO, JR., J.:

Citizenship is a priceless possession. Former U.S. Chief Justice Earl Warren fittingly emphasized its crowning value when he wrote that it is mans basic right for it is nothing less than to have rights.[1] When a person loses citizenship, therefore, the State sees to it that its reacquisition may only be granted if the former citizen fully satisfies all conditions and complies with the applicable law. Without doubt, repatriation is not to be granted simply based on the vagaries of the former Filipino citizen.

The Case

The instant petition for review[2] under Rule 45 of the 1997 Rules of Civil Procedure contests the denial by the Court of Appeals (CA) of the Petition forHabeas Corpus interposed by petitioner Joevanie Arellano Tabasa from the Order of Summary Deportation issued by the Bureau of Immigration and Deportation (BID) for his return to the United States.

The Facts

The facts as culled by the CA from the records show that petitioner Joevanie Arellano Tabasa was a natural-born citizen of the Philippines. In 1968,[3] when petitioner was seven years old,[4] his father, Rodolfo Tabasa, became a naturalized citizen[5] of the United States. By derivative naturalization (citizenship derived from that of another as from a person who holds citizenship by virtue of naturalization[6]), petitioner also acquired American citizenship.

Petitioner arrived in the Philippines on August 3, 1995, and was admitted as a balikbayan for one year. Thereafter, petitioner was arrested and detained by agentWilson Soluren of the BID on May 23, 1996, pursuant to BID Mission Order No. LIV-96-72 in Baybay, Malay, Aklan; subsequently, he was brought to the BIDDetention Center in Manila.[7]

Petitioner was investigated by Special Prosecutor Atty. Edy D. Donato at the Law and Investigation Division of the BID on May 28, 1996; and on the same day, Tabasa was accused of violating Section 8, Chapter 3, Title 1, Book 3 of the 1987 Administrative Code, in a charge sheet which alleged:

1. That on 3 August 1995, respondent (petitioner herein [Tabasa]) arrived in the Philippines and was admitted as a balikbayan;

2. That in a letter dated 16 April 1996, Honorable Kevin Herbert, Consul General of [the] U.S. Embassy, informed the Bureau that respondents Passport No. 053854189 issued on June 10, 1994 in S an Francisco, California, U.S.A., had been revoked by the U.S. Department of State;

3. Hence, respondent [petitioner Tabasa] is now an undocumented and undesirable alien and may be summarily deported pursuant to Law and Intelligence Instructions No. 53 issued by then Commissioner Miriam Defensor Santiago to effect his deportation (Exhibit 3).[8]

The pertinent portion of the Herbert letter is as follows:

The U.S. Department of State has revoked U.S. passport 053854189 issued on June 10, 1994 in San Francisco, California under the name of Joevanie Arellano Tabasa, born onFebruary 21, 1959 in the Philippines. Mr. Tabasas passport has been revoked because he is the subject of an outstanding federal warrant of arrest issued on January 25, 1996 by the U.S. District Court for the Northern District of California, for violation of Section 1073, Unlawful Flight to Avoid Prosecution, of Title 18 of the United States Code. He is charged with one count of a felon in possession of a firearm, in violation of California Penal Code, Section 12021(A)(1), and one count of sexual battery, in violation of California Penal Code, Section 243.4 (D).[9]

The BID ordered petitioners deportation to his country of origin, the United States, on May 29, 1996, in the following summary deportation order: Records show that on 16 April 1996, Mr. Kevin F. Herbert, Consul General of the U.S. Embassy in Manila, filed a request with the Bureau to apprehend and deport the abovenamed [sic] respondent [petitioner Tabasa] on the ground that a standing warrant for several federal charges has been issued against him, and that the respondents Passport No. 053854189 has been revoked.

By reason thereof, and on the strength of Mission Order No. LIV-96-72, Intelligence operatives apprehended the respondent in Aklan on 23 May 1996.

In Schonemann vs. Commissioner Santiago, et al., (G.R. No. 81461 [sic, 81461 should be 86461], 30 May 1989), the Supreme Court ruled that if a foreign embassy cancels the passport of an alien, or does not reissue a valid passport to him, the alien loses the privilege to remain in the country. Further, under Office Memorandum Order No. 34 issued on 21 August 1989, summary deportation proceedings lie where the passport of the alien has expired.

It is, thus, apparent that respondent has lost his privilege to remain in the country.[10]

Petitioner filed before the CA a Petition for Habeas Corpus with Preliminary Injunction and/or Temporary Restraining Order[11] on May 29, 1996, which was docketed as CA-G.R. SP No. 40771. Tabasa alleged that he was not afforded due process; that no warrant of arrest for deportation may be issued by immigration authorities before a final order of deportation is made; that no notice of the cancellation of his passport was made by the U.S. Embassy; that he is entitled to admission or to a change of his immigration status as a nonquota immigrant because he is married to a Filipino citizen as provided in Section 13, paragraph (a) of the Philippine Immigration Act of 1940; and that he was a natural-born citizen of the Philippines prior to his derivative naturalization when he was seven years old due to the naturalization of his father, Rodolfo Tabasa, in 1968.

At the time Tabasa filed said petition, he was already 35 years old.[12] On May 30, 1996, the CA ordered the respondent Bureau to produce the person of the petitioner on June 3, 1996 and show the cause of petitioners detention, and restrained the Bureau from summarily deporting him. On June 3, 1996, the BID presented Tabasa before the CA; and on June 6, 1996, the CA granted both parties ten (10) days within which to file their memoranda, after which the case would be considered submitted for decision.[13] Meanwhile, the Commissioner of Immigration granted the petitioners temporary release on bail on a PhP 20,000.00 cash bond.[14]

However, on June 13, 1996, petitioner filed a Supplemental Petition alleging that he had acquired Filipino citizenship by repatriation in accordance with Republic Act No. 8171 (RA 8171), and that because he is now a Filipino citizen, he cannot be deported or detained by the respondent Bureau.[15]

The Ruling of the Court of Appeals

The CA, in its August 7, 1996 Decision,[16] denied Tabasas petition on the ground that he had not legally and successfully acquired by repatriationhis Filipino citizenship as provided in RA 8171. The court said that although he became an American citizen by derivative naturalization when his father was naturalized in 1968, there is no evidence to show that he lost his Philippine citizenship on account of political or economic necessity, as explicitly provided in Section 1, RA 8171the law governing the repatriation of natural-born Filipinos who have lost their citizenship. The affidavit does not state that political or economic necessity was the compelling reason for petitioners parents to give up their Filipino citizenship in 1968. Moreover, the court a quo found that petitioner Tabasa did not dispute the truth of the April 16, 1996 letter of the United States Consul General Kevin F. Herbert or the various warrants issued for his arrest by the United States court. The court a quo noted that after petitioner was ordered deported by the BID on May 29, 1996, he successively executed an Affidavit of Repatriation on June 6, 1996 and took an oath of allegiance to the Republic of the Philippines on June 13, 1996more than ten months after his arrival in the country on August 3, 1995. The appellate court considered petitioners repatriation as a last ditch effort to avoid deportation and prosecution in the United States. The appellate court concluded that his only reason to want to reacquire Filipino citizenship is to avoid criminal prosecution in the United States of America. The court a quo, therefore, ruled against Tabasa, whose petition is now before us.

The Issue

The only issue to be resolved is whether petitioner has validly reacquired Philippine citizenship under RA 8171. If there is no valid repatriation, then he can be summarily deported for his being an undocumented alien.

The Courts Ruling

The Court finds no merit in this petition.

RA 8171, An Act Providing for the Repatriation of Filipino Women Who Have Lost Their Philippine Citizenship by Marriage to A liens and of Natural-Born Filipinos, was enacted on October 23, 1995. It provides for the repatriation of only two (2) classes of persons, viz:

Filipino women who have lost their Philippine citizenship by marriage to aliens and natural-born Filipinos who have lost their Philippine citizenship, including their minor children, on account of political or economic necessity, may reacquire Philippine citizenship through repatriation in the manner provided in Section 4 of Commonwealth Act No. 63, as amended: Provided, That the applicant is not a:

(1) Person opposed to organized government or affiliated with any association or group of persons who uphold and teach doctrines opposing organized government;

(2) Person defending or teaching the necessity or propriety of violence, personal assault, or association for the predominance of their ideas;

(3)

Person convicted of crimes involving moral turpitude; or

(4)

Person suffering from mental alienation or incurable contagious diseases.[17] (Emphasis supplied.)

Does petitioner Tabasa qualify as a natural-born Filipino who had lost his Philippine citizenship by reason of political or economic necessity under RA 8171?

He does not.

Persons qualified for repatriation under RA 8171

To reiterate, the only persons entitled to repatriation under RA 8171 are the following:

a.

Filipino women who lost their Philippine citizenship by marriage to aliens; and

b.

Natural-born Filipinos including their minor children who lost their Philippine citizenship on account of political or economic necessity.

Petitioner theorizes that he could be repatriated under RA 8171 because he is a child of a natural-born Filipino, and that he lost his Philippine citizenship by derivative naturalization when he was still a minor.

Petitioner overlooks the fact that the privilege of repatriation under RA 8171 is available only to natural-born Filipinos who lost their citizenship on account of political or economic necessity, and to the minor children of said natural-born Filipinos. This means that if a parent who had renounced his Philippine citizenship due to political or economic reasons later decides to repatriate under RA 8171, his repatriation will also benefit his minor children according to the law. This includes a situation where a former Filipino subsequently had children while he was a naturalized citizen of a foreign country. The repatriation of the former Filipino will allow him to recover his natural-born citizenship and automatically vest Philippine citizenship on his children of jus sanguinis or blood relationship:[18] the children acquire the citizenship of their parent(s) who are natural-born Filipinos. To claim the benefit of RA 8171, however, the children must be of minor age at the time the petition for repatriation is filed by the parent. This is so because a child does not have the legal capacity for all acts of civil life much less the capacity

to undertake a political act like the election of citizenship. On their own, the minor children cannot apply for repatriation or naturalization separately from their parents. In the case at bar, there is no dispute that petitioner was a Filipino at birth. In 1968, while he was still a minor, his father was naturalized as an American citizen; and by derivative naturalization, petitioner acquired U.S. citizenship. Petitioner now wants us to believe that he is entitled to automatic repatriation as a child of natural-born Filipinos who left the country due to political or economic necessity. This is absurd. Petitioner was no longer a minor at the time of his repatriation on June 13, 1996. The privilege under RA 8171 belongs to children who are of minor age at the time of the filing of the petition for repatriation.

Neither can petitioner be a natural-born Filipino who left the country due to political or economic necessity. Clearly, he lost his Philippine citizenship by operation of law and not due to political or economic exigencies. It was his father who could have been motivated by economic or political reasons in deciding to apply for naturalization. The decision was his parents and not his. The privilege of repatriation under RA 8171 is extended directly to the natural-born Filipinos who could prove that they acquired citizenship of a foreign country due to political and economic reasons, and extended indirectly to the minor children at the time of repatriation.

In sum, petitioner is not qualified to avail himself of repatriation under RA 8171. However, he can possibly reacquire Philippine citizenship by availing of the Citizenship Retention and Re-acquisition Act of 2003 (Republic Act No. 9225) by simply taking an oath of allegiance to the Republic of the Philippines.

Where to file a petition for repatriation pursuant to RA 8171

Even if we concede that petitioner Tabasa can avail of the benefit of RA 8171, still he failed to follow the procedure for reacquisition of Philippine citizenship. He has to file his petition for repatriation with the Special Committee on Naturalization (SCN), which was designated to process petitions for repatriation pursuant to Administrative Order No. 285 (A.O. No. 285) dated August 22, 1996, to wit:

SECTION 1. Composition.The composition of the Special Committee on Naturalization, with the Solicitor General as Chairman, the Undersecretary of Foreign Affairs and the Director-General of the National Intelligence Coordinating Agency, as members, shall remain as constituted.

SEC. 2. Procedure.Any person desirous of repatriating or reacquiring Filipino citizenship pursuant to R.A. No. 8171 shall file a petition with the Special Committee on Naturalization which shall process the same. If their applications are approved[,] they shall take the necessary oath of allegiance to the Republic of the Philippines, after which they shall be deemed to have reacquired Philippine citizenship. The Commission on Immigration and Deportation shall thereupon cancel their certificate of registration (emphasis supplied).

SEC. 3. Implementing Rules.The Special Committee is hereby authorized to promulgate rules and regulations and prescribe the appropriate forms and the required fees for the processing of petitions.

SEC. 4. Effectivity.This Administrative Order shall take effect immediately.

In the Amended Rules and Regulations Implementing RA 8171 issued by the SCN on August 5, 1999, applicants for repatriation are required to submit documents in support of their petition such as their birth certificate and other evidence proving their claim to Filipino

citizenship.[19] These requirements were imposed to enable the SCN to verify the qualifications of the applicant particularly in light of the reasons for the renunciation of Philippine citizenship. What petitioner simply did was that he took his oath of allegiance to the Republic of the Philippines; then, executed an affidavit of repatriation, which he registered, together with the certificate of live birth, with the Office of the Local Civil Registrar of Manila. The said office subsequently issued him a certificate of such registration.[20] At that time, the SCN was already in place and operational by virtue of the June 8, 1995 Memorandum issued by President Fidel V. Ramos.[21] Although A.O. No. 285 designating the SCN to process petitions filed pursuant to RA 8171 was issued only on August 22, 1996, it is merely a confirmatory issuance according to the Court in Angat v. Republic.[22] Thus, petitioner should have instead filed a petition for repatriation before the SCN.

Requirements for repatriation under RA 8171

Even if petitionernow of legal agecan still apply for repatriation under RA 8171, he nevertheless failed to prove that his parents relinquished their Philippine citizenship on account of political or economic necessity as provided for in the law. Nowhere in his affidavit of repatriation did he mention that his parents lost their Philippine citizenship on account of political or economic reasons. It is notable that under the Amended Rules and Regulations Implementing RA 8171, the SCN requires a petitioner for repatriation to set forth, among others, the reason/s why petitioner lost his/her Filipino citizenship, whether by marriage in case of Filipino woman, or whether by political or economic necessity in case of [a] natural-born Filipino citizen who lost his/her Filipino citizenship. In case of the latter, such political or economic necessity should be specified.[23]

Petitioner Tabasa asserts, however, that the CA erred in ruling that the applicant for repatriation must prove that he lost his Philippine citizenship on account of political or economic necessity. He theorizes that the reference to political or economic reasons is merely descriptive, not restrictive, of the widely accepted reasons for naturalization in [a] foreign country.[24]

Petitioners argument has no leg to stand on.

A reading of Section 1 of RA 8171 shows the manifest intent of the legislature to limit the benefit of repatriation only to natural-born Filipinos who lost their Philippine citizenship on account of political or economic necessity, in addition to Filipino women who lost their Philippine citizenship by marriage to aliens. The precursor of RA 8171, Presidential Decree No. 725 (P.D. 725),[25] which was enacted on June 5, 1975 amending Commonwealth Act No. 63, also gives to the same groups of former Filipinos the opportunity to repatriate but without the limiting phrase, on account of political or economic necessity in relation to natural -born Filipinos. By adding the said phrase to RA 8171, the lawmakers clearly intended to limit the application of the law only to political or economic migrants, aside from the Filipino women who lost their citizenship by marriage to aliens. This intention is more evident in the following sponsorship speech of Rep. Andrea B. Domingo on House Bill No. 1248, the origin of RA 8171, to wit:

Ms. Domingo: x x x

From my experience as the Commissioner of the Bureau of Immigration and Deportation, I observed that there are only four types of Filipinos who leave the country.

The first is what we call the economic refugees who go abroad to work because there is no work to be found in the country. Then we have the political refugees who leave the country for fear of their lives because they are not in consonance with the prevailing policy of government. The third type is those who have committed crimes and would like to escape from the punishment of said crimes. Lastly, we have those Filipinos who feel that they are not Filipinos, thereby seeking other citizenship elsewhere.

Of these four types of Filipinos, Mr. Speaker, the first two have to leave the country not of choice, but rather out of sacrifice to look for a better life, as well as for a safer abode for themselves and their families. It is for these two types of Filipinos that this measure is being proposed for approval by this body. (Emphasis supplied.)

xxxx

x x x [I]f the body would recall, I mentioned in my short sponsorship speech the four types of Filipinos who leave their country. And the two typesthe economic and political refugeesare the ones being addressed by this proposed law, and they are not really Filipino women who lost their citizenship through marriage. We had a lot of problems with these people who left the country because of political persecution or because of pressing economic reasons, and after feeling that they should come back to the country and get back their citizenship and participate as they should in the affairs of the country, they find that it is extremely difficult to get their citizenship back because they are treated no different from any other class of alien.[26]

From these two sources, namely, P.D. 725 and the sponsorship speech on House Bill No. 1248, it is incontrovertible that the intent of our legislators in crafting Section 1 of RA 8171, as it is precisely worded out, is to exclude those Filipinos who have abandoned their country for reasons other than political or economic necessity.

Petitioner contends it is not necessary to prove his political or economic reasons since the act of renouncing allegiance to ones native country constitutes a necessary and unavoidable shifting of his political allegiance, and his fathers loss of Philippine citizenship through naturalization cannot therefore be said to be for any reason other than political or economic necessity.[27]

This argument has no merit.

While it is true that renunciation of allegiance to ones native country is necessarily a political act, it does not follow t hat the act is inevitably politically or economically motivated as alleged by petitioner. To reiterate, there are other reasons why Filipinos relinquish their Philippine citizenship. The sponsorship speech of former Congresswoman Andrea B. Domingo illustrates that aside from economic and political refugees, there are Filipinos who leave the country because they have committed crimes and would like to escape from punishment, and those who really feel that they are not Filipinos and that they deserve a better nationality, and therefore seek citizenship elsewhere.

Thus, assuming petitioner Tabasa is qualified under RA 8171, it is incumbent upon him to prove to the satisfaction of the SCN that the reason for his loss of citizenship was the decision of his parents to forfeit their Philippine citizenship for political or economic exigencies. He failed to undertake this crucial step, and thus, the sought relief is unsuccessful.

Repatriation is not a matter of right, but it is a privilege granted by the State. This is mandated by the 1987 Constitution under Section 3, Article IV, which provides that citizenship may be lost or reacquired in the manner provided by law. The State has the power to prescribe by law the qualifications, procedure, and requirements for repatriation. It has the power to determine if an applicant for repatriation meets the requirements of the law for it is an inherent power of the State to choose who will be its citizens, and who can reacquire citizenship once it is lost. If the applicant, like petitioner Tabasa, fails to comply with said requirements, the State is justified in rejecting the petition for repatriation.

Petitioner: an undocumented alien subject to summary deportation

Petitioner claims that because of his repatriation, he has reacquired his Philippine citizenship; therefore, he is not an undocumented alien subject to deportation.

This theory is incorrect. As previously explained, petitioner is not entitled to repatriation under RA 8171 for he has not shown that his case falls within the coverage of the law.

Office Memorandum No. 34 dated August 21, 1989 of the BID is enlightening on summary deportation: 2. The Board of Special Inquiry and the Hearing Board IV shall observe summary deportation proceedings in cases where the charge against the alien is overstaying, or the expiration or cancellation by his government of his passport. In cases involving overstaying aliens, BSI and the Hearing Board IV shall merely require the presentation of the aliens valid passport and shall decide the case on the basis thereof. 3. If a foreign embassy cancels the passport of the alien, or does not reissue a valid passport to him, the alien loses the privilege to remain in the country, under the Immigration Act, Sections 10 and 15 (Schonemann v. Santiago, et al., G.R. No. 81461 [sic, should be 86461], 30 May 1989). The automatic loss of the privilege obviates deportation proceedings. In such instance, the Board of Commissioners may issue summary judgment of deportation which shall be immediately executory.[28]

In addition, in the case of Schonemann v. Defensor Santiago, et al., this Court held: It is elementary that if an alien wants to stay in the Philippines, he must possess the necessary documents. One of these documents is a valid passport. There are, of course, exceptions where in the exercise of its sovereign prerogatives the Philippines may grant refugee status, refuse to extradite an alien, or otherwise allow him or her to stay here even if he [the alien] has no valid passport or Philippine visa. Boat people seeking residence elsewhere are examples. However, the grant of the privilege of staying in

the Philippines is discretionary on the part of the proper authorities. There is no showing of any grave abuse of discretion, arbitrariness, or whimsicality in the questioned summary judgment. x x x [29]

Petitioner Tabasa, whose passport was cancelled after his admission into the country, became an undocumented alien who can be summarily deported. His subsequent repatriation cannot bar such deportation especially considering that he has no legal and valid reacquisition of Philippine citizenship.

WHEREFORE, this petition for review is DISMISSED, and the August 7, 1996 Decision of the Court of Appeals is AFFIRMED. No costs to the petitioner.

SO ORDERED.

Tabasa vs CA Tabasa vs CA In 1968, when petitioner was seven years old, his father, Rodolfo Tabasa, became a naturalized citizen of the United States. By derivative naturalization (citizenship derived from that of another as from a person who holds citizenship by virtue of naturalization), petitioner also acquired American citizenship. Petitioner theorizes that he could be repatriated under RA 8171 because he is a child of a natural-born Filipino, and that he lost his Philippine citizenship by derivative naturalization when he was still a minor. ISSUE: Is Jeovanie Tabasa a natural-born Filipino who had lost his Philippine citizenship by reason of political or economic necessity under RA 8171? HELD: He does not. The only persons entitled to repatriation under RA 8171 are the following: a. Filipino women who lost their Philippine citizenship by marriage to aliens; and b. Natural-born Filipinos including their minor children who lost their Philippine citizenship on account of political or economic necessity. Petitioner overlooks the fact that the privilege of repatriation under RA 8171 is available only to natural-born Filipinos who lost their citizenship on account of political or economic necessity, and to the minor children of said natural-born Filipinos. Petitioner overlooks the fact that the privilege of repatriation under RA 8171 is available only to natural-born Filipinos who lost their citizenship on account of political or economic necessity, and to the minor children of said natural-born Filipinos. The privilege under RA 8171 belongs to children who are of minor age at the time of the filing of the petition for repatriation.

EN BANC

NESTOR A. JACOT, Petitioner, G.R. No. 179848 Present: PUNO, C.J., QUISUMBING, YNARES-SANTIAGO, CARPIO, AUSTRIA-MARTINEZ, CORONA, CARPIO MORALES, AZCUNA, - versus TINGA, CHICO-NAZARIO, VELASCO, JR., NACHURA, REYES, DE CASTRO,* and BRION,** JJ.

Promulgated: ROGEN T. DAL and COMMISSION ON ELECTIONS, November 27, 2008 Respondents. x---------------------------- ---------------------x

DECISION

CHICO-NAZARIO, J.:

Petitioner Nestor A. Jacot assails the Resolution[1] dated 28 September 2007 of the Commission on Elections (COMELEC) En Banc in SPA No. 07-361, affirming the Resolution dated 12 June 2007 of the COMELEC Second Division[2] disqualifying him from running for the position of Vice-Mayor of Catarman,Camiguin, in the 14 May 2007 National and Local Elections, on the ground that he failed to make a personal renouncement of his United States (US) citizenship.

Petitioner was a natural born citizen of the Philippines, who became a naturalized citizen of the US on 13 December 1989. [3]

Petitioner sought to reacquire his Philippine citizenship under Republic Act No. 9225, otherwise known as the Citizenship Retention and Re-Acquisition Act. He filed a request for the administration of his Oath of Allegiance to the Republic of the Philippines with the Philippine Consulate General (PCG) of Los Angeles,California. The Los Angeles PCG issued on 19 June 2006 an Order of Approval[4] of petitioners request, and on the same day, petitioner took his Oath of Allegiance to the Republic of the Philippines before Vice Consul Edward C. Yulo. [5] On 27 September 2006, the Bureau of Immigration issued Identification Certificate No. 06-12019 recognizing petitioner as a citizen of the Philippines.[6]

Six months after, on 26 March 2007, petitioner filed his Certificate of Candidacy for the Position of Vice-Mayor of the Municipality of Catarman, Camiguin. [7]

On 2 May 2007, respondent Rogen T. Dal filed a Petition for Disqualification[8] before the COMELEC Provincial Office in Camiguin against petitioner, arguing that the latter failed to renounce his US citizenship, as required under Section 5(2) of Republic Act No. 9225, which reads as follows:

Section 5. Civil and Political Rights and Liabilities.Those who retain or reacquire Philippine citizenship under this Act shall enjoy full civil and political rights and be subject to all attendant liabilities and responsibilities under existing laws of the Philippines and the following conditions:

xxxx

(2) Those seeking elective public office in the Philippines shall meet the qualifications for holding such public office as required by the Constitution and existing laws and, at the time of the filing of the certificate of candidacy, make a personal and sworn renunciation of any and all foreign citizenship before any public officer authorized to administer an oath.

In his Answer[9] dated 6 May 2007 and Position Paper[10] dated 8 May 2007, petitioner countered that his Oath of Allegiance to the Republic of the Philippinesmade before the Los Angeles PCG and the oath contained in his Certificate of Candidacy operated as an effective renunciation of his foreign citizenship.

In the meantime, the 14 May 2007 National and Local Elections were held. Petitioner garnered the highest number of votes for the position of Vice Mayor.

On 12 June 2007, the COMELEC Second Division finally issued its Resolution[11] disqualifying the petitioner from running for the position of Vice-Mayor ofCatarman, Camiguin, for failure to make the requisite renunciation of his US citizenship. The COMELEC Second

Division explained that the reacquisition of Philippine citizenship under Republic Act No. 9225 does not automatically bestow upon any person the privilege to run for any elective public office. It additionally ruled that the filing of a Certificate of Candidacy cannot be considered as a renunciation of foreign citizenship. The COMELEC Second Division did not consider Valles v. COMELEC[12] and Mercado

v. Manzano[13] applicable to the instant case, since Valles and Mercado were dual citizens since birth, unlike the petitioner who lost his Filipino citizenship by means of naturalization. The COMELEC, thus, decreed in the aforementioned Resolution that:

ACCORDINGLY, NESTOR ARES JACOT is DISQUALIFIED to run for the position of Vice-Mayor of Catarman, Camiguin for the May 14, 2007 National and Local Elections. If proclaimed, respondent cannot thus assume the Office of Vice-Mayor of said municipality by virtue of such disqualification.[14]

Petitioner filed a Motion for Reconsideration on 29 June 2007 reiterating his position that his Oath of Allegiance to the Republic of the Philippines before the Los Angeles PCG and his oath in his Certificate of Candidacy sufficed as an effective renunciation of his US citizenship. Attached to the said Motion was an Oath of Renunciation of Allegiance to the United States and Renunciation of Any and All Foreign Citizenship dated 27 June 2007, wherein petitioner explicitly renounced hisUS citizenship.[15] The COMELEC en banc dismissed petitioners Motion in a Resolution[16] dated 28 September 2007 for lack of merit.

Petitioner sought remedy from this Court via the present Special Civil Action for Certiorari under Rule 65 of the Revised Rules of Court, where he presented for the first time an Affidavit of Renunciation of Allegi ance to the United States and Any and All Foreign Citizenship[17] dated 7 February 2007. He avers that he executed an act of renunciation of his US citizenship, separate from the Oath of Allegiance to the Republic of the Philippines he took before the Los Angeles PCG and his filing of his Certificate of Candidacy, thereby changing his theory of the case during the appeal. He attributes the delay in the presentation of the affidavit to his former counsel, Atty. Marciano Aparte, who allegedly advised him that said piece of evidence was unnecessary but who, nevertheless, made him execute an identical document entitled Oath of Renunciation of Allegiance to the United States and Renunciation of Any and All Foreign Citizenship on 27 June 2007 after he had already filed his Certificate of Candidacy.[18]

Petitioner raises the following issues for resolution of this Court:

WHETHER OR NOT PUBLIC RESPONDENT EXERCISED GRAVE ABUSE OF DISCRETION WHEN IT HELD THAT PETITIONER FAILED TO COMPLY WITH THE PROVISIONS OF R.A. 9225, OTHERWISE KNOWN AS THE CITIZENSHIP RETENTION AND RE-ACQUISITION ACT OF 2003, SPECIFICALLY SECTION 5(2) AS TO THE REQUIREMENTS FOR THOSE SEEKING ELECTIVE PUBLIC OFFICE;

II

WHETHER OR NOT PUBLIC RESPONDENT EXERCISED GRAVE ABUSE OF DISCRETION WHEN IT HELD THAT PETITIONER FAILED TO COMPLY WITH THE PROVISIONS OF THE COMELEC RULES OF PROCEDURE AS REGARDS THE PAYMENT OF THE NECESSARY MOTION FEES; AND

III

WHETHER OR NOT UPHOLDING THE DECISION OF PUBLIC RESPONDENT WOULD RESULT IN THE FRUSTRATION OF THE WILL OF THE PEOPLE OF CATARMAN, CAMIGUIN.[19]

The Court determines that the only fundamental issue in this case is whether petitioner is disqualified from running as a candidate in the 14 May 2007 local elections for his failure to make a personal and sworn renunciation of his US citizenship.

This Court finds that petitioner should indeed be disqualified.

Contrary to the assertions made by petitioner, his oath of allegiance to the Republic of the Philippines made before the Los Angeles PCG and his Certificate of Candidacy do not substantially comply with the requirement of a personal and sworn renunciation of foreign citizenship because these are distinct requirements to be complied with for different purposes.

Section 3 of Republic Act No. 9225 requires that natural-born citizens of the Philippines, who are already naturalized citizens of a foreign country, must take the following oath of allegiance to the Republic of the Philippines to reacquire or retain their Philippine citizenship:

SEC. 3. Retention of Philippine Citizenship.Any provision of law to the contrary notwithstanding, naturalborn citizens of the Philippines who have lost their Philippine citizenship by reason of their naturalization as citizens of a foreign country are hereby deemed to have reacquired Philippine citizenship upon taking the following oath of allegiance to the Republic:

I __________ solemnly swear (or affirm) that I will support and defend the Constituti on of the Republic of the Philippines and obey the laws and legal orders promulgated by the duly constituted authorities of the Philippines; and I hereby declare that I recognize and accept the supreme authority of the Philippines and will maintain true faith and allegiance thereto; and that I impose this obligation upon myself voluntarily, without mental reservation or purpose of evasion.

Natural-born citizens of the Philippines who, after the effectivity of this Act, become citizens of a foreign country shall retain their Philippine citizenship upon taking the aforesaid oath.

By the oath dictated in the afore-quoted provision, the Filipino swears allegiance to the Philippines, but there is nothing therein on his renunciation of foreign citizenship. Precisely, a situation might arise under Republic Act No. 9225 wherein said Filipino has dual citizenship by also reacquiring or retaining his Philippine citizenship, despite his foreign citizenship.

The afore-quoted oath of allegiance is substantially similar to the one contained in the Certificate of Candidacy which must be executed by any person who wishes to run for public office in Philippine elections. Such an oath reads:

I am eligible for the office I seek to be elected. I will support and defend the Constitution of the Philippines and will maintain true faith and allegiance thereto; that I will obey the laws, legal orders and decrees promulgated by the duly constituted authorities of the Republic of the Philippines; and that I impose this obligation upon myself voluntarily, without mental reservation or purpose of evasion. I hereby certify that the facts stated herein are true and correct of my own personal knowledge.

Now, Section 5(2) of Republic Act No. 9225 specifically provides that:

Section 5. Civil and Political Rights and Liabilities.Those who retain or reacquire Philippine citizenship under this Act shall enjoy full civil and political rights and be subject to all attendant liabilities and responsibilities under existing laws of the Philippines and the following conditions:

xxxx

(2) Those seeking elective public office in the Philippines shall meet the qualifications for holding such public office as required by the Constitution and existing laws and, at the time of the filing of the certificate of candidacy, make a personal and sworn renunciation of any and all foreign citizenship before any public officer authorized to administer an oath.

The law categorically requires persons seeking elective public office, who either retained their Philippine citizenship or those who reacquired it, to make a personal and sworn renunciation of any and all foreign citizenship before a public officer authorized to administer an oath simultaneous with or before the filing of the certificate of candidacy.[20]

Hence, Section 5(2) of Republic Act No. 9225 compels natural-born Filipinos, who have been naturalized as citizens of a foreign country, but who reacquired or retained their Philippine citizenship (1) to take the oath of allegiance under Section 3 of Republic Act No. 9225, and (2) for those seeking elective public offices in the Philippines, to additionally execute a personal and sworn renunciation of any and all foreign citizenship before an authorized public officer prior or simultaneous to the filing of their certificates of candidacy, to qualify as candidates in Philippine elections.

Clearly Section 5(2) of Republic Act No. 9225 (on the making of a personal and sworn renunciation of any and all foreign citizenship) requires of the Filipinos availing themselves of the benefits under the said Act to accomplish an undertaking other than that which they have presumably complied with under Section 3 thereof (oath of allegiance to the Republic of the Philippines). This is made clear in the discussion of the Bicameral Conference Committee on Disagreeing Provisions of House Bill No. 4720 and Senate Bill No. 2130 held on 18

August 2003 (precursors of Republic Act No. 9225), where the Hon. Chairman Franklin Drilon and Hon. Representative Arthur Defensor explained to Hon. Representative Exequiel Javier that the oath of allegiance is different from the renunciation of foreign citizenship:

CHAIRMAN DRILON. Okay. So, No. 2. Those seeking elective public office in the Philippines shall meet the qualifications for holding such public office as required by the Constitution and existing laws and, at the time of the filing of the certificate of candidacy, make a personal and sworn renunciation of any and all foreign citizenship before any public officer authorized to administer an oath. I think its very good, ha? No problem?

REP. JAVIER. I think its already covered by the oath.

CHAIRMAN DRILON. Renouncing foreign citizenship.

REP. JAVIER. Ah but he has taken his oath already.

CHAIRMAN DRILON. Nono, renouncing foreign citizenship.

xxxx

CHAIRMAN DRILON. Can I go back to No. 2. Whats your problem, Boy? Those seeking elective office in the Philippines.

REP. JAVIER. They are trying to make him renounce his citizenship thinking that ano

CHAIRMAN DRILON. His American citizenship.

REP. JAVIER. To discourage him from running?

CHAIRMAN DRILON. No.

REP. A.D. DEFENSOR. No. When he runs he will only have one citizenship. When he runs for office, he will have only one. (Emphasis ours.)

There is little doubt, therefore, that the intent of the legislators was not only for Filipinos reacquiring or retaining their Philippine citizenship under Republic Act No. 9225 to take their oath of allegiance to the Republic of the Philippines, but also to explicitly renounce their foreign citizenship if they wish to run for elective posts in the Philippines. To qualify as a candidate in Philippine elections, Filipinos must only have one citizenship, namely, Philippine citizenship.

By the same token, the oath of allegiance contained in the Certificate of Candidacy, which is substantially similar to the one contained in Section 3 of Republic Act No. 9225, does not constitute the personal and sworn renunciation sought under Section 5(2) of Republic Act No. 9225. It bears to emphasize that the said oath of allegiance is a general requirement for all those who wish to run as candidates in Philippine elections; while the renunciation of foreign citizenship is an additional requisite only for those who have retained or reacquired Philippine citizenship under Republic Act No. 9225 and who seek elective public posts, considering their special circumstance of having more than one citizenship.

Petitioner erroneously invokes the doctrine in Valles[21] and Mercado,[22] wherein the filing by a person with dual citizenship of a certificate of candidacy, containing an oath of allegiance, was already considered a renunciation of foreign citizenship. The ruling of this Court in Valles and Mercado is not applicable to the present case, which is now specially governed by Republic Act No. 9225, promulgated on 29 August 2003.

In Mercado, which was cited in Valles, the disqualification of therein private respondent Manzano was sought under another law, Section 40(d) of the Local Government Code, which reads:

SECTION 40. Disqualifications. The following persons are disqualified from running for any elective local position:

xxxx

(d) Those with dual citizenship.

The Court in the aforesaid cases sought to define the term dual citizenship vis--vis the concept of dual allegiance. At the time this Court decided the cases ofValles and Mercado on 26 May 1999 and 9 August 2000, respectively, the more explicitly worded requirements of Section 5(2) of Republic Act No. 9225 were not yet enacted by our legislature.[23]

Lopez v. Commission on Elections[24] is the more fitting precedent for this case since they both share the same factual milieu. In Lopez, therein petitioner Lopez was a natural-born Filipino who lost his Philippine citizenship after he became a naturalized US citizen. He later reacquired his Philippine citizenship by virtue of Republic Act No. 9225. Thereafter, Lopez filed his candidacy for a local elective position, but failed to make a personal and sworn renunciation of his foreign citizenship. This Court unequivocally declared that despite having garnered the highest number of votes in the election, Lopez is nonetheless disqualified as a candidate for a local elective position due to his failure to comply with the requirements of Section 5(2) of Republic Act No. 9225.

Petitioner presents before this Court for the first time, in the instant Petition for Certiorari, an Affidavit of Renunciation of Allegiance to the United States and Any and All Foreign Citizenship,[25] which he supposedly executed on 7 February 2007, even before he filed his Certificate of Candidacy on 26 March 2007. With the said Affidavit, petitioner puts forward in the Petition at bar a new theory of his case that he complied with the requirement of making a personal and sworn renunciation of his foreign citizenship before filing his Certificate of Candidacy. This new theory constitutes a radical change from the earlier position he took before the COMELEC that he complied with the requirement of renunciation by his oaths of allegiance to the Republic of the Philippines made before the Los Angeles PCGand in his Certificate of Candidacy, and that there was no more need for a separate act of renunciation.

As a rule, no question will be entertained on appeal unless it has been raised in the proceedings below. Points of law, theories, issues and arguments not brought to the attention of the lower court, administrative agency or quasi-judicial body need not be considered by a reviewing court, as they cannot be raised for the first time at that late stage. Basic considerations of fairness and due process impel this rule.[26] Courts have neither the time nor the resources to accommodate parties who chose to go to trial haphazardly.[27]

Likewise, this Court does not countenance the late submission of evidence.[28] Petitioner should have offered the Affidavit dated 7 February 2007 during the proceedings before the COMELEC.

Section 1 of Rule 43 of the COMELEC Rules of Procedure provides that In the absence of any applicable provisions of these Rules, the pertinent provisions of the Rules of Court in the Philippines shall be applicable by analogy or in suppletory character and effect. Section 34 of Rule 132 of the Revised Rules of Court categorically enjoins the admission of evidence not formally presented:

SEC. 34. Offer of evidence. - The court shall consider no evidence which has not been formally offered. The purpose for which the evidence is offered must be specified.

Since the said Affidavit was not formally offered before the COMELEC, respondent had no opportunity to examine and controvert it. To admit this document would be contrary to due process. [29] Additionally, the piecemeal presentation of evidence is not in accord with orderly justice.[30]

The Court further notes that petitioner had already presented before the COMELEC an identical document, Oath of Renunciation of Allegiance to the United States and Renunciation of Any and All Foreign Citizenship executed on 27 June 2007, subsequent to his filing of his Certificate of Candidacy on 26 March 2007. Petitioner attached the said Oath of 27 June 2007 to his Motion for Reconsideration with the COMELEC en banc. The COMELEC en banc eventually refused to reconsider said document for being belatedly executed. What was extremely perplexing, not to mention suspect, was that petitioner did not submit the Affidavit of 7 February 2007 or mention it at all in the proceedings before the COMELEC, considering that it could have easily won his case if it was actually executed on and in existence before the filing of his Certificate of Candidacy, in compliance with law.

The justification offered by petitioner, that his counsel had advised him against presenting this crucial piece of evidence, is lame and unconvincing. If the Affidavit of 7 February 2007 was in existence all along, petitioners counsel, and even petitioner himself, could have easily adduced it to be a crucial piece of evidence to prove compliance with the requirements of Section 5(2) of Republic Act No. 9225. There was no apparent danger for petitioner to submit as much evidence as possible in support of his case, than the risk of presenting too little for which he could lose.

And even if it were true, petitioners excuse for the late presentation of the Affidavit of 7 February 2007 will not change the outcome of petitioners case.

It is a well-settled rule that a client is bound by his counsels conduct, negligence, and mistakes in handling the case, and the client cannot be heard to complain that the result might have been different had his lawyer proceeded differently. [31] The only exceptions to the general rule -- that a client is bound by the mistakes of his counsel -- which this Court finds acceptable are when the reckless or gross negligence of counsel deprives the client of due process of law, or when the application of the rule results in the outright deprivation of ones property through a technicality.[32] These exceptions are not attendant in this case.

The Court cannot sustain petitioners averment that his counsel was grossly negligent in deciding against the presentation of the Affidavit of 7 February 2007during the proceedings before the COMELEC. Mistakes of attorneys as to the competency of a witness; the sufficiency, relevancy or irrelevancy of certain evidence; the proper defense or the burden of proof, failure to introduce evidence, to summon witnesses and to argue the case -- unless they prejudice the client and prevent him from properly presenting his case -- do not constitute gross incompetence or negligence, such that clients may no longer be bound by the acts of their counsel.[33]

Also belying petitioners claim that his former counsel was grossly negligent was the fact that petitioner continuously used his former counsels theory of the case. Even when the COMELEC already rendered an adverse decision, he persistently argues even to this Court that his oaths of allegiance to the Republic of thePhilippines before the Los Angeles PCG and in his Certificate of Candidacy amount to the renunciation of foreign citizenship which the law requires. Having asserted the same defense in the instant Petition, petitioner only demonstrates his continued reliance on and complete belief in the position taken by his former counsel, despite the formers incongruous allegations that the latter has been grossly negligent.

Petitioner himself is also guilty of negligence. If indeed he believed that his counsel was inept, petitioner should have promptly taken action, such as discharging his counsel earlier and/or insisting on the submission of his Affidavit of 7 February 2007 to the COMELEC, instead of waiting until a decision was rendered disqualifying him and a resolution issued dismissing his motion for reconsideration; and, thereupon, he could have heaped the blame on his former counsel. Petitioner could not be so easily allowed to escape the consequences of his former counsels acts, because, otherwise, it would render court proceedings indefinite, tentative, and subject to reopen ing at any time by the mere subterfuge of replacing counsel. [34]

Petitioner cites De Guzman v. Sandiganbayan,[35] where therein petitioner De Guzman was unable to present a piece of evidence because his lawyer proceeded to file a demurrer to evidence, despite the Sandiganbayans denial of his prior leave to do so. The wrongful

insistence of the lawyer in filing a demurrer to evidence had totally deprived De Guzman of any chance to present documentary evidence in his defense. This was certainly not the case in the Petition at bar.

Herein, petitioner was in no way deprived of due process. His counsel actively defended his suit by attending the hearings, filing the pleadings, and presenting evidence on petitioners behalf. Moreover, petitioners cause was not defeated by a mere technicality, but because of a mistaken reliance on a doctrine which is not applicable to his case. A case lost due to an untenable legal position does not justify a deviation from the rule that clients are bound by the acts and mistakes of their counsel.[36]

Petitioner also makes much of the fact that he received the highest number of votes for the position of Vice-Mayor of Catarman during the 2007 local elections. The fact that a candidate, who must comply with the election requirements applicable to dual citizens and failed to do so, received the highest number of votes for an elective position does not dispense with, or amount to a waiver of, such requirement.[37] The will of the people as expressed through the ballot cannot cure the vice of ineligibility, especially if they mistakenly believed that the candidate was qualified. The rules on citizenship qualifications of a candidate must be strictly applied. If a person seeks to serve the Republic of the Philippines, he must owe his loyalty to this country only, abjuring and renouncing all fealty and fidelity to any other state.[38] The application of the constitutional and statutory provisions on disqualification is not a matter of popularity. [39]

WHEREFORE, the instant appeal is DISMISSED. The Resolution dated 28 September 2007 of the COMELEC en banc in SPA No. 07361, affirming the Resolution dated 12 June 2007 of the COMELEC Second Division, is AFFIRMED. Petitioner is DISQUALIFIED to run for the position of Vice-Mayor of Catarman, Camiguin in the 14 May 2007 National and Local Elections, and if proclaimed, cannot assume the Office of Vice-Mayor of said municipality by virtue of such disqualification. Costs against petitioner.

SO ORDERED.

You might also like